SSC Combined Higher Secondary Level (10+2) Examination (1st Shift) Tier-I Held on 21 October, 2012 Question Paper With Answer Key

SSC Combined Higher Secondary Level (10+2) Examination (1st Shift) Tier-I Held on 21 October, 2012
SSC Combined Higher Secondary Level (10+2) Examination (1st Shift) Tier-I Held on 21 October, 2012 Question Paper With Answer Key

Staff Selection Commission Combined Higher Secondary Level (10+2) Examination

(1st Shift) Tier-I Held on 21 October, 2012

Part-I

General Intelligence

Directions-(Q. 1-6) select the related letter/word/number from the given alternatives.

1. Jewellery : Gold : : Furniture : ?

(A)  Table

(B)  Tree

(C)  Wood

(D)  Paint

Answer: (C)

2. Author : Novel : : Choreographer : ?

(A)  Music

(B)  Picture

(C)  Make-up

(D)  Dance

Answer: (D)

3. Rig : Ofd : : Met : ?

(A)  Jbq

(B)  Kcr

(C)  Jer

(D)  Kbq

Answer: (A)

4. BDAC : FHEG : : NPMO : ?

(A)  RQTS

(B)  QTRS

(C)  RTQS

(D)  TRQS

Answer: (C)

5. 49 : 64 : : 144 : ?

(A)  186

(B)  121

(C)  256

(D)  169

Answer: (D)

6. SHOE : NCJZ : : REWA : ?

(A)  WJBF

(B)  CITY

(C)  MZRV

(D)  CAAR

Answer: (C)

   Directions-(Q. 7 to 10) Select the one which is different from the other three responses.

7.

(A)  Tongue

(B)  Teeth

(C)  Nose

(D)  Ear

Answer: (B)

8.

(A)  Petrol – Car

(B)  Electricity – Television

(C)  Ink – Pen

(D)  Dust – Vacuum cleaner

Answer: (D)

9.

(A)  Light – Heavy

(B)  Crime – Blame

(C)  Short – Long

(D)  Man – Woman

Answer: (B)

10.

(A)  343

(B)  512

(C)  729

(D)  144

Answer: (D)

   Directions-(Q. 11 and 12) Which one of the given responses would be a meaningful order of the following?

11. (1) Destination (2) Booking

(3) Boarding      (4) Travel

(5) Planning

(A)  1, 2, 3, 4, 5

(B)  3, 4, 5, 1, 2

(C)  4, 3, 1, 2, 5

(D)  5, 2, 3, 4, 1

Answer: (D)

12. (1) Diagnosis (2) Doctor

(3) Sick              (4) Treatment

(5) Recovery

(A)  1, 2, 3, 4, 5

(B)  3, 2, 1, 4, 5

(C)  2, 1, 3, 4, 5

(D)  4, 5, 1, 3, 2

Answer: (B)

13. Arrange the following words as per order in the dictionary-

(1) Genuine        (2) Genesis

(3) Gender         (4) Gentle

(5) General

(A)  4, 5, 3, 2, 1

(B)  1, 5, 4, 3, 2

(C)  3, 5, 2, 4, 1

(D)  2, 5, 3, 1, 4

Answer: (C)

   Directions-(Q. 14 to 17) A series is given, with one term/number/letters missing. Choose the correct alternative from the given ones that will complete the series.

14. V, VIII, XI, XIV, ?, XX

(A)  IX

(B)  XXIII

(C)  XV

(D)  XVII

Answer: (D)

15.

3463       (2218)              1245

5324       ( ? )                  3626

(A)  2312

(B)  1142

(C)  1698

(D)  1592

Answer: (C)

16. XGH, WIJ, VKL, UMN, ?

(A)  HOW

(B)  UJI

(C)  TOP

(D)  SOP

Answer: (C)

17. SCD, TEF, UGH, ?, WKL

(A)  CMN

(B)  UJI

(C)  VIJ

(D)  VJI

Answer: (C)

18. K is more beautiful than B. B is not as beautiful as Y. J is not as beautiful as B or Y. Whose beauty is in the least degree?

(A)  Y

(B)  K

(C)  B

(D)  J

Answer: (D)

19. A person’s present age is two-fifth of the age of his mother. After 8 He will be one-half of the age of his mother. How old is the mother at present?

(A)  32 years

(B)  36 years

(C)  40 years

(D)  48 years

Answer: (C)

20. Raghu and Babu are twins. Babu’s sister is Reema. Reema’s husband is Rajan. Raghu’s mother is Lakshmi. Lakshmi’s husband is Rajsh. How is Rajesh related to Rajan?

(A)  Uncle

(B)  Son-in-law

(C)  Father-in-law

(D)  Cousin

Answer: (C)

21. If – stands for ÷, + stands for ×, ÷ stands for – and × stands for +, find out which one is correct?

(A)  49 + 7 – 3 × 5 ÷ 8 = 20

(B)  49 – 7 + 3 ÷ 5 × 8 = 24

(C)  49 × 7 + 3 ÷ 5 – 8 = 16

(D)  49 ÷ 7 ×3 + 5 – 8 = 26

Answer: (B)

   Directions-(Q. 22 and 23) From the given alternatives select the word which cannot be formed using the letters of the given word.

22. ANNIVERSARY

(A)  SAVE

(B)  VIEW

(C)  YARN

(D)  VERY

Answer: (B)

23. REMEMBRANCE

(A)  REMEMBER

(B)  MEMBRANE

(C)  NUMBER

(D)  EMBRACE

Answer: (C)

24. If MADRAS is written as DAMSAR, how can MUMBAI be written in that code?

(A)  BAIUMM

(B)  MUMIAB

(C)  IABMUM

(D)  MBIAUM

Answer: (B)

25. If DREAM is coded as 78026 and CHILD is coded as 53417, how can LEADER be coded?

(A)  102078

(B)  102708

(C)  102087

(D)  102780

Answer: (B)

26. If Rat is called Dog, Dog is called Mongoose, Mongoose is called Lion, Lion is called Snake and Snake is called Elephant, which is reared as a pet?

(A)  Rat

(B)  Dog

(C)  Mongoose

(D)  Lion

Answer: (C)

   Directions-(Q. 27 to 31) Select the missing number from the given responses.

27. 7, 8, 10, 13, 17, ?

(A)  23

(B)  20

(C)  22

(D)  21

Answer: (C)

28. 69, 72, 78, 87, ?, 114

(A)  93

(B)  96

(C)  111

(D)  99

Answer: (D)

29. 1 8  ?  2  7

(A)  41

(B)  64

(C)  35

(D)  61

Answer: (B)

30. 48, 82, 44, 77, 40, 72, ?

(A)  36

(B)  40

(C)  76

(D)  70

Answer: (A)

31. If 36   (146)  437

then        574   (?)     641

(A)  236

(B)  356

(C)  250

(D)  134

Answer: (D)

32. The number of students in a course increases every year in a college. Find out the number in 2010 from the following information-

(A)  55

(B)  65

(C)  70

(D)  75

Answer: (B)

33. Prakash travelled 6 km northward, then turned left and travelled 4 km, then turned left and travelled 6 km. How far was Prakash from the starting point?

(A)  10 km

(B)  8 km

(C)  6 km

(D)  4 km

Answer: (D)

34. Two cars leave the same place at the same time. One runs at 20 km/hr towards north and the other at 15 km/hr towards east. What will be the distance between them (in km) after 2 hours of travelling?

(A)  70

(B)  10

(C)  25

(D)  50

Answer: (D)

35. If Police is called Teacher, Teacher is called Politician, Politician is called Doctor, Doctor is called Surgeon, who will arrest the criminals?

(A)  Police

(B)  Lawyer

(C)  Teacher

(D)  Doctor

Answer: (C)

   Directions-(Q. 36 and 37) Two statements are given followed by two/four conclusions I, II, III and IV. You have to consider the two statements to be true even if they seem to be at variance from commonly known facts. You have to decide which of the given conclusions, if any, follow from the given statements.

36. Statements :

(1) Some buses are four wheelers.

(2) All four wheelers are vans.

Conclusions:

(I) Some vans are buses.

(II) Some buses are vans.

(A)  Only conclusion I follows

(B)  Only conclusion II follows

(C)  Either conclusion I or II follows

(D)  Both conclusions I and II follow

Answer: (D)

37. Statements:

(1) All peacocks are lions.

(2) Some tigers are peacocks.

Conclusions:

(I) Some lions are not tigers.

(II) All tigers are lions.

(III) Some tigers are lions.

(IV) All peacocks are tigers

(A)  Only conclusion I follows

(B)  Only conclusion II follows

(C)  Only conclusion III follows

(D)  Only conclusion IV follows

Answer: (A)

   Directions –(Q. 38 and 39) If a mirror is placed on the line XY, then which of the answer figures is the correct image of the given question figure?

38. 

Answer: (B)

39. 

Answer: (C)

40. How many cubes are there in this diagram?

(A)  16

(B)  12

(C)  10

(D)  8

Answer: (B)

41. How many educated people are employed?

(A)  18

(B)  26

(C)  24

(D)  16

Answer: (A)

42. From the details, find out the number of people who do not read any newspaper-

(A)  195

(B)  135

(C)  175

(D)  75

Answer: (A)

43. Find out the number of people who do not play any game-

(A)  9

(B)  24

(C)  18

(D)  15

Answer: (A)

44. Which answer figure will complete the pattern in the question figure?

Answer: (D)

   Directions-(Q. 45 to 47) From the given answer figures, select the one in which the question figure is hidden/embedded.

45. 

Answer: (A)

46. 

Answer: (B)

47. 

Answer: (C)

48. If the following pattern is drawn on a transparent rectangular sheet and folded along the dotted line, how does it appear?

Answer: (D)

49. A triangular sheet of paper has been folded and punched as shown in the following series of figures. How will it appear when opened?

Answer: (D)

50. A word is represented by only one set of numbers as given in any one of the alternatives. The sets of numbers given in the alternatives are represented by two classes of alphabets as in two matrices given below. The columns and rows of Matrix I are numbered from 0 to 4 and that of Matrix II are numbered from 5 to 9. A letter from these matrices can be represented first by its row and next by its column, e.g., M can be represented by 14, 21, etc. and P can be represented by 59, 78, etc. Similarly, you have to identify the set for the word MIST.

(A)  02, 58, 03, 86

(B)  40, 77, 34, 98

(C)  14, 89, 22, 88

(D)  40, 58, 03, 56

Answer: (D)

For Visually Handicapped Candidates Only

Directions-(Q. 38 and 39) Select the related word from the given alternatives.

38. Peacock : India : : Bear : ?

(A)  Australia

(B)  Russia

(C)  America

(D)  England

Answer: (B)

39. Menu : Food : : Catalogue : ?

(A)  Library

(B)  Newspaper

(C)  Rack

(D)  Books

Answer: (D)

   Directions-(Q. 40 and 41) Select the missing number/letters from the given responses.

40. 544, 509, 474, 439, ?

(A)  404

(B)  414

(C)  420

(D)  445

Answer: (A)

41. prt, ?, bdf, hjl, npr

(A)  vwc

(B)  vya

(C)  xzb

(D)  vxz

Answer: (D)

   Directions-(Q. 42 and 43) Select the one which is different from the other three responses.

42.

(A)  Scythe

(B)  Knife

(C)  Pillar

(D)  Saw

Answer: (C)

43.

(A)  Guitar

(B)  Violin

(C)  Veena

(D)  Keyboard

Answer: (D)

44. Arrange the following words as per order in the dictionary-

(1) Devious        (2) Devout

(3) Devolution   (4) Devotional

(5) Development

(A)  2, 5, 1, 3, 4

(B)  4, 2, 3, 5, 1

(C)  5, 2, 3, 1, 4

(D)  5, 1, 3, 4, 2

Answer: (D)

45. Which will appear third in the dictionary?

(A)  Serif

(B)  Sergeant

(C)  Serous

(D)  Serjeant

Answer: (D)

46. How is my brother’s grandfather’s only son’s only boy related to me?

(A)  Brother

(B)  Sister

(C)  Cousin

(D)  Mother

Answer: (B)

   Directions-(Q. 47 and 48) From the given alternative words, select the word which cannot be formed using the letters of the given word.

47. MISCELLANEOUS

(A)  MISCHIEF

(B)  CELL

(C)  CANE

(D)  MISS

Answer: (A)

48. SUBORDINATE

(A)  NEAT

(B)  BORDER

(C)  EAT

(D)  SEAT

Answer: (B)

49. If STAMPEDE can be written as PESTAMDE, then TAMPERED can be written as-

(A)  ERTAMPED

(B)  RETAMPED

(C)  RETAMPDE

(D)  ERTAMPDE

Answer: (A)

50. In a certain code, RAINTREE is written as ARNIRTEE. How is DAHLIA written in that code?

(A)  DALHAI

(B)  HADLAI

(C)  ADLHAI

(D)  LADHAI

Answer: (C)

Part-II

English Language

   Directions-(Q. 51-56) Some parts of the sentences have errors and some have none. Find out which part of a sentence has an error and blacken the oval (•) corresponding to the appropriate letter (A, B, C). If a sentence is free from error, blacken the oval corresponding to (D) in the Answer Sheet.

51. India has got (A)/ freedom (B)/in 1947. (C)/ No error (D)

Answer: (A)

52. Every scientific invention (A)/ has proved (B)/ much harmful to society than beneficial. (C)/ No error (D)

Answer: (C)

53. She is preparing (A)/ for this examination (B)/ since 2004. (C)/ No error (D)

Answer: (A)

54. I can depend upon (A)/ your help, (B)/ can I? (C)/ No error (D)

Answer: (A)

55. I am tired (A)/ so I’ll lay down (B)/ and take rest. (C)/ No error (D)

Answer: (B)

56. If her grandfather (A)/ would have lived three more days (B)/ he would have been 100 years old. (C)/ No error (D)

Answer: (B)

   Directions-(Q. 57-61) Sentences are given with blanks to be filled in with an appropriate word(s). Four alternatives are suggested for each question. Choose the correct alternative out of the four and indicate it by blackening the appropriate oval (•) in the Answer Sheet.

57. She thanked Vishal as she could reach the station on time ……. .his help.

(A)  since

(B)  for

(C)  with

(D)  in

Answer: (C)

58. My …….. brother is called Arhaan.

(A)  older

(B)  oldest

(C)  senior

(D)  elder

Answer: (D)

59. The lawyer has plenty of ……….

(A)  criminals

(B)  buyers

(C)  customers

(D)  clients

Answer: (D)

60. “I have brought the book. It’s ……. !” Ravi said assertively to all the boys present.

(A)  mine

(B)  my

(C)  me

(D)  myself

Answer: (A)

61. “The project is good, but there is ……… missing to make it an excellent work,” the engineer commented.

(A)  everything

(B)  anything

(C)  something

(D)  nothing

Answer: (C)

   Directions-(Q. 62-64) Out of the four alternatives, choose the one which best expresses the meaning of the given word and mark it in the Answer Sheet.

62. Fortitude

(A)  Prudence

(B)  Support

(C)  Courage

(D)  Sincerity

Answer: (C)

63. Imply

(A)  Conclude

(B)  Connote

(C)  Confirm

(D)  Comply

Answer: (B)

64. Vigilant

(A)  Intelligent

(B)  Ambitious

(C)  Smart

(D)  Watchful

Answer: (D)

   Directions-(Q. 65-67) Choose the word opposite in meaning to the given word and mark it in the Answer Sheet.

65. Feeble

(A)  Rickety

(B)  Weak

(C)  Infirm

(D)  Robust

Answer: (D)

66. Adulterate

(A)  Contaminate

(B)  Purify

(C)  Wash

(D)  Stain

Answer: (B)

67. Vain

(A)  Conceited

(B)  Egotistic

(C)  Humble

(D)  Proud

Answer: (C)

   Directions-(Q. 68-72) Four alternatives are given for the Idiom/Phrase. Choose the alternative which best expresses the meaning of the Idiom/Phrase and mark it in the Answer Sheet.

68. A cry in wilderness

(A)  a cry in vain

(B)  an unpleasant situation

(C)  a cry in disgrace

(D)  a cry with a laughter

Answer: (A)

69. To rock the boat

(A)  to conspire against

(B)  to create difficulties

(C)  to agitate against

(D)  to upset the balance

Answer: (B)

70. To beat the air

(A)  to make a great effort

(B)  to act intelligently

(C)  to make efforts that are useless and/or vain

(D)  to make every possible effort

Answer: (C)

71. See through

(A)  to persist with something

(B)  to see off

(C)  to detect the true nature

(D)  to ignore something

Answer: (C)

72. To give airs

(A)  exhale

(B)  inhale

(C)  boast

(D)  humble

Answer: (C)

   Directions-(Q. 73-79) A part of the sentence is bold. Below are given alternatives to the bold part at (A), (B), (C) which may improve the sentence. Choose the correct alternative. In case no improvement is needed, your answer is (D). Mark  your answer in the Answer Sheet.

73. She is my better half.

(A)  wife

(B)  Mrs.

(C)  partner

(D)  No improvement

Answer: (A)

74. The Prime Minister has gone to Brazil, isn’t it?

(A)  has he ?

(B)  hasn’t he?

(C)  didn’t he?

(D)  No improvement

Answer: (B)

75. They were congratulated him for his birthday.

(A)  congratulated him for

(B)  were congratulated him on

(C)  congratulated him on

(D)  No improvement

Answer: (C)

76. My cousin sister is a teacher.

(A)  cousin

(B)  cousin’s sister

(C)  cousin brother

(D)  No improvement

Answer: (A)

77. The news is so good but it can’t be true.

(A)  too good to

(B)  very good to

(C)  rather good to

(D)  No improvement

Answer: (A)

78. I had lived in this house since 2005.

(A)  am living

(B)  have been living

(C)  have lived

(D)  No improvement

Answer: (A)

79. Keeping away from controversy is best policy.

(A)  a better policy

(B)  the best policy

(C)  most best policy

(D)  No improvement

Answer: (B)

   Directions-(Q. 80-83) Out of the four alternatives choose the one which can be substituted for the given words/sentence.

80. Impossible to decipher, make out or read-

(A)  eligible

(B)  intelligible

(C)  illegible

(D)  ambiguous

Answer: (D)

81. Careful not to harm or inconvenience others-

(A)  humble

(B)  considerate

(C)  obstinate

(D)  rash

Answer: (B)

82. One who finds it easy to produce new and original ideas and things.

(A)  impulsive

(B)  creative

(C)  hospitable

(D)  bright

Answer: (B)

83. Done with good judgement-

(A)  eminent

(B)  judicious

(C)  enviable

(D)  judicial

Answer: (B)

   Directions-(Q. 84 and 85) There are four different words out of which one is correctly spelt. Find the correctly spelt word and indicate it by blackening the appropriate oval (•) in the Answer Sheet.

84. 

(A)  grammar

(B)  gramer

(C)  grammer

(D)  gramar

Answer: (A)

85.

(A)  imidate

(B)  imidiate

(C)  immediate

(D)  imidiat

Answer: (C)

   Directions-(Q. 86-100) You have following two brief passage with 10 questions in passage I and 5 questions in passage II. Read the passages carefully and fill in the blanks with suitable words out of the four alternatives given.

Passage I

(Q. 86-95)

   Beggars have found a new way of making money. They seek …(86)… via SMS, requesting to credit sums …(87)… from Rs 10 to Rs 100. They explain their …(88)… and end the message with a statement of …(89)…  “Those who are God fearing will definitely …(90)… to their request and will be heaped with …(91)… as a reward for their good …(92)…” Many also send heavenly pictures of …(93)… Quite a few people …(94)… and give away alms. They …(95)… that they are giving money in the name of God, irrespective of who the receiver is.

86.

(A)  donation

(B)  loan

(C)  alms

(D)  favour

Answer: (C)

87.

(A)  differing

(B)  ranging

(C)  fluctuating

(D)  producing

Answer: (B)

88.

(A)  problem

(B)  difficulty

(C)  task

(D)  duty

Answer: (A)

89.

(A)  dependence

(B)  morality

(C)  immorality

(D)  faith

Answer: (B)

90.

(A)  leap

(B)  heed

(C)  forward

(D)  think

Answer: (B)

91.

(A)  curse

(B)  cruelty

(C)  blessings

(D)  tensions

Answer: (C)

92.

(A)  deed

(B)  work

(C)  task

(D)  job

Answer: (A)

93.

(A)  river

(B)  hell

(C)  garden

(D)  paradise

Answer: (D)

94.

(A)  take action

(B)  respond

(C)  argue

(D)  quarrel

Answer: (B)

95.

(A)  argue

(B)  consider

(C)  believe

(D)  imagine

Answer: (C)

Passage II

(Q. 96-100)

   Left-handed persons can do certain things better than those who are right-handed. They generally find it more …(96)… to learn languages and mathematics, but have an advantage when it …(97)… to music or sports. Recent research shows that the reason for …(98)… or right-handedness, and the qualities …(99)… go with each of these, …(100)… lie in differences in the construction of the brain.

96.

(A)  easy

(B)  difficult

(C)  hard

(D)  rigid

Answer: (B)

97.

(A)  come

(B)  came

(C)  comes

(D)  coming

Answer: (C)

98.

(A)  left

(B)  right

(C)  lame

(D)  handicapped

Answer: (A)

99.

(A)  those

(B)  these

(C)  who

(D)  that

Answer: (D)

100.

(A)  might

(B)  must

(C)  may

(D)  need

Answer: (C)

Part-III

Quantitative Aptitude

101. The wrong number in the series is 2, 9, 28, 65, 126, 216, 344

(A)  9

(B)  65

(C)  216

(D)  None of these

Answer: (C)

102. Eight consecutive numbers are given. If the average of the two numbers that appear in the middle is 6, then the sum of the eight given numbers is-

(A)  36

(B)  48

(C)  54

(D)  64

Answer: (B)

103. If  then  is equal to-

(A) 

(B)   

(C)   

(D)   

Answer: (A)

104. A and B together can do a piece of work in 12 days, while B alone can finish it in 30 days. A alone can finish the work in-

(A)  15 days

(B)  18 days

(C)  20 days

(D)  25 days

Answer: (C)

105. A and B can do a job together in 12 days. A is 2 times as efficient as B. In how many days can B alone complete the work?

(A)  36

(B)  12

(C)  18

(D)  9

Answer: (A)

106. The marked price is 20% higher than cost price. A discount of 20% is given on the marked price. By this type of sale, there is-

(A)  no loss or gain

(B)  4% gain

(C)  4% loss

(D)  2% loss

Answer: (C)

107. A chair listed at Rs 350 is available at successive discounts of 25% and 10%. The selling price of the chair is-

(A)  Rs 240.25

(B)  Rs 242.25

(C)  Rs 236.25

(D)  Rs 230.25

Answer: (C)

108. A tradesman marks his goods at such a price that after allowing a discount of 15%, he makes a profit of 20%. What is the marked price of an article whose cost price is Rs 170?

(A)  Rs 220

(B)  Rs 200

(C)  Rs 240

(D)  Rs 260

Answer: (C)

109. In two types of stainless steel, the ratio of chromium and steel are 2 : 11 and 5 : 21 respectively. In what proportion should the two types be mixed so that the ratio of chromium to steel in the mixed type become 7 : 32?

(A)  1 : 2

(B)  1 : 3

(C)  2 : 3

(D)  3 : 4

Answer: (A)

110. A sum of Rs 7,000 is divided among A, B, C in such a way that the shares of A and B are in the ratio 2 : 3 and those of B and C are in the ratio 4 : 5. The share of B is-

(A)  Rs 1,600

(B)  Rs 2,000

(C)  Rs 2,400

(D)  Rs 3,000

Answer: (C)

111. Tea worth Rs 126 per kg and Rs 135 per kg are mixed with a third variety in the ratio 1 : 1 : 2. If the mixture is worth Rs 153 per kg, the price of the third variety per kg will be-

(A)  Rs 169.5

(B)  Rs 170.0

(C)  Rs 175.5

(D)  Rs 180.0

Answer: (C)

112. In the afternoon, a student read 100 pages at the rate of 60 pages per hour. In the evening, when she was tired, she read 100 more pages at the rate of 40 pages per hour. What was her average rate of reading, in pages per hour?

(A)  48

(B)  50

(C)  60

(D)  70

Answer: (A)

113. The mean weight of 34 students of a school is 42 kg. If the weight of the teacher be included, the mean rises by 400 gram. Find the weight of the teacher (in kg).

(A)  66

(B)  56

(C)  55

(D)  57

Answer: (B)

114. A cricketer has a mean score of 60 runs in 10 innings. Find out how many runs are to be scored in the eleventh innings to raise the mean score to 62?

(A)  80

(B)  81

(C)  83

(D)  82

Answer: (D)

115. A trader purchases a watch and a wall clock for Rs 390. He sells them making a profit of 10% on the watch and 15% on the wall clock. He earns a profit of Rs 51.50. The difference between the original prices of the wall clock and the watch is equal to-

(A)  Rs 110

(B)  Rs 100

(C)  Rs 80

(D)  Rs 120

Answer: (A)

116. A salesman expects a gain of 13% on his cost price. If in a month his sale was Rs 7,91,000, what was his profit?

(A)  Rs 91,000

(B)  Rs 97,786

(C)  Rs 85,659

(D)  Rs 88,300

Answer: (A)

117. A merchant fixed the selling price of his articles at Rs 700 after adding 40% profit to the cost price. As the sale was very low at this price level, he decided to fix the selling price at 10% profit. Find the new selling price.

(A)  Rs 450

(B)  Rs 490

(C)  Rs 500

(D)  Rs 550

Answer: (D)

118. A saves 20% of his monthly salary. If his monthly expenditure is Rs 6,000, then his monthly savings is-

(A)  Rs 1,200

(B)  Rs 4,800

(C)  Rs 1,500

(D)  Rs 1,800

Answer: (A)

119. From 2008 to 2009, the sales of a book decreased by 80%. If the sales in 2010 were the same in 2008, by what percent did it increase from 2009 to 2010?

(A)  80%

(B)  100%

(C)  120%

(D)  400%

Answer: (D)

120. The speed of a bus is 72 km/hr. The distance covered by the bus in 5 seconds is-

(A)  50 m

(B)  74.5 m

(C)  100 m

(D)  60 m

Answer: (C)

121. Two men start together to walk a certain distance, one at 4 km/h and another at 3 km/h. The former arrives half an hour before the latter. Find the distance-

(A)  6 km

(B)  9 km

(C)  8 km

(D)  7 km

Answer: (A)

122. A person invests Rs 12,000 as fixed deposit at a bank at the rate of 10% per annum simple interest. But due to some pressing needs he has to withdraw the entire money after 3 years, for which the bank allowed him a lower rate of interest. If he gets Rs 3,320 less than what he would have got at the end of 5 years, the rate of interest allowed by the bank is-

(A)   

(B)   

(C)   

(D)   

Answer: (D)

123. The compound interest on Rs 30,000 at 7% per annum for a certain time is Rs 4,347. The time is-

(A)  2 years

(B)  2.5 years

(C)  3 years

(D)  4  years

Answer: (A)

124. A prism has as the base a right-angled triangle whose sides adjacent to the right angles are 10 cm and 12 cm long. The height of the prism is 6 gm/cubic cm. The weight of the prism is-

(A)  3.4 kg

(B)  4.8 kg

(C)  6.4 kg

(D)  7.2 kg

Answer: (D)

125. Three circles of radii 4 cm, 6 cm and 8 cm touch each other pairwise externally. The area of the triangle formed by the line segments joining the centres of the three circles is-

(A)  6√6 sq. cm

(B)  24√6 sq. cm

(C)  144√13 sq. cm

(D)  12√105 sq. cm

Answer: (B)

126. The radius of the base of a right circular cone is doubled. To keep the volume fixed, the height of the cone will be-

(A)  half of the previous height

(B)  one-third of the previous height

(C)  one-fourth of the previous height

(D)  1/√2 times of the previous height

Answer: (C)

127. The base of a cone and a cylinder have the same radius 6 cm; they have also the same height 8 cm. The ratio of the curved surfaces of the cylinder to that of the cone is-

(A)  4 : 3

(B)  5 : 3

(C)  8 : 5

(D)  8 : 3

Answer: (C)

128. The ratio of length of each equal side and the third side of an isosceles triangle is 3 : 4. If the area of the triangle is 18√5 square unit, the third side is-

(A)  8√2 unit

(B)  12 unit

(C)  16 unit

(D)  5√10 unit

Answer: (B)

129. In a circle of radius 21 cm, an arc subtends an angle of 72° at the centre. The length of the arc is-

(A)  13.2 cm

(B)  19.8 cm

(C)  21.6 cm

(D)  26.4 cm

Answer: (D)

130. The x-intercept of the graph of 7x – 3y = 2 is-

(A)  2/5

(B)  2/7

(C)  3/4

(D)  3/7

Answer: (B)

131. If x = √3 + √2, then the value of  is-

(A)  2

(B)  3

(C)  2√2

(D)  2√3

Answer: (D)

132. If p + q = 10 and pq = 5, then the numerical value of  will be-

(A)  22

(B)  18

(C)  16

(D)  20

Answer: (B)

133. If  then the value of  will be-

(A)  3√3

(B)  5

(C)  1

(D)  2

Answer: (D)

134. An equilateral triangle TQR is drawn inside a square PQRS. The value of the angle PTS, in degrees, is-

(A)  75

(B)  90

(C)  120

(D)  150

Answer: (D)

135. If a, b and c are the sides of a triangle and a2 + b2 + c2 = ab + bc + ca, then the triangle is-

(A)  equilateral

(B)  isosceles

(C)  right-angled

(D)  obtuse-angled

Answer: (A)

136. The distance between the centres of two equal circles, each of radius 3 cm, is 10 cm. The length of a transverse common tangent is-

(A)  4 cm

(B)  6 cm

(C)  8 cm

(D)  10 cm

Answer: (C)

137. If the perimeter of a right-angled triangle is 56 cm and area of the triangle is 84 sq. cm, then the length of the hypotenuse is (in cm) –

(A)  7 cm

(B)  24 cm

(C)  25 cm

(D)  50 cm

Answer: (C)

138. In ∆ ABC, ∠A = 30°, ∠B = 60°. Find ∠C in circular measure-

(A)  πc/6

(B)  πc/2

(C)  2πc/3

(D)  3πc/4

Answer: (B)

139. If cos θ + sec θ = 2, the value cos6θ + sec6θ is-

(A)  1

(B)  2

(C)  4

(D)  8

Answer: (B)

140. A man standing at a point P is watching the top of a tower, which makes an angle of elevation of 30°. The man walks some distance towards the tower and then his angle of elevation of the top of the tower is 60°. If the height of the tower is 30 m, then the distance he moves is-

(A)  20 m

(B)  20√3 m

(C)  22 m

(D)  22√3 m

Answer: (B)

   Directions-The following pie-chart shows the performance in an examination in a particular year for 360 students. Study the pie-chart and answer the questions no. 141-145.

141. The number of students who passed in first division is-

(A)  45

(B)  54

(C)  64

(D)  74

Answer: (B)

142. The number of students who passed in second division is more than those in first division by-

(A)  111

(B)  112

(C)  109

(D)  108

Answer: (D)

143. The ratio of successful students to the failed students is-

(A)  9 : 1

(B)  5 : 1

(C)  1 : 9

(D)  2 : 7

Answer: (A)

144. The percentage of students who have failed in the examination is-

(A)  20%

(B)  36%

(C)  10%

(D)  30%

Answer: (C)

145. The total number of students who have passed in 2nd or 3rd division is-

(A)  162

(B)  270

(C)  108

(D)  None of these

Answer: (B)

   Directions: The following is a horizontal bar diagram showing the accidents in which two-wheelers are involved with other objects. Study the diagram and answer the questions no. 146-150.

146. The percentage of accidents in which pedestrians and cyclists are involved is-

(A)  60

(B)  20.4

(C)  24

(D)  6

Answer: (C)

147. The percentage by which the accidents involving buses is less than the accidents involving tanker lorry is-

(A)  40

(B)  28

(C)  6

(D)  4

Answer: (D)

148. The difference in percentage between the accidents involving two-wheelers and two-wheelers and two-wheelers and other objects is respectively.

(A)  54, more

(B)  54, less

(C)  77, more

(D)  77, less

Answer: (B)

149. 60% of the accidents are involved due to-

(A)  two-wheelers, cars, buses and stationary vehicles

(B)  two-wheelers, cars, buses and tanker lorry

(C)  cars, buses, tanker lorry and pedestrians

(D)  cars, tanker, lorry, bicycles and stationary vehicles

Answer: (A)

150. If the data of the bar diagram is represented by a pie-chart, and the angle of a sector of the pie-chart is 36°, then this sector represents the accidents involving-

(A)  buses

(B)  stationary vehicles

(C)  pedestrians

(D)  bicycles

Answer: (B)

For Visually Handicapped Candidates Only

141. A number on subtracting 12 from it, reduces to its 80%. 30% of that number is-

(A)  18

(B)  24

(C)  30

(D)  36

Answer: (A)

142. If a, b, c are in continued proportion, then the value of  is-

(A)  2

(B)  1/2

(C)  1

(D)  1/3

Answer: (C)

143. A shopkeeper sells all articles at a discount of 7.5% but increases the selling price of each article by 20%. His gain on each article is-

(A)  5%

(B)  9.5%

(C)  11%

(D)  12.5%

Answer: (C)

  1. A train 220 metres long is running at a speed of 60 km/hr. In how many seconds will it pass a man who is running at 6 km/hr in the direction opposite to that in which the train is going?

(A)  6 second

(B)  9 second

(C)  12 second

(D)  15 second

Answer: (C)

145. If x + y = √3 and x – y = √2, then the value of 8xy(x2 + y2) will be-

(A)  5/2

(B)  √6

(C)  5/9

(D)  5

Answer: (D)

146. The average of consecutive odd numbers up to 300 is-

(A)  300

(B)  100

(C)  150

(D)  250

Answer: (C)

147. If  is-

(A)  3/4

(B)  2/3

(C)  −2/3

(D)  −3/4

Answer: (A)

148. In the series 2 + 5 + 8 … + 152, what will be the middle term?

(A)  77

(B)  71

(C)  74

(D)  80

Answer: (C)

149. PT is a tangent at T of a circle with centre at O. TA is diameter. AP cuts the circle at B. If PT = 6 cm, PB = 3 c, then AB is-

(A)  3 cm

(B)  3√3 cm

(C)  9 cm

(D)  6 cm

Answer: (C)

150. The length of the minute hand of a wall clock is 7 cm. The area swept by it in 30 minutes is-

(A)  147 cm2

(B)  154 cm2

(C)  77 cm2

(D)  210 cm2

Answer: (C)

Part-IV

General Awareness

151. The basic characteristic off a capitalistic economy is-

(A)  Full employment

(B)  The private ownership of the means of production

(C)  Absence of monopoly

(D)  Large-scale production in primary industries

Answer: (B)

152. Which one of the following taxes is not a direct tax?

(A)  Gift tax

(B)  Wealth tax

(C)  Sales tax

(D)  Estate duty

Answer: (C)

153. UNDP prepares-

(A)  Index Number of Price Level

(B)  Physical Quality Index

(C)  Human Development index

(D)  Standard of living index

Answer: (C)

154. Fiscal policy refers to-

(A)  Sale and purchase of securities by RBI

(B)  Government taxes, expenditure and borrowings

(C)  Government borrowings from abroad

(D)  Sharing of its revenue by Central Government with States

Answer: (B)

155. Public opinion gets an authoritative expression in a democracy through-

(A)  Newspapers

(B)  Parliament

(C)  Pressure groups

(D)  Public meetings

Answer: (B)

156. Which one of the following is not a determining factor of a country’s foreign policy?

(A)  National interests

(B)  Interdependence

(C)  Cultural conditions

(D)  Religious conditions

Answer: (B)

157. Who will act as the Chairman of Public Accounts Committee?

(A)  The Leader of the Opposition in Lok Sabha

(B)  The Leader of the House

(C)  The Speaker of the Lok Sabha

(D)  The Vice-President of India

Answer: (A)

158. Who was the chosen unanimously as the President of India?

(A)  Dr. S. Radhakrishnan

(B)  Neelam Sanjiva Reddy

(C)  K. R. Narayanan

(D)  Dr. A.P.J. Abdul Kalam

Answer: (B)

159. The Constitution of India was passed by the Constituent Assembly on-

(A)  17th October, 1949

(B)  14th November, 1949

(C)  26th November, 1949

(D)  26th January, 1949

Answer: (C)

160. The power to decide an election petition is vested in the-

(A)  Parliament

(B)  Supreme Court

(C)  High Court

(D)  Election Commission

Answer: (C)

161. The 1857 Mutiny failed mainly because-

(A)  the British got French support

(B)  the British numbered more

(C)  of lack of planning and leadership

(D)  it was premature

Answer: (C)

162. The Story of ‘My Experiments with Truth’ is the autobiography of-

(A)  Lala Lajpat Rai

(B)  Gopal Krishna Gokhale

(C)  Bal Gangadhar Tilak

(D)  Mahatma Gandhi

Answer: (D)

163. Who among the following made the Ganapati festival very popular in Maharashtra?

(A)  Gopal Krishna Gokhale

(B)  Annie Besant

(C)  Mahadev Ranade

(D)  Bal Gangadhar Tilak

Answer: (D)

164. The Mughal ruler who built the Buland Darwaza was-

(A)  Akbar

(B)  Babur

(C)  Humayun

(D)  Bahadur Shah

Answer: (A)

165. Diarchy in the provinces was introduced through the-

(A)  Indian Councils Act, 1861

(B)  Indian Councils Act, 1892

(C)  Government of India Act, 1919

(D)  Government of India Act, 1935

Answer: (C)

166. Which of the following is the world’s largest desert?

(A)  Gobi

(B)  Sahara

(C)  The Great Australian Desert

(D)  Arabian Desert

Answer: (B)

167. The rate of erosion in a stream is lowest where-

(A)  breadth is greater

(B)  velocity is more

(C)  the river joins  the sea

(D)  depth is greater

Answer: (B)

168. The name ‘Sahyadri’ is related to-

(A)  Western Ghats

(B)  Cyclone hazards

(C)  A rain-bearing wind

(D)  Himalayan Peak

Answer: (A)

169. Which of the following is not correctly matched?

(A)  Himachal Pradesh – Shillong

(B)  Andhra Pradesh – Hyderabad

(C)  Uttar Pradesh – Lucknow

(D)  Arunachal Pradesh – Itanagr

Answer: (A)

170. Tides in the sea have stored in them-

(A)  Hydraulic energy

(B)  Kinetic energy

(C)  Gravitational potential energy

(D)  A combination of all the three forms of energy

Answer: (D)

171. Delivery of developed fetus is scientifically called as-

(A)  Parturition

(B)  Oviposition

(C)  Abortion

(D)  Ovulation

Answer: (A)

172. Thyroxine hormone is secreted by-

(A)  Pituitary gland

(B)  Thyroid gland

(C)  Adrenal gland

(D)  Testes

Answer: (B)

173. The digestive juice which has no enzyme is-

(A)  Bile

(B)  Saliva

(C)  Intestinal juice

(D)  Gastric juice

Answer: (A)

174. An essential feature of seed germination is the presence of-

(A)  Minerals

(B)  Water

(C)  Light

(D)  Temperature

Answer: (B)

175. Plants that grow on stones and rocks are-

(A)  Halophytes

(B)  Aerophytes

(C)  Psammophytes

(D)  Lithophytes

Answer: (D)

176. Tactile hair is found in the body of-

(A)  Insects

(B)  Mammals

(C)  Reptiles

(D)  Birds

Answer: (A)

177. The source of energy in the Sun is-

(A)  nuclear fission

(B)  nuclear fusion

(C)  radioactivity

(D)  electrical energy

Answer: (B)

178. Which one of the following materials is used as controller in a nuclear reactor power generator?

(A)  Cadmium

(B)  Beryllium

(C)  Graphite

(D)  Heavy water

Answer: (A)

179. Banking of curves on road or railway track is done to provide-

(A)  centripetal force

(B)  centrifugal force

(C)  gravitational force

(D)  angular velocity

Answer: (A)

180. ………. Is a type of application software used for communication.

(A)  FTP

(B)  Word processing

(C)  Database

(D)  Image editing

Answer: (A)

181. A floppy disk is-

(A)  a semiconductor random-access memory

(B)  an EPROM

(C)  used as the primary memory in computer systems

(D)  made up of magnetic material

Answer: (D)

182. MDI stands for-

(A)  Multiple Document Interface

(B)  Multiple Design Interface

(C)  Multiple Design Interaction

(D)  Multiple Document Interaction

Answer: (A)

183. People die in an atmosphere of carbon dioxide because-

(A)  it is a poisonous gas

(B)  it destroys tissues

(C)  of want of oxygen

(D)  of suffocation

Answer: (C)

184. Which of the following acts as photosensitizer during photosynthesis?

(A)  Oxygen

(B)  Nitrogen

(C)  Chlorophyll

(D)  Chlorine

Answer: (C)

185. What happens when bleaching powder is left exposed to air?

(A)  It turns dark brown in colour

(B)  It turns yellow in colour

(C)  It gradually loses its oxygen

(D)  It gradually loses its chlorine

Answer: (D)

186. Arsenic pollution leads to-

(A)  White foot disease

(B)  Black foot disease

(C)  Dyslexia

(D)  Allergy

Answer: (B)

187. Which one of the following does not contribute to pollution?

(A)  Thermal Power Plant

(B)  Nuclear Power Plant

(C)  Hydroelectric Power Plant

(D)  Atomic Power Plant

Answer: (B)

188. Which of the following are the two major components of dry air (by volume)?

(A)  Nitrogen and Oxygen

(B)  Oxygen and Argon

(C)  Nitrogen and Ammonia

(D)  Oxygen and Carbon dioxide

Answer: (A)

189. Which one of the following is not an example of Lotic ecosystem?

(A)  Stream

(B)  Lagoon

(C)  Pond

(D)  Estuary

Answer: (C)

190. Permissible noise level at industrial area during daytime is-

(A)  40 dB (A)

(B)  75 dB (A)

(C)  120 dB (A)

(D)  140 dB (A)

Answer: (B)

191. Benazir Bhutto, the former Pakistan Prime Minister was assassinated in-

(A)  Hyderabad

(B)  Karachi

(C)  Rawalpindi

(D)  Islamabad

Answer: (C)

192. Which of the following is correctly matched?

       Research Institutes      Headquarters

(A)  Leather Research Institute – Lucknow

(B)  Rice Research Institute – Cuttack

(C)  Silk Research Institute – Bangalore

(D)  Sugar Research Institute – Chennai

Answer: (B)

193. Who was affectionately known as the “Grand Old Man of India”?

(A)  Gopal Krishna Gokhale

(B)  Mahatma Gandhi

(C)  Bal Gangadhar Tilak

(D)  Dadabhai Naoroji

Answer: (D)

194. India test fired successfully its Agni-V, surface-to-surface ICBM from Wheeler Island on-

(A)  7th March, 2012

(B)  7th April, 2012

(C)  17th March, 2012

(D)  19th April, 2012

Answer: (D)

195. Find the odd one out.

(A)  IDBI – Industrial Finance

(B)  SIDBI – Financial assistance to small industries

(C)  FCI – Financial assistance to commercial institutions

(D)  EXIM Bank – Financing of export import trade

Answer: (C)

196. The Twelfth Five Year Plan will be operative for the period-

(A)  2010-2015

(B)  2011-2016

(C)  2012-2017

(D)  2013-2018

Answer: (C)

197. The State with largest gap in male and female literacy is-

(A)  Uttar Pradesh

(B)  Madhya Pradesh

(C)  Rajasthan

(D)  Kerala

Answer: (C)

198. “Better to reign in hell than serve in heaven.” Who said these words?

(A)  William Shakespeare

(B)  Milton

(C)  William Wordsworth

(D)  Lord Tennyson

Answer: (B)

199. Comprehensive Test Ban Treaty (CTBT) is associated with the ban on which of the following?

(A)  Ban on certain organizations under UN laws

(B)  Ban on money laundering activities

(C)  Ban on nuclear tests for developing arsenals

(D)  Ban on terrorism

Answer: (C)

200. The Pulitzer Prize is associated with which of the following?

(A)  Environmental Protection

(B)  Civil Aviation

(C)  Journalism

(D)  Olympic Games

Answer: (C)

SSC Combined Higher Secondary Level (10+2) Examination (Second Shift) Tier-I Held on 28 October, 2012 Question Paper With Answer Key

SSC Combined Higher Secondary Level (10+2) Examination (Second Shift) Tier-I Held on 28 October, 2012
SSC Combined Higher Secondary Level (10+2) Examination (Second Shift) Tier-I Held on 28 October, 2012 Question Paper With Answer Key

Staff Selection Commission Combined Higher Secondary Level (10+2) Examination

(Second Shift) Tier-I Held on 28 October, 2012

Part-I

General Intelligence

Directions-(Q. 1-6) select the related letter/word/number from the given alternatives.

1. NMKH : HIKN : : UTRO : ?

(A)  OPQR

(B)  ORTU

(C)  OPRU

(D)  ORPQ

Answer: (C)

2. ACEG : KMOQ : : CEGI : ?

(A)  HILN

(B)  FHJM

(C)  QSUV

(D)  IKMO

Answer: (D)

3. 5 : 124 : : 8 : ?

(A)  65

(B)  64

(C)  512

(D)  511

Answer: (D)

4. 164 : 143 : : 211 : ?

(A)  180

(B)  190

(C)  194

(D)  188

Answer: (B)

5. Never : Seldom : : Always : ?

(A)  Often

(B)  Everytime

(C)  Occasional

(D)  Usual

Answer: (A)

6. Ape : Man : : Cat : ?

(A)  Mouse

(B)  Pet-animal

(C)  Monkey

(D)  Tiger

Answer: (D)

Directions-(Q. 7-10) Select the one which is different from the other three responses.

7.

(A)  Teacher

(B)  Drill Master

(C)  Student

(D)  Registrar

Answer: (C)

8.

(A)  DZ

(B)  AR

(C)  JT

(D)  SK

Answer: (B)

9.

(A)  JIG

(B)  FEB

(C)  TSP

(D)  NMJ

Answer: (A)

10.

(A)  Oliver Twist

(B)  Macbeth

(C)  Othello

(D)  King Lear

Answer: (A)

11. 13, 16, 19, 22, 24, 28.

(A)  24

(B)  28

(C)  19

(D)  13

Answer: (A)

12. 1, 4, 9, 16, 23, 25, 36.

(A)  25

(B)  36

(C)  23

(D)  9

Answer: (C)

13. (1) Ream         (2) Riot

(3)   Ruin              (4) Route

(A)  1, 2, 4, 3

(B)  1, 2, 3, 4

(C)  2, 1, 3, 4

(D)  2, 3, 1, 4

Answer: (A)

14. (1) Appear       (2) Appraise

(3) Approval         (4) Opposite

(A)  1, 3, 2, 4

(B)  1, 2, 3, 4

(C)  1, 4, 3, 2

(D)  1, 4, 2, 3

Answer: (B)

15. Which one set of letters when sequentially placed at the gaps in the given letter series shall complete it?

cb_eb_e_edc_

(A)  dafd

(B)  bbcd

(C)  cbec

(D)  dceb

Answer: (B)

Directions-(Q. 16-18) A series is given, with one term/number missing. Choose the correct alternative from the given ones that will complete the series.

16. 4, 7, 11, 18, 29, 47, ?, 123, 199

(A)  84

(B)  102

(C)  76

(D)  70

Answer: (C)

17. ABNO, CDPQ, EFRS, ?

(A)  GHTU

(B)  KLYZ

(C)  IJVW

(D)  TUVW

Answer: (A)

18. 121, 112, ?, 97, 91, 86

(A)  99

(B)  104

(C)  102

(D)  108

Answer: (B)

19. If my mother’s maternal uncle is my wife’s grandfather, in what way is he related to my grandmother?

(A)  Husband

(B)  No direct relation

(C)  Brother

(D)  Cousin

Answer: (B)

20. Five books are placed on a table. E is on A and C is under B. A is above B and D is under C. Which book is lying at the bottom?

(A)  C

(B)  D

(C)  A

(D)  B

Answer: (B)

21. A man is standing facing the sun during sunset. Where will his shadow fall?

(A)  To his left

(B)  To his right

(C)  To his front

(D)  To his back

Answer: (D)

Directions-(Q. 22 and 23) In each question from the given alternatives words, select the word which cannot be formed using the letters of the given word.

22. MUSICAL

(A)  LASIUM

(B)  CLAIM

(C)  SLIM

(D)  CALCIUM

Answer: (D)

23. DAUGHTER

(A)  AUGHT

(B)  TRUTH

(C)  GATHER

(D)  DEARTH

Answer: (B)

24. If BRAKE is written as EUDNH, how can BRIDGE be written in that code?

(A)  EUDHLD

(B)  EULGJH

(C)  EUIGH

(D)  EUJNQ

Answer: (B)

25. If A = 1 and CAP = 20, what is HEN = ?

(A)  28

(B)  29

(C)  26

(D)  27

Answer: (D)

26. If ‘MEMORY’ is written as ‘848729’ in a certain code, then hos is ‘ROME’ written in that code?

(A)  4287

(B)  2487

(C)  2874

(D)  2784

Answer: (D)

Directions-(Q. 27 to 29) Select the  missing number from the given responses.

27.

22   32      42

2     2        2

20   ?        34

(A)  28

(B)  30

(C)  23

(D)  22

Answer: (A)

28.

3     5        7          105

4     6        8          192

9     2        3          ?

(A)  297

(B)  87

(C)  54

(D)  14

Answer: (C)

29.

3     5        9

8     6        2

6     ?        6

4     10      3

(A)  5

(B)  2

(C)  3

(D)  4

Answer: (C)

30. A statement is given below, followed by four inferences. Select the appropriate inference.

Statements:

(1) All students are graduates.

(2) Some students are swimmers.

(3) All swimmers are drivers.

Inferences  :

(A)  Some graduates are swimmers

(B)  All swimmers are graduates

(C)  All graduates are swimmers

(D)  All students are drivers

Answer: (A)

31. Find out the correct answer for the unsolved equation based on a certain system.

6 + 4 + 5 = 465; 7 + 3 + 4 = 374; 9 + 5 + 2 = ?

(A)  592

(B)  295

(C)  925

(D)  952

Answer: (A)

32.

If    0        1          2          3          = 9

1        2          3          4          =15

2        3          4          5          = 23

3        4          5          6          = ?

(A)  37

(B)  36

(C)  26

(D)  33

Answer: (D)

33. If H stands for +, Q stands for −, A stands for × and T stands for ÷, then

14 A 10 H 42 T 2 Q 8 = ?

(A)  153

(B)  251

(C)  158

(D)  216

Answer: (A)

34. A man started from his house and travelled 10 km towards East. He then turned left and travelled 5 km and then again turned left and cycled 10 km. How far is he from his home now?

(A)  15 km

(B)  20 km

(C)  5 km

(D)  10 km

Answer: (C)

35. A man starts from a point and cycles one km North. He then turns right and cycles 3 km and again turns right and cycles 5 km. How far and in which direction is he from his starting point?

(A)  4 km North

(B)  5 km North East

(C)  4 km East

(D)  5 km South East

Answer: (D)

36. Select the correct statement from the given responses.

Statement:

Environment air pollution is a product of modernization due to-

(I) lot of vehicles emitting smoke.

(II) draining of chemical waste into rivers.

(III) drying of animal wastes and fishes on roads.

(IV) drying of animal wastes and fishes on roads.

(A)  III is correct

(B)  IV is correct

(C)  I is correct

(D)  II is correct

Answer: (C)

37. A statement is given with four inferences. Select the correct inference.

Statement :

Spelling errors are common among children with learning abilities.

Inferences:

(I) Auditory reinforcement should be provided to them.

(II) Specialized individualized attention should be given.

(III) Repeat and tell the child to spell the word again and again.

(IV) Tell a sequences and improve speech sound

(A)  Either II or III is correct

(B)  I and II are correct

(C)  Only I is correct

(D)  III and IV are correct

Answer: (D)

38. Which number appears on the face opposite to 5?

(A)  4

(B)  1

(C)  6

(D)  2

Answer: (C)

39. Find the figure that represents the group-

Answer: (D)

40. The diagram represents Players, Teachers and Singers. Study the diagram and fund out how many Singer’s are also Players.

(A)  21

(B)  9

(C)  17

(D)  4

Answer: (A)

41. Find out the number of all those people who can speak English and Telugu.

(A)  62

(B)  22

(C)  34

(D)  4

Answer: (B)

42. Which number appears on the face opposite to 4?

(A)  2

(B)  6

(C)  5

(D)  1

Answer: (A)

Directions-(Q. 43 and 44) Which answer figure will complete the pattern of the question figure?

43. 

Answer: (D)

44. 

Answer: (A)

     Directions-(Q. 45 and 46) In each question from the given answer figures, select the one in which the question figure is hidden embedded.

45. 

Answer: (A)

46. 

Answer: (C)

47. If a mirror is placed on the line XY, then which of the answer figures is the correct image of the given question figure?

Answer: (D)

48. A piece of paper is folded and cut as shown below in the question figures. From the given answer figures, indicate how it will appear when opened.

Answer: (A)

49. A sheet of paper when folded, punched and opened will appear as in the question figure. Choose from the answer figures which punched hole pattern gives the question figure.

Answer: (B)

50. Find out from the given alternatives a suitable figure that would complete the figure matrix.

Answer: (B)

For Visually Handicapped Candidates Only

     Directions-(Q. 38 and 39) Each question a series is given, with one term/number missing. Choose the correct alternative from the given ones that will complete the series.

38. ABD, EFH, IJL, ?

(A)  OPQ

(B)  SPQ

(C)  UPQ

(D)  OPR

Answer: (D)

39. 109, 74, 46, 25, 11, ?

(A)  11

(B)  4

(C)  3

(D)  36

Answer: (B)

40. Which one of the given responses would be a meaningful order of the following?

(1) Stem               (2) Leaf

(3) Flower            (4) Branch

(5) Fruit

(A)  1, 4, 3, 5, 2

(B)  1, 4, 3, 2, 5

(C)  1, 2, 3, 4, 5

(D)  1, 4, 2, 3, 5

Answer: (D)

41. Arrange the following words as per order in the dictionary-

(1) succeed           (2) shuffle

(3) subtle              (4) subway

(A)  2, 3, 4, 1

(B)  2, 1, 3, 4

(C)  1, 3, 4, 2

(D)  2, 4, 3, 1

Answer: (A)

Directions-(Q. 42 to 44) Find the odd number/number pair/word from the given alternatives.

42. 6, 11, 16, 17, 21, 26

(A)  21

(B)  26

(C)  17

(D)  16

Answer: (C)

43.

(A)  59-48

(B)  33-27

(C)  68-85

(D)  121-134

Answer: (A)

44.

(A)  Butter

(B)  Curd

(C)  Cheese

(D)  Milk

Answer: (D)

45. Find out the correct answer for the unsolved equation based on a certain system.

2 + 3 + 6 + 7 = 9, 12 + 13 + 16 + 17 = 99, 102 + 103 + 106 + 107 = ?

(A)  999

(B)  9999

(C)  109

(D)  1009

Answer: (A)

Directions-(Q. 46 to 50) Select the related letters/word/number from the given alternatives.

46. Writer : Book : : ? : Song.

(A)  Singer

(B)  Sound engineer

(C)  Musician

(D)  Composer

Answer: (D)

47. 14 : 9 : : 26 : ?

(A)  31

(B)  15

(C)  12

(D)  13

Answer: (B)

48. DOG : z : : CAT : ?

(A)  x

(B)  y

(C)  v

(D)  w

Answer: (A)

49. KLMN : PONM : : HIJK : ?

(A)  UVWX

(B)  SRQP

(C)  DEFG

(D)  OPQR

Answer: (B)

50. Smile : Laugh : : ? : Shout

(A)  Scream

(B)  Hoot

(C)  Talk

(D)  Whisper

Answer: (D)

Part-II

English Language

     Directions-In questions no. 51 to 56, each sentence has been marked as (A), (B), (C) and (D) Some parts of the sentences have errors and some have none. Find out which part of a sentence has an error. If a sentence is free from error, blacken the oval corresponding to (D) in the Answer Sheet.

51. I doubt (A)/ that this is (B)/ acceptable to all. (C)/ No error (D)

Answer: (B)

52. I have (A)/ a lot of problems, (B)/ haven’t I ? (C)/ No error (D)

Answer: (D)

53. We are (A)/ hearing songs (B)/ from the tap-recorder. (C)/ No error (D)

Answer: (B)

54. A holy man accepts (A)/ with all the humility in the world (B)/ whatever God has provided him. (C)/ No error (D)

Answer: (B)

55. The teacher asked him (A)/ which English novel (B)/ did he like the most. (C)/ No error (D)

Answer: (C)

56. Students must (A)/ give the ear to (B)/ what the teacher tells them. (C)/ No error (D)

Answer: (B)

Directions- In questions no. 57 to 61, sentences are given with blanks to be filled in with an appropriate word(s). Four alternatives are suggested for each question. Choose the correct alternative out of the four options given with each of them.

57. In your younger years, our minds are ……….. and soft.

(A)  amenable

(B)  impressionable

(C)  flexible

(D)  pliable

Answer: (A)

58. She doesn’t ………. her money.

(A)  care over

(B)  careful of

(C)  care for

(D)  take care of

Answer: (C)

59. All clothes were soaked ……. water.

(A)  in

(B)  on

(C)  with

(D)  under

Answer: (A)

60. You stand a good chance ………. the match.

(A)  for winning

(B)  in winning

(C)  of winning

(D)  to win

Answer: (D)

61. All shell ……….. broken loose after the bomb blast.

(A)  has

(B)  are

(C)  have

(D)  is

Answer: (D)

Directions-In questions no. 62 to 64, out of the four alternatives, choose the one which best expresses the meaning of the given words and mark it in the Answer Sheet.

62. Gape

(A)  Struggle

(B)  Exclaim

(C)  Stare

(D)  Hide

Answer: (C)

63. Elude

(A)  Tell

(B)  Create

(C)  Avoid

(D)  Refer

Answer: (C)

64. Innocuous

(A)  Harmless

(B)  Faultless

(C)  Malicious

(D)  Offensive

Answer: (A)

Directions-In questions no. 65 to 67, choose the word opposite in meaning to the given word and mark in the Answer Sheet.

65. Repellent

(A)  Repulsive

(B)  Abhorrent

(C)  Attractive

(D)  Offensive

Answer: (C)

66. Gorgeous

(A)  Shabby

(B)  Ordinary

(C)  Untidy

(D)  Beautiful

Answer: (A)

67. Surfeit

(A)  Fullness

(B)  Deficiency

(C)  Redundancy

(D)  Excess

Answer: (B)

Directions-In questions no. 68 to 72, four alternatives are given for the Idiom/Phrase bold in the sentence. Choose the alternative which best expresses the meaning of the Idiom/Phrase and mark it in the Answer Sheet.

68. Hid did me a good turn by recommending for the post of Principal.

(A)  Returned my kindness

(B)  Changed my future

(C)  Did an act of kindness

(D)  Became suddenly good

Answer: (C)

69. If he is lazy, why don’t you turn him off.

(A)  Dismiss

(B)  Warn

(C)  Scold

(D)  Punish

Answer: (A)

70. Having walked twenty miles, I am feeling quite done up.

(A)  Energetic

(B)  Relaxed

(C)  Fresh

(D)  Exhausted

Answer: (D)

71. The sum and substance of our Principal’s speech was that students should learn discipline.

(A)  Outline

(B)  Theme

(C)  Conclusion

(D)  Gist

Answer: (D)

72. He is an honest man, he will take the bull by the horns and handle the management.

(A)  Face the problem boldly

(B)  Run away seeing the bull

(C)  Fight the bull holding its horns

(D)  Face the problem becoming nervous

Answer: (A)

Directions-In questions no. 73 to 79, a part of the sentence is bold. Below are given alternatives to the bold part at (A), (B) and (C) which may improve the sentence. Choose the correct alternative. In case no improvement is needed, your answer is (D). Mark your answer in the Answer Sheet.

73. Do you appraise any difficulty in your neighbourhood after the recent theft?

(A)  Apprise

(B)  Approach

(C)  Apprehend

(D)  No improvement

Answer: (C)

74. The driver drove slowly so as not to over-turn his load.

(A)  In order that not to overturn his load

(B)  That he may not overturn his load

(C)  Not to overturn his load

(D)  No improvement

Answer: (D)

75. A bird in a hand is worth two in the bush.

(A)  Hands

(B)  Hand

(C)  The hand

(D)  No improvement

Answer: (B)

76. He nobly acquitted himself in the battle.

(A)  Acquitted himself nobly

(B)  Nobly acquitted in the battle

(C)  Acquitted nobly in the battle

(D)  No improvement

Answer: (A)

77. While at Mumbai I stayed in a hotel and mush of my time was spent in interviewing people.

(A)  Spent much of my time

(B)  I had spent much of my time

(C)  Much of my time had been spent

(D)  No improvement

Answer: (A)

78. None as brave deserve the fair.

(A)  Though

(B)  But

(C)  If

(D)  No improvement

Answer: (B)

79. Where had you been playing since we last played together?

(A)  Are your playing

(B)  Have  you been playing

(C)  Would you play

(D)  No improvement

Answer: (B)

Directions-In questions no. 80 to 83 out of the four alternatives, choose the one which can be substituted for the given words/sentence.

80. One who possesses many talents.

(A)  Dexterous

(B)  Versatile

(C)  Gifted

(D)  Exceptional

Answer: (B)

81. One who derives pleasure from inflicting pain on others.

(A)  Recluse

(B)  Hedonist

(C)  Sadist

(D)  Maniac

Answer: (C)

82. A person who rides horses in races as an occupation.

(A)  Equestrain

(B)  Horseman

(C)  Horse-racer

(D)  Jockey

Answer: (D)

83. A widely accepted rule of conduct or general truth briefly expressed.

(A)  Maxim

(B)  Tenet

(C)  Syllogism

(D)  Doctrine

Answer: (B)

Directions-In questions no. 84 and 85, there are four different words out of which one is correctly spelt. Find the correctly spelt from the given options.

84.

(A)  Heirerchy

(B)  Hierarchy

(C)  Heirarchy

(D)  Hairarchy

Answer: (B)

85.

(A)  Carboretor

(B)  Carburettor

(C)  Carborator

(D)  Carrburator

Answer: (B)

Directions-In questions 86 to 100, you have following two brief passages with 10 questions in Passage 1 and 5 questions in Passage II. Read the passage carefully and fill up the blanks with suitable words out of the four alternatives gives.

Passage-I

(Questions No. 86-95)

     Books are the chief carriers of civilization; because of them ideas …(86)… and spread. How important books are,  you can judge from the …(87)… that very hot countries have …(88)… civilization. There are many …(89)… for this, but one of the most important is that white ants in the tropics eat …(90)… all the books. Where there are no books, there are no …(91)… and no literature. The ideas and knowledge of one …(92)… are not handed …(93)… to the next, and it is much more …(94)… for the race to progress and …(95)… civilized.

86.

(A)  improve

(B)  fall

(C)  live

(D)  extend

Answer: (D)

87.

(A)  thought

(B)  fact

(C)  reason

(D)  feeling

Answer: (B)

88.

(A)  the little

(B)  less

(C)  little

(D)  a little

Answer: (C)

89.

(A)  accounts

(B)  consequences

(C)  effects

(D)  reasons

Answer: (D)

90.

(A)  in

(B)  out

(C)  into

(D)  up

Answer: (D)

91.

(A)  memories

(B)  traces

(C)  records

(D)  ideas

Answer: (C)

92.

(A)  generation

(B)  nation

(C)  culture

(D)  race

Answer: (A)

93.

(A)  out

(B)  on

(C)  in

(D)  off

Answer: (B)

94.

(A)  practicable

(B)  desirable

(C)  easy

(D)  difficult

Answer: (D)

95.

(A)  grow

(B)  become

(C)  go

(D)  stay

Answer: (B)

Passage II

(Questions No. 96-100)

     The gorilla is something of a paradox …(96)… the African scene. One thinks one knows him very well. He has been …(97)… and imprisoned in zoos. His bones have been …(98)… in museums everywhere and he has always exerted a strong …(99)… upon scientists. He is an obvious …(100)… with our past.

96.

(A)  from

(B)  at

(C)  in

(D)  to

Answer: (C)

97.

(A)  captured

(B)  killed

(C)  attacked

(D)  untreated

Answer: (A)

98.

(A)  set up

(B)  kept

(C)  mounted

(D)  installed

Answer: (C)

99.

(A)  attraction

(B)  influence

(C)  effect

(D)  fascination

Answer: (B)

100.

(A)  link

(B)  monster

(C)  attachment

(D)  connection

Answer: (A)

Part-III

Quantitative Aptitude

101. 3 cubes whose edges are 3 cm, 4 cm and 5 cm respectively are melted to form a single cube. The surface area of the new cube will be-

(A)  215 sq. cm

(B)  216 sq. cm

(C)  115 sq. cm

(D)  150 sq. cm

Answer: (B)

102. There is a rebate of 15% if electric bills are paid in time. A man got a rebate of Rs 54 by paying the bill in time. His electric bill was-

(A)  Rs 360

(B)  Rs 380

(C)  Rs 300

(D)  Rs 350

Answer: (A)

103. After successive discounts of 20%, 10% and 10%, a washing machine was sold for Rs 6,480. Then the original marked price of the washing machine was-

(A)  Rs 10,800

(B)  Rs 12,000

(C)  Rs 8,960

(D)  Rs 10,000

Answer: (D)

104. A dealer allows his customers a discount of 25% and still gains 25%. If the cost price of the article is Rs 720, then the marked price is-

(A)  Rs 1,200

(B)  Rs 1,100

(C)  Rs 1,400

(D)  Rs 1,300

Answer: (A)

105. If ab, bc, x and c2 are in proportion, then find x, if none of these is equal to zero.

(A)  ac

(B)  a2c2

(C)  a2c

(D)  c2a

Answer: (A)

106. A watermelon is cut into two pieces in the ratio of 3 : 5 by weight. The bigger of the two is further cut in the ratio of 5 : 7 by weight. Find the ratio of each of the three pieces.

(A)  15 : 25 : 26

(B)  5 : 7 : 9

(C)  3 : 5 : 7

(D)  36 : 25 : 35

Answer: (D)

107. Divide Rs 450 among A, B and C such that A : B = 2 : 3 and B : C = 6 : 5. Share of C is-

(A)  Rs 150

(B)  Rs 120

(C)  Rs 90

(D)  Rs 100

Answer: (A)

108. The LCM of two given numbers is 6 times the GCD of the numbers. If the smaller of the two numbers is 6, then the other number is-

(A)  15

(B)  18

(C)  9

(D)  12

Answer: (C)

109. The population of a city has been increasing at the rate of 10% every year. If the present population is 4840000, what was it 2 years ago?

(A)  4100000

(B)  4200000

(C)  3600000

(D)  4000000

Answer: (D)

110. (0.98)333333333 + (0.02)3 + 0.98 ×06 – 1 is-

(A)  1.09

(B)  1.98

(C)  0

(D)  1

Answer: (C)

111. A monkey wanted to climb on the smooth vertical pole of height 35 metres. In the first one minute, he climbs up 5 metres, in the next one minute, he slips down by 2 metres. Further, he repeated the same process till he has reached onto the top of the pole. How many times it has to go upward to reach the apex of the pole?

(A)  11

(B)  27

(C)  35

(D)  12

Answer: (A)

112. If a cistern generally takes 20 minutes to be filled by a pipe but due to a leak, it takes 10 extra minutes to be filled, then find the time in w hich the leak can empty the full cistern.

(A)  50 minutes

(B)  60 minutes

(C)  30 minutes

(D)  40 minutes

Answer: (B)

113. The wheel of a bus 0.75 m in radius, makes 84 revolutions in half minute. Then the speed of the bus in km per hour is-

(A)  23.76

(B)  33.26

(C)  17.82

(D)  47.52

Answer: (D)

114. In a vocational course in a college, 15% seats increase annually. If there were 800 students in 2012, how many seats will be there in 2014?

(A)  1058

(B)  1178

(C)  920

(D)  1040

Answer: (A)

115. If I walks at 4 km/hr, I miss the bus by 10 minutes. If I walk at 5 km/hr, I am 5 minutes early. How far do I walk to reach the bus stand?

(A)  4 km

(B)  6 km

(C)  3 km

(D)  5 km

Answer: (D)

116. Two trains 180 metres and 220 metres long are running in opposite directions at a speed of 40 km/hr and 50 km/hr respectively. They cross each other in-

(A)  17 seconds

(B)  20 seconds

(C)  16 seconds

(D)  18 seconds

Answer: (C)

117. The simple interest on a certain sum for 2 years at 10% per annum is Rs 100. The corresponding compound interest is-

(A)  Rs 105

(B)  Rs 85

(C)  Rs 100

(D)  Rs 95

Answer: (A)

118. A man deposits Rs 5,600 in a bank at 15/4% simple interest. After 6 months he withdraws Rs 2400 together with interest and after 6 months, he withdraws the remaining money. The total amount, he gets as interest, is-

(A)  Rs 150

(B)  Rs 180

(C)  Rs 100

(D)  Rs 125

Answer: (A)

119. The diameters of the internal and external surfaces of a hollow spherical shell are 5 cm and 10 cm respectively. If it is melted and recast into a solid cylinder of length 8/3 cm, then the diameter of the cylinder is-

(A)  14 cm

(B)  16 cm

(C)  7 cm

(D)  10 cm

Answer: (A)

120. The average of runs of a cricket player in 15 matches is 33. If the average of the first 10 matches is 45, then the average of the last 5 matches is-

(A)  15

(B)  23

(C)  9

(D)  13.5

Answer: (C)

121. The average of ten numbers is 30. If 5 is added to every number, the new average is-

(A)  30

(B)  30.5

(C)  35

(D)  53

Answer: (C)

122. Average age of 38 students is 14 years. If the age of the teacher is included, the average becomes 14 years and 4 months. Then teacher’s age is-

(A)  25 years

(B)  29 years

(C)  23 years

(D)  27 years

Answer: (D)

123. I bought a picture for Rs 225, spent Rs 15 to decorate it and sold it for Rs 300. My profit percentage is-

(A)  

(B)  25

(C)  52

(D)  50

Answer: (B)

124. Sharma sold his bike to Mr. Kelkar at a loss of 20%. Mr. Kelkar spends Rs 2,000 on its repair and sells it for Rs 22,000, thus making a profit of 10%. Then the cost price of the bike for Mr. Sharma is-

(A)  Rs 22,500

(B)  Rs 15,000

(C)  Rs 18,500

(D)  Rs 25,000

Answer: (A)

125. A dishonest dealer claims that he sells his goods at the cost price but uses a false weight of 920 gm instead of 1 kg. Find his gain per cent.

(A)  9%

(B)  8.5%

(C)  8%

(D)  8.7%

Answer: (D)

126. 75% of a number when added to 75 is equal to the number. The number is-

(A)  225

(B)  300

(C)  150

(D)  200

Answer: (B)

127. If a = (√3 + √2)3 and b = (√3 − √2)3, then the value of (a + 1)1 + (b + 1)1 is-

(A)  48√2

(B)  50√3

(C)  1

(D)  5

Answer: (C)

128. If 5a + 3b : 4a + 7b = 3 : 4, then a : b is equal to-

(A)  9 : 8

(B)  7 : 11

(C)  8 : 9

(D)  11 : 9

Answer: (A)

129. The value of k for which the expression 9x2 – kx + 16 be a perfect square is-

(A)  ±12

(B)  ±24

(C)  ±2

(D)  ±6

Answer: (B)

130. The solution(s) of  is/are-

(A)  +8

(B)  −6

(C)  ±2

(D)  ±4

Answer: (C)

131. The water contained by a hemispherical cup having diameter 13.5 cm is poured into an empty right circular cylindrical glass of diameter 9 cm. Then the depth of the water in the glass is-

(A)  8 cm

(B)  

(C)  

(D)  

Answer: (C)

132. The area of the base of a cuboidal box is 21 sq. cm and the area of one of the faces is 30 sq. cm. The numerical value of each of the dimensions of this box is an integer greater than 1. Then the volume of the cuboidal box, in cu. cm, is-

(A)  210

(B)  630

(C)  105

(D)  120

Answer: (A)

133. A chord of a circle of radius 14 cm makes a right angle at the centre. The area of the minor segment of the circle is (take π = 22/7)

(A)  154 cm2

(B)  252 cm2

(C)  56 cm2

(D)  98 cm2

Answer: (A)

134. Eliminating t from the equations, x = cos t, y = 2 sin t cos t, we get-

(A)  y2 = 2x2 + 4x4

(B)  y2 = 2x2 – 4x4

(C)  y2 = 4x2 + 4x4

(D)  y2 = 4x2 – 4x4

Answer: (D)

135. If tan4 θ + tan2 θ = 1, then value of cos4 θ + cos2 θ is equal to-

(A)  3

(B)  10

(C)  1

(D)  2

Answer: (C)

136. The value of sin(45° + θ) – cos(45° − θ) is equal to-

(A)  1

(B)  0

(C)  2 cos θ

(D)  2 sin θ

Answer: (B)

137. In a ∆ ABC,  is a median. Which is the correct relation?

(A)  AB + AC > 2AD

(B)  AB + AC < 2AD

(C)  AB + AC = 2AD

(D)  None of these

Answer: (A)

138. If ∆ ABC and ∆ DEF one similar such that 2AB = DE, BC = 8 cm, then EF is-

(A)  4 cm

(B)  8 cm

(C)  16 cm

(D)  12 cm

Answer: (C)

139. From an external point, a tangent to a circle is drawn. If length of this tangent be 8 cm and radius of the circle is 6 cm, the distance of the external point from the centre of the circle is-

(A)  12/√3 cm

(B)  10 cm

(C)  8√3 cm

(D)  6√3 cm

Answer: (B)

140. A chord of length 16 cm is drawn in a circle of radius 10 cm. The distance of the chord from the centre of the circle is-

(A)  9 cm

(B)  12 cm

(C)  6 cm

(D)  8 cm

Answer: (C)

Directions-(Q. 141-145) Study the following frequency polygon showing the family size (i.e. the number of numbers in a family) alongwith the corresponding frequency (i.e. number of the families) and answers these questions.

141. There are 10 families having the family size; the family size is-

(A)  4

(B)  5

(C)  3

(D)  6

Answer: (D)

142. The family size possessed by maximum number of families is-

(A)  3

(B)  8

(C)  4

(D)  5

Answer: (C)

143. The number of families having size ‘4’ or less is-

(A)  50

(B)  40

(C)  35

(D)  45

Answer: (B)

144. The percentage of families having size ‘5’ is-

(A)  20

(B)  25

(C)  30

(D)  15

Answer: (B)

145. The difference between the number of families having size ‘3’ and size ‘6’ is-

(A)  15

(B)  8

(C)  5

(D)  10

Answer: (C)

Directions-(Q. 146-150) Study the following  histogram showing frequency distribution of marks of a group of students and answer these questions.

146. The percentage of students obtaining marks above 60 is-

(A)  

(B)  40

(C)  30

(D)  

Answer: (D)

147. The maximum number of students secured marks between-

(A)  40 and 60

(B)  80 and 100

(C)  20 and 40

(D)  60 and 80

Answer: (A)

148. The proportion of students getting marks in the range 60 to 80 is-

(A)  1/6

(B)  1/5

(C)  2/3

(D)  1/10

Answer: (B)

149. Exactly 100 students have secured their marks above-

(A)  60

(B)  80

(C)  40

(D)  20

Answer: (D)

150. The total number of students is-

(A)  150

(B)  180

(C)  100

(D)  120

Answer: (A)

For Visually Handicapped Candidates Only

141. A florist bought 240 roses at Rs 9 per dozen. If he sells all of them at Rs 1 each, his profit percentage is-

(A) 

(B) 

(C) 

(D) 

Answer: (D)

142. Equilateral triangles are drawn on one of the sides and one of the diagonals of a square as the box. The ratio of the areas of the two triangles is-

(A)  √3 : 4

(B)  1 : 4

(C)  1 : √2

(D)  1 : 2

Answer: (D)

143. If 13 sin θ – 12 = 0 and θ is an acute angle, then the value of  is-

(A)  1306/159

(B)  1036/195

(C)  1063/195

(D)  1036/159

Answer: (B)

144. One angle of a triangle is 108°, the angle included between the internal bisectors of the two acute angles of the triangle is-

(A)  144°

(B)  54°

(C)  72°

(D)  136°

Answer: (A)

145. The value of 

(A) 

(B)

(C)  1.20

(D)  

Answer: (B)

146. If (3x – 2y) : (x + 3y) = 5 : 6, then the value of x : y is-

(A)  17/13

(B)  5/13

(C)  3/13

(D)  27/13

Answer: (D)

147. If  then the value of  is-

(A)  5

(B)  3

(C)  4

(D)  8/3

Answer: (C)

148. Three persons X, Y, Z start a partnership business investing Rs 8,000, Rs 6,000 and Rs 4,000. They agree that the profit will be distributed in the ratio of their capitals. If the profit be Rs 15,453, then the share of X is-

(A)  Rs 7,475

(B)  Rs 3,964

(C)  Rs 5,854

(D)  Rs 6,868

Answer: (D)

149. 16 children take 12 days to complete a work which can be completed by 6 adults in 16 days. 16 adults started working and after 3 days, 10 adults left and 4 children joined them. How many days will they take to complete the remaining work?

(A)  12 days

(B)  15 days

(C)  6 days

(D)  9 days

Answer: (C)

150. A fisherman can row 2 km against the stream in 20 minutes and return in 15 minutes. Then the speed of the current is-

(A)  3 km/hr

(B)  4 km/hr

(C)  1 km/hr

(D)  2 km/hr

Answer: (C)

Part-IV

General Awareness

151. Which of the following Fundamental Rights is only for the citizens of India?

(A)  Right to life and liberty

(B)  Freedom of religion

(C)  Right to equality

(D)  Freedom of expression

Answer: (C)

152. The origin of Indian music could be traced back to which of the following Vedic Samhits?

(A)  Sama Veda

(B)  Yajur Veda

(C)  Rig Veda

(D)  Atharva Veda

Answer: (A)

153. Which one of the following was the most famous center of learning during the Mauryan period?

(A)  Ujjain

(B)  Vallabhi

(C)  Nalanda

(D)  Taxila

Answer: (A)

154. Kharvela, the Kalinga ruler patronized-

(A)  Hinduism (Viashnavism)

(B)  Shaivism

(C)  Buddhism

(D)  Janinism

Answer: (D)

155. Which of the following is not a reform measure adopted by Akbar?

(A)  Dag

(B)  Manasbdari System

(C)  Iqta system

(D)  Zabti

Answer: (C)

156. The first Anglo-Maratha War came to an end by the treaty of

(A)  Surat

(B)  Bassein

(C)  Salbai

(D)  Purandar

Answer: (C)

157. The name of India’s Permanent Research Station situated in the Antarctica is-

(A)  Dakshin Bharat

(B)  Dakshin Nivas

(C)  Dakshin Chitra

(D)  Maitri

Answer: (D)

158. The biggest copper producing country in Africa is-

(A)  Kenya

(B)  Tanzania

(C)  Union of South Africa

(D)  Zambia

Answer: (D)

159. Which one of the following is not possible under short-run?

(A)  Increasing the working time

(B)  Technology upgradation

(C)  Hiring additional workers

(D)  Procuring more raw materials

Answer: (D)

160. Value added tax was first introduced in-

(A)  Germany

(B)  Spain

(C)  France

(D)  Russia

Answer: (C)

161. “Supply creates its own demand” is the-

(A)  Law of Market

(B)  Law of Demand and Supply

(C)  Law of Demand

(D)  Law of Supply

Answer: (A)

162. Demand pull inflation is a situation of-

(A)  Excess demand

(B)  Parity between demand and supply

(C)  Excess supply

(D)  None of the above

Answer: (A)

163. Gandhi is known as-

(A)  Philosophical anarchist

(B)  Radical anarchist

(C)  Marxist

(D)  Fabianist

Answer: (A)

164. Indian Federalism is close to-

(A)  Nigeria

(B)  Australia

(C)  Canada

(D)  USA

Answer: (C)

165. Fundamental Rights are enshrined in which Part of the Indian Constitution?

(A)  IV

(B)  V

(C)  II

(D)  III

Answer: (D)

166. Which Article of the Indian Constitution provides special status to Jammu and Kashmir?

(A)  371

(B)  390

(C)  370

(D)  369

Answer: (C)

167. Which type of writs are not specifically provided in the Indian Constitution?

(A)  Mandamus

(B)  Quo warranto

(C)  Injunction

(D)  Prohibition

Answer: (C)

168. Which of the following algae is being used in space research?

(A)  Hydrodictyon

(B)  Cladophora

(C)  Chlamydomonas

(D)  Chlorella

Answer: (D)

169. Which one of the following is not a green house gas?

(A)  Ethane

(B)  Water vapour

(C)  Chlorofluorocarbon

(D)  Carbon dioxide

Answer: (A)

170. Which who basic forces are able to provide an attractive force between two neutrons?

(A)  Gravitational and electrostatic

(B)  Some other forces

(C)  Gravitational and nuclear

(D)  Electrostatic and nuclear

Answer: (C)

171. Which of the following phenomenon cannot take place in sound waves?

(A)  Diffraction

(B)  Polarization

(C)  Reflection

(D)  Interference

Answer: (A)

172. Identify the sequential-access media-

(A)  Magnetic tape

(B)  Digital video disk

(C)  Floppy disk

(D)  Optical disk

Answer: (A)

173. Which one of the following is more suitable for objective type examination?

(A)  Optical Mark Reading and Recognition

(B)  Optical Text Recognition

(C)  Magnetic Ink Character Recognition

(D)  Optical Character Recognition

Answer: (A)

174. The _____ service allows as group of Internet users to exchange their views on some common topic.

(A)  Nicnet

(B)  Milnet

(C)  Telnet

(D)  Usenet

Answer: (D)

175. In graphite, layers are held together by-

(A)  van der Waals forces

(B)  metallic bond

(C)  ionic bond

(D)  covalent bond

Answer: (A)

176. Which of the following places is the destination of iron-ore (slurry) carried from Kudremukh through pipeline?

(A)  Koyali

(B)  Jagdishpur

(C)  Mangalore

(D)  Udaipur

Answer: (C)

177. Which of the following geomorphical pairs is not correct?

(A)  Cirque-Glacier

(B)  Tombolo-Wave

(C)  Delta-River

(D)  Sinkholes-Wind

Answer: (D)

178. Sideite is an ore of-

(A)  Copper

(B)  Tin

(C)  Aluminium

(D)  Iron

Answer: (D)

179. Common cold is caused by-

(A)  Virus

(B)  Bacteria

(C)  Protozoa

(D)  Unicellular algae

Answer: (A)

180. Stilt roots are present in-

(A)  Banyan

(B)  Maize

(C)  Mango

(D)  China rose

Answer: (B)

181. Vitamin D is essential for-

(A)  prevention of osteoarthritis

(B)  absorption of magnesium from diet

(C)  absorption of calcium from diet

(D)  making strong and healthy bones

Answer: (C)

182. Which pair of structures are usually found in both plant and animal cells?

(A)  Cell wall and nucleus

(B)  Nucleus and chloroplast

(C)  Endoplasmic reticulum and cell membrane

(D)  Cell membrane and cell wall

Answer: (C)

183. Which of the following are homologous organs?

(A)  Wings of insects and birds

(B)  Tail of fish and cat

(C)  Skin of crocodile and feather of birds

(D)  Forelimb of horse and man

Answer: (A)

184. Who among the following Indian Prime Ministers received “NISHAN-E-PAKISTAN”, the highest civilian award in Pakistan?

(A)  Morarji Desai

(B)  Rajiv Gandhi

(C)  Atal Behari Vajpayee

(D)  Indira Gandhi

Answer: (A)

185. Mohammed Azharuddin, former Indian Cricket Team Captain represents ______ constituency of Uttar Pradesh in the Lok Sabha.

(A)  Mirzapur

(B)  Moradabad

(C)  Meerut

(D)  Mathura

Answer: (B)

186. Which country won the Gold in Sultan Azlan Shah Cup Hockey Tournament in 2012?

(A)  New Zealand

(B)  South Korea

(C)  Argentina

(D)  India

Answer: (A)

187. Where is Netaji Subhash National Institute of Sports?

(A)  Kolkata

(B)  Pune

(C)  Patiala

(D)  Gwalior

Answer: (C)

188. The only leader who got the Bharat Ratna award while holding the office of the Union Home Minister is-

(A)  Sardar Vallabhbhai Patel

(B)  Gulzari Lal Nanda

(C)  Lal Bahadur Shastri

(D)  Govind Ballabh Pant

Answer: (D)

189. Mullaperiyar Dam is a disputed issue between-

(A)  Tamil Nadu and Andhra Pradesh

(B)  Tamil Nadu and Karnataka

(C)  Kerala and Tamil Nadu

(D)  Kerala and Karnataka

Answer: (C)

190. Margaret Alva is the Governor of-

(A)  Tamil Nadu

(B)  Uttarakhand

(C)  Gujarat

(D)  Sikkim

Answer: (B)

191. The Union Territory of Puducherry does not have a common border with-

(A)  Karnataka

(B)  Tamil Nadu

(C)  Andhra Pradesh

(D)  Kerala

Answer: (A)

192. Fullerene, a newly discovered crystalline carbon allotrope, contains-

(A)  100 C atoms

(B)  80 C atoms

(C)  60 C atoms

(D)  40 C atoms

Answer: (C)

193. The metal which is protected by a layer of its own oxide is-

(A)  Gold

(B)  Iron

(C)  Aluminium

(D)  Silver

Answer: (C)

194. Filaria is caused by-

(A)  Wuchereria bancrofti

(B)  Plasmodium vivax

(C)  Ascaris lubricoides

(D)  Entamoeba histolytica

Answer: (A)

195. The maximum moisture content of the composting mixture for vermicomposting is-

(A)  40%

(B)  65%

(C)  30%

(D)  35%

Answer: (A)

196. The process of ozonation in water treatment is known as-

(A)  Ionization

(B)  Sedimentation

(C)  Precipitation

(D)  Disinfection

Answer: (D)

197. Permissible concentration of residual chlorine in drinking water in mg/L is-

(A)  1.0

(B)  5.0

(C)  0.2

(D)  0.05

Answer: (B)

198. Lead poisoning in vertebrates is not characterized by-

(A)  Anaemia

(B)  Methemoglobinemia

(C)  Neurological defects

(D)  Kidney dysfunction

Answer: (B)

199. Who is the Chief of the Indian Army?

(A)  Gen. J. J. Singh

(B)  Gen. Dalbir Singh

(C)  Gen. V. K. Singh

(D)  Gen. Bikram Singh

Answer: (B)

200. India is not a member of-

(A)  G-20

(B)  G-8

(C)  SAARC

(D)  U.N.

Answer: (B)

SSC Combined Higher Secondary Level (10+2) Examination (1st Shift) Tier-I Held on 04 November, 2012 Question Paper With Answer Key

SSC Combined Higher Secondary Level (10+2) Examination (1st Shift) Tier-I Held on 04 November, 2012
SSC Combined Higher Secondary Level (10+2) Examination (1st Shift) Tier-I Held on 04 November, 2012 Question Paper With Answer Key

Staff Selection Commission Combined Higher Secondary Level (10+2) Examination

(1st Shift) Tier-I Held on 04 November, 2012

Part-I

General Intelligence

Directions-(Q. 1-6) select the related letter/word/number from the given alternatives.

1. Play : Actor : : Concert : ?

(A)  Piano

(B)  Percussion

(C)  Symphony

(D)  Musician

Answer: (D)

2. QDXM : SFYN : : UIOZ : ?

(A)  QNLA

(B)  WKPA

(C)  PAQM

(D)  LPWA

Answer: (B)

3. CDEF : UVWX : : IJKL : ?

(A)  OPQR

(B)  MNOP

(C)  ABCD

(D)  NOPQ

Answer: (A)

4. 5 : 100 : : 7 : ?

(A)  91

(B)  98

(C)  49

(D)  196

Answer: (D)

5. 16 : 22 : : 36 : ?

(A)  44

(B)  26

(C)  24

(D)  46

Answer: (D)

6. Book : Paper : : Bread : ?

(A)  Cake

(B)  Butter

(C)  Flour

(D)  Biscuit

Answer: (C)

Directions-(Q. 7-11) Select the one which is different from the other three responses.

7.

(A)  342

(B)  511

(C)  126

(D)  215

Answer: (C)

8.

(A)  IKN

(B)  MOR

(C)  ACF

(D)  EGI

Answer: (D)

9.

(A)  Energy

(B)  Erg

(C)  Power

(D)  Pressure

Answer: (B)

10.

(A)  NEM

(B)  MAL

(C)  QRP

(D)  RUQ

Answer: (C)

11.

(A)  Kerosene

(B)  LPG

(C)  Petrol

(D)  Diesel

Answer: (B)

Directions-(Q. 12 and 13) Which one of the given responses would be a meaningful order of the following?

12. (1) wrist (2) arm

(3) shoulder          (4) finger

(5) palm                (6) nail

(A)  5, 4, 6, 1, 2, 3

(B)  3, 2, 1, 5, 4, 6

(C)  6, 4, 5, 2, 1, 3

(D)  3, 2, 5, 1, 4, 6

Answer: (B)

13. (1) Flower (2) Seed

(3) Plant               (4) Fruits

(5) Branches         (6) Tree

(A)  2, 1, 4, 6, 5, 3

(B)  2, 1, 3, 6, 5, 4

(C)  2, 3, 6, 5, 1, 4

(D)  2, 6, 3, 1, 4, 5

Answer: (C)

Directions-(Q. 14 and 15) A series is given, with one number/letter missing. Choose the correct alternative from the given ones that will complete the series.

14. ELFA, GLHA, ILJA, ……?………, MLNA

(A)  LLMA         

(B)  KLLA

(C)  OLPA

(D)  KLMA

Answer: (B)

15. 15, 39, 74, 122, …….?……, 265.

(A)  171

(B)  185

(C)  151

(D)  165

Answer: (B)

16. Which one set of letters when sequentially placed at the gaps in the given letter series shall complete it?

l…….b………..ub………ubt…………blu…………..tub

(A)  t u l b u

(B)  b u t l u

(C)  u b t l u

(D)  u t l u b

Answer: (D)

Directions-(Q. 17-21) Select the missing number/letter from the given responses.

17. 

?      27

21   13

(A)  64

(B)  63

(C)  49

(D)  37

Answer: (A)

18.

49   100    64

9     36      4

81   1        25

19   ?        15

(A)  17

(B)  18

(C)  14

(D)  16

Answer: (A)

19. 17, 19, 23, 29, ………

(A)  31

(B)  39     

(C)  33

(D)  35

Answer: (A)

20. 23, 24, 27, 28, 31, 32, ……..

(A)  29

(B)  35

(C)  30

(D)  33

Answer: (B)

21. 

E     G       I

R     U       X

J      N       ?

(A)  O

(B)  Q

(C)  P

(D)  R

Answer: (D)

22. The total number of students in class A and class B is 120. If 15 students from class A are taken and added to class B, the number in class B becomes twice that of A. Find out the number of students in class A in the beginning-

(A)  55

(B)  40

(C)  80

(D)  65

Answer: (A)

23. Kannan is the brother of Kumar. Lakshmi is the daughter of Kumar. Kalai is sister of Kannan and Govind is the brother of Lakshmi. Who is the uncle of Govind?

(A)  Lakshmi

(B)  Kannan

(C)  Kumar

(D)  Kalai

Answer: (B)

24. Find out the pair of numbers that do not belong to the group for each of common property.

(A)  196-121

(B)  100-225

(C)  144-81

(D)  169-600

Answer: (D)

25. Introducing a girl, Vein said, “Her mother

(A)  Brother

(B)  Husband

(C)  Uncle

(D)  Father

Answer: (D)

Directions-(Q. 26 and 27) from the given alternatives select the word which cannot be formed using the letters of the given word.

26. ‘RECOMMENDATION’

(A)  COMMEND

(B)  MEND

(C)  COMMENTARY

(D)  COMMON

Answer: (C)

27. OPERATION

(A)  RATIO

(B)  NATION

(C)  RATION

(D)  OPERA

Answer: (B)

28. In the morning, I walked 2 kms facing the Sun and stopped, From there, I walked 4 kms on my right hand side, from there I walked I km again facing the Sun, from there I walked on my right hand side. Which direction I will be walking now?

(A)  South West

(B)  Right hand side

(C)  East

(D)  South East

Answer: (B)

29. If ‘+’ and ‘−’ signs are interchanged, similarly ‘×’ and ‘÷’ are interchanged, then find the answer of the following equation-

10 – 2 + 12 × 1 ÷ 0

(A)  1

(B)  0

(C)  8

(D)  12

Answer: (D)

30. If ‘BASKET’ is written as ‘TEKSAB’, how can ‘PILLOW’ we written in that code?

(A)  LOWLIP

(B)  WOLPIL

(C)  LOWPIL

(D)  WOLLIP

Answer: (D)

31. In certain code, TOPPER is written as POTREP. In that code, which word will be written as RUBREG?

(A)  BURGET

(B)  BEURGR

(C)  BURGER

(D)  BLURBE

Answer: (C)

32. If EARN is written as GCTP, how NEAR can be written in that code?

(A)  PGCT

(B)  PCGT

(C)  CTGP

(D)  GPTC

Answer: (A)

33. A, B, C, D, E and F are sitting in a circle facing towards centre. D is between F and B. A is second to the left of D and second to the right of E. Who is facing towards D?

(A)  E

(B)  B

(C)  C

(D)  A

Answer: (C)

34. Choose the appropriate combination of signs to solve the equation.

(23 – 5) * (12 ÷ 2) * 3 * 6

(A)  × ÷ =

(B)  + − =

(C)  ÷ + =

(D)  − ÷ =

Answer: (C)

Directions-(Q. 35 to 37) One/two statements are given followed by two conclusions I and II. You have to consider the two statements to be true even if they seem to be at variance from commonly known facts. You have to decide which of the given conclusions, if any, follow from the given statements.

35. Statements : Rakesh is senior to Rajesh.

And Rajesh is senior to Rahul.

Conclusions : (i) Rakesh is senior to Rahul.

(ii) Rahul is not senior to Rakesh.

(A)  Both conclusion (i) and (ii) follow

(B)  Neither conclusion (i) nor (ii) follows

(C)  Only conclusion (i) follows

(D)  Only conclusion (ii) follows

Answer: (C)

36. Statements : (1) All crows are black.

(2) Some black things are beautiful.

Conclusions: (i) Some crows are beautiful.

(ii) Some beautiful things are black.

(A)  Both conclusion (i) and (ii) follow

(B)  Neither conclusion (i) nor (ii) follows

(C)  Only conclusion (i) follows

(D)  Only conclusion (ii) follows

Answer: (D)

37. Statement : Uneducated people are exploited by others in the society.

Conclusions : (i) People take advantage of uneducated people only.

(ii) Educated people exploit uneducated persons in society.

(A)  Both conclusion (i) and (ii) follow

(B)  Neither conclusion (i) nor (ii) follows

(C)  Only conclusion (i) follows

(D)  Only conclusion (ii) follows

Answer: (B)

38. Which number is on the face opposite to 6 in the dice whose four views are given below?

Answer: (C)

39. How many backward uneducated people are employed?

(A)  7

(B)  11

(C)  14

(D)  5

Answer: (D)

40. From the following two different appearances of a dice, find out which colour is opposite to white?

(A)  Yellow

(B)  Red

(C)  Blue

(D)  Black

Answer: (C)

41. Find the best relationship among the three classes in the following diagrams:

Answer: (B)

42. Which of the following answer figure indicates best relation between aeroplanes, train, travel?

Answer: (C)

Directions-(Q. 43 and 44) which answer figure completes the pattern given in the question figure?

43. 

Answer: (D)

44. 

Answer: (A)

45. How many triangles are there in the given question figure?

(A)  16

(B)  20

(C)  15

(D)  14

Answer: (C)

46. From the given answer figures, select the one in which the question figure is hidden/embedded:

Answer: (C)

Directions-(Q. 47 and 48) A piece of paper is folded and cut as shown below in the question figures. From the given answer figures, indicate how it will appear when opened.

47. 

Answer: (D)

48. 

Answer: (A)

49. Find out the mirror image of the given figure :

Answer: (B)

50. Identify the missing figure to complete the matrix :

Answer: (B)

For Visually Handicapped Candidates Only

Directions-(Q 38 to 40) Select the related letter/word/number from the given alternatives.

38. DONKEY : ENOJFX : : FLOWER : ?

(A)  GKPVFQ

(B)  GMPVDQ

(C)  GMPXFS

(D)  GKNVFQ

Answer: (A)

39. 2 : 32 : : 3 : ?

(A)  183

(B)  143

(C)  243

(D)  293

Answer: (C)

40. Plane : Air : : Car : ?

(A)  Road

(B)  Atmosphere

(C)  Motor cycle

(D)  Engine

Answer: (A)

Directions- (Q. 41 to 42) which one is different from the rest three?

41. 

(A)  24-57

(B)  56-70

(C)  35-66

(D)  71-94

Answer: (D)

42.

(A)  Nose

(B)  Tears

(C)  Eyes

(D)  Ears

Answer: (B)

43. From the given alternative words, select the word which cannot be formed using the letters of the given word?

CHARECTERISATION

(A)  FRICTION

(B)  CHARTER

(C)  ERECTION

(D)  RATION

Answer: (A)

44. Find the missing letters of the series:

ABC, GHI, MNO ……….?………

(A)  RST

(B)  STU

(C)  PQR

(D)  QRS

Answer: (B)

45. Which one set of letters when sequentially placed at the gaps in the given letter series shall complete it?

a a….b c c a a b b c ……a…b….c c

(A)  c c a a

(B)  b c c a

(C)  a b a b

(D)  b c a b

Answer: (D)

46. Arrange the following words according to the dictionary order:

(1) Corporate      (2) Correspond

(3) Correlation    (4) Convenience

(5) Co-operate

(A)  5, 1, 2, 4, 3

(B)  4, 5, 1, 3, 2

(C)  1, 2, 3, 5, 4

(D)  4, 1, 5, 2, 3

Answer: (B)

47. Divide Rs 600 among A : B : C in the ratio 3 : 5 : 7. Then B’s share is:

(A)  Rs 280

(B)  Rs 400

(C)  Rs 120

(D)  Rs 200

Answer: (D)

48. Find the wrong number in the given series :

3, 5, 12, 39, 154, 772, 4634

(A)  39

(B)  154

(C)  5

(D)  3

Answer: (A)

49. If EXAMINATION is written as AMINATIONEX, How can EXTRAPOLATION be written in that code?

(A)  TRAPOLATIONEX

(B)  POLATIONEXTRA

(C)  POLARATIONEX

(D)  NOITALOPARTXE

Answer: (A)

50. If 1 = 6, 2 = 7, 3 = 8, 4 = 9, and 5 = 0, then 786 = ?

(A)  321

(B)  123

(C)  213

(D)  231

Answer: (D)

Part-II

English Language

Directions-(Q. 51-53) Choose the word opposite in meaning to the given word and mark it in the Answer Sheet.

51. incessant

(A)  continuous

(B)  intermittent

(C)  unceasing

(D)  constant

Answer: (B)

52. eternal

(A)  usual

(B)  active

(C)  realistic

(D)  temporary

Answer: (D)

53. comedy

(A)  tragedy

(B)  trilogy

(C)  limerick

(D)  clergy

Answer: (A)

   Directions-(Q. 54-56) Out of the four alternatives, choose the one which best expresses the meaning of the given word and mark it in the Answer Sheet.

54. bliss

(A)  pleasure

(B)  fantasy

(C)  happiness

(D)  laughter

Answer: (C)

55. candid

(A)  able

(B)  quiet

(C)  fearless

(D)  frank

Answer: (D)

56. meagre

(A)  plenty

(B)  inadequate

(C)  sufficient

(D)  limited

Answer: (D)

Directions-(Q. 57-61) Four alternatives are given for the idiom/phrase bold in the sentence. Choose the alternative which best expresses the meaning of the idiom/phrase and mark it in the Answer Sheet.

57. I took exception to his remarks and left the meeting.

(A)  objected

(B)  suggested

(C)  heard

(D)  excused

Answer: (A)

58. He turned a deaf ear to his parents’ advice.

(A)  listen carefully

(B)  refused to obey

(C)  big help

(D)  attentively

Answer: (B)

59. Most people live from hand to mouth these days because of inflation.

(A)  lavishly

(B)  happily

(C)  comfortably

(D)  miserably

Answer: (D)

60. There are no hard and fast rules for admission to this college.

(A)  easy

(B)  strict

(C)  fixed

(D)  slow

Answer: (C)

61. Please hold your tongue in this matter, otherwise you will repent.

(A)  be silent

(B)  give advice

(C)  defend

(D)  argue

Answer: (A)

   Directions-(Q. 62-68) A part of the sentence is bold. Below are given alternatives to the bold part at (A), (B) and (C) which may improve the sentence. Choose the correct alternative. In case no improvement is needed your answer is (D). Mark your answer in the Answer Sheet.

62. He has left for London last night.

(A)  was leaving

(B)  hand left

(C)  left

(D)  No improvement

Answer: (C)

63. I hope we have taught our children to know right from wrong.

(A)  know right and wrong

(B)  understand right and wrong

(C)  see what is right

(D)  No improvement

Answer: (B)

64. He is one of those who believe in God.

(A)  of those who believes

(B)  who believe

(C)  who has believe

(D)  No improvement

Answer: (A)

65. This box of chocolates belonged to me.

(A)  belong

(B)  belongs

(C)  is belonging

(D)  No improvement

Answer: (B)

66. It is not wise to do the work at the eleventh hour.

(A)  on the eleventh hour

(B)  at the moment

(C)  at the end

(D)  No improvement

Answer: (D)

67. Books are ones best friends.

(A)  ones’

(B)  once

(C)  one’s

(D)  No improvement

Answer: (C)

68. The Principal was praised on his fiftieth birthday.

(A)  rewarded

(B)  awarded

(C)  honoured

(D)  No improvement

Answer: (C)

   Directions- (Q. 69-72) Out of the four alternatives choose the one which can be substituted for the given words/sentence.

69. Rebellion against lawful authority-

(A)  Mutiny

(B)  Coup

(C)  Revolution

(D)  Dissidence

Answer: (A)

70. Soldiers who fight on horseback-

(A)  Infantry

(B)  Artillery

(C)  Cavalry

(D)  Armoured

Answer: (C)

71. One who speaks for others-

(A)  Spokesman

(B)  Leader

(C)  Supporter

(D)  Naming

Answer: (A)

72. A man who doesn’t know how to read or write-

(A)  Uneducated

(B)  Illiterate

(C)  Ignorant

(D)  Oblivious

Answer: (B)

   Directions-(Q. 73 & 74) There are four different words out of which one is correctly spelt. Find the correctly spelt word and indicate it by blackening the appropriate oval (•) in the Answer Sheet.

73.

(A)  Embraus

(B)  Embarrass

(C)  Embarass

(D)  Embaruss

Answer: (C)

74.

(A)  Release

(B)  Relese

(C)  Rilese

(D)  Releise

Answer: (A)

   Directions- (Q. 75-89) You have following two brief passages with 10 questions in Passage-1 and 5 questions in Passage – II. Read the passages carefully and fill in the blanks with, out of the four alternatives given.

PASSAGE-1

   Without water …(75)… animal can survive. In desert regions, the greatest …(76)… to life is drying  up. But many creatures are able to make use of …(77)… little water that  exists in arid areas. One of nature’s masterpieces …(78)… creatures equipped to …(79)…  with desert life is the hardy camel. There are several stories describing the …(80)… endurance of these animals. It is said that camels can …(81)… a distance of about 800 miles in eight days through continuous travel …(82)… an intake of a single drop of water. The popular …(83)… in a way is …(84)… Water is indeed stored there but in the form of fat.

75. 

(A)  every

(B)  no

(C)  any

(D)  desert

Answer: (B)

76.

(A)  want

(B)  inadequacy

(C)  requirement

(D)  worry

Answer: (C)

77.

(A)  what

(B)  the

(C)  very

(D)  that

Answer: (B)

78.

(A)  for

(B)  about

(C)  among

(D)  with

Answer: (C)

79.

(A)  live

(B)  resist

(C)  bear

(D)  cope

Answer: (D)

80.

(A)  remarkable

(B)  little

(C)  tolerable

(D)  popular

Answer: (A)

81.

(A)  measure

(B)  reduce

(C)  cover

(D)  reach

Answer: (D)

82.

(A)  for

(B)  without

(C)  accepting

(D)  receiving

Answer: (B)

83.

(A)  proverb

(B)  guess

(C)  belief

(D)  version

Answer: (C)

84.

(A)  baseless

(B)  wrong

(C)  misleading

(D)  correct

Answer: (D)

PASSAGE-II

   About a million different species …(85)… insects have been identified. Yet little is …(86)… about some of them. Although insects vary so much in their shape and …(87)… they all have the …(88)… structure. All insects normally …(89)… six legs.

85.

(A)  for

(B)  of

(C)  about

(D)  in

Answer: (B)

86.

(A)  known

(B)  observed

(C)  seen

(D)  found

Answer: (D)

87.

(A)  colour

(B)  height

(C)  size

(D)  habits

Answer: (C)

88.

(A)  different

(B)  alike

(C)  same

(D)  single

Answer: (A)

89.

(A)  has

(B)  have

(C)  had

(D)  got

Answer: (B)

Directions-(Q. 90-95) Some parts of the sentences have errors and some have none. Find out which part of a sentence has an error and blacken the oval () corresponding to the appropriate letter (A, B, C). If a sentence is free from error, blacken the oval corresponding to (D) in the Answer Sheet.

90. The teacher told to (A)/ the students that (B)/ they must attend school regularly. (C)/ No error (D)

Answer: (A)

91. Modern youth pay more attention (A)/ to seeing films (B)/ than to read books. (C)/ No error (D)

Answer: (B)

92. Old habits (A)/ die (B)/ hardly. (C)/ No error (D)

Answer: (D)

93. I (A)/ have been studying (B)/ since four hours. (C)/ No error (D)

Answer: (C)

94. If he had walked (A)/ fast enough (B)/ he will get the bus. (C)/ No error (D)

Answer: (C)

    

95. Speakers after speakers (A)/ came on the stage (B)/ to perform. (C)/ No error (D)

Answer: (D)

Directions-(Q. 96-100) Sentences are given with blanks to be filled in with an appropriate word(s). Four alternatives are suggested for each question. Choose the correct alternative out of the four and indicate it by blackening the appropriate oval (•) in the Answer Sheet.

96. He ………. His living by hard work.

(A)  earns

(B)  wins

(C)  creates

(D)  ables

Answer: (A)

97. Bread is usually made …….. wheat.

(A)  of

(B)  from

(C)  with

(D)  by

Answer: (A)

98. Did the child ……….. from the chair?

(A)  fell

(B)  fallen

(C)  falling

(D)  fall

Answer: (D)

99. Do you ………. to see my collection?

(A)  cared

(B)  like

(C)  want

(D)  got

Answer: (C)

100. After initial setback, all ………. Programmes were successful due to the initiative of the new dynamic team.

(A)  consequent

(B)  subsequently

(C)  consequently

(D)  subsequent

Answer: (D)

Part-III

Quantitative Aptitude

101. In a triangle, distance from centroid to vertices are respectively 4 cm, 6 cm and 8 cm. Find medians-

(A)  6 cm, 9 cm, 12 cm

(B)  20 cm, 21 cm, 25 cm

(C)  16 cm, 4 cm, 18 cm

(D)  4 cm, 6 cm, 8 cm

Answer: (A)

102. The numerical values of-

 is-

(A)  9

(B)  1

(C)  4

(D)  5

Answer: (A)

103. If  then the value of  is-

(A)  1/5

(B)  3/5

(C)  2

(D)   

Answer: (A)

104. Three medians  of ∆ ABC Intersect at G. Area of ∆ ABC is 108 sq. cm. The area of ∆ AGB is-

(A)  48 cm2

(B)  24 cm2

(C)  54 cm2

(D)  36 cm2

Answer: (D)

105. A train 150 metres long passes a tree in 12 seconds. It will pass a tunnel of 250 metres long in-

(A)  32 seconds

(B)  26 seconds

(C)  20 seconds

(D)  25 seconds

Answer: (A)

106. ABCD is a rectangle with AB = h2 and AD = 3p. If h is doubled and p is halved, then the-

(A)  area is halved

(B)  area remains the same

(C)  area is multiplied by 4

(D)  area is doubled

Answer: (D)

107. A refrigerator listed at Rs 4000. Due to the festival season a shopkeeper announces a discount of 5%. Then the selling price of refrigerator (in Rs) is-

(A)  3600

(B)  3500

(C)  3800

(D)  3900

Answer: (C)

108. The circumference of a circle is equal to the perimeter of a square of side 22 cm. The area of the circle is-

(A)  28π cm2

(B)  196π cm2

(C)  49π cm2

(D)  

Answer: (B)

109. When a = 4/3 the value of 27a3 – 108a2 + 144a – 317 is-

(A)  −245

(B)  0

(C)  261

(D)  −253

Answer: (D)

110. If A, B and C denote respectively the number of vertices, edges and faces of a cube, then A + B + C is-

(A)  24

(B)  26

(C)  20

(D)  22

Answer: (B)

111. If a = 1, b = 2 and c = −3, then the value of  is-

(A)  0

(B)  1

(C)  3

(D)  2

Answer: (A)

112. D and E be two points on the sides  of the ∆ABC, such that  and  Find are  

(A)  4 : 9

(B)  21 : 25

(C)  4 : 25

(D)  2 : 5

Answer: (B)

113. The number of bricks required for a wall which is 8 m long, 6 m high and 22.5 cm thick, if each brick measures 25 cm ×25 cm × 6 cm, is-

(A)  7000

(B)  6400

(C)  6200

(D)  6550

Answer: (B)

114. A shopkeeper offered a discount of 9% for an article, but he marked it at 25% higher than the cost price. Find his profit percentage-

(A)  13.75%

(B)  16%

(C)  12.50%

(D)  13%

Answer: (A)

115. If 1/3 of A = 75% of B = 0.6 of C, then A : B : C is-

(A)  9 : 5 : 4

(B)  9 : 4 : 5

(C)  4 : 5 : 9

(D)  5 : 9 : 4

Answer: (B)

116. Three pipes A, B and C can fill a cistern in 10, 12 and 15 hours respectively, while working alonge. If all the three pipes are opened together, then find the time taken to fill the ccistern-

(A)  5 hours

(B)  6 hours

(C)  3 hours

(D)  4 hours

Answer: (D)

117. A paper is in the form of a rectangle ABCD where AB = 22 cm and BC = 14 cm. A semi-circular portion with BC as diameter is removed. Find the remaining area of the paper, (in cm2)

(A)  241

(B)  211

(C)  221

(D)  231

Answer: (D)

118. A bus can complete a journey in 6 hours if it travels at 60 km/hr. At what speed (km/hr) the bus must travel in order to complete the journey in 9 hours?

(A)  30

(B)  35

(C)  60

(D)  40

Answer: (D)

119. A chord of length 16 cm is at a distance of 6 cm from the centre of a circle. The radius of the circle is-

(A)  16 cm

(B)  22 cm

(C)  8 cm

(D)  10 cm

Answer: (D)

120. If 7n + 9 > 100 and n is an integer, the smallest possible value of n is-

(A)  14

(B)  15

(C)  13

(D)  12

Answer: (A)

121. Two men and 3 boys can do a piece of work in 10 days. While 3 men and 2 boys can do the same work in 8 days. In how many days can 2 men and 1 boy do the work?

(A)  11.5 days

(B)  12.5 days

(C)  11 days

(D)  10.5 days

Answer: (B)

122. The average age of 100 workers in a factory is 36.5. The average age of the men is 45 and that of the women is 28. The no. of women working in the factory is-

(A)  40

(B)  60

(C)  50

(D)  45

Answer: (C)

123. If p = 3/5, q = 7/9, r = 5/7, then which of the following inequality is true?

(A)  p < r < q

(B)  r < q < p

(C)  p < q < r

(D)  q < r < p

Answer: (A)

124. Three-fifth of two-third of three-seventh of number is 150. What is 60% of that

(A)  750

(B)  525

(C)  52.5

(D)  875

Answer: (B)

125. The ratio of father’s age to his son’s age is 7 : 3. The product of their age is 756. The ratio of their ages after 6 years will be-

(A)  11 : 7

(B)  13 : 9

(C)  2 : 1

(D)  5 : 2

Answer: (C)

126. In how many years shall Rs 2500, invested at the rate of 8% simple interest per annum, amount to Rs 3300?

(A)  6

(B)    

(C)  5

(D)  4

Answer: (D)

127. If  is the mean of n observations x1, x2, ……, xn, then the mean of   is-

(A)    

(B)    

(C)   

(D)   

Answer: (A)

128. If sec θ + cosec (90° − θ) = 4, (0 < θ < 90°), then the value of tan θ is –

(A)  1/√3

(B)  1

(C)  √3

(D)  1/√2

Answer: (C)

129. Ram sold a cow for Rs 136 and thus lost 15%. At what price he should have sold it to gain 15%.

(A)  Rs 180

(B)  Rs 184

(C)  Rs 204

(D)  Rs 150

Answer: (B)

130. Given that the mean of five numbers is 27; if one of them is excluded, the mean gets reduced by 2. Determine the excluded number-

(A)  25

(B)  35

(C)  45

(D)  55

Answer: (B)

131. A spherical balloon whose radius is r, subtends an angle α at the eye of an observer on the ground, when the angle of elevation of its centre is β. The height of its centre is-

(A)   

(B)  r sin α sin β

(C)  r cosec α sin β

(D)   

Answer: (D)

132. A mixture of milk and water is such that the quantity of milk is 3/5 that of water. The proportion of milk in the mixture is-

(A)  3/8

(B)  5/8

(C)  1/8

(D)  1/2

Answer: (A)

133. A horse is tied with a rope of length 7 m at one corner of a square field having side equal to 10 m. The minimum possible area of the square field that is left ungrazed is- [given π = 22/7]

(A)  56.1 m2

(B)  51.6 m2

(C)  65.1 m2

(D)  61.5 m2

Answer: (D)

134. If p = 99, then value of p(p2 + 3p + 3) is-

(A)  100089

(B)  999999

(C)  10000

(D)  99999

Answer: (B)

135. The compound interest on Rs 1800 at 10% per annum, for a certain period of time is Rs 378. Find the time in years-

(A)  2 years

(B)  3 years

(C)  1.5 years

(D)  2.5 years

Answer: (A)

136. A sells an article to B at a profit of 20%. B sells it to C at a profit of 10%. How much per cent will C pay more than what A pays?

(A)  32%

(B)  35%

(C)  28%

(D)  30%

Answer: (A)

137. A trader allows 10% discount on market price and gains 25%. If the market price of the article is Rs 50, what is its cost price?

(A)  Rs 25

(B)  Rs 55

(C)  Rs 36

(D)  Rs 45

Answer: (C)

138. A certain company has 80 engineers. If the engineers constitute 40% of its workers, then the number of people employed in the company is-

(A)  200

(B)  3200

(C)  150

(D)  800

Answer: (A)

139. A sofa-set is marked at Rs 20,000. The shopkeeper allows successive discount of 10%, 5% and 2% on it. What is the net selling price?

(A)  Rs 17598

(B)  Rs 16758

(C)  Rs 17768

(D)  Rs 16648

Answer: (B)

140. A train running at 36 km/hr crosses a pole in 25 seconds. Length of the train is-

(A)  225 metre

(B)  275 metre

(C)  250 metre

(D)  300 metre

Answer: (C)

Directions-The following pie-charts shows the contents of insects and rodents in an average Indian Household. Examine the chart and answer the questions number 141 to 145-

141. If the percentage of rat is y% of the total percentage of ant and cockroach; then ye is equal to-

(A)  25

(B)  20

(C)  30

(D)  35

Answer: (B)

142. If the difference of the percentage of rat and miscellaneous be z% of the percentage of cockroach, then z is equal to-

(A)  20%

(B)  15%

(C)  25%

(D)  35%

Answer: (C)

143. The total percentage of spider, rat and cockroach is greater than the percentage of ant by-

(A)  28

(B)  24

(C)  34

(D)  32

Answer: (C)

144. The total percentage of common lizard, spider and cockroach is greater than the percentage of rat by-

(A)  35

(B)  25

(C)  37

(D)  47

Answer: (D)

145. If the percentage of spider is x% of the percentage of cockroach, then x is equal to-

(A)    

(B)    

(C)   

(D)   

Answer: (A)

Directions- Study the following bar diagram carefully and answer question numbers 146 to 150.

146. The ratio between the number of students passed from University ‘A’ in the year 2007 and that from University ‘B’ in 2004 is-

(A)  5 : 6

(B)  3 : 5

(C)  5 : 4

(D)  5 : 3

Answer: (D)

147. Number of students passed from University ‘B’ in the year 2008 expressed as a percentage of the total number of students passed from University ‘A’ over the 6 years is-

(A)  25

(B)  35

(C)  30

(D)  20

Answer: (A)

148. The difference between the total number of students passed from both the Universities in the year 2008 combined together and the total number of students passed in the year 2004 from both the Universities taken together is-

(A)  20000

(B)  30000

(C)  35000

(D)  25000

Answer: (B)

149. The sum of the students passed from University ‘B’ in  years 2003, 2005 and 2006 combined together is-

(A)  50000

(B)  75000

(C)  80000

(D)  60000

Answer: (C)

150. The respective ratio between the number of students passed in year 2007, 2008 and 2004 from University ‘B’ is-

(A)  3 : 5 : 6

(B)  6 : 3 : 3

(C)  6 : 5 : 3

(D)  6 : 3 : 5

Answer: (C)

For Visually Handicapped Candidates Only

141. The side BC of an equilateral triangle ABC is produced upto D in such a way that BC = CD. Then the value of ∠BAD is-

(A)  90°

(B)  145°

(C)  120°

(D)  135°

Answer: (A)

142. The number of voters in a town is 1,20,000. Out of them, 75% go for poll for an election between A and B. If B gets 45% of the votes polled, votes polled for A is-

(A)  49,000

(B)  47,900

(C)  49,500

(D)  47,000

Answer: (C)

143. If  is-

(A)   

(B)   

(C)  0

(D)    

Answer: (C)

144. There is a loss of 4% if an article is sold at Rs x and gain of 12% if it is sold at Rs y. Then x : y is –

(A)  6 : 7

(B)  8 : 9

(C)  2 : 3

(D)  4 : 5

Answer: (A)

145. The sides of two squares are in the ratio 3 : 4. If their combined area is 12.25 sq. m, the side of larger square in metres is-

(A)  3.5

(B)  2.0

(C)  2.8

(D)  0.7

Answer: (C)

146. A train ‘B’ with a speed of 120 km/h another train ‘C’, running in same direction in 3 minutes. If the lengths of the train B and C be 200 m and 250 m respectively, the speed (in km/h) of the train C is-

(A)  127

(B)  135

(C)  111

(D)  123

Answer: (C)

147. The value of  is-

(A)  2

(B)  1

(C)  4

(D)  3

Answer: (B)

148. If x = ay, y = bx, then the value of  is-

(A)  −1

(B)  2

(C)  0

(D)  1

Answer: (D)

149. A can do a piece of work in 15 days and B alone can do it in 25 days. B works at it for 10 days and then leaves. A alone can finish the remaining work in-

(A)  9 days

(B)  12 days

(C)  6 days

(D)   

Answer: (*)

150. If  then the value of  is-

(A)  2

(B)  3

(C)  0

(D)  1

Answer: (A)

Part-IV

General Awareness

151. With which fort is Madras associated?

(A)  Fort St. George

(B)  Fort William

(C)  Fort St. David

(D)  Fort St. John

Answer: (A)

152. The longest National Highway that connects Varanasi and Kanya Kumari is-

(A)  NH-7

(B)  NH-10

(C)  NH-17

(D)  NH-14

Answer: (A)

153. The Emergency powers of the Indian president are borrowed from the constitution of-

(A)  Irish

(B)  Weimar Republic Germany

(C)  Australia

(D)  Canada

Answer: (B)

154. Identify the spreadsheet application-

(A)  MS-Access

(B)  MS-Power point

(C)  PL/SQL

(D)  MS-Excel

Answer: (D)

155. Which among the following is no more a fundamental right in India?

(A)  Right to Religion

(B)  Right to Property

(C)  Right to Equality

(D)  Right to Freedom

Answer: (B)

156. The term Khalisa in Mughal administration signified the-

(A)  Religious grants

(B)  Land given in lieu of salary

(C)  The land owned by emperor himself

(D)  Entire imperial establishment

Answer: (C)

157. Water gas is a mixture of-

(A)  Oxygen and carbon dioxide

(B)  Carbondixoide and water vapour

(C)  Carbon monoxide and hydrogen

(D)  Carbon monoxide and nitrogen

Answer: (C)

158. Jute fibres are-

(A)  Fibres of seed

(B)  Bast fibres from secondary phloem

(C)  Bast fibres from stem pericycle

(D)  Fibrous mesocarp

Answer: (C)

159. Microbial utilization of dissolved organics can be accomplished by-

(A)  Hydration

(B)  Carbonation

(C)  Oxidation

(D)  Reduction

Answer: (A)

160. The common plant disease caused by fungus is-

(A)  red not disease of sugarcane

(B)  tobacco mosaic disease

(C)  brown rust of wheat

(D)  citrus cauker

Answer: (C)

161. When a body is disturbed from its position of neutral equilibrium, what happens to its potential energy?

(A)  It decreases

(B)  It increases

(C)  It becomes zero

(D)  It remains constant

Answer: (D)

162. Which is the capital of Myanmar?

(A)  Moulmein

(B)  Naypyidaw

(C)  Yangona

(D)  Mandalaya

Answer: (C)

163. The headquarters of the Central Railway is at-

(A)  Mumbai

(B)  Jaipur

(C)  Nagpur

(D)  Vijayawada

Answer: (A)

164. Which of the following soils is suitable for cotton production?

(A)  Kankar

(B)  Regur

(C)  Bhangar

(D)  Khadar

Answer: (B)

165. Who is the Chief Minister of Maharashtra?

(A)  Sushil Kumar Shinde

(B)  Ashok Chavan

(C)  Shivaraj Singh Chouhan

(D)  Prithviraj Chavan

Answer: (D)

166. The most potent auxin for callus formations is-

(A)  NAA

(B)  2, 4-D

(C)  IAA

(D)  IBA

Answer: (C)

167. Who among the following is not a recipient of Rajiv Gandhi Khel Ratna for shooting?

(A)  Anjali Ved Pathak Bhagwat

(B)  Gagan Narang

(C)  P. Gopichand

(D)  Abhinav Bindra

Answer: (C)

168. The area-wise largest fresh water lake in Africa is-

(A)  Lake Tanganyika

(B)  Lake Malawi

(C)  Lake Victoria

(D)  Lake Rudolf

Answer: (C)

169. Accuracy of vision of is the function of-

(A)  Amacrine cells

(B)  Granuler cells

(C)  Rod cells

(D)  Cone cells

Answer: (D)

170. The presence of starch in a mixture can be detected by-

(A)  Bromine

(B)  Iodine

(C)  Fluorine

(D)  Chlorine

Answer: (B)

171. Car and Petrol are-

(A)  Necessaries

(B)  Luxuries

(C)  Complementaries

(D)  Substitutes

Answer: (C)

172. Who is authorized to impose tax on Interstate trade?

(A)  President

(B)  Finance Minister

(C)  Parliament

(D)  State Legislature

Answer: (D)

173. Where is the International Rice Research Institute?

(A)  Manila

(B)  Dhaka

(C)  Cuttack

(D)  Bangkok

Answer: (A)

174. Which teams of the first Indian Volley League (IVL) secured the I and II Rank Positions?

(A)  Yanam Tigers-I, Chennai Spikers-II

(B)  Chennai Spikers-I, Hyderabad Chargers-II

(C)  Chennai Spikers-I, Kerala Killer-II

(D)  Hyderbad Chargers-I Chennai Spikers-II

Answer: (B)

175. Of the following, which one corresponds to the fixed cost?

(A)  Transportation charges

(B)  Insurance Premium on Property

(C)  Payments for raw materials

(D)  Labour cost

Answer: (B)

176. The aim of Green and cleaner production is-

(A)  Biopesticides

(B)  Waste minimization

(C)  Agricultural productivity

(D)  Afforestation

Answer: (C)

177. Which is the competent authority to create New All-India services?

(A)  Lok Sabha

(B)  Union Public Service Commission

(C)  Parliament

(D)  Rajya Sabha

Answer: (D)

178. Which of the following Article of the Indian constitution prescribes constitution of Panchayats in villages?

(A)  Article-41

(B)  Article-38

(C)  Article-40

(D)  Article-39

Answer: (C)

179. Light appears to travel in straight line since-

(A)  Its wavelength is very small

(B)  Its velocity is very large

(C)  It is not absorbed by the atmosphere

(D)  It is reflected by the atmosphere

Answer: (A)

180. Who was the commander of the Maratha artillery in the third battle of Panipat?

(A)  Malhar Rao Holkar

(B)  Ibrahim Khan Gardi

(C)  Viswasa Rao

(D)  Sadasiva Rao

Answer: (D)

181. Which gas is responsible for Ozone hole?

(A)  Carbon dioxide

(B)  Nitrous oxide

(C)  Chloro fluro carbon

(D)  Carbon monoxide

Answer: (C)

182. Which one of the following place became the bone of contention between Vijayanagar Empire and Bahmani Kingdom?

(A)  The fort of Mudgal

(B)  Goa

(C)  Raichur Doab

(D)  Telengana

Answer: (C)

183. The claim for higher wages would be helped by-

(A)  increases in unemployment

(B)  increases in labour productivity

(C)  decrease in consumers demand

(D)  decrease in company’s profit

Answer: (B)

184. Which was the first state in India to have a woman as Chief Minister?

(A)  Odisha

(B)  Tamil Nadu

(C)  Punjab

(D)  Uttar Pradesh

Answer: (D)

185. The speed of a ship is measured in-

(A)  Horse power

(B)  Angstrom

(C)  Joule

(D)  Knot

Answer: (D)

186. Golgi bodies originated from-

(A)  Mitochondria

(B)  Zymogen granules

(C)  Cell wall

(D)  Endoplasmic reticulum

Answer: (C)

187. Atmospheric pressure generated on the earth surface is due to-

(A)  Revolution of earth

(B)  Gravitational pull of the earth

(C)  Rotation of earth

(D)  Heating by solar radiation

Answer: (B)

188. Western ghats are-

(A)  Plateau

(B)  Hills

(C)  Mountain

(D)  Escarpment of Plateau

Answer: (D)

189. Which one of the following is not included as a world heritage site?

(A)  Kalka-Shimla Railway

(B)  Nilgiri Mountain Railway

(C)  Chennai Central Station

(D)  Chhatrapati Shivaji Terminus Station in Mumbai

Answer: (C)

190. Cortisone is a-

(A)  Enzyme

(B)  Hormone

(C)  Protein

(D)  Lipid

Answer: (B)

191. Most of the fuels are carbon compounds with-

(A)  phosphorus

(B)  nitrogen

(C)  hydrogen

(D)  sulphur

Answer: (C)

192. A foreign compound, not normally considered as a constituent of a specified biological system is known as-

(A)  Xenobiotics

(B)  Additive

(C)  Residue

(D)  Substrata

Answer: (A)

193. The natural disease includes two phases as-

(A)  Prepathogenesis and pathogenesis

(B)  Pathogenesis and post pathogenesis

(C)  Contamination and contact

(D)  Incubation and infection

Answer: (A)

194. Meeting point of aggregate demand and aggregate supply is called as-

(A)  equilibrium demand

(B)  effective demand

(C)  market equilibrium

(D)  market demand

Answer: (A)

195. A ………. is pictorial representation of an algorithm.

(A)  flow-chart

(B)  structure chart

(C)  pseudo code

(D)  data flow diagram

Answer: (A)

196. Which of the following recommended on centre state relations?

(A)  Shri Krishna Commission

(B)  Sarkaria Commission

(C)  Sachar Commission

(D)  Kothari Commission

Answer: (B)

197. Which one of the following can perform both translation and execution?

(A)  Interpreter

(B)  Loader

(C)  Assembler

(D)  Compiler

Answer: (A)

198. Where was the first Buddhist council held?

(A)  Patliputra

(B)  Kundalvana

(C)  Rajagriha

(D)  Vaishali

Answer: (C)

199. When did Suu Kyi receive her Nobel Prize for peace awarded in 1991?

(A)  2011

(B)  2012

(C)  1992

(D)  2005

Answer: (C)

200. Where is the Headquarters of WTO?

(A)  Vienna

(B)  Paris

(C)  Berne

(D)  Geneva

Answer: (D)

SSC Combined Higher Secondary Level (10+2) IInd Shift Tier-I Examination Held on 20 October, 2013 Question Paper With Answer Key

SSC Combined Higher Secondary Level (10+2) IInd Shift Tier-I Examination Held on 20 October, 2013
SSC Combined Higher Secondary Level (10+2) IInd Shift Tier-I Examination Held on 20 October, 2013 Question Paper With Answer Key

Staff Selection Commission Combined Higher Secondary Level (10+2) IInd Shift

Tier-I Examination Held on 20 October, 2013

Part-I

General Intelligence

Directions-(Q. 1-7) Select the related word/letters/number from the given alternatives-

1. Squint : Vision : : Stammering : ?

(A)  Mouth

(B)  Hearing

(C)  Speech

(D)  Tongue

Answer: (C)

2. Racket : Tennis :: ?

(A)  Ball : Football

(B)  Glove : Cricket

(C)  Board : Chess

(D)  Bat : Cricket

Answer: (D)

3. Hand : Glove : : Head : ?

(A)  Hair

(B)  Hat

(C)  Neck

(D)  Hair-pin

Answer: (B)

4. TCEJBO : OBJECT : : TCEJORP : ?

(A)  ROPJECT

(B)  JCCTROP

(C)  PROJECT

(D)  ROJOECT

Answer: (C)

5. FLRX : DJPV : : EKQW : ?

(A)  BHMT

(B)  CIOU

(C)  AGMS

(D)  WQKE

Answer: (B)

6. 4 : 17 : : 7 : ?

(A)  48

(B)  49

(C)  50

(D)  51

Answer: (C)

7. 45 : 9 : : 94 : ?

(A)  17

(B)  14

(C)  13

(D)  10

Answer: (C)

Direction-(Q. 8-11) Find the odd words/letters/number pair from the given alternatives.

8.

(A)  Prince : Princess

(B)  Vamp : Vampire

(C)  Hero : Heroine

(D)  King : Queen

Answer: (B)

9.

(A)  Carrot

(B)  Potato

(C)  Beetroot

(D)  Radish

Answer: (B)

10.

(A)  UROL

(B)  QNKH

(C)  YVSP

(D)  USPM

Answer: (D)

11.

(A)  106 – 36

(B)  76 – 46

(C)  86 – 56

(D)  120 – 90

Answer: (A)

12. Find out the set among the four sets which is like the given set-

(11 – 18 – 25)

(A)  (4 – 12 – 16)

(B)  (19 – 26 – 33)

(C)  (18 – 28 – 33)

(D)  (9 – 14 – 24)

Answer: (B)

13. Which one of the given responses would be a meaningful order of the following?

(1) Ploughing     (2) Weeding

(3) Sowing         (4) Harvesting

(A)  1, 2, 3, 4

(B)  1, 3, 2, 4

(C)  2, 3, 1, 4

(D)  1, 2, 4, 3

Answer: (B)

14. Arrange the following words according to t he English Dictionary-

(1) Voyage         (2) Voice

(3) Vocation       (4) Volume

(A)  3, 2, 4, 1

(B)  2, 3, 4, 1

(C)  3, 4, 2, 1

(D)  2, 4, 3, 1

Answer: (A)

15. Which one set of letters when sequentially placed at the gaps in the given letter series shall complete it?

A_b a b_a b_a b b

(A)  b a a

(B)  a a a

(C)  b b b

(D)  b b a

Answer: (C)

Directions-(Q. 16-18) A series is given, with one term missing. Choose the correct alternative from the given ones that will complete the series.

16. 4, 7, 13, 22, 34, ?

(A)  47

(B)  52

(C)  49

(D)  45

Answer: (C)

17. FLA, GMB, HNC, IOD, ?

(A)  NSD

(B)  JPE

(C)  TOE

(D)  OTK

Answer: (B)

18. 135, 226, 353, ?, 739

(A)  522

(B)  622

(C)  632

(D)  722

Answer: (A)

19. If day before yesterday was Sunday, what day will fall on day after tomorrow?

(A)  Wednesday

(B)  Thursday

(C)  Friday

(D)  Tuesday

Answer: (B)

20. Prakash jumps higher than Pinakin, but not as much as Ketan jumps. Rohan jumps lower than Pranav but not less than Ketan. Who jumps lowest?

(A)  Pinakin

(B)  Ketan

(C)  Rohan

(D)  Pranav

Answer: (A)

21. Leela and her grandmother differ in their ages by 60 years. After 10 years, if the sum of their ages is 100 years, what are their present ages?

(A)  20, 80

(B)  10, 70

(C)  15, 75

(D)  25, 85

Answer: (B)

22. Sarika is 19th from either end of a row of girls. How many girls are there in that row?

(A)  38

(B)  39

(C)  37

(D)  19

Answer: (C)

23. Mohan is husband of Sona. Sona is aunt of Shilpa. Ankit is brother of Sona. How is Ankit related to Mohan?

(A)  Uncle

(B)  Brother

(C)  Cousin

(D)  Brother-in-law

Answer: (D)

24. If A = 1, AIR = 28, then RIB = ?

(A)  28

(B)  29

(C)  35

(D)  26

Answer: (B)

25. In the following series of English alphabets, how many ‘a b c’ occur in such a way that ‘b’ is in the middle and both ‘a’ and ‘c’ are on any one side-

q b c a j l c b I d a e c h k a m b c n o

a b c p k d c r b c h a q g a b c f e b c t u

(A)  6

(B)  4

(C)  3

(D)  2

Answer: (C)

26. If ‘PSYCHOLOGY’ is coded as ‘9834215173’; and ‘SOCIOLOGY’ is coded as ‘812015173’; then ‘PHILOSOPHY’ can be coded as-

(A)  9205182923

(B)  9206181923

(C)  9205181923

(D)  9105282913

Answer: (C)

27. If ‘ETHICS’ can be written as ‘SECTIH’; then how can ‘AFECTION’ be written-

(A)  NAOFETIC

(B)  NAOFICTE

(C)  NAOFIETC

(D)  NAOFITCE

Answer: (C)

28. Some letters are given below in the first line and numbers are given below them in the second time. Numbers are the codes for the alphabets and vice-vera. Choose the correct number-code for the given set of alphabets-

A  F  E  H  I  K  O  M  U  R

9   3   8   4  0  5   1   6   2    7

               M A E I O U

(A)  6 9 8 1 0 2

(B)  6 9 8 0 1 2

(C)  6 9 8 2 1 0

(D)  6 9 8 2 0 1

Answer: (B)

29. From the given alternatives select the word which can be formed using the letters given in the word-

ONSLAUGHT

(A)  HOUSE

(B)  GHOST

(C)  ENOUGH

(D)  STRONG

Answer: (B)

30. From the given alternatives select the word which cannot be formed using the letters of the given word-

TRANSLATION

(A)  TRANSIT

(B)  TRANSMIT

(C)  NATION

(D)  RATION

Answer: (B)

Directions-(Q. 31 an 32) Select the missing number from the given responses.

31.

7      56    8

9      63    7

12    ?      5

(A)  17

(B)  25

(C)  60

(D)  119

Answer: (C)

32. 

4      5      8

3      ?      2

4      10    4

48    200  64

(A)  185

(B)  40

(C)  20

(D)  4

Answer: (D)

Directions-(Q. 33 and 34) Some equations are solved on the basis of a certain system. On the same basis, find out the correct answer for the unsolved equation.

33. 2 × 7 × 5 = 527, 1 × 4 × 9 = 914, 6 × 3 × 8 = ?

(A)  683

(B)  863

(C)  638

(D)  836

Answer: (B)

34. If 624 = 9

and 3732 = 20, then

4523 = ?

(A)  26

(B)  16

(C)  23

(D)  21

Answer: (D)

35. Select the correct combination of mathematical signs to replace ‘*’ signs and to balance the given equation-

6 * 5 * 8 * 4 * 32

(A)  + + + =

(B)  ÷ + × +

(C)  × + ÷ =

(D)  − + × ×

Answer: (C)

36. If ‘+’ means division, ‘−’ means multiplication, ‘×’ means subtraction and ‘÷’ means addition then,

(280 + 10 × 20) – 8 ÷ 6 = ?

(A)  58

(B)  70

(C)  112

(D)  392

Answer: (B)

37. Seema started early in the morning on the road towards the Sun. After some time she turned to her left. Again after some time she turned to her right. After moving come distance she again turned to her right and began to move. At this time, in what direction was she moving?

(A)  North-West

(B)  North-East

(C)  East

(D)  South

Answer: (D)

38. ‘A’ walks 10 m towards East and then 10 m to his right. Then every time turning to his left, he walks 5, 15 and 15 m, respectively. How far is he now from his starting point?

(A)  5 m

(B)  10 m

(C)  15 m

(D)  20 m

Answer: (A)

Directions-(Q. 39 and 40) Two statements are given, followed by two conclusions I and II. You have to consider the statements to be true, even if they seem to be at variance from commonly known facts. You have to decide which of the given conclusions, if any, follow from the given statements. Indicate your answer.

39. Statements : All trams are prams, No pram can carry passengers.

Conclusions: I. No tram can carry passengers.

(II) Some trams cannot carry passengers.

(A)  Only conclusions I follows

(B)  Only conclusion II follows

(C)  Both the conclusions I and II follows

(D)  Neither conclusion I nor II follows

Answer: (A)

40. Statements : This world is neither good nor bad. Each individual manufactures world for

Conclusions : I. Some people find this world quite good.

(II) Some people find this world quite bad.

(A)  Only conclusion I follows

(B)  Only conclusion II follows

(C)  Both conclusions I and II follow

(D)  Neither conclusion I nor II follows

Answer: (D)

41. Five girls E, F, G, H and I are standing in a row. H is on the right of G. I is on the left of G, but is on the right of F. H is on the left of E. Who is standing on the extreme right?

(A)  F

(B)  G

(C)  H

(D)  E

Answer: (D)

42. There are 120 students in a class. 25% of the students play volleyball. 40% of the remaining students play cricket. 50% of the remaining students play football. How many students in the class do not play any game?

(A)  29

(B)  30

(C)  27

(D)  32

Answer: (C)

43. Select the related figure from the given alternatives.

Answer: (C)

44. Find the odd figure from the given alternatives.

Answer: (B)

45. Choose the correct alternative from the given ones that will complete the series.

Answer: (D)

46. Find the missing number from the given responses-

(A)  7

(B)  192

(C)  193

(D)  98

Answer: (C)

47. Identify the diagram that best represents the relationship among the classes given below:

Potato, Vegetables and Eatables

Answer: (A)

48. Which answer figure will complete the pattern in the question figure?

Answer: (A)

49. A piece of paper is folded and cut/punched as shown below in the question figures. From the given answer figures, indicate how it will appear when opened.

Answer: (D)

50. From the given answer figures, select the one in which the question figure is hidden/embedded.

Answer: (A)

For Visually Handicapped Candidates Only

43. Find the wrong number in the given series from the given alternatives-

445, 221, 109, 46, 25

(A)  25

(B)  46

(C)  221

(D)  109

Answer: (C)

44. A sum of money is to be distributed among A, B, C, D in the proportion of 5 : 2 : 4 : 3. If C gets Rs 1,000 more than D, what is B’s share?

(A)  Rs 500

(B)  Rs 1,500

(C)  Rs 2,000

(D)  Rs 2,500

Answer: (C)

45. If P stand for +, Q for ×, B for ÷ G for −, then find the value of the following equation.

19 G 9 P 15 B 5 Q 2 = ?

(A)  16

(B)  20

(C)  32

(D)  40

Answer: (A)

46. In a peculiar mathematical operation, before any operation is started, the numbers are changed as under-

1 to 4 changed to 1

5 to 9 changed to 2

Zero remains zero.

Applying the above condition, find the answer of the following equation-

769 + 2085 + 6083 – 8306 = ?

(A)  1163

(B)  1009

(C)  4033

(D)  2825

Answer: (A)

47. Usha ranks 21st in a class of 37 students. What is the her rank from the last?

(A)  16

(B)  17

(C)  18

(D)  19

Answer: (B)

48. The sum of two numbers is 21 and their product is 90. Then sum of their squares will be?

(A)  441

(B)  728

(C)  261

(D)  860

Answer: (C)

49. In the following series of numbers, how many 0’s are followed by 9 and preceded by 5?

1 0 9 1 1 5 0 9 2 0 1 2 3 2 8 1 3 9 3

5 0 9 4 8 1 4 7 1 6 7 5 5 0 9 6 1 5

(A)  7

(B)  3

(C)  5

(D)  4

Answer: (B)

50. LOVE is coded as 1357, and HATE is coded as 2467. What do the figures 6417 stand for?

(A)  HEAT

(B)  LEAD

(C)  TALE

(D)  LATE

Answer: (C)

Part-II

English Language

Directions-(Q. 51 to 55) Some parts of the sentences have errors and some are correct. Find out which part of a sentence has an error and blacken the oval () corresponding  to the appropriate letter (A), (B), (C). If a sentence is free from error, blacken the oval corresponding to (D) in the Answer-Sheet.

51. I want to get (A)/ a M.A. degree (B)/ (C)/ from a reputed university located in India. No error (D)

Answer: (B)

52. Not only she makes (A)/ home-made cakes (B)/ she also sells them. (C)/ No error (D)

Answer: (A)

53. Pandit Hariprasad Chaurasia played (A)/ a flute at the Music Conference (B)/ in a small town. (C)/ No error (D)

Answer: (D)

54. They are (A)/ ignorant of (B)/ what is happening here. (C)/ No error (D)

Answer: (D)

55. My mother made (A)/ the servant to (B)/ complete the work. (C)/ No error (D)

Answer: (B)

Directions-(Q. 56 to 60) Sentences are given with blanks to be filled in with an appropriate word(s). Four alternatives are suggested for each question. Choose the correct alternative out of the four and indicate it by blackening the appropriate oval (●) in the Answer-Sheet.

56. The doctors and the nurses worked hard to …….. the dying man.

(A)  bring up

(B)  bring out

(C)  bring back

(D)  bring round

Answer: (C)

57. They tried to bribe the peon but he …….. them.

(A)  has been too clever for

(B)  was too clever for

(C)  was being clever for

(D)  was clever for

Answer: (B)

58. She asked me ……….. .

(A)  what was the time

(B)  what is the time

(C)  what was time

(D)  what the time was

Answer: (D)

59. A legislation was passed to punish brokers who …………. their client’s funds.

(A)  devastate

(B)  devour

(C)  embezzle

(D)  defalcate

Answer: (C)

60. He fell down a flight of stairs and broke ………. rib.

(A)  a

(B)  the

(C)  his

(D)  up

Answer: (C)

Directions-(Q. 61 and 62) Out of the four alternatives, choose the one which best expresses the meaning of the given word and mark it in the Answer-Sheet.

61. Stratagem

(A)  Stratum

(B)  Ruby

(C)  Trick

(D)  Strait

Answer: (C)

62. Forsake

(A)  Abandon

(B)  Forego

(C)  Disintegrate

(D)  Separate

Answer: (A)

Directions-(Q. 63 and 64) Choose the word opposite in meaning to the given word and mark it in the Answer-Sheet.

63. Dreary

(A)  Solitary

(B)  Driblle

(C)  Unusual

(D)  Interesting

Answer: (D)

64. Keen

(A)  Impassioned

(B)  Indifferent

(C)  Clever

(D)  Immobile

Answer: (B)

Directions-(Q. 65 and 66) Four words are given in each question, out of which only one word is correctly spelt. Find the correctly spelt word and mark your answer in the Answer-Sheet.

65.

(A)  privelege

(B)  privilege

(C)  priviledge

(D)  privalege

Answer: (A)

66.

(A)  hipopotamus

(B)  hippopotamus

(C)  hippopatemus

(D)  hippopotomous

Answer: (B)

Directions-(Q. 67 to 69) Four alternatives are given for the Idiom/Phrase underlined in the sentence. Choose the alternative which best expresses the meaning of the Idiom/Phrase and mark it in the Answer-Sheet.

67. We could sit here and talk till the cows come home.

(A)  for a very long time

(B)  for a very short span of time

(C)  if the cows come home quickly

(D)  if the cows come home slowly

Answer: (A)

68. They earn so little that it is very difficult for them to keep the wolf from the door.

(A)  have less money to avoid hunger and need

(B)  have enough money to avoid hunger and need

(C)  to  hide the wolf behind the door

(D)  to display the wolf before the door

Answer: (B)

69. Arvind finally got through with his doctoral thesis.

(A)  finally cleared his last examination

(B)  left the thesis half way through

(C)  finally finished his thesis after hard toil

(D)  was awarded his doctoral degree

Answer: (D)

Directions-(Q. 70 to 72) Out of the four alternatives choose the one which can be substituted for the given words/sentence.

70. A place of shelter for ships.

(A)  Helipad

(B)  Port

(C)  Harbour

(D)  Barrack

Answer: (C)

71. A shopkeeper who sells fresh and green vegetables.

(A)  Butcher

(B)  Shopkeeper

(C)  Vendor

(D)  Greengrocer

Answer: (D)

72. That which cannot be consumed by fire.

(A)  Inflammable

(B)  Inexhaustible

(C)  Incombustible

(D)  Invicible

Answer: (C)

Directions-(Q. 73 and 74) A part of the sentence is underlined. Below are given alternatives to the underlined part at (A), (B), (C) which may improve the sentence. Choose the correct alternative. In case no improvement is needed your answer is (D). Mark your answer in the Answer-Sheet.

73. Historians feel there is an earnest need for the review of history books every five years and a revision of the same every ten years.

(A)  indispensable

(B)  urgent

(C)  imperative

(D)  No improvement

Answer: (A)

74. When describing the accident, he was in tears.

(A)  In describing

(B)  When he was describing

(C)  As describing

(D)  No improvement

Answer: (B)

Directions-(Q. 75 to 79) A sentence has been given in Active/Passive Voice. Out of the four alternatives suggested, select the one which best expresses the same sentence in Passive/Active Voice and mark your answer in the Answer-Sheet.

75. Differential Calculus is being learnt by the girl.

(A)  The girl is learning Differential Calculus

(B)  The girl learns Differential Calculus

(C)  The girl was learning Differential Calculus

(D)  The girl has learnt Differential Calculus

Answer: (A)

76. He arranged the furniture in the house according to the decorator’s plans.

(A)  The furniture was being arranged in the house according to the decorator’s plans

(B)  The furniture had been arranged in the house according to the decorator’s plans

(C)  The furniture have been arranged in the house according to the decorator’s plans

(D)  The furniture was arranged in the house according to the decorator’s plans

Answer: (D)

77. To make the batter add one tablespoonful of baking powder to the cake mixture.

(A)  Let one tablespoonful of baking powder be added to the cake mixture to make the batter

(B)  Let only one tablespoonful of baking powder be added to the cake mixture to make the batter

(C)  One tablespoonful of baking powder may be added to the cake mixture to make the batter

(D)  One tablespoonful of baking powder could be added to the cake mixture to make the batter

Answer: (A)

78. By whom is John looked after?

(A)  Who looks after John?

(B)  John looks after whom?

(C)  Whom looks after John?

(D)  Who is looking after John?

Answer: (D)

79. The horse jumped over the fence.

(A)  The fence might be jumped over by the horse

(B)  The fence was jumped over by the horse

(C)  The fence by the horse was jumped over

(D)  The fence was jumped by the horse

Answer: (B)

Directions-(Q. 80 to 84) A sentence has been given in Direct/Indirect. Out of the four alternatives suggested, select the one which best expresses the same sentence in Indirect/Direct and mark your answer in the Answer-Sheet.

80. The mother urged the child to come on the footpath.

(A)  The mother said to the child. “Come on the footpath.”

(B)  The mother said, “Come, child come. Come on the footpath.”

(C)  The mother said, “Please come on the footpath, child.”

(D)  The mother said to the child, “You should come on the footpath.”

Answer: (A)

81. Sushma said, “We shall celebrate the festival of Janmashtami tomorrow.”

(A)  Sushma said that we should celebrate the festival of Janmashtami the next day

(B)  Sushma said that we shall celebrate the festival of Janmashtami the next day

(C)  Sushma said that they should celebrate the festival of Janmashtami the next day

(D)  Sushma said that they would celebrate the festival of Janmashtami the next day

Answer: (C)

82. “I’m so tiny that I can’t reach your branches,” said the little boy to the tree.

(A)  The little boy said that the tree was so tiny that he couldn’t reach its branches

(B)  The little boy said that he was so tiny that he couldn’t reach its branches

(C)  The little boy told to the tree that he was so tiny that he couldn’t reach its branches

(D)  The little boy told the tree that he was so tiny that he couldn’t reach its branches

Answer: (C)

83. I’m sorry I cannot stay,” said the swallow. “I’m waited for in Egypt.”

(A)  The swallow told that he is sorry he cannot stay and added that he is waited for in Egypt

(B)  The swallow said he was sorry he couldn’t stay and added that someone was waiting for him in Egypt

(C)  The swallow regretted he couldn’t stay as he was waited for in Egypt

(D)  The swallow regretted that he was waited for in Egypt and couldn’t stay

Answer: (C)

84. The notice reads, “Trespassers will be prosecuted.”

(A)  The notice read that trespassers will be prosecuted

(B)  The notice read that trespassers would be prosecuted

(C)  The notice reads that trespassers would be prosecuted

(D)  The notice reads that trespassers will be prosecuted

Answer: (D)

Directions-(Q. 85 to 90) The 1st and the last sentence of the passage/parts of the sentence are numbered 1 and 6. The rest of the passage/sentence is split into four parts and named P, Q, R and S. These four parts are not given in their proper order. Read the passage/sentence and find out which of the four combinations is correct. Then find the correct answer and indicate it by blackening the appropriate oval (●) in the Answer Sheet.

85. (1) Self help means doing our own work ourselves.

(P) They prefer to have things done for them.

(Q) The most successful man is he who depends the least on others.

(R) There are men in our society who don’t exert themselves.

(S) Such people are rarely successful.

(6) He has a strength of mind and determination to succeed on his own.

(A)  RPSQ

(B)  PQRS

(C)  QPSR

(D)  QSRP

Answer: (A)

86. (1) At the time when the city of Venice in Italy was rich and famous, one of her chief citizens was a merchant named Antonio.

(P) He had many friends, among them a young Venetian nobleman called Bassanio.

(Q) But Antonio loved him and gave him money whenever he needed it.

(R) Although he came from a noble family, Bassanio was poor and spent more money than he could afford.

(S) He was well known for his kindness.

(6) For the merchant of Venice was a rich man, who owned many ships which traded with foreign countries.

(A)  PQRS

(B)  SQRP

(C)  SPRQ

(D)  PRSQ

Answer: (C)

87. (1) Drug addiction has now become one of the major problems of our society.

(P) There should be joint effort of family, government and social organizations for prevention and cure this curse.

(Q) The possibilities of these young men and women are nipped in the bud.

(R) In such an alarming situation, we cannot remain idle onlookers.

(S) Many youngsters are becoming victims of this deadly curse.

(6) We should take the pledge-“Let us unite for a drug-free world”.

(A)  SQRP

(B)  PQRS

(C)  PSQR

(D)  RQPS

Answer: (A)

88. (1) ‘Tsunami’ which means harbour wave’

(P) usually an ocean

(Q) of a large volume of a body of water

(R) is a series of water waves

(S) caused by the displacement

(6) though it can occur in large lakes.

(A)  RQPS

(B)  RSQP

(C)  SRQP

(D)  PSQR

Answer: (B)

89. (1) Dante was of moderate height

(P) clad always in such sober dress

(Q) with a slow and gentle pace

(R) was accustomed to walk somewhat bowed

(S) and after maturity

(6) as befitted his ripe years.

(A)  SRQP

(B)  RQSP

(C)  PSRQ

(D)  QSRP

Answer: (B)

90. (1) Queen Elizabeth had the mind

(P) and nurtured her country

(Q) through careful leadership

(R) of a political genius

(S) and by choosing capable men

(6) to assist her.

(A)  RQSP

(B)  RPQS

(C)  PSRQ

(D)  SPRQ

Answer: (B)

   Directions-(Q. 91 to 95) In the following passage some of the words have been left out. Read the passage carefully and choose the correct answer to each question out of the four alternatives and fill in the blanks.

   The sun had set but there was still some light in the sky. Martin …91… on his elbow and looked …92… through the leaves. In the waters of the lake, close to the shore, he saw a …93… of alligators floating quietly. One of the creatures, …94… huge one, was lying on a high …95… of sand, a few  yards from the water.

91.

(A)  crossed

(B)  leaned

(C)  lay

(D)  sat

Answer: (B)

92.

(A)  down

(B)  at

(C)  for

(D)  into

Answer: (D)

93.

(A)  family

(B)  volume

(C)  number

(D)  semblance

Answer: (C)

94.

(A)  a

(B)  one

(C)  single

(D)  an

Answer: (A)

95.

(A)  peak

(B)  pit

(C)  pile

(D)  bank

Answer: (C)

   Directions-(Q. 96 to 100) You have a brief passage with 5 questions following the passage. Read the passage carefully and choose the best answer to each question out of the four alternatives and mark it by blackening the appropriate oval () in the Answer-Sheet.

   I could not help thinking, as I looked at the works of Shakespeare on the shelf, that it would have been impossible, completely and entirely, for any woman to have written the plays of Shakespeare in the age of Shakespeare. Let me imagine, since facts are so hard to come by, what would have happened had Shakespeare had a wonderfully gifted sister, called Judith, let us say. Shakespeare himself went, very probably-his mother was an heiress-to the grammar school, where he may have learnt Latin – Ovid, Virgil and Horace – and the elements of grammar and logic. He was, it is well known, a wild boy who poached rabbits, perhaps shot a deer, and had, rather sooner than he should have done, to marry a woman in the neighbourhood, who bore him a child rather quicker than was right. That escapade sent him to seek his fortune in London. He had, it seemed, a taste for the theatre; he began by holding horses at the stage door.

96. Which of the following statements is true?

(A)  Shakespeare’s mother was poor

(B)  Shakespeare learnt Roman

(C)  Shakespeare did not have a sister

(D)  Judith was Shakespeare’s wife

Answer: (B)

97. ‘Come by’ means-

(A)  maintain

(B)  manage to get

(C)  invent

(D)  search

Answer: (B)

98. The passage is about-

(A)  Shakespeare’s sister

(B)  Shakespeare’s theatre

(C)  Shakespeare’s writing

(D)  Shakespeare’s life

Answer: (D)

99. Shakespeare’s marriage reveals that he was-

(A)  impulsive

(B)  talented

(C)  wise

(D)  stealthy

Answer: (D)

100. Shakespeare’s sister was-

(A)  wise

(B)  imaginary

(C)  famous

(D)  ordinary

Answer: (B)

Part-III

Quantitative Aptitude

101. The angles of depression of the top and the bottom of a building of height h units, from the top of a monument of height H units are complementary. If the distance between the building and the monument is ‘a’ units, then it is always rule that-

(A)   

(B)    

(C)   

(D)    

Answer: (C)

102. If  then the value of θ in circular measure will be-

(A)  π/4

(B)  π/12

(C)  π/3

(D)  π/6

Answer: (C)

103. If  then the value of tan 47° is-

(A)   

(B)  y/x

(C)  x/y

(D)   

Answer: (C)

104. The value of  is equal to-

(A)  1

(B)  1/4

(C)  1/2

(D)  2

Answer: (A)

105. If sec θ – cos θ = 3/2 (θ is a positive acute angle), then sec θ is equal to-

(A)  −1/2

(B)  0

(C)  1/2

(D)  2

Answer: (D)

106. The base of a pyramid of volume 48√3 cc is an equilateral triangle. If the height of the pyramid is 4 cm, then each side of the equilateral triangle at the base is-

(A)  3 cm

(B)  4 cm

(C)  6 cm

(D)  12 cm

Answer: (D)

107. In a triangle ABC, AB = 3 cm, BC = 4 cm and ∠ABC = 90°. The triangle is first rotated around the side AB and then around the side BC. The volume of the first cone, thus formed, is x% more than second cone. Then x is equal to-

(A)  25

(B)    

(C)  20

(D)   

Answer: (B)

108. The base of a right-angled triangle is 5 units and hypotenuse is 13 units. Then the area is-

(A)  15 sq. units

(B)  30 sq. units

(C)  12 sq. units

(D)  60 sq. units

Answer: (B)

109. A wall 9 m long, 6 m high and 20 cm thick is to be constructed using bricks of dimension 30 cm × 15 cm × 10 cm. How many bricks will be required?

(A)  2800

(B)  3600

(C)  3200

(D)  2400

Answer: (D)

110. The diameter of a sphere is 6 cm. It is melted and drawn into a wire of diameter 0.1 cm. Find the length of the wire-

(A)  24 cm

(B)  28 cm

(C)  36 cm

(D)  42 cm

Answer: (*)

111. The greatest number of solid cubes with 0.2 metre edges that can be packed in a box whose inside dimensions are 1 m by 1 m by 2 m are-

(A)  10

(B)  25

(C)  50

(D)  250

Answer: (D)

112. The length of a wire is 66 m. Then the number of circles of circumference 1.32 cm that can be made from this wire is-

(A)  50

(B)  100

(C)  1000

(D)  5000

Answer: (D)

113. Suppose O is the incentre of ∆ABC and the circle touches the sides BC, CA and AB at the points P, Q, R respectively. If ∠A = 60°, then the magnitude of ∠QPR is-

(A)  60°

(B)  90°

(C)  120°

(D)  100°

Answer: (A)

114. In an isosceles triangle ABC, AB = AC; a circle drawn with AB as diameter intersects BC at D. If BD = 3 cm, then the length of CD is-

(A)  1.5 cm

(B)  5 cm

(C)  3 cm

(D)  6 cm

Answer: (C)

115. A vertical stick, 12 cm long, casts a shadow 8 cm long on the ground. At the same time a tower casts a shadow 40 cm long on the ground. Determine the height of the tower-

(A)  40 cm

(B)  80 cm

(C)  60 cm

(D)  50 cm

Answer: (C)

116. If O is the circumcentre of a right-angled triangle ABC, right-angled at B, the position of point O is-

(A)  Mid-point of CA

(B)  Within triangle ABC

(C)  Outside the triangle ABC

(D)  Lies on BC

Answer: (A)

117. If in a triangle PQR, PQ = 3 cm, QR = 5 cm and PR = 4 cm, then the angles of the triangle in descending order of magnitude are-

(A)  ∠P > ∠Q > ∠R

(B)  ∠Q > ∠P > ∠R

(C)  ∠P > ∠R > ∠Q

(D)  ∠Q > ∠R  > ∠P

Answer: (A)

118. The graphs of x + y = 4 and 3x + 3y = 6-

(A)  Intercept at the point (2, 2)

(B)  Intersect at the point (1, 3)

(C)  Intersect at the point (3, 1)

(D)  Do not intersect

Answer: (D)

119. If yz : zx : xy = 1 : 2 : 3 (x, y, z ≠ 0), then  is-

(A)  1/4

(B)  1/2

(C)  3/2

(D)  4

Answer: (D)

120. If  then the value of  is-

(A)  4

(B)  8

(C)  12

(D)  34

Answer: (D)

121. If  then the value of x2 + 2x + 3 is-

(A)  3

(B)  0

(C)  4

(D)  1

Answer: (C)

122. If a + b + c = 4, then a3 + b3 + c3 – 12c2 + 48c – 64 is equal to-

(A)  3abc

(B)  3abc – 12ab

(C)  3abc + 12ab

(D)  abc + 12ab

Answer: (B)

123. 8 men can finish a piece of work in 40 days. If 2 more men join with them, then the work will be completed in-

(A)  30 days

(B)  32 days

(C)  36 days

(D)  25 days

Answer: (B)

124. The driver of a car driving @ 36 kmph locates a bus 40 m ahead of him. After 20 seconds the bus is 60 m behind. The speed of the bus is-

(A)  36 kmph

(B)  20 m/sec

(C)  72 m/sec

(D)  18 kmph

Answer: (D)

125. A producer of tea blends two varieties of tea from two tea gardens, one costing Rs 240 per kg and the other Rs 300 per kg, in the ratio 5 : 3. If he sells the blended variety at Rs 315 per kg then his gain per cent is-

(A)  10

(B)  20

(C)    

(D)   

Answer: (B)

126. A and B entered into a partnership with capitals in the ratio 4 : 5. After 3 months A withdrew 1/4 of his capital and B withdrew 1/5 of his capital of his capital. The gain at the end of 10 months was Rs 760. A’s share in this profit is-

(A)  Rs 330

(B)  Rs 360

(C)  Rs 450

(D)  Rs 540

Answer: (A)

127. A shopkeeper earns a profit of 15% on selling a book at 10% discount on the printed price. The ratio of the cost price and the printed price is-

(A)  16 : 21

(B)  18 : 21

(C)  17 : 23

(D)  18 : 23

Answer: (D)

128. The selling price of an article is Rs 39. If its cost price is numerically equal to its profit per cent, then its cost price (in Rs) is-

(A)  25

(B)  30

(C)  35

(D)  37

Answer: (B)

129. A man sold two tables for Rs 720 each. One he gained 20% and on the other be lost 20%. Find his total loss for gain in this transaction-

(A)  No gain no loss

(B)  Loss of Rs 60

(C)  Gain of Rs 60

(D)  Loss of Rs 50

Answer: (B)

130. The difference between simple and compound interest @ 10% per annum of a sum of Rs 1,000 after 4 years is-

(A)  Rs 31

(B)  Rs 32.10

(C)  Rs 40.40

(D)  Rs 64.10

Answer: (D)

131. The average of 15 numbers is 7. If the average of the first 8 numbers is 6.5 and the average of the last 8 numbers is 8.5, then the middle number is-

(A)  10

(B)  23

(C)  13

(D)  15

Answer: (D)

132. If  find   –

(A)  13.625

(B)  13.6

(C)  12.5

(D)  2.375

Answer: (A)

133. The sum of three numbers is 121. The ratio of the second to the 3rd is 9 : 6 and the first to the 3rd is 1 : 4. The second number is-

(A)  60

(B)  62

(C)  64

(D)  66

Answer: (D)

134. Two numbers are in the ratio 3 : 5 and their L.C.M. is 75. What is their H.C.F.?

(A)  3

(B)  5

(C)  10

(D)  15

Answer: (B)

135. A’s salary is 30% higher than B’s salary, The per cent that B’s salary is less than A’s salary is-

(A)  30%

(B)   

(C)  20%

(D)  25%

Answer: (B)

136. 1 minute 48 seconds in the percentage of 1 hour is represented by –

(A)  2%

(B)  3%

(C)  4%

(D)  5%

Answer: (B)

137. simplifies to-

(A)  √5 + √6

(B)  2√5 + √6

(C)  √5 – √6

(D)  2√5 – 3√6

Answer: (C)

138. H. C.F. of 1/2, 3/4, 5/6, 7/8, 9/10 is-

(A)  1/2

(B)  1/10

(C)  9/120

(D)  1/120

Answer: (D)

139. The sum of three numbers is 132. If the first number is twice the second and the third number is one-third of the first, then the second number is-

(A)  60

(B)  48

(C)  36

(D)  32

Answer: (C)

140. The least perfect square number which is divisible by 3, 4, 5, 6 and 8 is-

(A)  900

(B)  1600

(C)  2500

(D)  3600

Answer: (D)

141. 24 is divided into two parts such that 7 times the first part added to 5 times the second part makes 146. The first part is-

(A)  11

(B)  13

(C)  15

(D)  17

Answer: (B)

142. A number when divided by 5, 9, 13 leaves remainders 2, 6 and 10 respectively. The least such number is-

(A)  572

(B)  592

(C)  602

(D)  582

Answer: (D)

143. The base of a prism is an equilateral triangle of side 12 cm. If the volume of the prism is 1080 cc, then the total surface area of the prism in sq. cm in-

(A)  72√3

(B)  360√3

(C)  432√3

(D)  1080

Answer: (C)

144. If  then  is equal to-

(A)   

(B)  x

(C)   

(D)   

Answer: (A)

145. In the sequence 4, 13, 38, … the 4th term is-

(A)  58

(B)  96

(C)  111

(D)  123

Answer: (C)

146. If x2 + a2 = y2 + b2 = ax + by = 1, then the numerical value of a2 + b2 is-

(A)  0

(B)  1

(C)  2

(D)  3

Answer: (B)

Directions-(Q. 147 and 148) The histogram below shows the marks of 50 students in an examination. Examine the diagram and answer.

147. How many students obtained less than 40?

(A)  2

(B)  6

(C)  4

(D)  5

Answer: (D)

148. If the minimum marks for getting scholarship is 80, how many students will get it?

(A)  13

(B)  12

(C)  10

(D)  11

Answer: (A)

Directions-(Q. 149 and 150) The following pie-chart shows the result of an examination of 360 students. Study the chart and answer.

149. The number of students who passed in second division is more than that in third division by-

(A)  54

(B)  50

(C)  51

(D)  64

Answer: (A)

150. The percentage of students who failed is-

(A)  5

(B)  20

(C)  10

(D)  15

Answer: (C)

For Visually Handicapped Candidates only

147. If the height is doubled and the base is decreased by 20%, then the ratio of old area of the triangle and new area of the triangle is-

(A)  2 : 3

(B)  3 : 10

(C)  4 : 5

(D)  5 : 8

Answer: (D)

148. If tan 2θ tan 4θ = 1, then the value of tan 3θ is-

(A)  0

(B)  1

(C)  √3

(D)  1/√3

Answer: (B)

149. If  then the value of  is

(A)  ac/b

(B)  a2/bc

(C)  c/ab

(D)  bc/a

Answer: (D)

150. A certain sum amounts to Rs 5,182 in 2 years and Rs 5,832 in 3 years. If the simple interest is calculated annually, then the principal is-

(A)  Rs 2,000

(B)  Rs 5,000

(C)  Rs 3,882

(D)  Rs 4,000

Answer: (C)

Part-IV

General Awareness

151. An index number measuring the average price of consumer goods and services purchased by households is-

(A)  Consumer price index

(B)  Human development index

(C)  Cost of living index

(D)  Wholesale price index

Answer: (A)

152. Oligopoly consists of-

(A)  Two sellers

(B)  Three sellers

(C)  Single seller

(D)  Few sellers

Answer: (D)

153. National Income in India is estimated by the-

(A)  product method

(B)  income method

(C)  expenditure method

(D)  product and income methods

Answer: (D)

154. The fixed cost in the short-run for a producing firm will be-

(A)  always declining

(B)  always constant and fixed

(C)  always increasing

(D)  always fluctuating

Answer: (A)

155. Machine tool is an example of ……. goods.

(A)  consumer

(B)  capital

(C)  free

(D)  intermediary

Answer: (B)

156. Gandhara art was the combination of-

(A)  Indian and greek styles of sculptures

(B)  Indian and Persian styles of sculptures

(C)  Indian and Chinese styles of sculptures

(D)  None of these

Answer: (A)

157. Mohammed Gawan was a famous Wazir and Vakil in the kingdom of-

(A)  Bahmani

(B)  Gujarat

(C)  Kashmir

(D)  Mysore

Answer: (A)

158. The first Governor of the Portuguese in India was-

(A)  Vasco-da-Gama

(B)  Bartholomew Diaz

(C)  Albuquerque

(D)  De Almeida

Answer: (D)

159. Which one of the following reformative measures was not introduced by William Bentinck?

(A)  Abolition of Sati

(B)  Removal of disabilities due to change of religion

(C)  Abolition of slavery

(D)  Suppression of thugs

Answer: (C)

160. Which work of Thomas Paine inspired the colonial people to fight against Britain during the American War of Independence?

(A)  What is the Third Estate

(B)  Common Sense

(C)  The Spirit of the Laws

(D)  Social Contract

Answer: (B)

161. Fascism believes in-

(A)  One party system

(B)  Bi-party system

(C)  Fragmented multi-party system

(D)  Dominant multi-party system

Answer: (A)

162. Which of the following commissions is not a constitutional body?

(A)  Staff Selection Commission

(B)  Election Commission

(C)  Finance Commission

(D)  Union Public Service Commission

Answer: (A)

163. Under the advisory jurisdiction, the Supreme Court gives its opinion on a question of law or fact of great public importance referred to it by-

(A)  The Prime Minister

(B)  The President

(C)  The Union Law Minister

(D)  Any High Court

Answer: (B)

164. How much time is given to Rajya Sabha to pass a Money Bill?

(A)  12 days

(B)  13 days

(C)  14 days

(D)  15 days

Answer: (C)

165. The electoral college to elect the President composes-

(A)  All members of both the Houses of Parliament

(B)  All elected members of both the Houses of Parliament

(C)  All members of Parliament and all members of Legislative Assemblies and Councils

(D)  Elected members of both the Houses of Parliament and Legislative Assemblies

Answer: (D)

166. Which one of the tree species does not belong to deciduous vegetation?

(A)  Teak

(B)  Sal

(C)  Sandalwood

(D)  Deodar

Answer: (D)

167. The predominant soil in the Coastal plains and North Indian plains is-

(A)  Black

(B)  Red

(C)  Alluvial

(D)  Arid

Answer: (C)

168. Rocks having large quantity of underground water and permitting ready flow of water are called-

(A)  Aquicludes

(B)  Aquifers

(C)  Porous

(D)  Permeable

Answer: (A)

169. Which one of the following is not a grassland?

(A)  Selvas

(B)  Steppes

(C)  Downs

(D)  Veld

Answer: (C)

170. Duncan Passage separates-

(A)  North and Middle Andamans

(B)  Middle and South Andamans

(C)  South Andamans and Little Andamans

(D)  Little Andamans and Car Nicobar Islands

Answer: (C)

171. In biology, water soluble substances are referred to as-

(A)  hydrophobic

(B)  hydrophilic

(C)  hydrokinetic

(D)  hydrodynamic

Answer: (B)

172. Amoeba acquires its food through the process of-

(A)  Exocytosis

(B)  Endocytosis

(C)  Plasmolysis

(D)  Exocytosis and endocytosis

Answer: (B)

173. Female Anopheles mosquito is the vector of-

(A)  Filariasis

(B)  Dengue fever

(C)  Yellow fever

(D)  Malaria

Answer: (D)

174. The animal association seen in a sucker fish attached to a shark is-

(A)  Mutualism

(B)  Commensalism

(C)  Parasitism

(D)  Neutralism

Answer: (B)

175. Which among the following is symbiotic Nitrogen-fixing bacteria?

(A)  Xanthomonas

(B)  Pseudomonas

(C)  Rhizobium

(D)  Azotobacter

Answer: (C)

176. Match correctly the crops listed in List-I with the diseases affecting them given in List-II-

List-I (Crops)                List—II(Diseases)

(a) Paddy                       1. Downy mildew

(b) Wheat                       2. Blast

(c) Mustard                    3. Red not

(d) Sugarcane                 4. Rust

(A)  (a) – 1; (b) – 2; (c) – 4; (d) – 3

(B)  (a) – 3; (b) – 1; (c) – 2; (d) – 4

(C)  (a) – 2; (b) – 4; (c) – 3; (d) – 1

(D)  (a) – 4; (b) – 2; (c) – 3; (d) – 1

Answer: (C)

177. A plant with green leaves viewed in red light will appear-

(A)  Black

(B)  Green

(C)  Red

(D)  Violet

Answer: (A)

178. A body moving in a circular path with constant speed has-

(A)  constant retardation

(B)  constant acceleration

(C)  variable acceleration

(D)  radially outward acceleration

Answer: (B)

179. Total internal reflection occurs when light travels from-

(A)  a rarer medium to a denser medium

(B)  a denser medium to a rarer medium

(C)  a rarer medium to a denser medium with angle of incidence greater than critical angle

(D)  a denser medium to a rarer medium with angle of incidence greater than critical angle

Answer: (B)

180. MCB, which cuts off the electricity supply in case of short-circuiting works on the-

(A)  chemical effect of current

(B)  heating effect of current

(C)  magnetic effect of current

(D)  electroplating effect of current

Answer: (B)

181. An operating system is a/an-

(A)  Application software

(B)  System software

(C)  Utility software

(D)  Accounting software

Answer: (B)

182. In DOS, the DIR Command is used to-

(A)  delete files

(B)  display list of files and subdirectories

(C)  copy files

(D)  display contents of a file

Answer: (B)

183. ………….. is the amount of work that the system is able to do per unit time.

(A)  Output

(B)  Transfer rate

(C)  Throughput

(D)  Turnaround

Answer: (C)

184. An example of a covalent molecule is-

(A)  Carbon tetrachloride

(B)  Potassium chloride

(C)  Sodium chloride

(D)  Lead chloride

Answer: (D)

185. The element common to all acids is-

(A)  Oxygen

(B)  Hydrogen

(C)  Nitrogen

(D)  Sulphur

Answer: (B)

186. In medicine bottles containing tablets or capsules, a small pouch of silica gel is kept to-

(A)  kill bacteria

(B)  absorb moisture

(C)  absorb gases

(D)  keep the bottle warm

Answer: (B)

187. Aluminium oxide is a/an-

(A)  acid oxide

(B)  basic oxide

(C)  neutral oxide

(D)  amphoteric oxide

Answer: (D)

188. Which one of the following is the most stable ecosystem?

(A)  Desert

(B)  Mountain

(C)  Ocean

(D)  Forest

Answer: (C)

189. Which of the following is a biodegradable pollutant?

(A)  Dye effluent

(B)  Bleaching effluent

(C)  Sewage

(D)  Heavy metals

Answer: (C)

190. The concentration of pollutants in atmosphere are generally expressed in-

(A)  parts per million

(B)  parts per billion

(C)  kilogram per square metre

(D)  parts per trillion

Answer: (A)

191. Which country won the Confederations Cup, 2013?

(A)  Spain

(B)  Brazil

(C)  Italy

(D)  Uruguay

Answer: (B)

192. Who is the Chief Justice of India?

(A)  Justice Altamas Kabir

(B)  Justice P. Sathasivam

(C)  Justice Markandey Singh

(D)  Justice K.G. Balakrishnan

Answer: (B)

193. Which one of the following days does not synchronize with the birth anniversary of any eminent Indian?

(A)  National Science Day

(B)  Teacher’s Day

(C)  Children’s Day

(D)  Gandhi Jayanti

Answer: (A)

194. Which one of the following novels was not written by Charles Dickens?

(A)  Hard Times

(B)  Pride and Prejudice

(C)  Oliver Twist

(D)  Pickwick Papers

Answer: (B)

195. Which country has given asylum to Julian Assange, Wiki Leaks founder?

(A)  Cuba

(B)  Ecuador

(C)  Russia

(D)  None of these

Answer: (B)

196. What is the currency of South Africa?

(A)  Real

(B)  Rial

(C)  Dirham

(D)  Rand

Answer: (D)

197. Match the two lists containing the name of poets and the language in which they wrote Ramayana-

Poets

(a) Kamban

(b) Krittivasa

(c) Madhava Kandali

(d) Tulsi Das

Language in which Ramayana was written

(1) Bengali         (2) Hindi

(3) Tamil (4) Assamese

(A)  (a) – 2; (b) – 3; (c) – 1; (d) – 4

(B)  (a) – 3; (b) – 1; (c) – 4; (d) – 2

(C)  (a) – 4; (b) – 2; (c) – 1; (d) – 3

(D)  (a) – 3; (b) – 4; (c) – 1; (d) – 2

Answer: (B)

198. Which one of the following countries is not a member of the Organization of Petroleum Exporting Countries?

(A)  Algeria

(B)  Brazil

(C)  Ecuador

(D)  Libya

Answer: (B)

199. The two events in which India won gold medals in the Asian Athletic Championships, 2013 are-

(A)  Women’s 400 m and Women’s Discuss Throw

(B)  Women’s Long Jump and Men’s Discuss Throw

(C)  Women’s Shot Put and Women’s 4 × 400 m Relay

(D)  Women’s 4 × 400 m Relay and Men’s Discuss Throw

Answer: (D)

200. For which one of the following films directed by Shyam Benegal was the story written jointly by him and Dr. Verghese Kurien?

(A)  Ankur

(B)  Mandi

(C)  Manthan

(D)  Zubeidaa

Answer: (C)

SSC Combined Higher Secondary Level (10+2) 1st Shift Tier-I Examination Held on 27 November, 2013 Question Paper With Answer Key

SSC Combined Higher Secondary Level (10+2) 1st Shift Tier-I Examination Held on 27 November, 2013
SSC Combined Higher Secondary Level (10+2) 1st Shift Tier-I Examination Held on 27 November, 2013 Question Paper With Answer Key

Staff Selection Commission Combined Higher Secondary Level (10+2) 1st Shift

Tier-I Examination Held on 27 November, 2013

Part-I

General Intelligence

1. Which one of the given responses would be a meaningful order of the following?

(1) Twilight          (2) Dawn

(3) Noon             (4) Night

(A)    2, 1, 3, 4

(B)    2, 3, 1, 4

(C)    1, 2, 3, 4

(D)    1, 3, 2, 4

Answer: (B)

2. Arrange the following words as per order in the dictionary-

(1) Aqueous         (2) Aquarium

(3) Aquiline          (4) Aquatic

(A)    4, 3, 2, 1

(B)    1, 2, 3, 4

(C)    2, 4, 1, 3

(D)    3, 1, 4, 2

Answer: (C)

3. Which one set of letters when sequentially placed at the gaps in the given letter series shall complete it?

mc_m_a_ca_ca_c_mc

Answer: (A)

Dirctions –(Q. 4-6) In the following questions, a series is given, with one term missing. Choose the correct alternative from the given ones that will complete the series.

4. AMN, BOP, CQR, ?

(A)    BAS

(B)    DST

(C)    FQP

(D)    FRS

Answer: (B)

5. 5, 6, 9, 14, 21, ?

(A)    28

(B)    30

(C)    31

(D)    29

Answer: (B)

6. 4, 11, 30, 67, 128, ?

(A)    219

(B)    228

(C)    231

(D)    237

Answer: (A)

7. If the day before yesterday was Wednesday when will Sunday be?

(A)    Today

(B)    Tomorrow

(C)    Day after tomorrow

(D)    Two days after tomorrow

Answer: (C)

8. Lakshmi is elder than Meenu. Leela is leder than Meenu but younger than Lakshmi. Latha is younger than both Meenu and Hari but Hari is younger than Meenu. Who is the  youngest?

(A)    Lakshmi

(B)    Meenu

(C)    Leela

(D)    Latha

Answer: (D)

9. Nikhil is 8 years younger than his brother Rohan. How old will Rohan be when he is twice as old as Nikhil?

(A)    4

(B)    6

(C)    8

(D)    16

Answer: (D)

10. In a row of letters, a letter is 5th from left end and 12th from the right end. How many letters are there in a row?

(A)    15

(B)    16

(C)    17

(D)    18

Answer: (B)

11. Pointing to a lady Simon said, “She is the daughter of the only sister of my father”. How is lady related to Simon?

(A)    Mother

(B)    Aunt

(C)    Sister

(D)    Cousin Sister

Answer: (D)

12. If A = 1, ACT = 24, then FAT = ?

(A)    26

(B)    25

(C)    27

(D)    24

Answer: (C)

13. If ‘GLOSSORY’ is coded as 97533562 and ‘GEOGRAPY’ = 915968402, then ‘GEOLOGY’ can be coded as-

(A)    915692

(B)    9157592

(C)    9057592

(D)    9157591

Answer: (B)

14. If ‘NEUROTIC’ can be written as ‘TICRONEU’ then how can ‘PSYCHOTIC’ be written?

(A)    TICOCHPSY

(B)    TICCHOPSY

(C)    TICCOHPSY

(D)    TICHCOPSY

Answer: (B)

15. Some letters are given below in the first line and numbers are given below them in the second line. Numbers are the codes for the alphabets and vice-versa. Choose the correct number-code for the given set of alphabets.

C     W        E          A         Z          X         J           Y         K         L

3      9          5          7          4          8          1          0          2          6

J W X C L Z

(A)    1 9 8 3 6 4

(B)    1 9 8 2 6 4

(C)    1 9 8 3 5 4

(D)    1 9 7 3 5 4

Answer: (A)

16. From the given alternatives select the word which can be formed using the letters given in the word.

PREPARATION

(A)    PAMPER

(B)    REPEAT

(C)    PARTITION

(D)    PARROT

Answer: (D)

17. From the given alternatives select the word which cannot be formed using the letters of the given word.

INTERVENTION

(A)    ENTER

(B)    INTENTION

(C)    INVENTION

(D)    ENTERTAIN

Answer: (D)

Directions-(Q. 18 and 19) In these questions, select the missing number from the given responses.

18.

2               24        28

3               9          27

4               ?          40

(A)    10

(B)    16

(C)    12

(D)    32

Answer: (A)

19.

81             36        25

49             100      36

9               64        16

139           200      ?

(A)    107

(B)    77

(C)    27

(D)    50

Answer: (B)

Directions-(Q. 20 and 21) In these questions, some equations are solved on the basis of a certain system. Find the correct answer for the unsolved equation on that basis.

20. 4 × 6 × 9 = 694, 5 × 3 × 2 = 325, 7 × 8 × 2 = ?

(A)    729

(B)    872

(C)    827

(D)    279

Answer: (C)

21. If 2463 = 36 and 5552 = 30, then 6732 = ?

(A)    32

(B)    36

(C)    34

(D)    39

Answer: (A)

22. Select the correct combination of mathematical signs to replace * signs and to balance the given equation.

5 * 6 * 5 * 8 * 14

(A)    + − × =

(B)    × ÷ + =

(C)    ÷ + = −

(D)    + × − ÷

Answer: (B)

23. If – mean ×, × means +, + means ÷ and ÷ means −, then −

(A)    44

(B)    16

(C)    7.95

(D)    4

Answer: (D)

24. After starting from a point, A walks 3 km towards East, then turning to his left he moves 3 km. After this he again turns left and moves 3 km. In which direction is ‘A’ from his starting point?

(A)    North

(B)    East

(C)    West

(D)    South

Answer: (A)

25. Ram goes 15 metres North then turns right and walks 20 metres, then again turns right and walks 10 metres, then again turns right and walks 20 metres. How far is he from his original position?

(A)    5

(B)    10

(C)    15

(D)    20

Answer: (A)

Directions-(Q. 26 and 27) In these questions, two statements are given followed by two conclusions I and II. You have to consider the statements/inferences to be true even if they seem to be at variance from commonly known facts. You are to decide which of the given conclusions, if any, follow from the given statements. Indicate your answer.

26. Statements:

(i) Some chairs are made up of wood.

(ii) Some tables are made up of wood.

Inferences :

(I) All wooden things are either chairs or tables.

(II) Some chairs are tables.

(A)    Only inference I follows

(B)    Only inference II follows

(C)    Both inferences I and II follow

(D)    None of the inference follows

Answer: (D)

27. Statements:

(i) For protection of Indian museums Central Govt. is responsible.

(ii) Victoria Memorial Hall is national property.

Conclusions:

(I) Indian museum is national property.

(II) Historical property of nation is protected by the Central Government.

(A)    Only Conclusion I follows

(B)    Only conclusion II follow

(C)    Both conclusions I and II follow

(D)    None of the conclusion follows

Answer: (D)

Directions-(Q. 28-34) In these questions, select the related words/letters/number from the given alternatives.

28. Cobbler : Leather : : Carpenter : ?

(A)    Furniture

(B)    Wood

(C)    Hammer

(D)    Chair

Answer: (B)

29. Foot : ? : : Hand : Wrist

(A)    Length

(B)    Shoe

(C)    Ankle

(D)    Leg

Answer: (D)

30. Chair : Wood : : ?

(A)    Book : Print

(B)    Mirror : Glass

(C)    Plate : Food

(D)    Purse : Money

Answer: (B)

31. EDUCATION : NOITACUDE : : INTELLIGENCE : ?

(A)    ECENGILLTEIN

(B)    ECNGEILLTENI

(C)    ECNEGILLETNI

(D)    ECNEGILLTENI

Answer: (C)

32. AZCX : BYDW : : HQJO : ?

(A)    GREP

(B)    IPKM

(C)    IPKN

(D)    GRJP

Answer: (C)

33. 08 : 66 : : ? : 38

(A)    2

(B)    6

(C)    12

(D)    19

Answer: (B)

34. 32 : 21 : : 35 : ?

(A)    51

(B)    53

(C)    32

(D)    26

Answer: (A)

Directions-(Q. 35-38) In these questions, find the odd words/letters/number pair from the given ahead alternatives-

35.

(A)    Barter

(B)    Purchase

(C)    Sale

(D)    Borrow

Answer: (D)

36.

(A)    Flag : Flagship

(B)    Court : Courtship

(C)    War : Worship

(D)    Friend : Friendship

Answer: (C)

37.

(A)    BDFH

(B)    MOQS

(C)    SUWY

(D)    TVZE

Answer: (D)

38.

(A)    8 – 15

(B)    25 – 36

(C)    49 – 64

(D)    81 – 100

Answer: (A)

39. Find out the set among the four sets which is like the given set.

(13 : 20 : 27)

(A)    (3 : 11 : 18)

(B)    (18 : 25 : 32)

(C)    (18 : 27 : 72)

(D)    (7 : 14 : 28)

Answer: (B)

40. How many Ms occur in the following series such that it is preceded by W and followed by V?

X U V M R S T M W N V M W O P M W U V M W A C W M V H P N V W M W T U N

(A)    3

(B)    2

(C)    1

(D)    5

Answer: (C)

41. In a gathering seven members are sitting in a row. ‘C’ is sitting left to ‘B’ but on the right to ‘D’. ‘A’ is sitting right to ‘B’, ‘F’ is sitting right to ‘E’ but left to ‘D’. ‘H’ is sitting left to ‘E’. Find the person sitting in the middle.

(A)    C

(B)    D

(C)    E

(D)    F

Answer: (B)

42. In 60 families 30% of the families own a car each. 50% of the remaining family own a motor cycle each. Balance of families own a bicycle each. How many families own bicycle?

(A)    12

(B)    21

(C)    18

(D)    42

Answer: (B)

43. Select the related figure from the given alternatives-

Answer: (D)

44. Find the odd figure from the given alternatives-

Answer: (D)

45. Choose the correct alternative from the given ones that will complete the series.

Answer: (D)

46. Select the missing number from the given responses.

(A)    2

(B)    52

(C)    66

(D)    28

Answer: (C)

47. Identify the diagram that best represents the relationship among classes given below-

Mammal, Dog and Bat

Answer: (D)

48. Which answer figure will complete the pattern in the question figure?

Answer: (B)

49. A piece of paper is folded and cut/punched as shown below in the question figures. From the given answer figures, indicate, how it will appear when opened?

Answer: (C)

50. From the given answer figures, select the one in which the question figure is hidden / embedded-

Answer: (A)

For Visually Handicapped Candidates Only

43. Find out the wrong number in the given series from the given alternatives :

196, 169, 144, 121, 101

(A)    101

(B)    121

(C)    169

(D)    196

Answer: (A)

44. A frog is at the bottom of a 20 feet well. Each day he jumps 3 feet and slips back two. At that rate, when will the frog reach the top of the well?

(A)    17th Day

(B)    18th Day

(C)    20th Day

(D)    21th Day

Answer: (B)

 

  45. If means ‘×’ means ‘−’, ‘∆’ means ‘÷’, then

(A)    12

(B)    16

(C)    28

(D)    33

Answer: (D)

46. In a peculiar mathematical operation, before any operations are done, the numbers are changed as per following rule-

1, 3, 5, 7, 9 → changed to 2

2, 4, 6, 8 → changed to 0

0 → changed to 1

Solve the following equation-

4792 + 6785 – 4071 – 968 = ?

(A)    122

(B)    100

(C)    211

(D)    200

Answer: (B)

47. Anil is fifteenth from both ends of a row of boys. How many boys are there in the row?

(A)    15

(B)    16

(C)    30

(D)    29

Answer: (D)

48. If the difference between 3/8th and 1/3rd of a number is 6, then the number is-

(A)    240

(B)    120

(C)    96

(D)    144

Answer: (D)

49. In the following numbers series, how many 9’s are followed by an odd number and preceded by an even number?

4713891694395432971954821495178

(A)    2

(B)    3

(C)    4

(D)    5

Answer: (B)

50. How many Tuesdays are possible if the last day of the month (excluding February) is Wednesday?

(A)    3

(B)    4

(C)    5

(D)    6

Answer: (C)

Part-II

English Language

Directions-(Q. 51 and 52) Four words are given in each question, out of which only one word is correctly spelt. Find the correctly spelt word and mark your answer in the Answer Sheet.

51.

(A)    Adalosent

(B)    Adolascent

(C)    Adolescent

(D)    Adaloscent

Answer: (C)

52.

(A)    Sovenir

(B)    Souvenir

(C)    Suvenir

(D)    Suvenior

Answer: (B)

Directions-(Q. 53-55) In these questions, four alternatives are given for the Idiom/Phrase underlined in the sentence. Choose the alternative which best expresses the meaning of the Idiom/Phrase and mark it in the Answer Sheet.

53. If you want to solve the problem for ever you must take the bull by the horns.

(A)    pull the bull’s horns.

(B)    face a difficulty or danger boldly.

(C)    run away from a difficulty or danger.

(D)    face a difficulty or danger confidently.

Answer: (D)

54. Rajib washed his hands of the matter long ago.

(A)    overcame the difficulties

(B)    refused to accept responsibility for

(C)    took the responsibility

(D)    refused to change the decision

Answer: (B)

55. All the neighbours gave her the cold shoulder because she ill-treated her daughter-in-law.

(A)    showed intentional unfriendliness to her company

(B)    felt her shoulder was cold

(C)    showed dislike for her company

(D)    showed liking for her company

Answer: (A)

Directions-(Q. 56-58) In these questions, out of the four alternatives choose the one which can be substituted for the given words/sentence.

56. A person who runs away from justice or the law-

(A)    Refugee

(B)    Martyr

(C)    Fugitive

(D)    Recluse

Answer: (C)

57. A place where bees are kept-

(A)    Aviary

(B)    Apiary

(C)    Granary

(D)    Dispensary

Answer: (B)

58. A hater of women-

(A)    Bigamist

(B)    Misogynist

(C)    Misanthrope

(D)    Polygynist

Answer: (B)

Directions-(Q. 59 and 60) In these questions, a part of the sentence is underlined. Below are given alternatives to the underlined part at (A), (B), (C) which may improve the sentence. Choose the correct alternative. In case in improvement is needed  your answer is (D). Mark  your answer in the Answer Sheet.

59. The criminal was ordered to be hanged to death.

(A)    hung to death

(B)    hanged till death

(C)    hung till death

(D)    No improvement

Answer: (B)

60. The expert was very intersecting in this candidate.

(A)    was having interest

(B)    was very interested

(C)    is having interest

(D)    no improvement

Answer: (B)

Directions-(Q. 61-65) In these questions, a sentence has been given in Active/Passive Voice. Out of the four alternatives suggested, select the one which best expresses the same sentence in Passive/Active Voice and mark  your answer in the Answer Sheet.

61. I took care of his dog.

(A)    His dog was taken care by me

(B)    His dog is taken care by me

(C)    His dog is taken care of by me

(D)    His dog was taken care of by me

Answer: (D)

62. The washerman will return the clothes in the evening.

(A)    The clothes will have to be returned by the washerman in the evening

(B)    The clothes will be returned by the washerman in the evening

(C)    The clothes will return by the washerman in the evening

(D)    In the evening the clothes will have been returned by the washerman

Answer: (B)

63. He picked up the telephone and dialed the police number

(A)    The telephone was being picked up and the police number dialled

(B)    The telephone was picked up and the police number dialled

(C)    The telephone had been picked up and the police number dialled

(D)    The telephone has been picked up and the police number dialled

Answer: (B)

64. An i-pod has been bought by Anubhav’s mother.

(A)    Anubhav’s mother was engaged in buying an i-pod.

(B)    Anubhav’s mother had bought an i-pad

(C)    Anubhav’s mother has been buying an i-pod

(D)    Anubhav’s mother has bought an i-pad

Answer: (D)

65. Some girls were cleaning their dormitories.

(A)    The dormitories were cleaned by some girls

(B)    The dormitories will have been cleaned by some girls

(C)    The dormitories were being cleaned by some girls

(D)    The dormitories are being cleaned by some girls

Answer: (C)

Directions-(Q. 66-70) In these questions, a sentence has been given in Direct/Indirect. Out of the for alternatives suggested, select the one which best expresses the same sentence in Indirect/Direct and mark your answer in the Answer Sheet.

66. He swore in the name of God that he was ignorant o the matter.

(A)    He said, “I’m ignorant by God !”

(B)    He declared, “God knows I was ignorant of the matter”

(C)    He said, “By God ! I’m ignorant of the matter”

(D)    He said he was ignorant of God

Answer: (C)

67. He proposed to his companions that they should not miss that chance to see the last show.

(A)    He said to the companions, ‘Let us not to miss the chance to see the last show’

(B)    He said to his companions, ‘Let us not miss that chance to see the last show’

(C)    He said to his companions, ‘We should not miss that chance to see the last show’

(D)    He said to his companions, ‘Let us not miss the chance to see the last show’

Answer: (B)

68. “Bring me an elephant with one golden tusk,” the with told her minion.

(A)    The witch requested her minion to bring her an elephant with one golden tusk

(B)    The witch told her minion to bring her the elephant with one golden tusk

(C)    The witch ordered her minion to bring her an elephant with one golden tusk

(D)    The witch advised her minion to bring her an elephant with one golden tusk

Answer: (C)

69. “How much will you pay for this cow?” Jack asked the strange man.

(A)    Jack asked the strange man how much he would pay for that cow

(B)    Jack asked the strange man how much would he pay for that cow

(C)    Jack asked the strange man how much would be pay for this cow

(D)    Jack asked the strange man how much he will pay for that cow

Answer: (A)

70. The teacher said, “Water boils at 100°”

(A)    The teacher told water boiled at 100°C

(B)    The teacher said that water boiled at 100°C

(C)    The teacher said that water boils at 100°C

(D)    The teacher told that water boiled at 100°C

Answer: (C)

Directions-(Q. 71-76) In these questions, the 1st and the last sentence/parts of the passage/sentence are numbered 1 and 6. The rest of the passage/sentence is split into four parts and named P, Q, R and S. These four parts are not given in their proper order. Read the sentence and find out which of the four combinations is correct. Then find the correct answer and indicate it by blackening the appropriate oval (•) in the Answer Sheet.

71. (1) Six robbers entered a Nationalised bank today at 9.50 a.m.

(P) One group kept an eye on the customers.

(Q) The assistant manager was forced to unlock the vault.

(R) The other group held the Manager at gun point.

(S) They cut off the telephone lines and deactivated the burglar alarm.

(6) The whole operation was over in 20 minutes.

(A)    SPRQ

(B)    PSRQ

(C)    PQSR

(D)    QPRS

Answer: (A)

72. (1) AIDS is an incurable disease which effects the resistance power of the body.

(P) People succumb within two years and often die inspite of any treatment.

(Q) AIDS is caused by a virus named HIV.

(R) In such a conditions the body is totally defenseless against all kinds of infection.

(S) HIV destroys the while blood cells which fight the bacterial virus.

(6) Thorough research is going on throughout the world for the prevention and cure of AIDS.

(A)    QPRS

(B)    QSRP

(C)    RQSP

(D)    SPQR

Answer: (B)

73. (1) Books are the best company.

(P) Through books are even seen them as if they are really alive.

(Q) They also give us the best society.

(R) We hear what such great men said or did.

(S) They help us in establishing contact with many great men.

(6) As we read, we share great thoughts with great men.

(A)    PQSR

(B)    QRSP

(C)    QSRP

(D)    SPQR

Answer: (A)

74. (1) One of the greatest advances.

(P) has been the invention of computers.

(Q) is modern technology

(R) in industry, administration and education.

(S) which are widely used.

(6) and in almost every sphere of human life.

(A)    PQRS

(B)    QPSR

(C)    QPRS

(D)    SPQR

Answer: (B)

75. (1) The amount of oxygen.

(P) to meet the increasing demands.

(Q) due to indiscriminate deforestation.

(R) is being diminished day-by-day.

(S) Which is released from the plant kingdom.

(6) of the human population.

(A)    RQSP

(B)    PSRQ

(C)    RPQS

(D)    SRQP

Answer: (A)

76. (1) A patient accusing a doctor.

(P) of malpractice will find it difficult.

(Q) without another doctor’s testimony.

(R) to prove damage.

(S) or even file a case in the court.

(6) about proper medical procedures.

(A)    RPQS

(B)    SQPR

(C)    QRPS

(D)    PRSQ

Answer: (D)

Directions-(Q. 77-81) In the following passage some of the words have been left out. Read the passage carefully and choose the correct answer to each question out of the four alternatives and fill in the blanks-

     In these days of cable television the radio has again become popular. As in the days before the …77… of television, people switch on their radios first thing in the morning. FM channels of every …78… can be heard broadcasting news and songs of every …79… More than anything else, people prefer to listen …80… which bring a refreshing change to the …81… routine of their everyday lives.

77.

(A)    advent

(B)    appearance

(C)    birth

(D)    genesis

Answer: (A)

78.

(A)    range

(B)    variety

(C)    frequency

(D)    category

Answer: (C)

79.

(A)    state

(B)    language

(C)    description

(D)    kind

Answer: (B)

80.

(A)    into the songs

(B)    from the songs

(C)    the songs

(D)    to the songs

Answer: (C)

81.

(A)    packed

(B)    monotonous

(C)    exciting

(D)    light

Answer: (B)

Directions-(Q. 82-86)  You have brief passage with 5 questions following the passage. Read the passage carefully and choose the best answer to each question out of the four alternatives and mark it by blackening the appropriate oval (•) in the Answer Sheet.

     Bringing up the baby is now a science and fine art ! Most of the maternal grandparents would agree with this statement. My mother lost fifteen Kilos in six months when my son was born and I gained that as much. Both ways it was because of the anxiety and shared responsibility. My colleagues welcome retirement when their daughter is ready to deliver or had just delivered. They most often befriend young experienced mother and spend long counseling sessions with them collecting tips on how to tackle a crisis.

It is a funny sight to see grey haired women browse ‘Bringing up Baby’ books at the local book store. One tends to think what they did with their own off springs. Ask them and pat comes the reply “it was so easy then. They grew up on their own”. The mother may wear many hats at the same time. But to her or her parents, the mind boggling thing for the first few years is diaper changes, circadian rhythm and continual feeding. At home social life is taboo. Phone calls give cause for irritation, music becomes noise, friends and relatives become total strangers, the domestic help a spirit of mercy and the pediatrician a fairy godmother. Raising a kid is akin to a guerilla warfare and it you hear a woman say that her bady ‘is beautiful and perfect and an angel all the time’ then she is the grandma.

82. What is difficult for maternal grandparents?

(A)    Raising grandchildren

(B)    Feeding children

(C)    Giving birth to grandchildren

(D)    Bringing up babies

Answer: (D)

83. How did the mother and the daughter share the same anxiety?

(A)    Both were uneasy with the new born

(B)    Both feared babies

(C)    Both were concerned about raising the baby

(D)    Both were nervous about shared responsibility

Answer: (C)

84. Why do female colleagues welcome retirement?

(A)    They get lot of free time

(B)    They become free to attend to household duties

(C)    They can be of help to their daughters and their offspring

(D)    They can cook good food for their children and grandchildren

Answer: (C)

85. ‘To wear many hats’ means-

(A)    Wear helmet

(B)    Wear head gear

(C)    Do hairstyling

(D)    Undertake much work

Answer: (D)

86. Why is social life taboo at home?

(A)    Babies are demanding

(B)    Babies do not like guests

(C)    Babies hate parties

(D)    Babies prefer to go out than stay at home

Answer: (A)

Directions-(Q. 87-91) In these questions, some parts of the sentence have errors and some are correct. Find out which part of a sentence has an error had blacken the oval (•) corresponding to the appropriate letter (A, B, C). If a sentence is free from errors, blacken the oval corresponding to (D) in the Answer Sheet.

87. The principal (A)/started his lecture (B)/ with a pessimistic note. (C)/ No error (D)

Answer: (D)

88. Rohan has been playing (A)/for his club for fifteen years, (B)/ when his elbow got injured. (C)/ No error (D)

Answer: (C)

89. None of (A)/ the twins (B)/ offered to help me. (C)/ No error (D)

Answer: (A)

90. My teacher (A)/ did not object (B)/ to me using the calculator. (C)/ No error (D)

Answer: (C)

91. The river (A)/ has overflown (B)/ its banks. (C)/ No error (D)

Answer: (B)

Directions-(Q. 92-96) In these questions, sentences are given with blanks to be filled in with an appropriate word(s). Four alternatives are suggested for each question. Choose the correct alternative out of the four and indicate it by blackening the appropriate oval (•) in the Answer Sheet.

92. The bird’s voice was speaking ……… all lying under stars.

(A)    to

(B)    for

(C)    at

(D)    in

Answer: (A)

93. ………….. my food and my repose were salted.

(A)    None of

(B)    All of

(C)    Both

(D)    Both of

Answer: (C)

94. India will ……… arch rival Pakistan in the finals.

(A)    takes after

(B)    looks after

(C)    taken back

(D)    take on

Answer: (D)

95. The shepherded was looking after the….. of sheep.

(A)    crew

(B)    gang

(C)    fleet

(D)    flock

Answer: (D)

96. Many of us believe that science is something modern ……… the truth is otherwise.

(A)    unless

(B)    thought

(C)    if

(D)    as if

Answer: (B)

Directions-(Q. 97 and 98) In these questions, out of the four alternatives, choose the one which best expresses the meaning of the given word and mark it in the Answer Sheet.

97. VIVACIOUS

(A)    Lively

(B)    Venomous

(C)    Clear

(D)    Superior

Answer: (A)

98. Staunch

(A)    Loyal

(B)    Benign

(C)    Good

(D)    Wise

Answer: (A)

Directions-(Q. 99 and 100) In these questions, choose the word opposite in meaning of the given word and mark it in the Answer Sheet.

99. Modesty

(A)    Honesty

(B)    Vanity

(C)    Originality

(D)    Variety

Answer: (B)

100. Spite

(A)    Spleen

(B)    Venom

(C)    Spirit

(D)    Affection

Answer: (D)

Part-III

Quantitative Aptitude

101. If 2x + 3y : 3x + 5y = 18 : 29, then x/y is equal to-

(A)    3/4

(B)    4/3

(C)    2/3

(D)    5/6

Answer: (A)

102. is-

(A)    24

(B)    2.4

(C)    0.024

(D)    2.04

Answer: (C)

103. In a particular type of alloy the ratio of lead to tin is 5 : 6. The amount of lead should be added to 12.1 kg of this material to make the ratio 1 : 1 is-

(A)    1.1 kg

(B)    1 kg

(C)    0.9 kg

(D)    1.5 kg

Answer: (A)

104. If the average of m numbers is n2 and that of n numbers is m2, then the average of (m + n) numbers is-

(A)    m/n

(B)    mn

(C)    m – n

(D)    m + n

Answer: (B)

105. The simple interest at x% for x years will be Rs x on a sum of-

(A)    Rs x

(B)    Rs 100x

(C)    Rs 100/x

(D)    Rs 100/x2

Answer: (C)

106. A shopkeeper sells an article at a loss of  Had he hold the article for Rs 51.80 more, he would have earned a profit of 6%. The cost price of the article is-

(A)    280

(B)    580

(C)    370

(D)    450

Answer: (A)

107. The cost price of 18 articles is equal to the selling price of 15 articles. The gain is-

(A)    25%

(B)    20%

(C)    18%

(D)    15%

Answer: (B)

108. Successive discounts of 50% and 50% is equivalent to-

(A)    100%

(B)    75%

(C)    50%

(D)    25%

Answer: (B)

109. A, B, C enter into a partnership with shares in the ratio  After 4 months, A increases his share by 50%. If the total profit at the end of the  year was Rs 43,200, then the B’s share in the profit is-

(A)    Rs 4,200

(B)    Rs 4,800

(C)    Rs 7,200

(D)    Rs 8,000

Answer: (D)

110. Two trains, of same length, are running in parallel tracks in opposite directions with speed 65 km/hour and 85 km/hour respectively. They cross each other in 6 seconds. The length of each train is-

(A)    100 metre

(B)    115 metre

(C)    125 metre

(D)    150 metre

Answer: (C)

111. 50 persons can do a work in 12 days, working 6 hours/day. 60 persons can do the same work in 8 days, working x hours per day. The value of x is-

(A)    15

(B)  

(C)  

(D)    5

Answer: (B)

112 If  is equal to-

(A)    2

(B)    1/2

(C)    1

(D)    0

Answer: (C)

113. If 4x – 5z = 16 and xz = 12, then value of 64x3 – 125z3 is equal to-

(A)    15610

(B)    15616

(C)    15618

(D)    15620

Answer: (B)

114. If x + y + z = 15, xy + yz + zx = 75, then  is equal to-

(A)    1

(B)    0

(C)    2

(D)    −1

Answer: (C)

115. If the graphs of 3x – 5y = −8 and 3x + 5y = 32 intersect at the point (p, q), then the value of p – q is-

(A)    3

(B)    2

(C)    1

(D)    0

Answer: (D)

116. If x = √3 + √2 and y = √3 – √2, then the value of 8xy(x2 + y2) is-

(A)    16√6

(B)    32

(C)    48√2

(D)    80

Answer: (D)

117. In a ∆ ABC, ∠A = 80°. If BD and CD are internal bisectors of ∠B and ∠C respectively, then the ∠BDC is-

(A)    100°

(B)    110°

(C)    120°

(D)    130°

Answer: (D)

118. The side BC of the triangle ABC is produced to D. If ∠ACD = 112° and ∠ABC = ¾ ∠BAC, then the ∠ABC is equal to-

(A)    68°

(B)    58°

(C)    48°

(D)    38°

Answer: (C)

119. In ∆ ABC, X and Y are points on sides AB and BC respectively such that XY || AC and XY divides triangular region ABC into two parts equal in area. Then AX/AB is equal to-

(A) 

(B) 

(C) 

(D) 

Answer: (C)

120. A circle is touching the side BC of ∆ ABC at P and is also touching AB and AC produced at Q and R respectively. If AQ = 6 cm, then perimeter of ∆ ABC is-

(A)    6 cm

(B)    10 cm

(C)    12 cm

(D)    18 cm

Answer: (C)

121. The length of two parallel chords of a circle are 6 cm and 8 cm. If the smaller chord is at a distance of 4 cm from the centre, the distance of the other chord from the centre is-

(A)    5 cm

(B)    6 cm

(C)    4 cm

(D)    3 cm

Answer: (D)

122. The area of an isosceles trapezium is 176 cm2 and the heights is 2/11th of the sum of its parallel sides. If the ratio of the length of the parallel sides be 4 : 7, then the length of a diagonal is-

(A)    √168 cm

(B)    2√147 cm

(C)    √137 cm

(D)    2√137 cm

Answer: (D)

123. The medians AD, BE and CF of a triangle ABC are of lengths 18 cm, 24 cm and 30 cm respectively. The area in cm2 of the triangle is-

(A)    96

(B)    192

(C)    288

(D)    374

Answer: (C)

124. A solid iron sphere of diameter 6 cm is melted and made to a hollow cylindrical pipe of external diameter 10 cm and of length 4 cm. The width of the sheet of pipe is-

(A)    1 cm

(B)    2 cm

(C)    3 cm

(D)    4 cm

Answer: (A)

125. If t he volumes of a sphere and a cylinder of equal radius are same, then ratio of the radius height of the cylinder will be-

(A)    4 : 3

(B)    3 : 2

(C)    3 : 4

(D)    2 : 3

Answer: (C)

126. If the base of a right pyramid is a triangle of sides 21 cm, 20 cm, 13 cm and its altitude is 30 cm, then its volume (in cm3) will be-

(A)    430

(B)    630

(C)    1068

(D)    1260

Answer: (D)

127. If the difference between an exterior angle of a regular polygon of n sides and an exterior angle of another regular polygon of (n + 1) sides is equal to 5°, then the value of n is-

(A)    5

(B)    6

(C)    7

(D)    8

Answer: (D)

128. If the base of a right prism of 20 cm high is a rhombus whose diagonal are 14 cm and 18 cm, then the volume in cm3 of the prism will be-

(A)    2520

(B)    2250

(C)    3520

(D)    1250

Answer: (A)

129. The value of sin2 1° + sin2 3° + sin2 5° + ….+ sin2 87° + sin2 89° is-

(A)    22

(B)  

(C)    23

(D)  

Answer: (B)

130. The minimum value of cos2 θ + sec2 θ is-

(A)    0

(B)    1

(C)    2

(D)    3

Answer: (C)

131. If cos θ + sec θ = 2 (0° ≤ θ ≤ 90°) then the value of cos 10 θ + sec 11 θ is-

(A)    0

(B)    1

(C)    2

(D)    −1

Answer: (C)

132. In a division sum, the divisor is 10 times the quotient and 5 times the remainder. If the remainder is 40, then the dividend is-

(A)    240

(B)    440

(C)    4040

(D)    4000

Answer: (C)

133. The value of  of is 2/5 is-

(A)    1/3

(B)    2/3

(C)    3/4

(D)    3/5

Answer: (B)

134. Let  The value of x is-

(A)    5 + √5

(B)    0

(C)    9/2

(D)    5

Answer: (C)

135. The total number of prime factors in (25)10 × (24)20 × (26)25 is-

(A)    1000

(B)    120

(C)    150

(D)    140

Answer: (C)

136. The value of  _

(A)    3825

(B)    3845

(C)    3925

(D)    3945

Answer: (A)

137. The value of  is_

(A)    0

(B)    4

(C)    3

(D)    2

Answer: (D)

138. Two numbers are respectively 25% and 40% less than a third number. What per cent of the first is the second?

(A)    60

(B)    70

(C)    40

(D)    80

Answer: (D)

139. A reduction of 20% in the price of sugar enables Mr. Bhadra to buy an extra 5 kg of it for Rs 320. The original rate of sugar is-

(A)    Rs 12 per kg

(B)    Rs 15 per kg

(C)    Rs 16 per kg

(D)    Rs 20 per kg

Answer: (C)

140. In two blends of mixed tea, the ratios of Darjeeling and Assam tea are 4 : 7 and 2 : 5. The ratio in which these two blends should be mixed to get the ratio of Darjeeling and Assam tea in the new mixture as 6 : 13 is-

(A)    2 : 7

(B)    5 : 13

(C)    4 : 14

(D)    22 : 35

Answer: (D)

141. If sec θ – cosec θ = 0, then the value of tan θ + cot θ_

(A)    0

(B)    1

(C)    −1

(D)    2

Answer: (D)

142. From the top of a tree of height 120 m the angle of depression of two boats in the same line with the foot of the tree and on the same side of it are 45° and 60° The distance between the boat is-

(A)    40(3 – √3)m

(B)    40(3√3 – 1)m

(C)    120(√3 – 1)m

(D)    120(3 – √3)m

Answer: (A)

143. (a + b – 2c)3 – (b + c – 2a)3 + (c + a – 2b)3 is equal to-

(A)    (a + b – 2c) (b + c – 2a) (c + a – 2b)

(B)    2(a + b – 2c) (b + c – 2a) (c + a – 2b)

(C)    3(a + b – 2c) (b + c – 2a) (c + a – 2b)

(D)    −3(a + b – 2c) (b + c – 2a) (c + a – 2b)

Answer: (C)

144. The sum of two numbers is 528 and their HCF is 33. How many pairs of such numbers can be?

(A)    1

(B)    2

(C)    3

(D)    4

Answer: (D)

145. A, B and C are three taps connected to a tank. A and B together can fill the tank in 6 hours, B and C together can fill it in hours and A and C together can fill it in In how much time will C alone can fill the tank?

(A)    10 hours

(B)    12 hours

(C)    20 hours

(D)    30 hours

Answer: (D)

146. A and B walk from one corner of a rectangular field to the corner just diagonally opposite. A walks along the diagonal and B walks along the sides, reaching the finishing point together. If the ratio of the length and breadth of t he field is 3 : 4, the ratio of their speeds is-

(A)    3 : 4

(B)    4 : 3

(C)    4 : 5

(D)    5 : 7

Answer: (D)

Directions-(Q. 147 and 148) Study the following table and answer the questions given below-

147. The increase in the number of candidates qualified under Engineering discipline from 2009 to 2010 was-

(A)    28

(B)    30

(C)    46

(D)    49

Answer: (C)

  1. The number of candidates qualified in Science discipline was the lowest in the year-

(A)    2006

(B)    2008

(C)    2009

(D)    2010

Answer: (B)

Directions-(Q. 149 and 150) Study the following graph and answer the questions given below-

149. The value of sales of finished goods in 1999 was approximately what per cent of the average amount invested in raw materials in the year 1997, 1998 and 1999?

(A)    33%

(B)    37%

(C)    49%

(D)    52%

Answer: (*)

150. The maximum difference between the amount invested in raw materials and the value of sales of finished goods in that year was in the year-

(A)    1995

(B)    1996

(C)    1997

(D)    1998

Answer: (C)

For Visually Handicapped Candidate Only

147. A particle goes form a point P to another point Q at an average speed of 60 metre/- minute and comes back to the point P at an average speed of 40 metres/minutes. The average speed of the particle for the whole journey is-

(A)    50 metre/minutes

(B)    48 metre/minutes

(C)    24 metre/minutes

(D)    44 metre/minutes

Answer: (B)

148. A and B are two stations 465 km apart. A car starts from A at 10 a.m. and travels towards B at 65 km/hr. Another car starts from B at 11 am and travels towards A at 35 km/hr. At what time two cars meet each other?

(A)    5 pm

(B)    4 pm

(C)    3 pm

(D)    2 pm

Answer: (C)

149. A car runs along the four sides of a square field at the speed of 20 km/hr, 30 km/hr, 40 km/hr and 60 km/hr. Find the average speed (in km/hr) of the car around the field-

(A)    32

(B)    34

(C)    38

(D)    42

Answer: (A)

150. A boat goes 30 km upstream and 44 km down stream in 10 hrs. In 13 hrs, it can go 40 km upstream and 55 km down stream, then the speed of the boat in still water is-

(A)    5 km/hr

(B)    8 km/hr

(C)    15 km/hr

(D)    13 km/hr

Answer: (B)

Part-IV

General Awareness

151. Who was the founder of the ‘Prarthana Samaj’?

(A)    Ramakrishna Paramahansa

(B)    Swami Vivekananda

(C)    Athmaram Panduranga

(D)    Dayananda Saraswati

Answer: (C)

152. Arrange the following historical events chronologically choosing the correct response:

(I) French Revolution

(II) Glorious Revolution

(III) American War of Independence

(IV) Russian Revolution

(A)    I, II, III, IV

(B)    II, III, I, IV

(C)    II, I, IV, III

(D)    III, II, II, IV

Answer: (B)

153. The Southernmost Point of India is-

(A)    Cape Comorin

(B)    Point Calimere

(C)    Indira Point

(D)    Port Blair

Answer: (C)

154. Which of the following soils is most ideal for paddy cultivation?

(A)    Laterite soil

(B)    Red soil

(C)    Alluvial soil

(D)    Black soil

Answer: (C)

155. The red colour of red soil is due to the rusting of-

(A)    Nitrogen

(B)    Humus

(C)    Iron

(D)    Copper

Answer: (C)

156. Which one of the following rivers crosses the equator two times?

(A)    Cango River

(B)    Amazon River

(C)    Niger River

(D)    Nile River

Answer: (A)

157. Match correctly the following countries and their capitals by choosing the correct response-

List-I (Country)       List-II (Capital)

(a) Argentina                 1. Bogota

(b) Colombia                  2. Harare

(c) Egypt                        3. Buenos Aires

(d) Zimbabwe                4. Cairo

(A)    (a) – 1; (b) – 3; (c) – 4; (d) – 1

(B)    (a) – 3; (b) – 4; (c) – 1; (d) – 2

(C)    (a) – 4; (b) – 1; (c) – 2; (d) – 3

(D)    (a) – 3; (b) – 1; (c) – 4; (d) – 2

Answer: (D)

158. The type of root formed in Betel vine is-

(A)    Stilt Root

(B)    Clinging Root

(C)    Climbing Root

(D)    Prop Root

Answer: (C)

159. Chromosomes are made up of-

(A)    DNA

(B)    Protein

(C)    DNA and Protein

(D)    RNA

Answer: (C)

160. Jersey bull used for cross breeding is an exotic variety from-

(A)    England

(B)    France

(C)    Holland

(D)    Switzerland

Answer: (A)

161. Which one of the following symbiotic associations forms a lichen?

(A)    An algae and a fungus

(B)    An algae and a bryophyte

(C)    A bacterium and a fungus

(D)    A bacterium and a gymnosperm

Answer: (A)

162. Which of the following crop combination is not adopted in mixed cropping in India?

(A)    Wheat and Mustard

(B)    Wheat and Chick Pea

(C)    Rice and Ground nut

(D)    Ground nut and Sun flower

Answer: (D)

163. Daltonism (Protanopia) is the form of colour blindness in which the patient cannot see-

(A)    Green colour

(B)    Red colour

(C)    Blue colour

(D)    All the above colours

Answer: (B)

164. The photoelectric effect is described as the ejection of electrons from the surface of a metal when-

(A)    It is heated

(B)    It is placed in strong electric field

(C)    Electrons of suitable velocity impinge on it

(D)    Light of suitable wavelength falls on it

Answer: (D)

165. X-rays are ……… waves.

(A)    longitudinal

(B)    transverse

(C)    electromagnetic

(D)    elastic

Answer: (C)

166. The shape of a rain drop is spherical due to-

(A)    Viscosity

(B)    Surface tension

(C)    Elasticity

(D)    Gravitation

Answer: (B)

167. The penetrating powers of α, β and γ-radiations, in decreasing order, are-

(A)    α, β, γ

(B)    γ, β, α

(C)    β, α, γ

(D)    γ, α, β

Answer: (B)

168. In MS-Excel, which of the following function is used to find the highest value within the list?

(A)    COUNT

(B)    PROPER

(C)    MAX

(D)    SUM

Answer: (C)

169. While the computer executes a program, the program is held in-

(A)    RAM

(B)    ROM

(C)    Hard Disk

(D)    Floppy Disk

Answer: (C)

170. A hybrid computer is the one having the combined properties of-

(A)    Super of micro computers

(B)    Mini and micro computers

(C)    Analog and digital computers

(D)    Super and mini computers

Answer: (C)

171. Tritium is an isotope of-

(A)    Oxygen

(B)    Hydrogen

(C)    Phosphorous

(D)    Nitrogen

Answer: (B)

172.  The three basis components of an atom are-

(A)    protons, neutrons and ions

(B)    protons, neutrons and electrons

(C)    protium, deuterium and tritium

(D)    protons, neturinos and ions

Answer: (B)

173. Alcohol obtained in the saponification process is-

(A)    Ethyl alcohol

(B)    Methyl alcohol

(C)    Wood spirit

(D)    Glycerol

Answer: (A)

174. The product of equivalent weight and valency of an element is equal to-

(A)    Vapour density

(B)    Specific heat

(C)    Atomic weight

(D)    Molecular weight

Answer: (C)

175. Chlorofluro carbons are widely used in-

(A)    Micro ovens

(B)    Solar heaters

(C)    Washing machines

(D)    Refrigerators

Answer: (D)

176. In which Eco-system Grassland is included ?

(A)    Marine

(B)    Fresh water

(C)    Terrestrial

(D)    Artificial

Answer: (C)

177. The man pollutant responsible for Bhopal Gas Tragedy is-

(A)    Methyl isocyanate

(B)    Bromine

(C)    Chlorofluro carbon

(D)    Chlorine

Answer: (A)

178. Presently the Union Minister of Minority Affairs is-

(A)    Dr. Farooq Abdullah

(B)    Ghulam Nabi Azad

(C)    K. Rahman Khan

(D)    Salman Khurshid

Answer: (C)

179. Who won the Women French Open Title in 2013?

(A)    Maria Sharapova

(B)    Victoria Azarenka

(C)    Serena Williams

(D)    Venus Williams

Answer: (C)

180. Which of the following places of Sikh religious heritage is not in India?

(A)    Nankana Sahib

(B)    Nanded

(C)    Paonta Sahib

(D)    Keshgarh Sahib

Answer: (A)

181. The headquarters of which one of the following organizations is not in Geneva?

(A)    Food and Agricultural Organization

(B)    World Meteorological Organization

(C)    World Health Organization

(D)    World  Trade Organization

Answer: (A)

182. Which one of the following museums in India and its location in wrongly matched?

(A)    National Railway Museum – New Delhi

(B)    The Calico Museum of Textiles – Mumbai

(C)    Salar Jung Museum – Hyderabad

(D)    Napier Museum – Thiruvananthapuram

Answer: (B)

183. Which one of the following European Union countries has its own currency and has not switched over to Euro?

(A)    France

(B)    Germany

(C)    Sweden

(D)    Spain

Answer: (C)

184. The pioneer of modern Hindi Literature is-

(A)    Srinivas Das

(B)    Munshi Premchand

(C)    Devaki Nandan Khatri

(D)    Bharatendu Harishchandra

Answer: (D)

185. Which country has been admitted in July 2013 as the 28th member of the European Union?

(A)    Bulgaria

(B)    Croasia

(C)    Cyprus

(D)    Romania

Answer: (B)

186. Which one of the following is wrongly paired?

Sports                     Ground

(A)    Athletics                 Track

(B)    Base Ball                Court

(C)    Golf                         Course

(D)    Wrestling                Ring

Answer: (B)

187. Which American President was a famous film/T.V. start before he became the President?

(A)    J. F. Kennedy

(B)    Gerald R. Fond

(C)    Richard Nixon

(D)    Ronald Regan

Answer: (D)

188. ‘Value added’ is equal to-

(A)    Output minus input

(B)    Manufacturing profit

(C)    Gross profit

(D)    Capital gain

Answer: (D)

189. Value of a commodity expressed in terms of money is called-

(A)    Price

(B)    Demand

(C)    Utility

(D)    Money value

Answer: (A)

190. Selling cost is an element of-

(A)    Monopolistic Competition

(B)    Perfect Competition

(C)    Pure Competition

(D)    Monopoly

Answer: (A)

191. The opportunity cost of a factor of production is-

(A)    what it earns in its present use

(B)    what it can earn in the long period

(C)    what it can earn in some other use

(D)    the cost of production

Answer: (C)

192. The demand for a product which is wanted for itself is known as-

(A)    Derived Demand

(B)    Industry Demand

(C)    Company Demand

(D)    Direct or Autonomous Demand

Answer: (D)

193. The Parliamentary form of Government in India has been adopted from the-

(A)    American Constitution

(B)    Russian Constitution

(C)    British Constitution

(D)    Swiss Constitution

Answer: (C)

194. Which of these are essentials of democratic system?

(A)    Free and fair elections

(B)    Equality of opportunity

(C)    Protection of rights

(D)    All of the above

Answer: (D)

195. Which Amendment Act is referred as mini constitution?

(A)    7th Constitutional Amendment Act, 1956

(B)    24th Constitutional Amendment Act, 1971

(C)    42d Constitutional Amendment Act, 1976

(D)    44th Constitutional Amendment Act, 1978

Answer: (C)

196. Which Article of the Indian Constitution stipulates that Directive Principles of State Policy shall not be enforceable by any Court?

(A)    Article 31

(B)    Article 38

(C)    Article 37

(D)    Article 39

Answer: (C)

197. Which one of the following languages is not specified in Schedule 8 of the Indian Constitution?

(A)    Sanskrit

(B)    Sindhi

(C)    English

(D)    Nepali

Answer: (B)

198. Which ancient Indian kingdom is wrongly matched below with its capital?

(A)    Maurya – Pataliputra

(B)    Pandya – Madurai

(C)    Pallava – Vellore

(D)    Kakatiya – Warangal

Answer: (C)

199. Which were the two dynasties, which ruled immediately before and after the Khiljis?

(A)    Slave and Lodi

(B)    Saiyyad and Lodi

(C)    Slave and Tughlaq

(D)    Tughlaq and Lodi

Answer: (C)

200. Who among the following did not participate in the 1857 Revolt against the British?

(A)    Tantia Tope

(B)    Tipu Sultan

(C)    Rani Lakshmi Bai

(D)    Nana Saheb

Answer: (B)

SSC Combined Higher Secondary Level (10+2) Examination (1st Shift) Tier-I Held on 10 November, 2013 Question Paper With Answer Key

SSC Combined Higher Secondary Level (10+2) Examination (1st Shift) Tier-I Held on 10 November, 2013
SSC Combined Higher Secondary Level (10+2) Examination (1st Shift) Tier-I Held on 10 November, 2013 Question Paper With Answer Key

Staff Selection Commission Combined Higher Secondary Level (10+2)

1st Shift Tier-I Examination Held on 10 November, 2013

Part-I

General Intelligence

Directions-(Q. 1-7) Select the related word/letters/number from the given alternatives.

1. BADC : XWZY : : QPSR : ?

(A)  LMON

(B)  NMPO

(C)  TUVW

(D)  GHIJ

Answer: (B)

2. 64 : 81 : : 8 : ?

(A)  9

(B)  6

(C)  7

(D)  8

Answer: (A)

3. 175 : 13 : : 108 : ?

(A)  8

(B)  9

(C)  6

(D)  7

Answer: (B)

4. Flower : Petal : : Book : ?

(A)  Author

(B)  Library

(C)  Page

(D)  Contents

Answer: (C)

5. Doctor : Hospital : : ?

(A)  Water : Reservoir

(B)  Farmer : Village

(C)  Plumber : Wrench

(D)  Chef : Kitchen

Answer: (D)

6. Patient : Hospital : : Car : ?

(A)  Garage

(B)  Office

(C)  Hanger

(D)  House

Answer: (A)

7. COMPREHENSION : NOISNEHERPMOC : : BEAUTY : ?

(A)  YTUAEB

(B)  UTYBEA

(C)  EAUTYB

(D)  AUTYBE

Answer: (A)

Directions – (Q. 8-11) Find the odd words/letters/number pair from the given alternatives.

8.

(A)  Simplicity

(B)  Vanity

(C)  Humility

(D)  Modesty

Answer: (B)

9.

(A)  GGHK

(B)  KKLO

(C)  AABE

(D)  MMNO

Answer: (D)

10.

(A)  19-61

(B)  24-76

(C)  12-39

(D)  16-52

Answer: (C)

11.

(A)  Industry : Chairman-cum-Managing Director

(B)  Newspaper : Reporter

(C)  University : Vice-Chancellor

(D)  College : Principal

Answer: (B)

12. Find out the set among the four sets which is like the given set.

Given set is (63, 54, 45)

(A)  (28, 17, 8)

(B)  (36, 45, 33)

(C)  (29, 20, 11)

(D)  (30, 21, 11)

Answer: (C)

13. Which one of the given responses would be a meaningful order of the following?

(1) Rain                (2) Sunlight

(3) Vapour            (4) Cloud

(A)  3, 2, 4, 1

(B)  2, 3, 4, 1

(C)  4, 1, 2, 3

(D)  2, 3, 1, 4

Answer: (C)

14. Arrange the following words as per order in the English dictionary-

(1) Activity          (2) Attention

(3) Arise               (4) Absent

(A)  2, 3, 4, 1

(B)  3, 2, 1, 4

(C)  4, 1, 3, 2

(D)  4, 3, 2, 1

Answer: (C)

15. Which one set of letters when sequentially placed at the gaps in the given letter series shall complete it?

Ba_abab_b_ba_aba

(A)  baab

(B)  bbaa

(C)  abab

(D)  aabb

Answer: (A)

16. In the following letter series, how many P Q S occur in such a way, Q is in the middle and P and S are on any one side?

MDRPQGKPQSLAPQS

TGKPQGMDGPQSTU

(A)  4

(B)  1

(C)  2

(D)  3

Answer: (D)

Directions-(Q. 17-19) A series is given, with one term missing. Choose the correct alternative from the given ones that will complete the series.

17. 10, 18, 28, 40, 54, 70, ?

(A)  87

(B)  88

(C)  85

(D)  86

Answer: (B)

18. 0, 6, 20, 42, ?

(A)  72

(B)  76

(C)  82

(D)  81

Answer: (A)

19. ZIP, WIP, TIP, ?

(A)  QIP

(B)  SIP

(C)  PIP

(D)  RIP

Answer: (A)

20. ‘X’ and ‘Y’ are brothers, ‘B’ is the daughter of ‘A’. ‘A’ is the wife of ‘Y’. What is the relation of ‘B’ to ‘X’?

(A)  Sister-in-law

(B)  Niece

(C)  Daughter

(D)  Sister

Answer: (B)

21. If W = 23, STRONG = 93, then WEAK = ?

(A)  40

(B)  41

(C)  38

(D)  44

Answer: (A)

22. If INTERNET is written as 1425423 and DISTANCE is written as 61738492, then what will CENTRE be written as?

(A)  852431

(B)  291534

(C)  124576

(D)  924352

Answer: (D)

23. If the day before yesterday was Sunday, when will Thursday be?

(A)  Day after tomorrow

(B)  Two days after tomorrow

(C)  Today

(D)  Tomorrow

Answer: (A)

24. Mohan is older than Prabir. Suresh is younger than Prabir. Mihir is plder than Suresh, but younger than Prabir. Who among the four is the youngest?

(A)  Mohan

(B)  Suresh

(C)  Prabir

(D)  Mihir

Answer: (B)

25. A man is 24 years older than his son. In two years, his age will be twice the age of his son. The present age of his son is-

(A)  20 years

(B)  18 years

(C)  14 years

(D)  22 years

Answer: (D)

26. If the seventh person from the front in a queue is the tenth person from behind, how many are there in the queue?

(A)  14

(B)  10

(C)  17

(D)  16

Answer: (D)

27. From the given alternatives select the word which cannot be formed using the letters of the given word.

APPEARANCE

(A)  PEAS

(B)  REAP

(C)  PANE

(D)  NEAR

Answer: (A)

28. If CIGARETTE is coded as GICERAETT, then DIRECTION will be coded as-

(A)  RIDTCEION

(B)  IRDCTIONE

(C)  RIDTCENOI

(D)  NORTECDII

Answer: (C)

29. Some capital letters are given below in the first and number are assigned to each of them in the second line. The numbers are the codes for the letters and vice-versa.

M              O         E          A         S          J           T          Z

3               5          7          6          2          9          4          0

Choose the correct number code for the given set of letters.

J E Z T M S

(A)  9 7 0 5 3 4

(B)  9 7 0 4 3 2

(C)  9 7 0 4 3 5

(D)  9 7 0 3 4 5

Answer: (B)

30. From the given alternatives select the word which can be formed using the letters given in the word.

PRONUNCIATION

(A)  PUNCTUATION

(B)  RATION

(C)  NATURAL

(D)  PRINTING

Answer: (B)

Directions-(Q. 31 and 32) Select the missing number from the given responses.

31.

15   11      20

2     3        1         

5     6        7

35   39      ?

(A)  27

(B)  24

(C)  30

(D)  35

Answer: (A)

32.

3     15      5

5     35      7

9     ?        5

(A)  45

(B)  25

(C)  42

(D)  35

Answer: (A)

Directions-(Q. 33 and 34) Some equations are solved on the basis of a certain system. On the same basis, find out the correct answer for the unsolved equation.

33. If 882 = 20

and           996 = 26,

then          729 = ?

(A)  20

(B)  21

(C)  24

(D)  30

Answer: (A)

34. If 8 – 3 + 4 = 348, 9 – 6 + 8 = 689, then 5 – 9 + 6 = ?

(A)  596

(B)  956

(C)  659

(D)  965

Answer: (D)

35. Medha moves towards South-East for 7 km, then moves towards West for 14 km. Then she moves towards North-West for 7 km and finally moves a distance of 4 m East and stands there. How far is the starting point from where she stands now?

(A)  7 km

(B)  10 km

(C)  3 km

(D)  4 km

Answer: (B)

36. Select the correct combination of mathematical signs to replace ‘∗’ signs and to balance the given equation.

8 ∗ 6 ∗ 7 ∗ 3 ∗ 18

(A)  − + + =

(B)  + − ÷ =

(C)  + − + =

(D)  + + − =

Answer: (D)

37. If ‘÷’ means ‘−’, ‘−’ means ‘×’, ‘×’ means ‘+’ and ‘+’ means ‘÷’, then

20 × 60 ÷ 40 – 20 + 10 = ?

(A)  60

(B)  80

(C)  0

(D)  40

Answer: (C)

38. A man starts walking from his college, walks 10 km towards North, then he t urns to his lefts and walks 10 km. From there he takes a right turn and walks 10 km. In which direction is he facing now?

(A)  East

(B)  West

(C)  South

(D)  North

Answer: (D)

39. Which of the conclusion can be drawn from the given statement?

Statement:

Many creative persons become artists.

(A)  A high level of creativity is needed to become an artist

(B)  Some artists are creative persons

(C)  A creative person will certainly become an artist

(D)  It is not  possible to become an artist without creativity

Answer: (B)

40. Manisha is sitting in between Kamini and Suvarna. Smita is right to Suvarna. Kamini is left to Manisha, Select the correct seating order.

(A)  Kamini, Manisha, Smita, Suvarna

(B)  Kamini, Manisha, Suvarna, Smita

(C)  Kamini, Suvarna, Smita, Manisha

(D)  Manisha, Suvarna, Smita, Kamini

Answer: (B)

41. Two statements are given followed by two conclusions I and II. You have to consider the statements to be true, even if they seem to be at variance from commonly known facts. You have to decide which of the given conclusions, if any, follow from the given statements. Indicate your answer.

Statements:

(1) Teachers are role models of the students.

(2) Teachers are responsible for developing scientific attitude in their students.

Conclusions:

(I) Students do not have scientific attitude.

(II) By and large teachers can influence scientific attitude of the students.

(A)  Both conclusions I and II follow

(B)  Neither conclusions I nor II follows

(C)  Only conclusions I follows

(D)  Only conclusion II follows

Answer: (D)

42. The population of a city doubles every 6 years. If in 2005, the population is 10,562, by which year will the population increase to 42,248?

(A)  2017

(B)  2029

(C)  2011

(D)  2023

Answer: (A)

43. Choose the correct alternative from the given ones t hat will complete the series.

Answer: (A)

44. Select the related figure from the given alternatives.

Answer: (A)

45. Select the figure which is different from the rest three.

Answer: (A)

46. Identify the diagram that best represents the relationship among the classes given below-

Sparrow, Bird, Cat

Answer: (C)

47. Which answer figure will complete the pattern in the question figure?

Answer: (B)

48. Select the missing number from the given responses.

(A)  28

(B)  30

(C)  24

(D)  26

Answer: (D)

49. From the given answer figures, select the one in which the questions figure is hidden/embedded.

Answer: (D)

50. A piece of paper is folded and cut/punched as shown below in the question figures. From the given answer figures, indicate how it will appear when opened.

Answer: (B)

For Visually Handicapped Candidates Only

43. In a row of trees, one tree is the fifth from either end of the row. How many trees are there in the row?

(A)  10

(B)  9

(C)  11

(D)  8

Answer: (B)

44. If ‘a’ is greater than ‘b’ by 2 and ‘b’ is greater than ‘c’ by 10, and a + b + c = 130, then (b + c) – a = ?

(A)  38

(B)  44

(C)  34

(D)  42

Answer: (C)

45. How many 7’s are there in the following number series which are preceded by an even number but not followed by any odd number?

4 3 7 5 2 3 7 2 1 3 6 7 5 4

2 7 4 2 7 1 2 2 7 6 5 7 2

(A)  3

(B)  4

(C)  1

(D)  2

Answer: (D)

46. In a board exam, 40% students fail in Maths 30% in Science and 15% in both. Find the passing percentage.

(A)  35%

(B)  50%

(C)  45%

(D)  65%

Answer: (C)

47. Find the wrong number in the given number series.

852, 213, 53, 13, 3

(A)  53

(B)  213

(C)  852

(D)  13

Answer: (C)

48. If rain is called water, water is called air, air is called cloud, cloud is called sky, sky is called sea, sea is called road, where do the aeroplanes fly?

(A)  Sea

(B)  Cloud

(C)  Water

(D)  Road

Answer: (A)

49. If ‘P’ denotes ‘+’, Q denotes ‘−’, ‘R’ denotes ‘÷’ and ‘S’ denotes ‘×’, then what is the value of the following equation?

18 S 36 R 12 Q 6 P 7 = ?

(A)  648

(B)  25

(C)  115

(D)  55

Answer: (D)

50. A, B, C, D and E share some mangoes among themselves so that A gates one less than B, C gets 4 more than D, E gets 3 more than B,, and D gets as many as B. Who gets the least number of mangoes?

(A)  D

(B)  E

(C)  A

(D)  B

Answer: (C)

Part-II

English Language

Directions-(Q. 51-55) Some parts of the sentences have errors and some are correct. Find out which part of a sentence has an error and blacken the oval (●) corresponding to the appropriate letter (A, B, C). If a sentence is free from error, blacken the oval corresponding to (D) in the Answer Sheet.

51. I had asked him (A)/ how he could go out (B)/if it started raining. (C)/ No error (D)

Answer: (A)

52. Eighty-five thousand rupees (A)/ is a large sum of money (B)/ to earn in a month. (C)/ No error (D)

Answer: (A)

53. His voice shook with emotion, (A)/ and it was so funny to hear him, (B)/ that all we longed to laugh and to cry. (C)/ No error (D)

Answer: (C)

54. It being a rainy day (A)/ I will decide to skip work (B)/ and stay at home. (C)/ No error (D)

Answer: (B)

55. Another baffling change (A)/ that I noticed in him nowdays (B)/ is that he avoids speaking to me. (C)/ No error (D)

Answer: (D)

Directions-(Q. 56-60) Sentences are given with blanks to be filed in with an appropriate word(s). Four alternatives are suggested for each question. Choose the correct alternative out of the four and indicate it by blackening the appropriate oval (●) in the Answer Sheet.

56. The …… of puppies is fond of milk.

(A)  group

(B)  litre

(C)  gang

(D)  litter

Answer: (D)

57. He ……………. his father.

(A)  takes off

(B)  takes out

(C)  takes back

(D)  takes after

Answer: (D)

58. Guard ………… spelling mistakes.

(A)  above

(B)  at

(C)  against

(D)  after

Answer: (C)

59. Make hay while the sum ……… .

(A)  was shining

(B)  has been shining

(C)  shines

(D)  is shining

Answer: (C)

60. The Prime Minister insisted …….. fuel prices in the Cabinet meeting.

(A)  in raising

(B)  at raising

(C)  to raise

(D)  on raising

Answer: (D)

Directions-(Q. 61 and 62) Out of the four alternatives, choose the one which best expresses the meaning of the given word and mark it in the Answer Sheet.

61. Evade-

(A)  Pretend

(B)  Avoid

(C)  Vacate

(D)  Cheat

Answer: (B)

62. Impetuous-

(A)  Hasty           

(B)  Disturbing

(C)  Impious

(D)  Impressive

Answer: (A)

Directions-(Q. 63 and 64) Choose the word opposite in meaning to the given word and mark it in the Answer Sheet.

63. Felicity-

(A)  Mimicry

(B)  Infidelity

(C)  Innocence

(D)  Sorrow

Answer: (D)

64. Indomitable-

(A)  Arrogant

(B)  Cowardly

(C)  Adamant

(D)  Certain

Answer: (B)

Directions-(Q. 65 and 66) Four words are given in the each questions, out of which only one word is correctly spelt. Find the correctly spelt word and mark your answer in the Answer Sheet.

65.

(A)  Detireorate

(B)  Detiorrate

(C)  Deteriarate

(D)  Deteriorate

Answer: (D)

66.

(A)  Acquiantance

(B)  Acquaintance

(C)  Acquaintence

(D)  Acquintance

Answer: (D)

Directions-(Q. 67-69) Four alternatives are given for the Idiom/Phrase bold in the sentence. Choose the alternative which best expresses the meaning of the Idiom/Phrase and mark it in the Answer Sheet.

67. He is in the habit of picking holes in

(A)  causing trouble to

(B)  asking questions about

(C)  delving deep into

(D)  finding fault with

Answer: (D)

68. The member took exception to the secretary’s remark about the current political situation in the State.

(A)  objected to

(B)  granted

(C)  accepted readily

(D)  laughed at

Answer: (A)

69. The die is cast and nothing can be done now.

(A)  The effect has worn out

(B)  The decision has been taken

(C)  The game has been played

(D)  There has been a failure

Answer: (B)

Directions-(Q. 70-72) Out of the four alternatives choose the one which can be substituted for the given words/sentence.

70. One who lives/survives on others/other lives-

(A)  Parasite

(B)  Refugee

(C)  Expatriate

(D)  Pesticide

Answer: (A)

71. One who possesses many talents-

(A)  Dexterous

(B)  Attractive

(C)  Versatile

(D)  Unique

Answer: (C)

72. Detailed plan of journey-

(A)  Schedule

(B)  Itinerary

(C)  Travelogue

(D)  Travel-kit

Answer: (B)

Directions-(Q. 73 and 74) A part of the sentence is bold. Below are given alternatives to the bold part at (A), (B), (C) which may improve the sentence. Choose the correct alternative. In case no improvement is needed your answer is (D). Mark your answer in the Answer Sheet.

73. The Sat on the tree to protect herself from the rain.

(A)  sat before

(B)  sat under

(C)  sat upon

(D)  No improvement

Answer: (B)

74. He argued endlessly with him on the subject.

(A)  argued all the time

(B)  argued continuously

(C)  argued on and on

(D)  No improvement

Answer: (B)

Directions-(Q. 75-79) A sentence has been given in Active/Passive Voice. Out of the four alternatives suggested, select the one which best expresses the same sent3ence in Passive/Active Voice and mark your answer in the Answer Sheet.

75. Her manners pleased us very much.

(A)  We pleased her very much with manners

(B)  We were very much pleased with her manners

(C)  We were very much pleased on her manners

(D)  We are very much pleased with her manners

Answer: (B)

76. They proposed to hold a meeting.

(A)  A meeting should be proposed to be held

(B)  It was proposed to hold a meeting

(C)  A meeting had to be proposed to be held by them

(D)  It was proposed by them to hold a meeting

Answer: (D)

77. I shall post these letters tomorrow.

(A)  These letters are to be posted tomorrow

(B)  These letters have to be posted tomorrow

(C)  These letters would be posted tomorrow

(D)  These letters will be posted tomorrow

Answer: (D)

78. A massive search operation has been launched to nab the suspects.

(A)  The police launched  a massive search operation to nab the suspects

(B)  The police had been launched a massive search operation to nab the suspects

(C)  The police had launched a massive search operation to nab the suspects

(D)  The police have launched a massive search operation to nab the suspects

Answer: (D)

79. The driver was blowing the horn.

(A)  The horn has been blown by the driver

(B)  The horn is being blown by the driver

(C)  The horn was blown by the driver

(D)  The horn was being blown by the driver

Answer: (D)

Directions-(Q. 80-84) A sentence has been given in Direct/Indirect. Out of the four alternatives suggested, select the one which best expresses the same in Indirect/Direct and mark your answer in the Answer Sheet.

80. I told her, “It was raining last night when you left.”

(A)  I told her that it has been raining the night before when she left

(B)  I told her that it had been raining last night when she had left

(C)  I told her that it had been raining the previous night when she had left

(D)  I told her that it was raining last night when she left

Answer: (C)

81. She said, “I shall try to bring you the books tomorrow.”

(A)  She said that she would try to bring me the books tomorrow

(B)  She said that she should try to bring me the books tomorrow

(C)  She said that she should try to bring me the books the next day

(D)  She said that she would try to bring me the books the next day

Answer: (D)

82. He said, “Alas ! The little puppy is run over by the car.”

(A)  He exclaimed sadly that the little puppy had been run over by the care

(B)  He exclaimed sadly that the little puppy is run over by the car

(C)  He exclaimed, alas, the little puppy was run over by the car

(D)  He exclaimed sadly that the little puppy was run over by the car

Answer: (D)

83. He said, “He took tea in the morning.”

(A)  He said the he took tea in the morning

(B)  He said that he would take tea in the morning

(C)  He said that he did take tea in the morning

(D)  He said that he had taken tea in the morning

Answer: (D)

84. He requested the boss to let him go on with his project.

(A)  He said to the boss, “Let me go on with my project please.”

(B)  He told the boss, “Go on with my project please.”

(C)  He said to the boss, “You better let me go on with my project.

(D)  He told the boss, “Please go on with my project.”

Answer: (A)

Directions-(Q. 85-90) The Ist and the last sentences of the passage/parts of the sentence are numbered 1 to 6. The rest of the passage/sentence is split into four parts and named P, Q, R and S. These four parts are not given in their proper order. Read the passage/sentence and find out which of the four combinations is correct. Then find the correct answer and indicate it by blackening the appropriate oval (•) in the Answer Sheet.

85. (1) Most men spend most of their lives looking after and waiting upon machines.

(P) They must be fed well and kept at right temperatures.

(Q) They might burst with rage and blow up causing ruin.

(R) If they do not get their meals they refuse to work.

(S) Machines are very stern masters.

(6) So, men need to wait upon machines attentively.

(A)  QSPR

(B)  RPQS

(C)  SPRQ

(D)  PQRS

Answer: (C)

86. (1) Sappho was one of the greatest and earliest of women poets.

(P) Whatever we know about her poetry today is from the quotations found in the works of others.

(Q) She lived on the island of Lesbos in the late 600’s BC.

(R) Most of Sappho’s works-about 10 books of verse have been destroyed.

(S) Only one of the her poems has survived in its complete form.

(6) Without doubt, she was one of the best lyric poets of Ancient Greece.

(A)  QSPR

(B)  QRSP

(C)  PRQS

(D)  PQSR

Answer: (B)

87. (1) I realize that peace and

(P) security are rightful aims,

(Q) violent adventure is probably

(R) and that my own desire for

(S) merely an adaptation to the

(6) age in which I live.

(A)  PRQS

(B)  SPQR

(C)  PQRS

(D)  SRQP

Answer: (A)

88. (1) Books have been present since the time the first scripts were formed-about 5000 years ago..

(P) The books of that time looked different from the ones available today.

(Q) Then an important invention, letter press printing, also known as ‘black art’ changed the world.

(R) Later, the  books came in the form of rollers, or texts were stapled together and covered with a wooden book cover.

(S) Initially, people bound the small clay tablets together with leather bands.

(6) There was no longer the need to write text by hand; instead copies of text could be made with the help of a printing press.

(A)  SPRQ

(B)  SRQP

(C)  PRQS

(D)  PSRQ

Answer: (D)

89. (1) The harnessing of our rivers

(P) is a great national problem

(Q) the waters of which

(R) which must be considered

(S) now mostly run to waste

(6) and dealt with on national lines.

(A)  QSRP

(B)  RPSQ

(C)  RPQS

(D)  QSPR

Answer: (D)

90. (1) Many people drive

(P) hopelessly jammed

(Q) their own cars to work

(R) the roads would become

(S) but if everyone did this

(6) especially during rush-hours.

(A)  RPSQ

(B)  QSPR

(C)  QSRP

(D)  PRQS

Answer: (C)

Directions-(Q. 91-95) In the following passage some of the words have been left out. Read the passage carefully and choose the correct answer to each question out of the four alternatives and fill in the blanks.

     Meeting old school friends can be a strange experience. Some have changed so much that you can …(91)… recognize them : they speak with a different …(92)… are interested in different things, and all you can do is make …(93)… talk and hope they’ll go soon. Others, though you might have been out of …(94)… with them for years, are just the same as they always were; it’s …(95)… if you saw them yesterday.

91.

(A)  hardly

(B)  easily

(C)  nearly

(D)  almost

Answer: (A)

92.

(A)  way

(B)  tongue

(C)  language

(D)  accent

Answer: (D)

93.

(A)  silly

(B)  gossip

(C)  small

(D)  little

Answer: (A)

94.

(A)  sound

(B)  feel

(C)  sight

(D)  touch

Answer: (C)

95.

(A)  so

(B)  as

(C)  just

(D)  like

Answer: (B)

Directions- (Q. 96-100) You have a brief passage with 5 questions following the passage. Read the passage carefully and choose the best answer to each question out of the four alternatives and mark it by blackening the appropriate oval (•) in the Answer Sheet.

     Earth is the only planet so far known with the suitable environment for sustaining life. Land, water, air, plants and animals are the major components of the global environment. Population, food and energy are the three fundamental problems facing mankind. Unemployment, inflation, crowding, dwindling resources and pollution are all due to the factors like increasing population, high standard of living, deforestation, etc.

     Man has been tampering with the Ecosphere for a very long time and is forced to recognize that environmental resources are scarce. Environmental  problems are really social problems. They begin with people as cause and end with people as victims. Unplanned  use of resources has resulted in the depletion of fossils, fuels, pollution of air and water, deforestation which has resulted in ecological imbalance and draining away of national wealth through heavy expenditure on oil and power generation.

96. We face three fundamental problems that are-

(A)  population, inflation and food

(B)  population, food and energy

(C)  inflation, deforestation and unemployment

(D)  population, deforestation and energy

Answer: (B)

97. Increase population causes-

(A)  dwindling resources

(B)  unemployment, inflation, crowding, dwindling resources and pollution

(C)  unemployment and crowding

(D)  inflation and pollution

Answer: (B)

98. National wealth is drained away by spending heavily on-

(A)  water and power generation

(B)  oil and power generation

(C)  power generation

(D)  fuels

Answer: (B)

99. The three major components of the global environment are-

(A)  land, water and air

(B)  plants, animals and mankind

(C)  food energy and population

(D)  high standard of living, crowding and inflation

Answer: (A)

100. Depletion of fossils and fuels, pollution of air and water and deforestation will never occur in case of-

(A)  unplanned use of resources

(B)  over use of resources

(C)  improper use of resources

(D)  planned use of resources

Answer: (D)

Part-III

Quantitative Aptitude

101. If a number is increased by 25% and the resulting number is decreased by 25%, then the percentage increase or decrease finally is-

(A)  increased by  

(B)  increased by 6%

(C)  no change

(D)  decreased by  

Answer: (D)

102. A reduction of 20% in the price of rice enables a customer to purchase 12.5 kg more for Rs 800. The original price of rice per kg is-

(A)  Rs 12

(B)  Rs 15

(C)  Rs 14

(D)  Rs 16

Answer: (D)

103. In two alloys A an B, the ratio of zinc to tin is 5 : 2 and 3 : 4 respectively. 7 kg of the alloy A and 21 kg of the alloy B are mixed together to form a new alloy. What will be the ratio of zinc and tin in the new alloy?

(A)  2 : 3

(B)  1 : 1

(C)  2 : 1

(D)  1 : 2

Answer: (B)

104. If A : B = 3 : 4 and B : C = 6 : 5, then C : A is-

(A)  8 : 9

(B)  9 : 8

(C)  10 : 9

(D)  9 : 10

Answer: (C)

105. If a, b are rational and 

 then the values of a, b are respectively.

(A)  2, 1

(B)  2, 3

(C)  1, 2

(D)  1, 3

Answer: (C)

106. The ratio of boys and girls in a college is 5 : 3. If 50 boys leave the college and 50 girls join the college, the ratio becomes 9 : 7. The number of boys in the college is-

(A)  500

(B)  600

(C)  300

(D)  400

Answer: (A)

107. When n is divided by 6, the remainder is 4, When 2n is divided by 6, the remainder is-

(A)  4

(B)  1

(C)  2

(D)  0

Answer: (C)

108. The value of  is-

(A)  3/20

(B)  7/20

(C)  1/10

(D)  3/5

Answer: (A)

109. Let  Then we have-

(A)  a = 18

(B)  a = 9

(C)  a < 18 but a ≠ 9

(D)  a > 18

Answer: (C)

110. The smallest among the numbers 2250, 3150, 5100 and 4200 is-

(A)  3150

(B)  2250

(C)  4200

(D)  5100

Answer: (D)

111. The greatest of the following numbers 0.16, √16, (0.16)2, 0.04 is-

(A)  0.04

(B)  (0.16)2

(C)  0.16

(D)  √0.16

Answer: (D)

112. If  part of a journey takes 10 minutes, then to complete 3/5th of that journey, it will take-

(A)  48 minute

(B)  36 minute

(C)  40 minute

(D)  45 minute

Answer: (A)

113. Two trains 180 metre and 120 metre in length are running towards each other on parallel tracks, one at the rate 65 km/hr and another at 55 km/hr. In how many seconds will they clear of each other from the moment they meet?

(A)  12

(B)  15

(C)  6

(D)  9

Answer: (D)

114. Three men can complete a piece of work in 6 days. Two days after they started the work, 3 more men joined them. How many days will they take to complete the remaining work?

(A)  3 days

(B)  4 days

(C)  1 day

(D)  2 days

Answer: (D)

115. If  then  equals-

(A)  4

(B)  8

(C)  1

(D)  2

Answer: (D)

116. If  then the value of a3 + b3 is-

(A)  1

(B)  2

(C)  0

(D)  −1

Answer: (C)

117. If a2 + b2 + c2 = ab + bc + ca, then  is equal to-

(A)  3

(B)  4

(C)  1

(D)  2

Answer: (D)

118. If the graphs of the equations x + y = 0 and 5y + 7x = 24 intersect at (m, n), then the value of m + n is-

(A)  0

(B)  −1

(C)  2

(D)  1

Answer: (A)

119. The average age of a class of 35 students is 15 years. If the teacher’s age is also included the average age increases by one year. Furthermore, if the average age of the teacher’s family having a wife and a son is 40 years and the son’s age is 80% less than his mother’s age, then the age of the teacher’s wife is-

(A)  47.5 years

(B)  55.5 years

(C)  57.5 years

(D)  50 years

Answer: (C)

120.  If the amount is the sum after 3 years at compound interest compounded annually, then the rate to interest per annum is-

(A)    

(B)   

(C)  25%

(D)  50%

Answer: (D)

121. A shopkeeper sells an article at 15% gain. Had he sold it for Rs 18 more, the would have gained 18%. The cost price (in Rs) of the article is-

(A)  600

(B)  350

(C)  540

(D)  318

Answer: (A)

122. A cloth merchant on selling 33 metre of cloth obtains a profit equal to the selling price of 11 metre of cloth. The profit is-

(A)  50%

(B)  11%

(C)  40%

(D)  22%

Answer: (A)

123. A single discount equivalent to successive discounts of 20%, 10% and 5% is-

(A)  31.6%

(B)  36.1%

(C)  35%

(D)  35.6%

Answer: (A)

124. A, B and C started a business with their investments in the ratio 1 : 2 : 4. After 6 months A increased his capital by 50% and B invested twice the amount as before, while C withdrew 1/4th of his own investment. The ratio of their profits at the end of the year was-

(A)  6 : 9 : 17

(B)  5 : 14 : 16

(C)  10 : 5 : 9

(D)  5 : 12 : 14

Answer: (D)

125. P and Q are centres of two circles with radii 9 cm and 2 cm respectively, where PQ = 17 cm. R is the centre of another circle of radius x cm, which touches each of the above two circles externally. If ∠PRQ = 90°, then the value of x is-

(A)  7 cm

(B)  8 cm

(C)  4 cm

(D)  6 cm

Answer: (D)

126. The area of a rhombus is 216 cm2 and the length of its one diagonal is 24 cm. The perimeter (in cm) of the rhombus is-

(A)  120

(B)  100

(C)  52

(D)  60

Answer: (D)

127. The perimeter of an isosceles triangle is 544 cm and each of the equal sides in 5/6 times the base. What is the area (in cm2) off the triangle?

(A)  31872

(B)  13872

(C)  38172

(D)  18372

Answer: (B)

128. A metallic sphere of radius 10.5 cm is melted and then recast into small cones each of radius 3.5 cm and height 3 cm. The number of cones thus formed is-

(A)  112

(B)  126

(C)  140

(D)  132

Answer: (B)

129. If the ratio of the diameters of two right circular cones of equal height be 3 : 4, then the ratio of their volumes will be-

(A)  16 : 9

(B)  27 : 64

(C)  3 : 4

(D)  9 : 16

Answer: (D)

130. A right pyramid stands on a square base of diagonal 10√2 cm. If the height of the pyramid is 12 cm, the area (in cm2) of its slant surface is-

(A)  360

(B)  260

(C)  520

(D)  420

Answer: (B)

131. If x = 5 – √21, then the value of  is-

(A)    

(B)   

(C) 

(D)    

Answer: (D)

132. In ∆ ABC, the internal bisectors of ∠ABC and ∠ACB meet at I and ∠BAC = 50°. The measure of ∠BIC is-

(A)  125°

(B)  130°

(C)  105°

(D)  115°

Answer: (D)

133. BL and CM are medians of ∆ABC right-angled at A and BC = 5 cm. If  then the length of CM is-

(A)  10√2 cm

(B)  4√5 cm

(C)  2√5 cm

(D)  5√2 cm

Answer: (C)

134. In ∆ABC, D and E are the points of sides AB and BC respectively such that DE || AC and AD : DB = 3 : 2. The ratio of area of trapezium ACED to that of ∆BED is-

(A)  4 : 21

(B)  21 : 4

(C)  4 : 15

(D)  15 : 4

Answer: (B)

135. A quadrilateral ABCD circumscribes a circle and AB = 6 cm, CD = 5 cm and AD = 7 cm. The length of side BC is-

(A)  3 cm

(B)  6 cm

(C)  4 cm

(D)  5 cm

Answer: (C)

136. From a tower 125 metre high , the angles of depression of two objects, which are in horizontal line through the base of the tower, are 45° and 30° and they are on the same side of the tower. The distance (in metre) between the objects is-

(A)  125/(√3 – 1)

(B)  125(√3 + 1)

(C)  125√3

(D)  125(√3 – 1)

Answer: (D)

137. If each interior angle of a regular polygon is 150°, the number of sides of the polygon is-

(A)  10

(B)  15

(C)  8

(D)  None of these

Answer: (D)

138. If the altitude of a right prism is 10 cm and its base is an equilateral triangle of side 12 cm, then its total surface area (in cm2) is-

(A)  360

(B)  72(5 + √3)

(C)  (5 + 3√3)

(D)  36√3

Answer: (B)

139. The value of the tan 10° tan 15° tan 75° tan 80° is-

(A)  −1

(B)  2

(C)  0

(D)  1

Answer: (D)

140. The minimum value of 4 tan2 θ + 9 cot2 θ is equal to-

(A)  12

(B)  13

(C)  0

(D)  5

Answer: (A)

141. If sin 7x = cos 11x, then the value of tan 9x + cot 9x is-

(A)  3

(B)  4

(C)  1

(D)  2

Answer: (D)

142. If tan2 α = 1 + 2 tan2 β(α, β are positive acute angles), then √2 cos α – cos β is equal to-

(A)  1

(B)  −1

(C)  0

(D)  √2

Answer: (C)

143. Two pipes X and Y can fill a cistern in 24 minute and 32 minute respectively. If both the pipes are opened together, then after how much time (in minutes) should Y be closed so that the tank is full in 18 minute?

(A)  6

(B)  5

(C)  10

(D)  8

Answer: (D)

144. The length and breadth of a rectangular field are in the ratio 7 : 4. A path 4 m wide running all round outside has an area of 416 m2. The breadth (in m) of the field is-

(A)  15

(B)  16

(C)  28

(D)  14

Answer: (B)

145. If a + b + c = 0, then a3 + b3 + c3 is equal to-

(A)  2 abc

(B)  3 abc

(C)  a +  b + c

(D)  abc

Answer: (B)

146. Two numbers 11284 and 7655, when divided by a certain number of three digits, leaves the same remainder. The sum of digits of such a three-digit number is-

(A)  10

(B)  11

(C)  8

(D)  9

Answer: (B)

Directions-Study the table and answer questions no. 147 and 148.

147. Average height (in cm) of the girls whose heights are 155 cm and above in about-

(A)  160.4

(B)  162.6

(C)  158.7

(D)  159.8

Answer: (D)

148. The number of girls whose height is above 150 cm is-

(A)  86

(B)  97

(C)  22

(D)  29

Answer: (C)

Directions-Student’s strength of a college in Arts, Science and Commerce from 2004-05 to 2007-08 sessions are shown in the following bar graph. Study the graph and answer questions no. 149 and 150.

149. The increase in Science students in 2007-08 over 2006-07 was-

(A)  16.7%

(B)  18.2%

(C)  10.1%

(D)  11.1%

Answer: (D)

150. The ratio of average number of students in Arts to the average number of students in Commerce is-

(A)  7 : 4

(B)  48 : 35

(C)  12 : 5

(D)  10 : 7

Answer: (C)

For Visually Handicapped Candidates Only

147. If  then the value of x72 + x60 + x48 + x36 is-

(A)  4

(B)  8

(C)  0

(D)  2

Answer: (A)

148. If ax = b, by = c, cz = a, then the value of xyz = ?

(A)  abc

(B)  1/abc

(C)  0

(D)  1

Answer: (D)

149. Let a = ∛2 + ∛3 + 1, then the value of  is-

(A)  163

(B)  164

(C)  165

(D)  162

Answer: (D)

150. If  and  then the value of x3 + y3 is-

(A)  a3

(B)  b3

(C)  a

(D)  b

Answer: (B)

Part-IV

General Awareness

151. Where do we find the ideals of Indian democracy in the Constitution?

(A)  Part IV

(B)  Part I

(C)  The Preamble

(D)  Part III

Answer: (C)

152. Comptroller and Auditor General of India is appointed by the-

(A)  Finance Minister

(B)  Lok Sabha

(C)  Prime Minister

(D)  President

Answer: (D)

153. Which Article of the Indian Constitution directs the State Governments to organize Village Panchayats?

(A)  Article 40

(B)  Article 51

(C)  Article 32

(D)  Article 37

Answer: (A)

154. The Attorney General of India has the right of audience in-

(A)  any Sessions Court

(B)  any Court of Law within India

(C)  the Supreme Court

(D)  any High Court

Answer: (B)

155. The capital of the ancient Chola kingdom was-

(A)  Thanjavur

(B)  Madurai

(C)  Uraiyur

(D)  Kaveripoompattinam

Answer: (C)

156. Arrange the dynasties of Delhi Sultnate given below in chronological order-

(1) Khilji            (2) Tughlaq

(3) Sayyad         (4) Slave

(A)  1, 2, 3, 4

(B)  4, 1, 2, 3

(C)  4, 1, 3, 2

(D)  1, 4, 2, 3

Answer: (B)

157. Production of a commodity mostly through the natural process is an activity of-

(A)  Tertiary Sector

(B)  Technology Sector

(C)  Primary Sector

(D)  Secondary Sector

Answer: (C)

158. The law of Demand is based on-

(A)  Supplier’s preference

(B)  Consumer’s preference

(C)  Manufacturer’s preference

(D)  Seller’s preference

Answer: (B)

159. Which one of the following is not a method for computing GNP?

(A)  Savings Approach

(B)  Value Added Approach

(C)  Income Approach

(D)  Expenditure Approach

Answer: (A)

160. A part of National Debt known as External Debt is the amount-

(A)  borrowed by its government from abroad

(B)  lent by its government to foreign government

(C)  borrowed by its citizens from abroad

(D)  lent by its citizens to foreign governments

Answer: (A)

161. The non-expenditure costs which arise when the producing firm itself owns and supplies certain factors of production are-

(A)  Implicit costs

(B)  Replacement costs

(C)  Explicit costs

(D)  Original costs

Answer: (A)

162. Who said “Rama Rajya through Grama Rajya”?

(A)  Jayaprakash Narayan

(B)  Jawaharlal Nehru

(C)  Mahatma Gandhi

(D)  Vinoba Bhave

Answer: (C)

163. Which of the biomes is called the ‘Bread Basket’ of the world?

(A)  Mediterranean

(B)  Tropical Savanna

(C)  Mid-latitude grasslands

(D)  Taiga

Answer: (C)

164. Asia and North America are separated by-

(A)  Bering Strait

(B)  Cook Strait

(C)  Bass Strait

(D)  Strait of Dover

Answer: (A)

165. Phototropic movement is controlled by-

(A)  Cytokinin

(B)  Ethylene

(C)  Auxin

(D)  Gibberellin

Answer: (C)

166. Lactogenic hormone is secreted by-

(A)  Ovary

(B)  Pituitary

(C)  Mammary glands

(D)  Placenta

Answer: (B)

167. An organism, which can monitor air pollution is-

(A)  Algae

(B)  Fungi

(C)  Bacteria

(D)  Lichen

Answer: (D)

168. In the human body, which of the following organs is responsible for water balance?

(A)  Kidneys

(B)  Lungs

(C)  Heart

(D)  Liver

Answer: (A)

169. Chlorophyll containing autotrophic thallophytes is called as-

(A)  Fungi

(B)  Bryophytes

(C)  Algae

(D)  Lichens

Answer: (C)

170. Which was the earliest settlement of the Dutch in India?

(A)  Surat

(B)  Ahmedabad

(C)  Masulipatnam

(D)  Pulicat

Answer: (C)

171. During British rule, who was instrumental for the introduction of the Ryotwari system in the then Madras Presidency?

(A)  Thomas Munro

(B)  John Lawrence

(C)  Macartney

(D)  Elphinstone

Answer: (A)

172. Who amongst the following was not associated with the Unification of Italy?

(A)  Mussolini

(B)  Mazzini

(C)  Cavour

(D)  Garibaldi

Answer: (A)

173. The Greater Himalayas is otherwise called as-

(A)  Assam Himalayas

(B)  Siwaliks

(C)  Himadirs

(D)  Sahyadri

Answer: (C)

174. The cup-shaped mouth of the volcano is-

(A)  Crater

(B)  Cinder cone

(C)  Focus

(D)  Epicentre

Answer: (A)

175. The cool temperate grasslands of South America are known as-

(A)  Veld

(B)  Savannah

(C)  Pampas

(D)  Prairies

Answer: (C)

176. A computer programming language often used by children is-

(A)  BASIC

(B)  JAVA

(C)  LOGO

(D)  PILOT

Answer: (C)

177. A portable, personal computer small enough to fit on your lap is called is-

(A)  Mainframe computer

(B)  Workstation

(C)  Note-book computer

(D)  PDA

Answer: (C)

178. Assembler is a program that translates the program from-

(A)  machine to low-level

(B)  low-level to high-level

(C)  high-level to assembly

(D)  assembly to machine

Answer: (D)

179. ‘Table sugar’ is which type of sugar?

(A)  Glucose

(B)  Sucrose

(C)  Fructose

(D)  Galactose

Answer: (B)

180. An alloy used in making heating elements for electric heating device is-

(A)  Nichorme

(B)  German Silver

(C)  Solder

(D)  Alloy Steel

Answer: (A)

181. The degree of dissociation of an electrolyte depends on-

(A)  atmospheric pressure

(B)  method of dissolution

(C)  dilution

(D)  impurities

Answer: (C)

182. Chlorophyll contains-

(A)  Cobalt

(B)  Zinc

(C)  Iron

(D)  Magnesium

Answer: (D)

183. The gas causing acid rain in an industrial area is-

(A)  Sulphur dioxide

(B)  Methane

(C)  Carbon dioxide

(D)  Carbon monoxide

Answer: (A)

  1. Match correctly the insect vectors in List-I with the diseases transmitted by them given in List-II-

List-I

(a) Anopheles (female)

(b) Culex

(c) Sand fly

(d) Tse-tse fly

List-II

(1) Kala-azar

(2) Sleeping sickness

(3) Filariasis

(4) Malaria

(A)  (a)-3; (b)-2; (c)-1; (d)-4

(B)  (a)-4; (b)-3; (c)-1; (d)-2

(C)  (a)-1; (b)-4; (c)-2; (d)-3

(D)  (a)-2; (b)-1; (c)-4; (d)-3

Answer: (B)

185. A white and smooth surface is-

(A)  good absorber and bad reflector of heat

(B)  bad absorber and bad reflector of heat

(C)  good absorber and good reflector of heat

(D)  bad absorber and good reflector of heat

Answer: (D)

186. When a body is immersed in a liquid, the forces acting on it are-

(A)  Upthrust

(B)  Weight

(C)  Mass

(D)  Both (A) and (B)

Answer: (D)

187. When two semiconductor of p and n-type are brought in contact, they form p-n junction which acts like a/an-

(A)  Rectifier

(B)  Amplifier

(C)  Conductor

(D)  Oscillator

Answer: (D)

188. The mass of body measured by a physical balance in a lift at rest is found to be m. If the lift is going up with an acceleration a, its mass will be measured as-

(A)  m

(B)  zero

(C)   

(D)   

Answer: (A)

189. Among the following political leaders of South India, who has not acted in any film?

(A)  N. T. Rama Rao

(B)  M. G. Ramachandran

(C)  C. N. Annadurai

(D)  Jayalalitha

Answer: (C)

190. Which one of the following days is not observed on a fixed date every year?

(A)  International Friendship Day

(B)  World Habitat Day

(C)  World Environment Day

(D)  International Women’s Day

Answer: (D)

191. Which one of the following novels was a source of inspiration for the freedom fighters in India?

(A)  Rangbhoomi

(B)  Padmarag

(C)  Pariksha Guru

(D)  Anandmath

Answer: (D)

192. Which prestigious award was given in 2013 to Aparajita Datta for her outstanding contribution for the conservation of hornbills?

(A)  Whitley Award

(B)  Rajiv Gandhi Ecology Award

(C)  Magsaysay Award

(D)  Right Livelihoood Award

Answer: (A)

193. Which presently serving State Chief Minister has been in office continuously for the longest period?

(A)  Bhupinder Singh Hooda

(B)  Manik Sarkar

(C)  Sheila Diskhit

(D)  Narendra Modi

Answer: (B)

194. Which one of the following monuments is the first inhabited World Heritage Monument?

(A)  Jaisalmer Fort

(B)  Amber Fort

(C)  Agra Fort

(D)  Red Fort

Answer: (A)

195. The famous three P’s of environmental awareness are-

(A)  Population, Politics, Price

(B)  Population, Poverty, Pollution

(C)  People, Poverty, Politics

(D)  Power, Production, Pollution

Answer: (B)

196. The stagnant water at the bottom of a lake is called-

(A)  Metalimnion

(B)  Hypolimnion

(C)  Epilimnion

(D)  Mesolimnion

Answer: (A)

197. Shri Jyotiraditya Madhavarao Scindia is the Minister of State with independent charge of the Ministry of-

(A)  Power

(B)  Information and Broadcasting

(C)  Tourism

(D)  Chemical and Fertilizers

Answer: (A)

198. Who won the Wimbledon Men’s Singles in 2013?

(A)  Juan Martin Del Potro

(B)  Fernando Verdasco

(C)  Andy Murray

(D)  Novak Djokovic

Answer: (C)

199. Who won the Silver Medal for the Women’s 400 metre race in the Asian Athletic Championships 2013?

(A)  Tintu Luka

(B)  Mayookha Johny

(C)  Zhao Yanmin

(D)  M.R. Poovamma

Answer: (D)

200. Where is Taksim Square, which witnessed in 2013 prolonged massive protests against the redevelopment of Gezi Park?

(A)  Istanbul

(B)  Teheran

(C)  Ankona

(D)  Cairo

Answer: (A)

SSC Combined Higher Secondary Level (10+2) Examination (1st Shift) Tier-I Held on 9 November, 2014 Question Paper With Answer Key

SSC Combined Higher Secondary Level (10+2) Examination (1st Shift) Tier-I Held on 9 November, 2014
SSC Combined Higher Secondary Level (10+2) Examination (1st Shift) Tier-I Held on 9 November, 2014 Question Paper With Answer Key

Staff Selection Commission Combined Higher Secondary Level (10+2)

Examination (1st Shift) Tier-I Held on 9 November, 2014

Paper-I General Intelligence

Directions-(Q. 1 to 5) A series is given, with one term missing. Choose the correct alternative from the given ones that will complete the series.

1. CE, GI, KM, OQ, ?

(A)  TW

(B)  TV

(C)  SU

(D)  RT

Answer: (C)

2. R, O, L, I, F, ?

(A)  C

(B)  A

(C)  E

(D)  I

Answer: (A)

3. 3, 15, 4, 16, 5, 17, 6, ?, 7.

(A)  12

(B)  13

(C)  15

(D)  18

Answer: (D)

4. 68, 81, 96, ?, 132.

(A)  105

(B)  110

(C)  113

(D)  130

Answer: (C)

5. 121, 253, 374, 495, ?.

(A)  565

(B)  523

(C)  5116

(D)  5102

Answer: (C)

6. Which one set of letters when sequentially placed at the gaps in the given letter series shall complete it?

_cb_ca_bacb_ca_bac_d

(A)  b a d d d b

(B)  b b b d d d

(C)  a d d d d b

(D)  a d d b b b

Answer: (C)

7. Mani is double the age of Prabhu. Ramona is half the age of Prabbu. If Mani is sixty, find out the age of Ramona-

(A)  20

(B)  15

(C)  10

(D)  24

Answer: (B)

8. A family consisted of a man, his wife, his three sons, their wives and three children in each son’s family. How many members are there in the family?

(A)  12

(B)  13

(C)  15

(D)  17

Answer: (D)

9. From the given alternatives select the word which cannot be formed using the letters of the given word-

INFLATIONARY

(A)  FLAIR

(B)  FAULTY

(C)  NATIONAL

(D)  RATION

Answer: (B)

10. If PALE is coded as 2134, EARTH is coded as 41590, how is PEARL coded as?

(A)  29530

(B)  24153

(C)  25413

(D)  25430

Answer: (B)

11. If the word PRINCIPAL is written as LAPICNIRP, how ADOLESCENCE can be written in that code?

(A)  ECNCESELODA

(B)  ECNECSLEODA

(C)  ECNSCEELODA

(D)  ECNECSELODA

Answer: (D)

12. Let, J = 1, K = 2, L = 5, M = 7, N = 11, O = 13, P = 17.

Find the letter to be inserted in the box in the relation given-

(N × □ + M) ÷ K = 31

(A)  L

(B)  P

(C)  J

(D)  O

Answer: (A)

13. Some equations are solved on the basis of a certain system. On the same basis, find out the correct answer for the unsolved equation-

2 × 3 × 4 = 432, 5 × 6 × 7 = 765

7 × 8 × 9 = 987, 2 × 5 × 7 = ?

(A)  572

(B)  752

(C)  725

(D)  257

Answer: (B)

14. The overall rainfall in certain region of India decreases year after Find out from the data the trend in decrease-

(A)  6 mm

(B)  7 mm

(C)  5 mm

(D)  8 mm

Answer: (C)

Directions-(Q. 15 and 16) Select the missing number from the given responses.

15. 

(A)  444

(B)  515

(C)  343

(D)  373

Answer: (C)

16. 

(A)  9

(B)  40

(C)  7

(D)  36

Answer: (A)

17. Raju was to go to the planetarium. So, he walked 1.5 km towards east from the place and then turned to right and walked 2.5 km and then turned towards east walked 1 km and turned to south and walked 4 km and reached the place by walking 2.5 km towards west. What distance is he from the starting point?

(A)  6.5 km

(B)  9.5 km

(C)  10 km

(D)  9 km

Answer: (A)

18. A cyclist rides 40 km to the east, turns north and rides 20 km, again turns left and rides 20 km. How far is he from the starting point?

(A)  28.28 km

(B)  10 km

(C)  20 km

(D)  30 km

Answer: (A)

Directions-(Q. 19 and 20) Three statements are given followed by two/four conclusions I, II, III and IV. You have to consider the three statements to be true even if they seem to be at variance from commonly known facts. You have to decide which of the given conclusions, if any, follow from the given statements.

19. Statements :

(1) Rabindranath Tagore wrote many poems

(2) Every poet has aesthetic knowledge.

(3) Aesthetic is a part of axiological study.

Conclusions:

(I) Rabindranath Tagore did different axiological study.

(II) He followed the base of logic and ethics.

(A)  Only conclusion I

(B)  Both conclusions I and II

(C)  Only conclusion II

(D)  None of these

Answer: (A)

20. Statements :

(1) All clerks are superintendents.

(2) All superintendents are managers.

(3) All managers are supervisors.

Conclusions:

(I) All supervisors are clerks.

(II) Some clerks are supervisors.

(III) Some managers are clerks.

(IV) All superintendents are clerks.

(A)  Only conclusions I

(B)  Only conclusion II

(C)  Only conclusion III

(D)  Only conclusion IV

Answer: (C)

Directions-(Q. 21 to 29) Select the related letter/word/number from the given alternatives.

21. Maharashtra : India :: Texas : ?

(A)  Canada

(B)  Mexico

(C)  Brazil

(D)  USA

Answer: (D)

22. Qualm : Nausea :: Burn : ?

(A)  Fresh

(B)  Sear

(C)  Sensible

(D)  Wet

Answer: (B)

23. Heart : Cardiologist :: Kidney : ?

(A)  Endocrinologist

(B)  Orthodontist

(C)  Nephrologist

(D)  Neurologist

Answer: (C)

24. DEF : EFD :: FGH : ?

(A)  FHG

(B)  HGF

(C)  HFG

(D)  GHF

Answer: (D)

25. AZB : CYD :: EXF : ?

(A)  GWH

(B)  FGV

(C)  TMR

(D)  QSV

Answer: (A)

26. HGUOR : HTOOMS :: ? : REDNET

(A)  TOUGH

(B)  THOUGH

(C)  HUGOT

(D)  HGUOT

Answer: (B)

27. 7 : 24 :: ?

(A)  30 : 100

(B)  23 : 72

(C)  19 : 58

(D)  11 : 43

Answer: (B)

28. 12 : 140 :: 156 : ?

(A)  1820

(B)  1500

(C)  1250

(D)  1121

Answer: (A)

29. 64 : 4 :: ? : 9

(A)  18

(B)  729

(C)  81

(D)  144

Answer: (B)

Directions-(Q. 30 to 37) Find the odd word/number/letters/number pair from the given alternatives.

30.

(A)  Poland

(B)  Korea

(C)  Spain

(D)  Greece

Answer: (B)

31.

(A)  Polaris

(B)  Nike

(C)  Crux

(D)  Phoenix

Answer: (D)

32.

(A)  Chameleon

(B)  Crocodile

(C)  Alligator

(D)  Locust

Answer: (D)

33.

(A)  B C D G

(B)  G I J L

(C)  P R S U

(D)  U W X Z

Answer: (A)

34.

(A)  M K H B D

(B)  G F K H C

(C)  B D F A T

(D)  X V R P I

Answer: (D)

35.

(A)  D C E B

(B)  P N Q S T

(C)  V K H G M

(D)  W P Z L H

Answer: (A)

36.

(A)  1625

(B)  3649

(C)  6481

(D)  5025

Answer: (D)

37.

(A)  512

(B)  625

(C)  1296

(D)  2401

Answer: (A)

38. Find the wrong number in the series-

30, 27, 36, 45, 72

(A)  30

(B)  27

(C)  36

(D)  72

Answer: (A)

Directions-(Q. 39 to 41) Which one of the given responses would be a meaningful order of the following?

39. (1) Reading (2) Listening

(3) Writing           (4) Speaking

(A)  4, 2, 1, 3

(B)  2, 4, 3, 1

(C)  2, 4, 1, 3

(D)  4, 3, 2, 1

Answer: (B)

40. (1) Adulthood (2) Infancy

(3) Childhood      (4) Adolescence

(A)  1, 3, 4, 2

(B)  2, 3, 4, 1

(C)  2, 4, 3, 1

(D)  1, 2, 3, 4

Answer: (B)

41. (1) Curd (2) Milk

(3) Butter milk     (4) Cow

(5) Ghee               (6) Butter

(A)  2, 5, 6, 4, 1, 3

(B)  4, 6, 2, 1, 3, 5

(C)  4, 2, 1, 3, 6, 5

(D)  2, 6, 4, 5, 3, 1

Answer: (C)

42. How many rectangles are there in the question figure?

(A)  6

(B)  7

(C)  8

(D)  9

Answer: (D)

43. Among the four answer figures, which figure can be formed from the cutpieces given below in the question figure?

Answer: (D)

Directions-(Q. 44 and 45) Which one of the following diagrams represents the correct relation-ship among?

44. Lion, Fox and Carnivorous

Answer: (C)

45. Manager, Labour Union and Worker

Answer: (A)

46. Which answer figure will complete the question figure?

Answer: (A)

47. Which of the answer figures is embedded in the question figure?

Answer: (C)

48. A piece of paper is folded and cut as shown below in the question figures. From the given answer figures, indicate how it will appear when opened.

Answer: (D)

49. If a mirror is placed on the line MN, then which of the answer figures is the right image of the given figure?

Answer: (C)

50. A word is represented by only one set of numbers as given in any one of the alternatives. The sets of numbers given in the alternatives are represented by two classes of alphabets as in two matrices given below. The columns and rows of Matrix-I are numbered from 0 to 4 and that of Matrix-II are numbered from 5 to 9. A letter from these matrices can be represented first by its row and next by its column e.g., A can be represented by 01, 20, 42 etc. and H can be represented by 65, 57, 98 etc. Similarly, you have to identify the set for the word given in the question.

‘FAITH’

(A)  24, 31, 10, 59, 57

(B)  12, 20, 40, 68, 65

(C)  31, 34, 23, 76, 79

(D)  43, 42, 41, 78, 89

Answer: (C)

For Visually Handicapped Candidates only

Directions-(Q. 42 to 45) Select the related word/letters/number from the given alternatives.

42. Earthworm : Mud :: Crab : ?

(A)  Sea

(B)  Water

(C)  Sand

(D)  Bank

Answer: (A)

43. Peacock : India :: Bear : ?

(A)  Australia

(B)  America

(C)  Russia

(D)  England

Answer: (C)

44. UTS : EDC :: WVU : ?

(A)  XWV

(B)  WYZ

(C)  SJM

(D)  RPO

Answer: (A)

45. 3 : 11 :: 5 : ?

(A)  18

(B)  27

(C)  15

(D)  31

Answer: (B)

Directions-(Q. 46 and 47) Find the odd word/number/letters/number pair from the given alternatives.

46.

(A)  Potassium

(B)  Gallium

(C)  Germanium

(D)  Zirconium

Answer: (C)

47.

(A)  Tomato

(B)  Cucumber

(C)  Gourd

(D)  Potato

Answer: (D)

48. Choose the correct alternative from the given ones that will complete the series-

FDBZ, GECA, MKIG, PNLJ, ?

(A)  WVTR

(B)  WUSQ

(C)  WUSR

(D)  JHFD

Answer: (B)

Directions-(Q. 49 and 50) Find the which one is different from the rest of the following.

49. 1, 4, 9, 16, 25, 36, 49, 64, 81, 100, 111.

(A)  25

(B)  111

(C)  36

(D)  100

Answer: (B)

50. 2, 2, 4, 12, 48, 192, 1440.

(A)  48

(B)  192

(C)  12

(D)  1440

Answer: (B)

Part-II

English Language

Directions-In Question No. 1 to 5 choose the word opposite in meaning to the given word and mark it in the Answer Sheet.

1. Veteran

(A)  Activist

(B)  Enthusiast

(C)  Novice

(D)  Master

Answer: (C)

2. Superfluous

(A)  Essential

(B)  Excess

(C)  Unwanted

(D)  Necessary

Answer: (D)

3. Equilibrium

(A)  Work out

(B)  Disturb

(C)  Imbalance

(D)  Unevenness

Answer: (C)

4. Immortal

(A)  Eternal

(B)  Permanent

(C)  Deathly

(D)  Temporary

Answer: (D)

5. Focus

(A)  Disappear

(B)  Disperse

(C)  Link

(D)  Layer

Answer: (B)

Directions-In question Nos. 6 to 10 out of the four alternatives, choose the one which best expresses the meaning of the given word and mark it in the Answer Sheet.

6. Vivacious

(A)  Imaginary

(B)  Lively

(C)  Perceptible

(D)  Languid

Answer: (B)

7. Sporadic

(A)  Timely

(B)  Scattered

(C)  Frequent

(D)  Irrelevant

Answer: (B)

8. Persevere

(A)  Fickle

(B)  Persist

(C)  Constant

(D)  Polite

Answer: (B)

9. Petition

(A)  Rotation

(B)  Administration

(C)  Appeal

(D)  Vocation

Answer: (C)

10. Proposition

(A)  Intimation

(B)  Protestation

(C)  Proposal

(D)  Invitation

Answer: (C)

Directions-In Questions Nos. 11 to 15, four alternatives are given for the Idiom/Phrase underlined in the sentence. Choose the alternative which best expresses the meaning of the Idiom/Phrase and mark it in the Answer Sheet.

11. In his salad days he was quite a dandy.

(A)  childhood

(B)  adolescence

(C)  school days

(D)  old age

Answer: (B)

12. He is cool about working at night.

(A)  ready to work

(B)  not ready to work

(C)  excited about working

(D)  grudgingly working

Answer: (A)

13. You cannot throw dust into my eyes.

(A)  terrify me

(B)  cheat me

(C)  hurt me

(D)  abuse me

Answer: (B)

14. He spoke well though it was his maiden speech.

(A)  long speech

(B)  first speech

(C)  brief speech

(D)  emotional speech

Answer: (B)

15. The students were all ears, when the speaker started talking about the changes in the exam.

(A)  smiling

(B)  silent

(C)  restless

(D)  attentive

Answer: (D)

Directions-In Question Nos. 16 to 20, apart of the sentence is underlined. Below are given alternatives to the underlined part at (A), (B), (C) which may improve the sentence. Choose the correct alternative. In case no improvement is needed your answer is (D). Mark your answer in the Answer Sheet.

16. One cannot be indifferent to one’s health, can’t one

(A)  can’t be ?

(B)  can one ?

(C)  isn’t it ?

(D)  No improvement

Answer: (B)

17. The mother with her children were

(A)  was

(B)  will

(C)  have

(D)  No improvement

Answer: (A)

18. Sohan is pleased at the news yesterday.

(A)  has been pleased

(B)  had been pleased

(C)  was pleased

(D)  No improvement

Answer: (C)

19. She did not like the movie, nor I did.

(A)  nor did I.

(B)  nor I like it.

(C)  nor did I like it.

(D)  No improvement

Answer: (A)

20. Old habits die hardly

(A)  hard

(B)  too hard

(C)  much hardly

(D)  No improvement

Answer: (D)

Directions-In Question Nos. 21 to 25, out of the four alternatives choose the one which can be substituted for the given words/sentence.

21. An allowance made to a wife by her husband, when they are legally separated.

(A)  Alimony

(B)  Parsimony

(C)  Matrimony

(D)  Honorarium

Answer: (A)

22. Wild imagination.

(A)  Whim

(B)  Fantasy

(C)  Fancy

(D)  Memory

Answer: (B)

23. A poem of fourteen lines.

(A)  Ballad

(B)  Psalm

(C)  Sonnet

(D)  Carol

Answer: (C)

24. Incapable of error.

(A)  Erroneous

(B)  Incorrigible

(C)  Unbeatable

(D)  Infallible

Answer: (D)

25. One who believes everything he or she hears.

(A)  Credulous

(B)  Credible

(C)  Creditable

(D)  Credential

Answer: (A)

Directions-In Question to Nos. 26 to 30, four words are given in each question, out of which only one word is correctly spelt. Find the correctly spelt word and mark your answer in the Answer Sheet.

26.

(A)  Laibertarian

(B)  Libertarian

(C)  Liebertarian

(D)  Liberterian

Answer: (B)

27.

(A)  Emphetic

(B)  Emphattic

(C)  Emphatick

(D)  Emphatic

Answer: (D)

28.

(A)  Mountainer

(B)  Mountaineer

(C)  Mounteener

(D)  Mountineer

Answer: (B)

29.

(A)  Happened

(B)  Happenned

(C)  Hapened

(D)  Hapenned

Answer: (A)

30.

(A)  Sentimantalist

(B)  Sentimentelist

(C)  Sentimentalist

(D)  Santimentalist

Answer: (C)

Directions – In Question Nos. 31 to 40, in the following passage some of the words have been left out. Read the passage carefully and choose the correct answer to each question out of the four alternatives and fill in the blanks.

     Delhi …(31)… the capital of India. People from all parts of the country and the world …(32)… to Delhi. There …(33)… many historical buildings. People …(34)… the Rajghat, Shantivan and Vijayghat. We visited Delhi last year …(35)… our cousins. There …(36)… many other historical cities. Agra …(37)… one of them. We …(38)… visit Agra and Jaipur next time. The Red Fort of Delhi and the Hawa Mahal of Jaipur were …(39)… famous for their Mughal …(40)… Rajasthani architecture respectively.

31.

(A)  was

(B)  are

(C)  is

(D)  were

Answer: (C)

32.

(A)  came

(B)  comes

(C)  come

(D)  coming

Answer: (C)

33.

(A)  has

(B)  were

(C)  is

(D)  are

Answer: (D)

34.

(A)  visit

(B)  visited

(C)  visiting

(D)  visits

Answer: (A)

35.

(A)  for

(B)  on

(C)  of

(D)  with

Answer: (D)

36.

(A)  is

(B)  are

(C)  were

(D)  was

Answer: (B)

37.

(A)  are

(B)  was

(C)  is

(D)  were

Answer: (C)

38.

(A)  will

(B)  would

(C)  could

(D)  can

Answer: (A)

39.

(A)  much

(B)  very

(C)  too

(D)  more

Answer: (A)

40.

(A)  either

(B)  because

(C)  or

(D)  and

Answer: (D)

Directions-In Question Nos. 41 to 45, some parts of sentences have errors and some are correct. Find out which part of a sentence has an error and blacken the circle [●] corresponding to (D) in the Answer Sheet.

41. A interesting book (A)/’A Tale of two cities’ (B)/ was written by Alexander Dumas. (C)/ No error (D)

Answer: (A)

42. In India (A)/ there are (B)/ many poors. (C)/ No error (D)

Answer: (C)

43. I worked (A)/ as medical representative (B)/ for eight months. (C)/ No error (D)

Answer: (B)

44. Shakespe are has written (A)/ many plays (B)/ as well as some poetries. (C)/ No error (D)

Answer: (C)

45. Neither of the girls (A)/ were willing to (B)/ (C)/ accept the proposal. No error (D)

Answer: (B)

Directions – In Question Nos. 46 to 50, sentences given with blanks to be filled in with an appropriate word(s). Four alternatives are suggested for each question. Choose the correct alternative out of the four and indicate it by blackening the appropriate circle [●] in the Answer Sheet.

46. There is no …….. evidence to support your assertion.

(A)  facile

(B)  fictitious

(C)  facetious

(D)  factual

Answer: (D)

47. Throw a stone ……….. the fierce dog.

(A)  at

(B)  upon

(C)  on

(D)  above

Answer: (A)

48. Is not learning superior ……… wealth?

(A)  than

(B)  from

(C)  by

(D)  to

Answer: (D)

49. A group of agitators …….. the mob to break down the Vice-Chancellor’s door.

(A)  wished

(B)  excited

(C)  threatened

(D)  incited

Answer: (D)

50. Turn the lights ……….. before you go to bed.

(A)  on

(B)  off

(C)  out

(D)  down

Answer: (B)

Part-III

Quantitative Aptitude

1. The average salary of all the workers in a workshop is Rs 8,000. The average salary of 7 technicians is Rs 12,000 and the average salary of the rest is Rs 6,000. The total number of workers in the workshop is-

(A)  20

(B)  21

(C)  22

(D)  23

Answer: (B)

2. 3 years ago the average age of a family of 5 members was 17 years. A baby having been born, the average age of the family is the same today. The present age of the baby is-

(A)  1 year

(B) 

(C)  2 years

(D)  3 years

Answer: (C)

3. A total profit of Rs 3,600 is to be distributed amongst A, B and C such that A : B = 5 : 4 and B : C = 8 : 9. The share of C in the profit is-

(A)  Rs 1,200

(B)  Rs 1,500

(C)  Rs 1,650

(D)  Rs 1,700

Answer: (A)

4. A man sold his watch at a loss of 5%. Had the sold it for Rs 56.25 more he would have gained 10%. What is the cost price of the watch (in Rs)?

(A)  370

(B)  365

(C)  375

(D)  390

Answer: (C)

5. 1% of 1% of 25% of 1000 is-

(A)  .025

(B)  .0025

(C)  .25

(D)  .000025

Answer: (A)

6. The population of a village increases by 5% annually. If its present population is 4410, then its population 2 years ago was-

(A)  4, 500

(B)  4,000

(C)  3,800

(D)  3,500

Answer: (B)

7. A is twice as fast as B and B is thrice as fast as C is. The journey covered by C in  will be covered by A in-

(A)  15 minutes

(B)  20 minutes

(C)  30 minutes

(D)  1 hour

Answer: (A)

8. A sum of Rs 210 was taken as a loan. This is to be paid back in two equal installments. If the rate of interest be 10% compounded annually, then the value of each installment is-

(A)  Rs 127

(B)  Rs 121

(C)  Rs 210

(D)  Rs 225

Answer: (B)

9. The area of a square park is 25 sq. km. The time taken to complete a round of the field once, at a speed of 3 km/hr is-

(A)  4 hours 60 minutes

(B)  4 hours 50 minutes

(C)  6 hours 40 minutes

(D)  5 hours 40 minutes

Answer: (C)

10. The external fencing of a circular path around a circular plot of land is 33 m more than its interior fencing. The width of the path around the plot is-

(A)  5.52 m

(B)  5.25 m

(C)  2.55 m

(D)  2.25 m

Answer: (B)

11. A horse takes  to compete a round around a circular field. If the speed of the horse was 66 m/sec, then the radius of the field is- [ Given π = 22/7]

(A)  25.62 m

(B)  26.52 m

(C)  25.26 m

(D)  26.25 m

Answer: (D)

12. A flask in the shape of a right circular cone of height 24 cm is filled with water. The water is poured in a right circular cylindrical flask whose radius is 1/3rd of the radius of the base of the circular cone. Then the height of the water in the cylindrical flask is-

(A)  32 cm

(B)  24 cm

(C)  48 cm

(D)  72 cm

Answer: (D)

13. If the three medians of a triangle are same, then the triangle is-

(A)  equilateral

(B)  isosceles

(C)  right-angled

(D)  obtuse-angled

Answer: (A)

14. If  then the value of   is –

(A)  4/3

(B)  3/2

(C)  5/2

(D)  5/3

Answer: (C)

15. Which one of the following is true?

(A)  √5 + √3 > √6 + √2

(B)  √5 + √3 < √6 + √2

(C)  √5 + √3 = √6 + √2

(D)  (√5 + √3) (√6 + √2) = 1

Answer: (A)

16. If  and x + y + z ≠ 0, then each ratio is-

(A) 

(B) 

(C) 

(D) 

Answer: (D)

17. If  then value of x4 – 2x2y2 + y4 is-

(A)  24

(B)  4

(C)  16

(D)  8

Answer: (C)

18. If x = 3 + 2√2, then  is equal to-

(A)  216

(B)  192

(C)  198

(D)  204

Answer: (D)

19. The perimeters of two similar triangles ∆ABC and ∆PQR are 36 cm and 24 cm respectively. If PQ = 10 cm, the AB is-

(A)  15 cm

(B)  12 cm

(C)  14 cm

(D)  26 cm

Answer: (A)

20. If the sides of a right-angled triangle are three consecutive integers, then the length of the smallest side is-

(A)  3 units

(B)  2 units

(C)  4 units

(D)  5 units

Answer: (A)

21. Two circles intersect each other at the points A and B. A straight line parallel to AB intersects the circle at C, D E and F. If CD = 4.5 cm, then the measure of EF is-

(A)  1.50 cm

(B)  2.25 cm

(C)  4.50 cm

(D)  9.00 cm

Answer: (C)

22. The sum of the interior angles of a polygon is 1440°. The number of sides of the polygon is-

(A)  6

(B)  9

(C)  10

(D)  12

Answer: (C)

23. In ∆ABC, D and E are two points on the sides AB and AC respectively so that DE || BC and  Then   is equal to-

(A)  5/9

(B)  21/25

(C) 

(D) 

Answer: (B)

24. The value of  is-

(A)  −1

(B)  0

(C)  1

(D)  2

Answer: (A)

25. If θ is a positive acute angle and 4cos2 θ – 4 cos θ + 1 = 0, then the value of tan (θ – 15°) is equal to-

(A)  0

(B)  1

(C)  √3

(D)  1/√3

Answer: (B)

26. If (r cos θ – √3)2 + (r sin θ – 1)2 = 0, then the value of  is equal to-

(A)  4/5

(B)  5/4

(C)  √3/4

(D)  √5/4

Answer: (A)

27. A vertical pole and a vertical tower are standing on the same level ground. Height of the pole is 10 metres. From the top of the pole the angle of elevation of the top of the tower and angle of depression of the foot of the tower are 60° and 30° The height of the tower is-

(A)  20 m

(B)  30 m

(C)  40 m

(D)  50 m

Answer: (C)

28. Ram left 1/3 of his property to his widow and 3/5 of the remainder to his daughter. He gave the rest to his son who received Rs 6,400. How much was h is original property worth?

(A)  Rs 16,000

(B)  Rs 32,000

(C)  Rs 24,000

(D)  Rs 1,600

Answer: (C)

29. The first term of an Arithmetic Progression is 22 and the last term is –11. If the sum is 66, the number of terms in the sequence are-

(A)  10

(B)  12

(C)  9

(D)  8

Answer: (B)

30. The H.C.F. and L.C.M. of two numbers are 44 and 264 respectively. If the first number is divided by 2, the quotient is 44. The other number is-

(A)  147

(B)  528

(C)  132

(D)  264

Answer: (C)

31. A teacher wants to arrange his students in an equal number of rows and columns. If there are 1369 students, the number of students in the last row are-

(A)  37

(B)  33

(C)  63

(D)  47

Answer: (A)

32. If  then the value of 

(A)  3/7

(B) 

(C)  1

(D)  2

Answer: (C)

33. A and B working separately can do a piece of work in 9 and 15 days respectively. If they work for a day alternately, with A beginning, then the work will be completed in-

(A)  10 days

(B)  11 days

(C)  9 days

(D)  12 days

Answer: (B)

34. Two pipes A and B can fill a tank in 36 min. and 45 min. Respectively. Another pipe C can empty the tank in 30 min. First A and B are opened. After 7 minutes, C is also opened. The tank is filled up in-

(A)  39 min.

(B)  46 min.

(C)  40 min.

(D)  45 min.

Answer: (B)

35. If the sum of the dimensions of a rectangular parallelepiped is 24 cm an the length of the diagonal is 15 cm, then the total surface area of it is-

(A)  420 cm2

(B)  275 cm2

(C)  351 cm2

(D)  378 cm2

Answer: (C)

36. Area of a regular hexagon with side ’a’ is-

(A) 

(B) 

(C) 

(D) 

Answer: (C)

37. The marked price of a saree is Rs 200. After allowing a discount of 20% on the marked price, the shopkeeper makes a profit of Rs 16. Find the gain per cent-

(A) 

(B) 

(C)  11%

(D)  8%

Answer: (A)

38. The marked price of an item is twice the cost price. For a gain of 15%, the discount should be-

(A)  7.5%

(B)  20.5%

(C)  32.5%

(D)  42.5%

Answer: (D)

39. Two numbers are in the ratio 3 : 5. If 9 is subtracted from each, the new numbers are in the ratio 12 : 23. The small number is-

(A)  27

(B)  33

(C)  49

(D)  55

Answer: (B)

40. If x : y = 5 : 2, then (8x + 9y) : (8x + 2y) is-

(A)  22 : 29

(B)  26 : 61

(C)  29 : 22

(D)  61 : 26

Answer: (C)

41. The length of the shadow of a vertical tower on level ground increase by 10 metres when the altitude of the sun changes from 45° to 30°. Then the height of the tower is-

(A)  5(√3 + 1) metres

(B)  5(√3 – 1) metres

(C)  5√3 metres

(D)  5/√3 metres

Answer: (A)

Directions-(Q. 42-45) Sales of Books (in thousands numbers) from Six Branches – B1, B2, B3, B4, B5 and B6 of a Publishing Company in 2000 and 2001. Study the graph and answer the questions.

42. Total sale of branches B1, B3 and B5 together for both the years (in thousand numbers) is-

(A)  250

(B)  310

(C)  435

(D)  560

Answer: (D)

43. Find the ratio of the total sales of branch B2 for both years to the total sales of branch B4 for both years-

(A)  2 : 3

(B)  3 : 5

(C)  4 : 5

(D)  7 : 9

Answer: (D)

44. Percentage of the average sale of branches B1, B2 and B3 in 2001 and the average sale of branches B1, B3 and B6 in 2000-

(A)  87.5

(B)  75

(C)  77.5

(D)  82.5

Answer: (A)

45. Find the percentage increase in the sales of books of branch B3 in the year 2001 than the branch B2-

(A)  69.2

(B)  50.8

(C)  40.9

(D)  65.7

Answer: (A)

Directions- (Q. 46-50) The diagram shows the age-distribution of the patients admitted to a hospital on a particular day. Study the diagram and answer the questions.

46. Number of patients of age between 55 years to 60 years, who got admitted to the hospital on that day is-

(A)  6

(B)  4

(C)  24

(D)  8

Answer: (B)

47. Total number of patients of age more than 55 years, who got admitted to the hospital is-

(A)  4

(B)  7

(C)  9

(D)  10

Answer: (D)

48. Number of patients of age more than 40 years and less than 55 years, who got admitted to the hospital on that day is-

(A)  20

(B)  30

(C)  15

(D)  12

Answer: (C)

49. Percentage of patients of age less than 45 years, who got admitted to the hospital on that day is approximately equal to-

(A)  14%

(B)  20%

(C)  37%

(D)  62%

Answer: (C)

50. About 11% of the patients who got admitted to the hospital on that particular day were of age-

(A)  either between 35 years and 40 years or between 55 years and 60 years

(B)  between 60 years and 65 years

(C)  between 35 years and 40 years

(D)  between 35 years and 40 years and between 55 years ad 60 years

Answer: (D)

For Visual Handicapped Candidates Only

42. If the simple interest and compound interest at the same rate of certain amount for 2 years are Rs 400 and Rs 420 respectively, then the rate of interest is-

(A)  12%

(B)  8%

(C)  10%

(D)  11%

Answer: (C)

43. Three friends divided Rs 624 among themselves in the ratio1/2 : 1/3 : 1/4 ; the share of third friend is-

(A)  Rs 288

(B)  Rs 192

(C)  Rs 148

(D)  Rs 144

Answer: (D)

44. In the expression xy2, the values of x and y are each decreased by 25%. The value of the expression is decreased by-

(A)  37/64 of its value

(B)  1/2 of its value

(C)  27/64 of its value

(D)  3/4 of its value

Answer: (A)

45. Rs 64,000 will amount to Rs 68,921 at 5% per annum and interest payable half yearly in-

(A) 

(B)  2 years 

(C) 

(D) 

Answer: (D)

46. What is the value on an angle included between x-axis and y-axis in radian?

(A)  π/6

(B)  π/3

(C)  π/4

(D)  π/2

Answer: (D)

47. The value of  is equal to-

(A)  0

(B)  1

(C)  2

(D)  3

Answer: (B)

48. The value of (1,000,001)2 – (999.999)2 is=

(A)  2.000,000

(B)  4,000,000

(C)  6,000,000

(D)  8,000,000

Answer: (B)

49. The sum of the square of three consecutive natural numbers is 194. The sum of the numbers is-

(A)  24

(B)  27

(C)  21

(D)  30

Answer: (A)

50. A vessel is in the form of an inverted cone. Its height is 11 cm and radius of its top, which is open, is 2.5 cm. It is filled with water upto the rim. When lead shots, each of which is a sphere of radius 0.25 cm are dropped into the vessel, 2/5 of the water flows out. The number of lead shots dropped into the vessel is-

(A)  880

(B)  440

(C)  220

(D)  110

Answer: (B)

Part-IV

General Awareness

1. The original founder of the Manuscripts and Editor of Kautilya’s Arthashastra was-

(A)  Srikanta Shastri

(B)  Srinivasa Iyangar

(C)  R. Shamashastri

(D)  William Jones

Answer: (C)

2. Which of the following is the largest Biosphere Reserves of India?

(A)  Nilgiri

(B)  Nandadevi

(C)  Sundarbans

(D)  Gulf of Mannar

Answer: (D)

3. ISRO’s Master Control Facility is in-

(A)  Andhra Pradesh

(B)  Odisha

(C)  Gujarat

(D)  Karnataka

Answer: (C)

4. India is the largest producer and exporter of-

(A)  Cotton

(B)  Copper

(C)  Tea

(D)  Mica

Answer: (A)

5. The soils which are rich in Calcium are known as-

(A)  Pedocals

(B)  Pedalfers

(C)  Podsols

(D)  Laterites

Answer: (A)

6. Cultivable land is defined as-

(A)  land actually under crops

(B)  cultivable waste land + fallow land

(C)  old fallow lands + current fallow lands

(D)  total fallow lands + net sown area

Answer: (A)

7. From which part of Opium plant we get morphine?

(A)  Leaves

(B)  Stem

(C)  Bark

(D)  Fruit coat

Answer: (D)

8. Which of the following is a Biological method of soil conservation?

(A)  Contour farming

(B)  Contour terracing

(C)  Gully control

(D)  Basin listing

Answer: (A)

9. Glucose is a type of-

(A)  Pentose sugar

(B)  Hexose sugar

(C)  Tetrose sugar

(D)  Diose sugar

Answer: (B)

10. Number of mitochondria in bacterial cell is-

(A)  one

(B)  two

(C)  many

(D)  zero

Answer: (D)

11. In cactus, the spines are the modified-

(A)  stem

(B)  stipules

(C)  leaves

(D)  buds

Answer: (C)

12. The smallest known prokaryotic organism is-

(A)  Microcystic

(B)  Mycoplasma

(C)  Bacteria

(D)  Chlorella

Answer: (B)

13. Rainbow is formed due to-

(A)  refraction and dispersion

(B)  scattering and refraction

(C)  diffraction and refraction

(D)  refraction and reflection

Answer: (D)

14. Golden view of sea shell is due to-

(A)  Diffraction

(B)  Dispersion

(C)  Polarization

(D)  Reflection

Answer: (C)

15. An object covers distance which is directly proportional to the square of the time. Its acceleration is-

(A)  increasing

(B)  decreasing

(C)  zero

(D)  constant

Answer: (D)

16. If the horizontal range of a projectile is four times its maximum height, the angle of projection is-

(A)  30°

(B)  45°

(C)  sin1 (1/4)

(D)  tan1 (1/4)

Answer: (B)

17. Which place is called as ‘Silicon Valley’ of India?

(A)  Delhi

(B)  Pune

(C)  Bengaluru

(D)  Hyderabad

Answer: (C)

18. Telnet stands for-

(A)  Telephone Network

(B)  Television Network

(C)  Teletype Network

(D)  Telefax Network

Answer: (A)

19. Which of the following metals has least melting point?

(A)  Gold

(B)  Silver

(C)  Mercury

(D)  Copper

Answer: (C)

20. The gas produced is marshy places due to decomposition of vegetation is-

(A)  Carbon monoxide

(B)  Carbon dioxide

(C)  Sulphur dioxide

(D)  Methane

Answer: (D)

21. The boiling point of water decreases at higher altitudes is due to-

(A)  low temperature

(B)  low atmospheric pressure

(C)  high temperature

(D)  high atmospheric pressure

Answer: (B)

22. The chemical name of ‘Hypo’ commonly used in photography is-

(A)  Sodium thiosulphate

(B)  Silver nitrate

(C)  Sodium nitrate

(D)  Silver iodide

Answer: (A)

23. With what bio-region is the term ‘Steppe’ associated?

(A)  Grasslands

(B)  Tropical forests

(C)  Savanna

(D)  Coniferous forests

Answer: (A)

24. About how much of the world’s land area is tropical rainforest?

(A)  2 per cent

(B)  7 per cent

(C)  10 per cent

(D)  15 per cent

Answer: (C)

25. According to your text, what can “be thought of as the genetic library that keeps life going on Earth”?

(A)  A bio-engineering lab

(B)  Human genes

(C)  The human genome project

(D)  Biodiversity

Answer: (B)

26. The world’s growing appetite for what food product is a leading cause of tropical deforestation?

(A)  Pork

(B)  Sugar

(C)  Lamb

(D)  Beef

Answer: (D)

27. ‘Life Divine’ is a book written by-

(A)  M. K. Gandhi

(B)  Rabindranath Tagore

(C)  S. Radhakrishnan

(D)  Shri Aurobindo

Answer: (D)

28. The Oscar Award was won 36 times by-

(A)  Charlie Chaplin

(B)  Alfred Hitchcock

(C)  Walt Disney

(D)  Akiro Kurosawa

Answer: (C)

29. ‘Meghdoot’ was written by-

(A)  Humayun Kabir

(B)  Khushwant Singh

(C)  Banabhatta

(D)  Kalidasa

Answer: (D)

30. Who among the following is a famous English writer?

(A)  Amrita Pritam

(B)  Mahadevi Verma

(C)  Ashapurna Devi

(D)  Mulk Raj Anand

Answer: (D)

31. The President of World Bank is-

(A)  Jim Yong Kim

(B)  Christine Lagarde

(C)  Prema Cariappa

(D)  Vijay L. Kelkar

Answer: (A)

32. Tulsidas wrote Ramcharitmanas in the reign of-

(A)  Babar

(B)  Akbar

(C)  Aurangzeb

(D)  Jahangir

Answer: (B)

33. Grammy Award is given in the field of-

(A)  Acting

(B)  Music

(C)  Singing

(D)  Boxing

Answer: (B)

34. The first woman to get the Bharat Ratna Award is-

(A)  Mother Teresa

(B)  Indira Gandhi

(C)  Lata Mangeshkar

(D)  Sarojini Naidu

Answer: (B)

35. Karl Marx wrote-

(A)  Asian Drama

(B)  Emma

(C)  Das Kapital

(D)  Good Earth

Answer: (C)

36. The religious text of the Jews is named is-

(A)  The Analectus

(B)  Torah

(C)  Tripitaka

(D)  Zend-Avesta

Answer: (B)

37. Which of the following statements is correct?

(A)  Most workers will work for less than their reservation wage.

(B)  The reservation wage is the maximum amount any firm will pay for a worker.

(C)  Economic rent is the different between the market wage and the reservation wage.

(D)  Economic rent is the amount one must pay to enter a desirable labour market.

Answer: (C)

38. Other things being equal, a decrease in quantity demanded of a commodity can be caused by-

(A)  a rise in the price of the commodity

(B)  a rise in the income of the consumer

(C)  a fall in the price of the commodity

(D)  a fall in the income of the consumer

Answer: (A)

39. Which of the following is not an economic problem?

(A)  Deciding between paid work and leisure.

(B)  Deciding between expenditure on one good and the other.

(C)  Deciding between alternative methods of personal saving.

(D)  Deciding between different ways of spending leisure time.

Answer: (D)

40. Which of the following occurs when labour productivity rises?

(A)  The equilibrium nominal wage falls

(B)  The equilibrium quantity of labour falls

(C)  Competitive firms will be induced to use more capital

(D)  The labour demand curve shifts to the right

Answer: (D)

41. Which of the following are consumer semidurable goods?

(A)  Cars and television sets

(B)  Milk and milk products

(C)  Foodgrains and other food products

(D)  Electrical appliance like fans and electric irons

Answer: (C)

42. In the provisional Parliament of India, how many members were there?

(A)  296

(B)  313

(C)  318

(D)  316

Answer: (B)

43. It was decided to observe Mahatma Gandhi’s birthday October 2 as the International Nonviolence Day at-

(A)  International Indology Conference

(B)  Satyagraha Centenary Conference

(C)  Congress Foundation Day Celebration

(D)  None of these

Answer: (D)

44. Who admits a new State to the Union of India?

(A)  President

(B)  Supreme Court

(C)  Prime Minister

(D)  Parliament

Answer: (D)

45. In which year were the States re-organized on a linguistic basis?

(A)  1951

(B)  1947

(C)  1950

(D)  1956

Answer: (D)

46. Who has got the power to create All India Services?

(A)  Supreme Court

(B)  The Parliament

(C)  Council of Ministers

(D)  Prime Minister

Answer: (B)

47. Which one of the following is the most lasting contribution of the Rastrakutas?

(A)  Kailasha Temple

(B)  Pampa, Ponna, Ranna, the three writers of Kannada Poetry and Kailasha Temple

(C)  Patronage of Jainism

(D)  Conquests

Answer: (A)

48. Ravikirti, a jain, who composed the Aihole Prashasti, was patronized by-

(A)  Pulakeshi I

(B)  Harsha

(C)  Pulakeshi II

(D)  Kharavela

Answer: (C)

49. The ‘Mein Kampf’ was written by-

(A)  Hitler

(B)  Mussolini

(C)  Bismarck

(D)  Mazzini

Answer: (A)

50. When did the reign of Delhi Sultanate came to an end?

(A)  1498 A.D.

(B)  1526 A.D.

(C)  1565 A.D.

(D)  1600 A.D.

Answer: (B)

SSC Combined Higher Secondary Level (10+2) Examination (Second Shift) Tier-I Held on 2 November, 2014 Question Paper With Answer Key

SSC Combined Higher Secondary Level (10+2) Examination (Second Shift) Tier-I Held on 2 November, 2014
SSC Combined Higher Secondary Level (10+2) Examination (Second Shift) Tier-I Held on 2 November, 2014 Question Paper With Answer Key

Staff Selection Commission Combined Higher Secondary Level (10+2)

Examination (Second Shift) Tier-I Held on 2 November, 2014

Part-I

General Intelligence

Directions – (Q. 1 to 9) Select the one which is different from the other three responses.

1. 

(A)  Square

(B)  Equilateral Triangle

(C)  Rhombus

(D)  Right Angle Triangle

Answer: (D)

2.

(A)  Aptitude

(B)  Altitude

(C)  Attitude

(D)  Behaviour

Answer: (B)

3.

(A)  SSA

(B)  RMSA

(C)  RUSA

(D)  NASA

Answer: (A)

4.

(A)  63

(B)  69

(C)  65

(D)  66

Answer: (C)

5.

(A)  108

(B)  91

(C)  144

(D)  225

Answer: (B)

6.

(A)  187 : 11

(B)  194 : 12

(C)  195 : 13

(D)  224 : 14

Answer: (B)

7.

(A)  D

(B)  G

(C)  H

(D)  J

Answer: (B)

8.

(A)  SRT

(B)  PON

(C)  KJL

(D)  VUW

Answer: (B)

9.

(A)  KVFU

(B)  CXDW

(C)  AZBX

(D)  GTHS

Answer: (C)

10. Which one of the given responses would be a meaningful order of the following?

(1) Earth   (2) Jupiter

(3) Venus (4) Mars

(5) Mercury

(A)  5, 3, 1, 2, 4

(B)  5, 3, 4, 1, 2

(C)  5, 3, 1, 4, 2

(D)  5, 3, 2, 4, 1

Answer: (C)

11. Find the odd one out-

(A)  January, May

(B)  April, June

(C)  July, August

(D)  January, December

Answer: (B)

Directions-(Q. 12 to 18) A series is given, with one term missing. Choose the correct alternative from the given ones that will complete the series.

12. CBA, ABC, ABCD, DCBA, ABCDE, ?

(A)  EDCBA

(B)  DBAC

(C)  CABD

(D)  BACD

Answer: (A)

13. AD, EI, JN, PS, ?

(A)  WY

(B)  XX

(C)  WX

(D)  WW

Answer: (C)

14. PON, RQP, TSR, VUT, ?

(A)  WUY

(B)  YXZ

(C)  XWV

(D)  UVW

Answer: (C)

15. ar, cs, et, ?

(A)  wy

(B)  gv

(C)  vb

(D)  gu

Answer: (D)

16. 9, 15, 23, 33, ?

(A)  44

(B)  36

(C)  38

(D)  45

Answer: (D)

17. 4, 7, 14, 24, 41, ?

(A)  71

(B)  68

(C)  72

(D)  51

Answer: (B)

18. 5, 16, 51, 158, ?

(A)  481

(B)  465

(C)  441

(D)  478

Answer: (A)

19. A shepherd had 17 sheep. All but nine died. How many sheep are left?

(A)  9

(B)  8

(C)  7

(D)  10

Answer: (A)

20. Present ages of father & son are in the ratio of 5 : 1 respectively. Seven years later this ratio becomes 3 : 1. What is the son’s present age in years?

(A)  8

(B)  7

(C)  6

(D)  5

Answer: (B)

21. From the given alternative words, select the word which can be formed using the letters of the given word-

TRADITIONAL

(A)  NATION

(B)  RADIO

(C)  ANIMAL

(D)  DIRTY

Answer: (B)

22. If ‘NEWS’ is written as ‘WENS’, then how ‘MATE’ will be written in this code?

(A)  TAME

(B)  META

(C)  EATM

(D)  AMET

Answer: (A)

23. If REASON is coded as 5 and BELIEVED as 7, what is the code number for GOVERNMENT?

(A)  6

(B)  8

(C)  9

(D)  10

Answer: (C)

24. Select the missing number from the given responses-

1 4 2 3 2 ?

(A)  2

(B)  3

(C)  4

(D)  5

Answer: (A)

25. Which of the following interchange of signs would make the give equation correct?

(6 + 3) + (4 × 7) = 29

(A)  +  and −

(B)  ÷ and +

(C)  × and +

(D)  ÷ and ×

Answer: (C)

26. Change the sign to find the equation :

28 – 7 + 2 × 2 = 0

(A)  Change + into ×

(B)  Change × into,

(C)  Change – into +

(D)  Change + into −

Answer: (A)

27. The price of onions is shown below for every fortnight. Find the price in Feb. 3rd week.

(A)  140

(B)  300

(C)  180

(D)  320

Answer: (B)

28. The population of a village in Madurai is increasing year after year. Find out the population (in lakhs) in 2013 from the following information-

(A)  49

(B)  46

(C)  45

(D)  43

Answer: (A)

29. Ram walk 12 kms to the North, then 10 kms to West, 12 kms South. How far is Ram from the starting point?

(A)  9 kms

(B)  13 kms

(C)  8 kms

(D)  10 kms

Answer: (D)

30. Town A and town B were 60 km apart. Joshua left town A for town B and travelled at an average speed of 65 km/h. At the same time, Menon left town B for town A. The two of them met 5 hours later. Find Menon’s average speed.

(A)  55 km/h

(B)  60 km/h

(C)  65 km/h

(D)  120 km/h

Answer: (A)

31. There are two statements labeled as Assertion (A) and Reason (R).

Assertion (A) : Dyslexia is a psychological disorder.

Reason (R) : Vitamin deficiency causes the disease.

Codes :

(A)  If both (A) and (R) are true

(B)  If both (A) and (R) are false

(C)  If (A) is false, but (R) is true

(D)  If (A) is true, but (R) is false

Answer: (D)

32. Two statements are given followed by two conclusions I and II. You have to consider the statements to be true even if they seem to be at variance from commonly known facts. You are to decide which of the given conclusions. If any, follow from the given statements. Indicate your answer.

Statements:

All Americans are English-speaking.

No Eskimos are English-Speaking.

Conclusions:

(I) No Eskimos are Americans.

(II) No English-speakers are Eskimos.

(A)  Only I follows

(B)  Neither I nor II follows

(C)  Only II follows

(D)  Both I and II follows

Answer: (D)

Directions –(Q. 33 to 41) Select the related letter/word/number from the given alternatives.

33. River : Tributary :: Tree : ?

(A)  Stem

(B)  Root

(C)  Branch

(D)  Flower

Answer: (C)

34. Lion : Forest :: Fish : ?

(A)  Cage

(B)  Nest

(C)  Sky

(D)  Water

Answer: (D)

35. Polio : Virus :: Anthrax : ?

(A)  Fungus

(B)  Bacteria

(C)  Virus

(D)  Insect

Answer: (B)

36. MONEY : YENOM :: RIGHT : ?

(A)  HTIRG

(B)  THGIR

(C)  GIRHT

(D)  IRGHT

Answer: (B)

37. NIIOLUSL : ILLUSION : TBERVARE : ?

(A)  STRANGE

(B)  VERTEBRA

(C)  VABTERE

(D)  ERAVEBT

Answer: (B)

38. RUST : 9687 :: TSUR : ?

(A)  7896

(B)  7869

(C)  7689

(D)  6789

Answer: (B)

39. 13 : 169 :: ?

(A)  12 : 140

(B)  3 : 6

(C)  11 : 111

(D)  9 : 81

Answer: (D)

40. 23 : 29 :: 41 : ?

(A)  43

(B)  45

(C)  47

(D)  49

Answer: (C)

41. 6 : 10 :: 9 : ?

(A)  10

(B)  12

(C)  15

(D)  18

Answer: (C)

42. The sides of a cube show the colours of rainbow. Two positions of the cube are shown below. Which of the colours of rainbow is left out?

(A)  Yellow

(B)  Green

(C)  Violet

(D)  Indigo

Answer: (C)

43. Based on the diagrams what is the letter which is opposite to A?

(A)  B

(B)  C

(C)  D

(D)  E

Answer: (D)

Directions-(Q. 44 ad 45) Identify the diagram that best represents the relationship among classes given below-

44. Singers, Boys, Dancers.

Answer: (C)

45. Ornaments, Gold, Silver.

Answer: (A)

46. Identify the answer figure from which the piece given in question figure has been cut.

Answer: (A)

47. From the given answer figures, select the one in which the question figure is hidden/embedded.

Answer: (A)

48. Find out from amongst the four alternatives as to how the pattern would appear when the transparent sheet is folded at the dotted line.

Answer: (B)

49. Choose the alternative which most closely resembles the mirror-image of the given combination.

DANIEL

Answer: (B)

50. A words is represented by only one set of numbers as given in any one of the alternatives. The sets of numbers given in the alternatives are represented by two classes of alphabets as in two matrices given below. The columns are rows of Matrix I are numbered from 0 to 4 and that of Matrix II are numbered from 5 to 9. A letter from these matrices can be represented first by its row and next by its column, e.g., ‘I’ can be represented by 13, 21 etc. and ‘B’ can be represented by 57, 65 etc. Similarly, you have to identify the set for the word given in question.

‘FADE’

(A)  76, 02, 75, 32

(B)  68, 20, 57, 14

(C)  55, 33, 65, 23

(D)  89, 10, 96, 41

Answer: (A)

For Visually Handicapped Candidates Only

Directions-(Q. 42 to 45) Select the related letter/word/number from the given alternatives.

42. Pen : ink :: ………

(A) Doctor : coat

(B) Tank : petrol

(C) Police : gun

(D) Fisherman : net

Answer: (B)

43. Coconut : Shell :: Letter : ?

(A)  Letter-box

(B)  Stamp

(C)  Mail

(D)  Envelope

Answer: (D)

44. AJT : CMW :: EJO : ?

(A)  GMR

(B)  GLR

(C)  GLQ

(D)  GKS

Answer: (A)

45. 393 : 857 :: 467 : ?

(A)  468

(B)  526

(C)  483

(D)  589

Answer: (D)

Directions-(Q. 46 to 49) Find the odd number/letters/pair from the given alternatives.

46.

(A)  Snake

(B)  Whale

(C)  Crocodile

(D)  Lizard

Answer: (B)

47.

(A)  Hurricane

(B)  Explosion

(C)  Earthquake

(D)  Tsunami

Answer: (B)

48.

(A)  631

(B)  862

(C)  530

(D)  357

Answer: (D)

49.

(A)  Dholak

(B)  Madal

(C)  Mridanga

(D)  Tabla

Answer: (B)

50. Find the odd one in the series-

4, 9, 19, 34, 79

(A)  34

(B)  79

(C)  9

(D)  19

Answer: (A)

Part-II

English Language

Directions-(Q. 1-5) Sentences are given with blanks to be filled in with an appropriate word(s). For alternatives are suggested for each question. Choose the correct alternative out of the four and indicate it by blackening the appropriate square (∎) in the answer sheet.

1. The priest was innocent ………. he could not prove it.

(A)  and

(B)  but

(C)  for

(D)  so

Answer: (B)

2. Many people who …… practicing photography as a hobby turn it eventually into a livelihood.

(A)  resume

(B)  prefer

(C)  imply

(D)  start

Answer: (D)

3. She is ……… of lizards.

(A)  fearful

(B)  frightened

(C)  afraid

(D)  terrified

Answer: (C)

4. His path was beset ……. difficulties.

(A)  by

(B)  with

(C)  of

(D)  from

Answer: (B)

5. The plan is worth considering, think it ……… carefully.

(A)  of

(B)  on

(C)  off

(D)  over

Answer: (D)

Directions- (Q. 6-10) In these question choose the word opposite in meaning to the given word and mark it in the Answer Sheet.

6. Reluctant

(A)  Hesitant

(B)  Reserved

(C)  Anxious

(D)  Willing

Answer: (C)

7. Novel

(A)  Naughty

(B)  Novelist

(C)  Banal

(D)  Nasty

Answer: (D)

8. Eloquent

(A)  Elegant

(B)  Lucid

(C)  Articulate

(D)  Inarticulate

Answer: (D)

9. Fluent

(A)  Inappropriate

(B)  Halting

(C)  Degrading

(D)  Insensitive

Answer: (B)

10. Adversity

(A)  Prosperity

(B)  Curiosity

(C)  Animosity

(D)  Sincerity

Answer: (A)

Directions-(Q. 11-15) In these question out of the four alternatives, choose the one which best expresses the meaning of the given word and mark it in the Answer Sheet.

11. Amiable

(A)  Rude

(B)  Curt

(C)  Friendly

(D)  Annoyed

Answer: (C)

12. Curious

(A)  Doubtful

(B)  Inquisitive

(C)  Sad

(D)  Suspicious

Answer: (B)

13. Candid

(A)  Honest

(B)  Greedy

(C)  Dishonest

(D)  Secretive

Answer: (A)

14. Forsaken

(A)  Nurtured

(B)  Neglected

(C)  Pardoned

(D)  Abandoned

Answer: (D)

15. Vivacious

(A)  Lonely

(B)  Lively

(C)  Beautiful

(D)  Bravely

Answer: (B)

Directions-(Q. 16-20) In these question four alternatives are given for the Idiom/Phrase bold in the sentence. Choose the alternative which best expresses the meaning of the idiom/Phrase and mark it in the Answer Sheet.

16. The detective went abroad in search of a culprit but his journey proved to be a wild goose chase.

(A)  expensive

(B)  full of difficulties

(C)  unprofitable adventure

(D)  ill advised

Answer: (C)

17. A close shave.

(A)  A narrow escape from danger.

(B)  Stubble

(C)  Very short hair

(D)  Hairless skin

Answer: (A)

18. Please be as brief as you, can, I have other fish to fry.

(A)  to cook the food

(B)  to take rest

(C)  some important work to attend to

(D)  to attend the friends

Answer: (C)

19. His argument does not hold water.

(A)  to have effect

(B)  to influence

(C)  to check the flow of water

(D)  sound logical fact

Answer: (D)

20. It was her maiden speech on the stage and she performed well.

(A)  unprepared speech

(B)  sudden speech

(C)  primary speech

(D)  first speech

Answer: (D)

Directions-(Q. 21-25) In these question a part of the sentence is bold. Below are given alternatives to the bold part at (A), (B), (C) which may improve the sentence. Choose the correct alternative. In case no improvement is needed your answer is (D). Your answer in the Answer Sheet.

21. When the little girl losses her doll, she began to cry.

(A)  losing her doll

(B)  loss her doll

(C)  lost her doll

(D)  No improvement

Answer: (C)

22. I brushed my teeth every day at 7 O’clock in the morning.

(A)  I brushes my teeth every day at 7 O’clock in the morning.

(B)  I brush my teeth every day at 7 O’clock in the morning.

(C)  I brush my tooth every day at 7 O’clock in the morning.

(D)  No improvement

Answer: (B)

23. More than 60% of India’s population live under the poverty line.

(A)  live by the poverty line

(B)  live below poverty line

(C)  live beside the poverty line

(D)  No improvement

Answer: (B)

24. Only me I can solve the problem.

(A)  Only me can

(B)  Only I can

(C)  I can only

(D)  No improvement

Answer: (B)

25. They congratulated me for my victory in the debate competition.

(A)  about

(B)  at

(C)  on

(D)  No improvement

Answer: (B)

Directions-(Q. 26-30) In these question out of the four alternatives choose the one which can be substituted for the given words/sentence.

26. Beyond Correction

(A)  Inverterate

(B)  Glib

(C)  Incorrigible

(D)  Incongruous

Answer: (C)

27. Widespread scarcity of food-

(A)  Hunger

(B)  Drought

(C)  Poverty

(D)  Famine

Answer: (B)

28. Inflammation of gums-

(A)  Gangerene

(B)  Gingivitis

(C)  Conjunctivitis

(D)  Orchitis

Answer: (B)

29. A book written by an unknown author-

(A)  Anonymous

(B)  Acrimonious

(C)  Audacious

(D)  Assiduous

Answer: (A)

30. An event that causes great harm or dams-

(A)  Problem

(B)  Disaster

(C)  Pain

(D)  Relief

Answer: (B)

Directions- (Q. 31-35) In these question four words are given in each question, out of which only one word is correctly spelt. Find the correctly spelt word and mark your answer in the Answer Sheet.

31.

(A)  nuisense

(B)  nuisanse

(C)  nuissance

(D)  nuisance

Answer: (D)

32.

(A)  saimultaneous

(B)  simultaenous

(C)  simultaneous

(D)  simultanious

Answer: (C)

33.

(A)  honourarium

(B)  honorarium

(C)  honourerium

(D)  honourrarium

Answer: (B)

34.

(A)  fortuneate

(B)  fortuneit

(C)  forchunate

(D)  fortunate

Answer: (D)

35.

(A)  misfourtune

(B)  miscelaneous

(C)  misdemenour

(D)  misspell

Answer: (B)

Directions- (Q. 36-45) In the following passage some of the words have been left out. Read the passage carefully and choose the correct answer to each question out of the four alternatives and fill in the blanks.

   Rajan is a salesman. He goes from door to door …(36)… vacuum cleaners. On his first day of work, he …(37)… to sell his goods by …(38)… on the doors of a private housing estate. “My first …(39)…,” he thought as he …(40)… the doorbell of the first house. A middle-aged woman with curlers in her hair opened the door. Rajan began to …(41)… who he was and the …(42)… he was selling. The …(43)… minute, the door was …(44)… shut. “Not interested !” John heard her shout …(45)… the closed door.

36.

(A)  purchasing

(B)  buying

(C)  gifting

(D)  selling

Answer: (D)

37.

(A)  decided

(B)  undecided

(C)  finalized

(D)  cancelled

Answer: (A)

38.

(A)  hacking

(B)  knocking

(C)  breaking

(D)  shaking

Answer: (B)

39.

(A)  consumer

(B)  customer

(C)  supplier

(D)  manufacturer

Answer: (B)

40.

(A)  pressed

(B)  impressed

(C)  depressed

(D)  suppressed

Answer: (A)

41.

(A)  complain

(B)  explain

(C)  refrain

(D)  plain

Answer: (B)

42.

(A)  produce

(B)  material

(C)  product

(D)  ingredients

Answer: (C)

43.

(A)  after

(B)  before

(C)  next

(D)  subsequent

Answer: (C)

44.

(A)  slammed

(B)  damned

(C)  tamed

(D)  crammed

Answer: (A)

45.

(A)  before

(B)  on

(C)  aside

(D)  behind

Answer: (D)

   Directions-(Q. 46-50) In these questions some parts of the sentences have errors and some are correct. Find out which part of a sentence has an error and blacken the oval () corresponding to the appropriate letter (A, B, C). If a sentence is free from errors, blacken the oval corresponding to (D) in the Answer Sheet.

46. No sooner did the rabbit (A)/ come out of the bush (B)/ when the hunter killed it. (C)/ No error (D)

Answer: (C)

47. ‘The Arabian Nights’ (A)/ (B)/ (C)/ No error (D)

Answer: (B)

48. She has not completed (A)/ her course. (B)/Isn’t it ? (C)/ No error (D)

Answer: (C)

49. Every citizen is (A)/ entitled to (B)/ the voting. (C)/ No error (D)

Answer: (C)

50. The collector (A)/ visits the office regularly (B)/Isn’t it? (C)/ No error (D)

Answer: (D)

Part-III

Quantitative Aptitude

1. 40 men can complete a work in 18 days. Eight days after they started working together, 10 more men joined them. How many days will they now take to complete the remaining work?

(A)  6

(B)  8

(C)  10

(D)  12

Answer: (B)

2. 16 women take 12 days to complete a work which can be completed by 12 men in 8 days. 16 men started working and after 3 days 10 men left and 4 women joined them. How many days will it take them to complete the remaining work?

(A)  4

(B)  6

(C)  8

(D)  10

Answer: (B)

3. The area of an equailateral triangle is 48 sq. cm. The length of the side is-

(A)  √8 × 4 cm

(B)  4√3 cm

(C)  8(3)1/4

(D)  16 cm

Answer: (C)

4. An elephant of length 4 m is at one corner of a rectangular cage of size (16 m × 30 m) and faces towards the diagonally opposite corner. If the elephant starts moving towards the diagonally opposite corner it taken 15 seconds to reach this corner. Find the speed of the elephant-

(A)  1 m/sec

(B)  2 m/sec

(C)  1.87 m/sec

(D)  1.5 m/sec

Answer: (B)

5. A shopkeeper allows a rebate 12% on the marked price of an article such that the selling price is Rs 440. Then the marked price of the article is-

(A)  Rs 490

(B)  Rs 500

(C)  Rs 600

(D)  Rs 550

Answer: (B)

6. A tradesman marks his goods at 25 p.c. above the cost price. If he reduces the marked price by  then his profit will be-

(A)  

(B) 

(C)   

(D)  

Answer: (A)

7. If A : B = 7 : 9 and B : C = 3 : 5, then A : B : C is equal to-

(A)  7 : 9 : 5

(B)  21 : 35 : 45

(C)  7 : 9 : 15

(D)  7 : 3 : 15

Answer: (C)

8. Three numbers are in the ratio 1 : 2 : 3 and their H.C.F. is 12. The numbers are-

(A)  4, 8, 12         

(B)  5, 10, 15

(C)  10, 20, 30

(D)  12, 24, 36

Answer: (D)

9. A cricketer whose bowling average is 12.4 runs per wicket, takes 5 wickets for 26 runs and thereby decreases his average by 0.4. The number of wickets taken by him till the last match was-

(A)  64

(B)  72

(C)  80

(D)  85

Answer: (D)

10. The average monthly income of P and Q is Rs 5,050. The average monthly income of Q and R is Rs 6,250 and the average monthly income of P and R is Rs 5,200. The monthly income of P is-

(A)  Rs 3,500

(B)  Rs 4,000

(C)  Rs 4,050

(D)  Rs 5,000

Answer: (B)

11. Y purchased a flat for Rs 9,25,000 and spent Rs 35,000 for its renovation. If he sold the flat for Rs 10,80,000, then this profit per cent is-

(A)  15.0

(B)  17.5

(C)  20.5

(D)  12.5

Answer: (D)

12. A table is sold at a profit of 13%. If it is sold for Rs 25 more, profit is 18%. Cost price of table is-

(A)  Rs 100

(B)  Rs 500

(C)  Rs 200

(D)  Rs 1,000

Answer: (B)

13. In an examination, 19% students fail in Mathematics and 10% students fail in English. If 7% of all students fail in both subjects, then the number of students passed in both subjects is-

(A)  36% of all students

(B)  64% of all students

(C)  71% of all students

(D)  78% of all students

Answer: (D)

14. X spends 35% of his salary on food and 5% of his salary on children education. In January 2011, he spent Rs 17,600 on these two items. His salary for that month is-

(A)  Rs 40,000

(B)  Rs 44,000

(C)  Rs 48,000

(D)  Rs 46,000

Answer: (B)

15. A motorist travels along the boundary of a square field of area 1476225 m2, at 36 km/hr. He returns to starting point in-

(A)  8 min., 8 sec.

(B)  6 min., 8 sec.

(C)  8 min., 6 sec.

(D)  6 min., 6 sec.

Answer: (C)

16. A sum of Rs 800 amounts to Rs 920 in 3 years at the simple interest rate. If the rate is increased by 3% p.a. what will be the sum amount to in the same period?

(A)  Rs 992

(B)  Rs 962

(C)  Rs 942

(D)  Rs 982

Answer: (A)

17. Each Edge of a regular tetrahedron is 4 cm. Its volume (in cubic cm) is-

(A)  16√3/2

(B)  16√3

(C)  16√2/3

(D)  16√2

Answer: (C)

18. If the numerical value of circumference and area of a circle is same, then the area is-

(A)  6 π sq. unit

(B)  4 π sq. unit

(C)  8 π sq. unit

(D)  12 π sq. unit

Answer: (B)

19. The median of an equilateral triangle is 6√3 cm. The area (in cm2) of the triangle is-

(A)  72

(B)  108

(C)  72√3

(D)  36√3

Answer: (D)

20. If the perimeter of an equilateral triangle be 18 cm, then the length of each median is-

(A)  3√2 cm

(B)  2√3 cm

(C)  3√3 cm

(D)  2√2 cm

Answer: (C)

21. If the radius of a cylinder is decreased by 50% and the height is increased by 50%, then the change in volume is-

(A)  52.5%

(B)  67.5%

(C)  57.5%

(D)  62.5%

Answer: (D)

22. The simplified value of-

 is

(A)  √2

(B)  1/√2

(C)  √3 − √2

(D)  0

Answer: (D)

23. If  then the value of  is-

(A)  0

(B)  1

(C)  2

(D)  6

Answer: (A)

24. If  then the value of a18 + a12 + a6 + 1 is-

(A)  0

(B)  1

(C)  −1

(D)  4

Answer: (A)

25. If x = 997, y = 998 and z = 999, then the value of x2 + y2 + z2 – xy – yz – zx is-

(A)  0

(B)  1

(C)  −1

(D)  3

Answer: (D)

26. If  then two values of x are-

(A)  1, 2

(B)  2, −1/2

(C)  0, 1

(D)  ½, 1

Answer: (B)

27. In a ∆ABC, ∠A + ∠B = 70° and ∠B + ∠C = 130°, value of ∠A is-

(A)  20°

(B)  50°

(C)  110°

(D)  30°

Answer: (B)

28. In a ∆ ABC, ∠A + ∠B = 70° and ∠B + ∠C = 130°, value of ∠A is-

(A)  20°

(B)  50°

(C)  110°

(D)  30°

Answer: (A)

29. If sec θ + tan θ = 4, (θ ≠ 90°), then the value of cos θ is-

(A)  0

(B)  8/17

(C)  17/8

(D)  4/5

Answer: (B)

30. Two circles touches externally at P. QR is a common tangent of the circles touching the circles at Q and R. Then measure of ∠QPR is-

(A)  60°

(B)  30°

(C)  90°

(D)  45°

Answer: (C)

31. If in a triangle ABC, D and E are on the sides AB and AC, such that, DE is parallel to BC and  If AC = 4 cm, then AE is-

(A)  1.5 cm

(B)  2.0 cm

(C)  1.8 cm

(D)  2.4 cm

Answer: (A)

32. A person has three iron bars whose lengths are 10, 15 and 20 metres respectively. He wants to cut pieces of same length from each of the three bars. What is the least number of total pieces if he is to cut without any wastage?

(A)  45

(B)  15

(C)  9

(D)  30

Answer: (C)

33. The 12th term of the series 

(A)    

(B)   

(C)    

(D)   

Answer: (A)

34. The value of  is close to-

(A)  0.4

(B)  0.8

(C)  1.0

(D)  1.4

Answer: (B)

35. If  equal to

(A)  .0025

(B)  .025

(C)  .25

(D)  .00025

Answer: (B)

36. If X is 20% less than Y, then find 

(A)  1/5, −4

(B)  5, −1/4

(C)  2/5, −5/2

(D)  3/5, −5/3

Answer: (A)

37. If θ is a positive acute angle and cosec θ = √3, then the value of cot θ – cosec θ is-

(A)   

(B)    

(C)    

(D)   

Answer: (A)

38. If 0° < A < 90°, then the value of tan2 A + cot2A – sec2 A cosec2 A is-

(A)  0

(B)  1

(C)  2

(D)  −2

Answer: (D)

39. If α and β are positive acute angles, sin (4α – β) = 1 and cos (2α + β) = 1/2, then the value of sin (α + 2β) is-

(A)  0

(B)  1

(C)  √3/2

(D)  1/√2

Answer: (D)

40. The angle of elevation of sun changes from 30° to 45°, the length of the shadow of a pole decreases by 4 metres, the height of the pole is- (Assume √3 = 1.732)

(A)  1.464 m

(B)  9.464 m

(C)  3.648 m

(D)  5.464 m

Answer: (D)

41. From an aeroplane just over a river, the angle of depression of two palm tree on the opposite bank of the river are found to be 60° and 30° If the breadth of the river is 400 metres, then the height of the aeroplane above the river at that instant is- (Assume √3 = 1.732)

(A)  173.2 metres

(B)  346.4 metres

(C)  519.6 metres

(D)  692.8 metres

Answer: (A)

Directions-(Q. 42 to 45) Study the graph carefully and answer the questions given below it-

42. The production of state D in 1993-94 is how many times it production in 1994-95?

(A)  1.33

(B)  0.75

(C)  0.56

(D)  1.77

Answer: (A)

43. Which of the following statement is false?

(A)  State A and E showed the same production in 1993-94.

(B)  There was no improvement in the production of cotton in state B during 1994.95.

(C)  State A has produced maximum cotton during the given period.

(D)  Production of state C and D together is equal to that of state B during 1993-94.

Answer: (C)

44. How many states showing below average production in 1992-93 showed above average production in 1993-94?

(A)  4

(B)  2

(C)  3

(D)  1

Answer: (C)

45. What is the average production of the five states in the year 1994-95 taken together?

(A)  12.3

(B)  14.6

(C)  15.6

(D)  16.3

Answer: (B)

Directions-(Q. 46 to 50) The subjectwise no. of applicants for the year 2008 and 2009 in a college is given in the following chart. Study the graph and answer.

46. The subject for which growing of demand is maximum is-

(A)  Chemistry

(B)  Mathematics

(C)  Computer

(D)  Biotechnology

Answer: (C)

47. The subject for which growing of demand is minimum is-

(A)  Statistics

(B)  Zoology

(C)  Botany

(D)  Microbiology

Answer: (C)

48. The number of Chemistry seeking applicants increased by-

(A)  17.26%

(B)  18.89%

(C)  19.25%

(D)  21.08%

Answer: (B)

49. The number of Physics seeking applicants increased by-

(A)  17.26%

(B)  18.89%

(C)  19.25%

(D)  21.08%

Answer: (A)

50. The number of Mathematics seeking applicants increased by-

(A)  17.26%

(B)  18.89%

(C)  19.25%

(D)  21.08%

Answer: (C)

For Visually Handicapped Candidates Only

42. 70 cattle can graze a piece of land for 28 days. How many cattle can graze a field three times as large in 70 days?

(A)  168

(B)  84

(C)  64

(D)  252

Answer: (B)

43. Five years ago, the ratio of A’s age and B’s age was 5 : 6. The ratio of their ages will be 7 : 8 after five years. The ratio of their present age is-

(A)  6 : 7

(B)  4 : 3

(C)  7 : 6

(D)  3 : 4

Answer: (A)

44. If 10% of x = 20% of y, then x : y is equal to-

(A)  1 : 2

(B)  3 : 1

(C)  2 : 1

(D)  1 : 4

Answer: (C)

45. A farmer borrowed Rs 2,400 at 12% interest per annum. At the end of  he cleared his account by paying Rs 1,200 and a cow. Then the cost of cos is-

(A)  Rs 720

(B)  Rs 3120

(C)  Rs 1920

(D)  Rs 2120

Answer: (C)

46. When the Sun’s altitude increases from 45° to 60°, the shadow of the vertical post is diminished by 6 metres. The height (in metres) of the post is- (Take √3 = 1.732)

(A)  7.608

(B)  9.464

(C)  14.196

(D)  28.392

Answer: (C)

47. The value of  is-

(A)  3/2

(B)  1/2

(C)  1

(D)  0

Answer: (C)

48. If a2 + b2 + c2 –ab – bc – ca = 0, then a relation between a, b and c is-

(A)  a + b = c

(B)  a + b + c = 0

(C)  b + c = a

(D)  a = b = c

Answer: (D)

49. If 10% of x = 15% of y = 20% of z, then x : y : z is-

(A)  2 : 3 : 4

(B)  3 : 4 : 6

(C)  6 : 4 : 3

(D)  4 : 3 : 2

Answer: (C)

50. A road which is 7 metre wide surrounds a circular park whose circumference is 352 metre. The area of the road in square metre is – (taking π = 22/7)

(A)  2518

(B)  2581

(C)  2618

(D)  2681

Answer: (C)

Part-IV

General Awareness

1. Who was the contemporary South Indian ruler of Harshavardhana?

(A)  Krishnadevaraya

(B)  Pulakeshin II

(C)  Mayuravarma

(D)  Chikkadevaraja Wodeyar

Answer: (B)

2. Nappe is a kind of-

(A)  fluvial feature

(B)  folded structure

(C)  erosional plain

(D)  delta region

Answer: (B)

3. The Panama Canal is different from the Suez Canal as it has-

(A)  Lock system

(B)  Shorter route

(C)  Busy route

(D)  Less busy route

Answer: (A)

4. The area which is resistant to any massive structural transformation is called-

(A)  rigid mass

(B)  ancient landmass

(C)  tectonic plate

(D)  shield

Answer: (A)

5. Llanos are the grasslands of-

(A)  Guyana Highlands

(B)  Brazilian Highlands

(C)  Argentina

(D)  Chile

Answer: (A)

6. The areas marked by internal drainage-

(A)  Plateau

(B)  Plains

(C)  Deserts

(D)  Mountains

Answer: (C)

7. The space retaining life in any form is called-

(A)  Biomass

(B)  Biosphere

(C)  Lithosphere

(D)  Hydrosphere

Answer: (C)

8. Rhizobium is a kind of

(A)  Photosynthetic bacteria

(B)  Symbiotic bacteria

(C)  Parasitic bacteria

(D)  Saprophytic bacteria

Answer: (D)

9. A Parenchyma cell which stores ergastic substance is known as-

(A)  Phragmoblast

(B)  Idioblast

(C)  Conidioplast

(D)  Chloroplast

Answer: (B)

10. Root Hairs arise from-

(A)  Cortex

(B)  Pericycle

(C)  Epidermis

(D)  Endodermis

Answer: (C)

11. The gametophyte is called pro-thallus in-

(A)  Pteridophyta

(B)  Bryophyta

(C)  Spermatophyta

(D)  Thallophyta

Answer: (A)

12. The best example of Polyembryony is-

(A)  Cocoa

(B)  Capsicum

(C)  Citrus

(D)  Cycas

Answer: (C)

13. A moving neutron collides with a stationary α-particle. The fraction of the kinetic energy lost by the neutron is-

(A)  1/4

(B)  1/16

(C)  9/25

(D)  16/25

Answer: (D)

14. Which of these waves can be polarized?

(A)  Sound waves in air

(B)  Longitudinal waves on a string

(C)  Transverse waves on a string

(D)  Light waves

Answer: (D)

15. The unit of Planck’s constant is-

(A)  Js

(B)  Js2

(C)  J/s

(D)  Js2

Answer: (A)

16. If a wire of resistance R is melted and recast to half of its length then the new resistance of the wire will be-

(A)  R/4

(B)  R/2

(C)  R

(D)  2R

Answer: (A)

17. Hard copies can be obtained from-

(A)  Scanner

(B)  Speaker

(C)  Printer

(D)  Recorder

Answer: (C)

18. In an E-R diagram, ellipse represents-

(A)  relation

(B)  entity

(C)  fields

(D)  key

Answer: (D)

19. Chemical name of common salt is-

(A)  Sodium Chloride

(B)  Sodium Bicarbonate

(C)  Sodium

(D)  Sodium Oxide

Answer: (A)

20. Which one of the following mineral does not contain oxygen?

(A)  Haematite

(B)  Bauxite

(C)  Cryolite

(D)  Calcite

Answer: (C)

21. Non-conductor of electricity is-

(A)  Iron

(B)  Gas Carbon

(C)  Copper Sulphate

(D)  Mercury

Answer: (C)

22. Which of the following will replace hydrogen from acids to form salts?

(A)  S

(B)  Na

(C)  Ag

(D)  P

Answer: (B)

23. Which type of lake is formed by volcanic activities?

(A)  Lagoon

(B)  Fresh water lake

(C)  Cladera lake

(D)  Karst lake

Answer: (C)

24. Green manure is obtained from-

(A)  Domestic vegetable waste

(B)  Oil seed husk cakes

(C)  Fresh animal excreta

(D)  Decomposing green legume plants

Answer: (D)

25. The environment includes-

(A)  Abiotic factors

(B)  Biotic factors

(C)  Oxygen and Nitrogen

(D)  Abiotic and Biotic factors

Answer: (D)

26. The temperate grasslands of North America are known as-

(A)  Pampas

(B)  Downs

(C)  Steppes

(D)  Prairies

Answer: (D)

27. The Jawaharlal Nehru Port is located at-

(A)  Paradip

(B)  Cochin

(C)  Mumbai

(D)  Kolkata

Answer: (C)

28. How many islands are there in Lakshadweep?

(A)  17

(B)  27

(C)  36

(D)  47

Answer: (C)

29. Tagore Award for Cultural Harmony 2013 has been given to-

(A)  Zubin Mehta

(B)  Narendra Kohli

(C)  Shalini Singh

(D)  Arvind Aadiga

Answer: (A)

30. Winner of the Indira Gandhi Peace Prize 2013-

(A)  Ela Bhatt

(B)  Angela Merkel

(C)  Lydia Davis

(D)  Christina Legarde

Answer: (B)

31. Pandit Shiv Kumar Sharma is an exponent of-

(A)  Mandolin

(B)  Santoor

(C)  Sitar

(D)  Veena

Answer: (B)

32. Moortidevi Award is given annually in which of the following fields?

(A)  Literature

(B)  Films

(C)  Journalism

(D)  Music

Answer: (A)

33. Nobel Prizes are distributed annually at-

(A)  Manila

(B)  Stockholm

(C)  Geneva

(D)  New York

Answer: (B)

34. ‘Freedom From Fear’ is a book written by-

(A)  Benazir Bhutto

(B)  Corazon Aquino

(C)  Aung San Suu Kyi

(D)  Nayantara Sehgal

Answer: (C)

35. Who is the author of the book ‘Nineteen Eighty Four’?

(A)  J. M. Barrie

(B)  Walter Scott

(C)  George Orwell

(D)  Thomas Hardy

Answer: (C)

36. The 28th Annual Lata Mangeshkar Samman Alankaran 2013 was conferred to-

(A)  Sonu Nigam

(B)  Kailash Kher

(C)  Hariharan

(D)  A. R. Rahman

Answer: (C)

37. The internal rate of return-

(A)  must be less than the interest rate if the firm is to invest

(B)  makes the present value of profits equal to the present value of costs

(C)  falls as the annual yield of an investment rises

(D)  is equal to the market interest rate of all the firm’s investment

Answer: (B)

38. As the number of investments made by a firm increases, its internal rate of return-

(A)  declines due to diminishing marginal productivity

(B)  declines because the market rate of interest will fall, ceteris paribus

(C)  increases to compensate the firm for the current consumption foregone

(D)  increase because the level of savings will fall

Answer: (C)

39. Capacity utilization-

(A)  is usually near 100 per cent

(B)  represents the per cent of the labour force that is employed

(C)  is a measure of the proportional of the existing capital stock used for current production

(D)  rises as the economy moves into a recession, since firms must replace unemployed workers with some other resources to maintain production

Answer: (C)

40. The theory of ‘Maximum Social Advantage’ in Public Finance was given by-

(A)  Robbins

(B)  Musgrave

(C)  Findley

(D)  Dalton

Answer: (D)

41. The value of investment multiplier relates to-

(A)  change in income due to change in autonomous investment

(B)  change in autonomous investment due to change is income

(C)  change in income due to change in consumption

(D)  change in the income due to change in induced investment

Answer: (A)

42. When did the Constituent Assembly adopt National Anthem?

(A)  25th January, 1950

(B)  26th January, 1950

(C)  24th January, 1950

(D)  29th January, 1950

Answer: (C)

43. The President of India is an integral part of the-

(A)  The Parliament

(B)  The Lok Sabha

(C)  Rajya Sabha

(D)  Council of Ministers

Answer: (A)

44. Union Public Service Commission has to submit the report to the-

(A)  President

(B)  Parliament

(C)  Lok Sabha

(D)  Rajya Sabha

Answer: (A)

45. How many times was the Preamble of the Constitution amended?

(A)  Three times

(B)  Two times

(C)  Once

(D)  Not amended

Answer: (C)

46. An ordinance issued by Governor is subject to approval by-

(A)  The President

(B)  The State Legislature

(C)  The State Council of Ministers

(D)  The Parliament

Answer: (A)

47. Which rebellion in Bengal was highlighted by Bankim Chandra Chatterjee in his novel ‘Anand Math’?

(A)  Chaur Uprising

(B)  Sanyasi Rebellion

(C)  Kol Uprising

(D)  Santhal Uprising

Answer: (B)

48. Name the Greek Ambassador at the Mauryan Court-

(A)  Alexander

(B)  Megasthanese

(C)  Plato

(D)  Aristotle

Answer: (B)

49. When the rule of the Delhi Sultan began?

(A)  1106 A.D.

(B)  1206 A.D.

(C)  1306 A.D.

(D)  1406 A.D.

Answer: (B)

50. Who introduced leather token currency in India?

(A)  Akbar

(B)  Mohammad-Bin-Tughlaq

(C)  Babur

(D)  Humayun

Answer: (D)

SSC Combined Higher Secondary Level (10+2) Examination-2015 (Ist Shift) Tier-I Held on 2 November, 2015 Question Paper With Answer Key

SSC Combined Higher Secondary Level (10+2) Examination-2015 (Ist Shift) Tier-I Held on 2 November, 2015
SSC Combined Higher Secondary Level (10+2) Examination-2015 (Ist Shift) Tier-I Held on 2 November, 2015 Question Paper With Answer Key

Staff Selection Commission Combined Higher Secondary Level (10+2)

Examination-2015 (Ist Shift) Tier-I Held on 2 November, 2015

Part-I

General Intelligence

Directions-(Q. 1-3) In the following questions, arrange the following words as per order in the dictionary.

1. (1) Intervention (2) Intertwine

(3) Interview            (4) Intervene

(A)  4, 3, 2, 1

(B)  2, 4, 1, 3

(C)  2, 3, 4, 1

(D)  3, 4, 1, 2

Answer: (B)

2. (1) Smoulder (2) Smother

(3) Smuggle    (4) Smudge

(A)  4, 3, 2, 1

(B)  1, 4, 3, 2

(C)  1, 2, 3, 4

(D)  2, 1, 4, 3

Answer: (D)

3. (1) Plate (2) Plane

(3) Plaster       (4) Plasma

(5) Plastic

(A)  2, 4, 3, 5, 1

(B)  1, 2, 3, 4, 5

(C)  2, 1, 5, 3, 4

(D)  2, 5, 3, 4, 1

Answer: (A)

Directions-(Q. 4 and 5) In the following questions, which one set of letters when sequentially placed at the gaps in the given letter series shall complete it?

4. b_ac_cc_cb_ab_ac

(A)  abbbc

(B)  aabba

(C)  cbaba

(D)  bbaac

Answer: (B)

5. _zy_zxy_yxzx_zyx_xy

(A)  yzxyx

(B)  xyzzy

(C)  yxzyz

(D)  zxyzy

Answer: (C)

Directions-(Q. 6-9) In the following questions, a series is given with one term missing. Choose the correct alternative from the given ones what will complete the series.

6. 1, 5, 13, 25, 41, 61, …?….

(A)  85

(B)  91

(C)  81

(D)  77

Answer: (A)

7. EZ FY XG …?…

(A)  GW

(B)  WG

(C)  HW

(D)  WH

Answer: (D)

8. 1, 16, 81, 256, 625 ….?….

(A)  2225

(B)  4163

(C)  1225

(D)  1296

Answer: (D)

9. Find out the incorrect term in the series-

2, 5, 10, 3, 6, 18, 4, 7, 30

(A)  6

(B)  30

(C)  7

(D)  10

Answer: (B)

10. Raghu jogged 2 km southwards, then he turned right and walked 5 km. He again turned right and walked and jogged 8 km. In which direction was he seen moving last?

(A)  North

(B)  West

(C)  South

(D)  East

Answer: (A)

11. In 5 years, Rs 5,000 amounts to Rs 9,000. In what time at the same rate will Rs 600 amounts to Rs 900?

(A)  5 years

(B)  3 years

(C)  6 years

(D)  2 years

Answer: (B)

12. If in a certain code ONE is coded as 231, FIVE is coded as 9641, then how will be NINE coded?

(A)  3631

(B)  3316

(C)  3613

(D)  3361

Answer: (A)

13. If PNLJ : 2468, then QOKL: …?…

(A)  1376

(B)  1276

(C)  3591

(D)  1367

Answer: (A)

14. Select the set of symbols which can be fitted correctly in the equation-

5 …….3……8……4…….2 = 21

(A)  +, ×, −, ÷

(B)  −, ×, +, ÷

(C)  ×, +, −, ÷

(D)  +, ×, +, ÷

Answer: (C)

15. If a means +, b means ×, c means ÷, d means −, then 20a10b45c5d12 = ?

(A)  88

(B)  98

(C)  68

(D)  74

Answer: (B)

Directions-(Q. 16 and 17) In the following questions, unscramble the letters to form a meaningful word. Then find out the correct numerical position of the letters.

16. H N R C A B

      1  2  3  4  5 6

(A)  6 5 3 4 1 2

(B)  6 3 5 2 4 1

(C)  3 5 6 4 1 2

(D)  4 1 5 6 2 3

Answer: (B)

17. N I  T  I  F E  I  N

     2  4  5  4  3 6  4  2

(A)  3 2 4 3 5 2 4 6

(B)  3 2 5 3 5 2 4 6

(C)  4 2 3 4 2 4 5 6

(D)  2 4 3 2 4 2 5 6

Answer: (C)

Directions-(Q. 18-21) In the following questions, select the missing number from the given responses.

18. 3 21  7

     4   ?    3

    2   16  8

(A)  12

(B)  5

(C)  10

(D)  1

Answer: (A)

19. 8 13  10

     7   12   9

     10  15   ?

(A)  19

(B)  5

(C)  8

(D)  12

Answer: (D)

20. 

(A)  75

(B)  100

(C)  25

(D)  50

Answer: (D)

21. 

(A)  6

(B)  8

(C)  1

(D)  12

Answer: (D)

22. Five students are standing one behind the other in the playground facing the instructor. Malini is behind Anjana, but in front of Gayatri. Meena is in front of Shena, but behind Gayatri. What is the position of Meena?

(A)  Second from first

(B)  Extreme last

(C)  Second front last

(D)  Extreme first

Answer: (C)

23. A watch reads 4 : 30. If the minute hand points East, in which direction will the hour hand point?

(A)  South-West

(B)  South

(C)  North-East

(D)  North

Answer: (C)

24. In the question, two statements are given. You have to answer considering the statements to be true, even if they seem to be at variance from commonly known facts.

Statements:

(I) Plants grow from seeds.

(II) Seeds are produced by flowers.

Conclusions:

(A)  Flowers are produced by plants.

(B)  Flowers and seeds are produced by plants.

(C)  Plants grow from flowers.

(D)  Seeds are produced by plants.

Answer: (C)

25. In the question, two statements are given followed by two conclusions I and II. You have to consider the two statements to be true even if they seem to be at variance from commonly known facts. You have to decide which of the given conclusions, if any, follow from the given statements.

Statements:

(I) Creative persons are learned people.

(II) Some doctors are creative persons.

Conclusions:

(I) Some creative persons are learned people.

(II) Some learned people are doctors.

(A)  I and II

(B)  Neither I nor II

(C)  I

(D)  II

Answer: (D)

Directions-(Q. 26-34) In the following questions, select the related word/letters/number from the given alternatives.

26. Governor : State : President : …?…

(A)  District

(B)  Constituency

(C)  Organization

(D)  Country

Answer: (D)

27. GFEH : RQPS : : :LKJM : ?

(A)  IHJG

(B)  VUWX

(C)  ONMP

(D)  LKNM

Answer: (C)

28. CEIM : DGLQ : : FGIO : ?

(A)  GMIS

(B)  GMSI

(C)  GILS

(D)  GLIS

Answer: (C)

29. HIJK : GFED : : NOPQ : ?

(A)  BCDE

(B)  FDEC

(C)  EFGH

(D)  MLKJ

Answer: (D)

30. 235 : 587 : : 435 : ?

(A)  988

(B)  989

(C)  788

(D)  789

Answer: (D)

31. 17 : 102 : : 23 : ?

(A)  138

(B)  413

(C)  112

(D)  216

Answer: (A)

32. 4 : 36 : : ? : 49

(A)  7

(B)  8

(C)  5

(D)  6

Answer: (C)

33. Heart : Cardiologist : : Brain : …?…

(A)  Neurologist

(B)  Gynaecologist

(C)  Nephrologist

(D)  Ophthalmologist

Answer: (A)

34. Car : Garrage : : ? : Hangar

(A)  Truck

(B)  Train

(C)  Bus

(D)  Aeroplane

Answer: (D)

Directions-(Q. 35-43) In the following questions, find out the word/letters/number/number pair from the giben alternatives.

35. Find out the odd word-

(A)  PEARS

(B)  HAMAM

(C)  CINTHOL

(D)  RIN

Answer: (D)

36. Find out the odd letters-

(A)  RFD

(B)  TIW

(C)  PET

(D)  CAP

Answer: (A)

37. Find out the odd letters-

(A)  STUA

(B)  HGFA

(C)  RQPA

(D)  MLKA

Answer: (A)

38. Find out the odd letters-

(A)  LORU

(B)  WZCF

(C)  ADGJ

(D)  IMQU

Answer: (D)

39. Find out the odd number-

(A)  742956

(B)  876321

(C)  368127

(D)  564327

Answer: (A)

40. Find out the odd number pair-

(A)  (54, 216)

(B)  (61, 244)

(C)  (24, 96)

(D)  (27, 135)

Answer: (D)

41. Find out the odd number pair-

(A)  12-21

(B)  57-75

(C)  15-41

(D)  34-43

Answer: (C)

42. Find out the odd word-

(A)  Sparrow

(B)  Humming bird

(C)  Kingfisher

(D)  Crane

Answer: (D)

43. Find out the odd pair-

(A)  Pen-Ink

(B)  Brush-Paint

(C)  Pencil-Lead

(D)  Crayon-Chart

Answer: (D)

44. Four positions of a dice are given below-

Find the number on the face opposite to the face showing 4.

(A)  1

(B)  5

(C)  3

(D)  6

Answer: (B)

45. Identify the diagram that best represents the relationship among classes given below-

Library, Books, Racks

Answer: (A)

46. Which answer the figure will complete the pattern in the question figure?

Answer: (C)

47. Identify the answer figure from which pieces given in the question figure have been cut.

Answer: (B)

48. A piece of paper is folded and punched is shown below in the question figures. From the given answer figures, indicate how it will appear when opened.

Answer: (C)

49. If a mirror is placed on the line MN, then which of the answer figures is the right image of the given figure?

Answer: (C)

50. A word is represented by only one set of numbers as given in any one of the alternatives. The sets of numbers given in the alternatives are represented by two classes of alphabets as in two matrices given below. The columns and rows of Matrix I are numbered from 0 to 4 and that of Matrix II are numbered from 5 to 9. A letter from these matrices can be represented first by its row and next by its column e.g., ‘M’ can be represented by 01, 10 etc., and ‘A’ can be represented by 56, 65 etc. Similarly, you have to identify the set for the word ‘ROD’.

(A)  56, 66, 86

(B)  58, 69, 65

(C)  58, 66, 78

(D)  67, 96, 57

Answer: (D)

For Visually Handicapped/Cerebral Palsy Candidates Only

Directions-(Q. 44-47) In the following questions, select the missing number/letters from the given responses.

44. AXE, ESI, IOM, ?

(A)  PST

(B)  MKQ

(C)  QLM

(D)  MQI

Answer: (B)

45. 3, …., 18, 33, 53, 78

(A)  6

(B)  9

(C)  8

(D)  12

Answer: (C)

46. 48       96      1440

      24         48      720

       8           16      240

        2           4        ?

(A)  180

(B)  120

(C)  60

(D)  6

Answer: (C)

47. 72 61 51 42 …?…

(A)  34

(B)  36

(C)  31

(D)  33

Answer: (A)

48. Select the set of symbols which can be fitted correctly in the question.

7…2…4…6…3 = 17

(A)  −, ×, +, ÷

(B)  −, +, ×, ÷

(C)  +, ×, +, ÷

(D)  ×, +, ÷, −

Answer: (C)

49. Select the related word from the given alternatives.

Turkey : Lira : Iran : …?…

(A)  Dinar

(B)  Rial

(C)  Yen

(D)  Taka

Answer: (B)

50. If FRIEND is coded as 56, then ENEMY is coded as-

(A)  60

(B)  61

(C)  62

(D)  58

Answer: (C)

Part-II

English Language

Directions-(Q. 51-60) In the following questions given below the following passage some of the words have been left out. Read the passage carefully and choose the correct answer to each question out of the four alternatives and fill in the blanks by blackening the appropriate circle [●] in the Answer Sheet.

    Did the Earth always …(51)… as it does today ? No, …(52)… its origin about 4.8 billion years ago, out planet has …(53)… a lot of changes. The initial ball of fire …(54)… down and the surface slowly …(55)… solid. Today we …(56)… that the Earth’s crust is divided …(57)… several plates, …(58)… move continuously. Entire continents …(59)… been displaced and mountains, islands …(60)… new seas have been formed or have disappeared.

51. 

(A)  looking

(B)  looks

(C)  looked

(D)  look

Answer: (D)

52

(A)  since

(B)  after

(C)  from

(D)  before

Answer: (A)

53.

(A)  undergoes

(B)  underwent

(C)  undergo

(D)  undergone

Answer: (D)

54.

(A)  cooling

(B)  cool

(C)  cools

(D)  cooled

Answer: (D)

55.

(A)  turn

(B)  turned

(C)  has turned

(D)  turning

Answer: (B)

56.

(A)  had known

(B)  knew

(C)  know

(D)  have known

Answer: (C)

57.

(A)  on

(B)  in

(C)  to

(D)  into

Answer: (D)

58.

(A)  who

(B)  whose

(C)  whom

(D)  which

Answer: (D)

59.

(A)  had

(B)  are

(C)  has

(D)  have

Answer: (D)

60.

(A)  and

(B)  so

(C)  or

(D)  but

Answer: (A)

Directions-(Q. 61-64) In the following questions, some parts of the sentences have errors and some are correct. Find out which part of the sentence has an error and blacken the circle [●] corresponding to the appropriate correct option. If a sentence is free from error, blacken the circle [●] corresponding to ‘No Error” in the Answer Sheet.

61. Forty minutes later James return to his family bearing an important piece of news.

(A)  bearing an important piece of news

(B)  No error

(C)  Forty minutes later

(D)  James return to his family

Answer: (D)

62. Tutankhamun’s death mask is still untarnished after thirty four centuries.

(A)  after thirty four centuries

(B)  No error

(C)  Tutankhamun’s death mask

(D)  is still untarnished

Answer: (D)

63. The flower and the vine are growing well.

(A)  are growing well

(B)  No error

(C)  The flower

(D)  and the vine

Answer: B

64. I Jumped with the opportunity of going to Kashmir.

(A)  of going to Kashmir

(B)  No error

(C)  I jumped

(D)  with the opportunity

Answer: (D)

Directions-(Q. 65-68) In the following questions, sentences are given with blanks to be filled in with an appropriate word. Four alternatives are suggested for each question. Choose the correct alternative out of the four and indicate it by blackening the appropriate circle [●] in the Answer Sheet.

65. I have no ……. riches.

(A)  desire for

(B)  desire to

(C)  desirous of

(D)  desire of

Answer: (A)

66. The doctor ……….. the woman that her son would recover.

(A)  assumed

(B)  assured

(C)  ensured

(D)  insured

Answer: (B)

67. He is only an ……. director in the organization but he does his work sincerely.

(A)  honourable

(B)  honorary

(C)  honorific

(D)  honour

Answer: (B)

68. A warm reception was …… him.

(A)  accorded to

(B)  allied to

(C)  given for

(D)  allotted to

Answer: (A)

Directions-(Q. 69-72) In the following questions, out of the four alternatives, choose the one which best expresses the meaning of the given word and mark it in the Answer Sheet.

69. Despair

(A)  Disrupt

(B)  Hopeless

(C)  Distress

(D)  Discourage

Answer: (B)

70. Luxuriant

(A)  Lush

(B)  Lovely

(C)  Class

(D)  Luxurious

Answer: (A)

71. Malady

(A)  Melody

(B)  Parody

(C)  Lady

(D)  Disease

Answer: (D)

72. Languid

(A)  Spirited

(B)  Lethargic

(C)  Luxurious

(D)  Lively

Answer: (B)

Directions-(Q. 73-76) In the following questions, choose the word opposite in meaning to the given word and mark it in the Answer Sheet.

73. Dubious

(A)  Double

(B)  Demise

(C)  Easy

(D)  Assured

Answer: (D)

74. Abhor

(A)  Love

(B)  Miss

(C)  Mad

(D)  Hate

Answer: (A)

75. Cautious

(A)  Alert

(B)  Thoughtful

(C)  Watchful

(D)  Careless

Answer: (D)

76. Mammoth

(A)  Monumental

(B)  Prodigious

(C)  Minute

(D)  Stupendous

Answer: (C)

Directions-(Q. 77-80) In the following questions, four alternatives are given for the Idiom/Phrase underlined. Choose the alternative which best expresses the meaning of the Idiom/Phrase and mark it in the Answer Sheet.

77. To eat humble pie

(A)  to pretend to be humble

(B)  to starve

(C)  to have a low estimate of oneself

(D)  to act apologetically

Answer: (D)

78. There was a definite element of fabrication in the story, but he believed the account hook, line and sinker.

(A)  not wholly

(B)  hardly

(C)  a little bit

(D)  completely

Answer: (D)

79. A wild goose chase

(A)  a good effort

(B)  a desperate effort

(C)  a useless effort

(D)  a baseless effort

Answer: (C)

80. Men like Vidyasagar and Bhagat Singh are the salt of the earth.

(A)  hardworking men

(B)  makers of the earth

(C)  famous men

(D)  ideal men

Answer: (D)

Directions-(Q. 81-84) In the following questions, a sentence/a part of the sentence is underlined. Below are given alternatives to the underlined part which may improve the sentence. Choose the correct alternative. In case no improvement is needed blacken the circle [●] corresponding to “No improvement” in the Answer Sheet.

81. Let you and I go together-

(A)  mine

(B)  No improvement

(C)  my

(D)  me

Answer: (B)

82. He summarily dismissed what his son told him in a fit of rage.

(A)  what his son told in a fit of rage he summarily dismissed

(B)  No improvement

(C)  in a fit of rage he summarily dismissed what his son told him

(D)  He summarily dismissed in a fit of rage what his son told him

Answer: (C)

83. There were times in his life when he struggled for one square meal a day.

(A)  he struggled one day for food

(B)  No improvement

(C)  he keeps struggling for meals

(D)  he struggled with one square meal a day

Answer: (B)

84. Each and everyone of you are guilty of some crime or the other.

(A)  everyone are guilty

(B)  No improvement

(C)  Each and everyone of you is guilty

(D)  Each and each of your are guilty

Answer: (C)

Directions-(Q. 85-88) In the following questions, out of the four alternatives, choose the one which can be substituted for the given words/sentences and indicate it by blackening the appropriate circle [●] in the Answer Sheet.

85. One who sells articles at public sales.

(A)  teller

(B)  milliner

(C)  juggler

(D)  auctioneer

Answer: (D)

86. A general act of forgiveness or pardon-

(A)  Gratification

(B)  Amnesty

(C)  Benediction

(D)  Emancipation

Answer: (B)

87. A partner in crime-

(A)  assistant

(B)  Helper

(C)  accomplice

(D)  alibi

Answer: (C)

88. A place where arms and weapons are stored.

(A)  tannery

(B)  kiln

(C)  arsenal

(D)  granary

Answer: (C)

Directions-(Q. 89-92) In the following questions, four words are given in each question, out of which only one word is correctly spelt. Find the correctly spelt word and mark your answer in the Answer Sheet.

89. Find the correctly spelt word-

(A)  Bouganvella

(B)  Bougenvillea

(C)  Bouganvilla

(D)  Bougainvillea

Answer: (D)

90. Find the correctly spelt word-

(A)  Occurance

(B)  Occurrance

(C)  Occurrence

(D)  Ocurrance

Answer: (C)

91. Find the correctly spelt word-

(A)  Diminutiv

(B)  Diminutive

(C)  Diminuitive

(D)  Dimenutive

Answer: (B)

92. Find the correctly spelt word-

(A)  Courteus

(B)  Courtoeus

(C)  Courtious

(D)  Courteous

Answer: (D)

Directions-(Q. 93-96) In the following questions, the 1st and the part of a sentence are numbered 1 and 6. The rest of the sentence is split into four parts and named P, Q, R and S. These four parts are not given in their proper order. Read the sentence and find out which of the four combinations is correct. Then find the correct answer and indicate it in the Answer Sheet.

93. (1) England’s Cath Webb

(P) with her resolution

(Q) to make a sponge cake every day for a year

(R) hit the headlines in January

(S) and give it to a friend, family member or stranger

(6) just to make them smile.

(A)  SRPQ

(B)  SPRQ

(C)  RQSP

(D)  RPQS

Answer: (D)

94. (1) Some people seem to imagine

(P) and that when you have filled it with anything

(Q) and it becomes as empty

(R) that the human mind is like a bottle

(S) you pour it out again

(6) as it was before.

(A)  RPSQ

(B)  PQRS

(C)  RSPQ

(D)  PRSQ

Answer: (A)

95. (1) The pilgrimage to Mecca

(P) giving alms

(Q) recognizing Allah as the only God

(R) is one of the five religious duties of each Muslim

(S) along with praying five times a day

(6) and fasting four weeks once a year.

(A)  RSQP

(B)  SQRP

(C)  RQPS

(D)  SPQR

Answer: (A)

96. (1) A social network is

(P) provides a clear way of analyzing

(Q) a set of actors and the

(R) a social structure made up of

(S) dyadic ties between these actors

(6) the structure of whole social entities.

(A)  SPRQ

(B)  PQRS

(C)  RQSP

(D)  QSPR

Answer: (C)

Directions-(Q. 97 and 98) In the following questions, a sentence has been given in Direct/Indirect Speech. Out of the four alternatives suggested, select t he one which best expresses the same sentence in Indirect/Direct Speech and mark  your answer in the Answer Sheet.

97. The maid confessed to the police that it was she who had taken the diamond ring.

(A)  The police were told, “I have taken the diamond ring.”

(B)  The maid told the police, “It is I who have taken the diamond ring.”

(C)  The maid said, “I have stolen the diamond ring.”

(D)  The maid said, “It is I who took the diamond ring.”

Answer: (B)

98. My father said to me, “Go now”.

(A)  My father ordered me to go then

(B)  My father ordered to go now

(C)  My father ordered me to go later

(D)  My father ordered me to go now

Answer: (A)

Directions-(Q. 99 and 100) In the following questions, a sentence has been given in Active/Passive Voice, Out of the four alternatives suggested, select the one which best expresses the same sentence in Passive/Active Voice and Mark your answer in the Answer Sheet.

99. He may have left a message for me.

(A)  A message may have been left for me

(B)  A message may have been left for me by him

(C)  He might have left a message for me

(D)  A message might be left for me by him

Answer: (B)

100. Was the question answered?

(A)  Were they answering the question?

(B)  Did the question get answered?

(C)  Did they answer the question?

(D)  Was the answer to the question given?

Answer: (C)

Part-III

Quantitative Aptitude

101. A vendor purchased 40 dozen bananas for Rs 250. Out of these 30 bananas were rotten and could not be sold. At what rate per dozen should be sell the remaining bananas to make a profit of 20%?

(A)  Rs 12

(B)  Rs 6

(C)  Rs 10

(D)  Rs 8

Answer: (D)

102. The average marks of 14 students was 71. It was later found that the marks of one of the students has been wrongly entered as 42 instead of 56 and of another as 74 instead of 32. What is the correct average?

(A)  69

(B)  71

(C)  67

(D)  68

Answer: (A)

103. Average age of seven persons in a group is 30 years, the average age of five persons of this group is 31 years. What is the average age of the other two persons in the group?

(A)  15 years

(B)  26 years

(C)  55 years

(D)  None of these

Answer: (D)

104. One-fifth of half of a number is 20. Then 20% of that number is-

(A)  60

(B)  80

(C)  20

(D)  40

Answer: (D)

105. A man rows to a place 60 km distant and back in 13 hour 30 minutes. He finds that he can row 5 km with the stream in the same time as he can row 4 km against the stream. Find the rate of the stream-

(A)  10 km/hr

(B)  8 km/hr

(C)  1 km/hr

(D)  1/2 km/hr

Answer: (C)

106. A man spends his three months in four months time. If his monthly income is Rs 1000, then his annual savings is-

(A)  Rs 6,000

(B)  Rs 9,000

(C)  Rs 3,000

(D)  Rs 4,000

Answer: (C)

107. In 2kg mixture of copper and aluminium 30% is copper. How much aluminium powder should be added to the mixture so that the proportion of copper becomes 20%?

(A)  1000 gms

(B)  1200 gms

(C)  800 gms

(D)  900 gms

Answer: (A)

108. A man rows 12 km in 5 hours against the stream, the speed of current being 4 kmph. What time will be taken by him to row 15 m with the stream?

(A) 

(B)    

(C)    

(D) 

Answer: (D)

109. Arvind deposited a sum of money with a bank on Ist January, 2012 at 8% simple interest per annum. He received an amount Rs 3,144 on 7th August, 2012. The money he deposited with the bank was-

(A)  Rs 2,500

(B)  Rs 3,000

(C)  Rs 3,100

(D)  Rs 3,080

Answer: (B)

110. Base of a prism of height 10 cm is square. Total surface area of the prism is 192 sq. cm. The volume of the prism is-

(A)  160 cm3

(B)  120 cm3

(C)  640 cm3

(D)  90 cm3

Answer: (A)

111. The time required for a sum of money to amount to four times itself at 15% simple interest p.a. will be-

(A)  15 years

(B)  20 years

(C)  10 years

(D)  30 years

Answer: (B)

112. The radii of a sphere and a right circular cylinder are 3 cm each. If their volumes are equal, then curved surface area of the cylinder is-

(Assume π = 22/7)

(A)    

(B)    

(C)   

(D)   

Answer: (B)

113. A right prism has triangular base. If ‘v’ be the number of vertices. ‘e’ be the number of edges and ‘f be the number of faces of the prism. The value of  is-

(A)  10

(B)  4

(C)  2

(D)  5

Answer: (D)

114. If  then the value of (1 – a + a2) (1 + a – a2) is-

(A)  0

(B)  −4

(C)  1

(D)  4

Answer: (A)

115. A low land, 48 m long and 31.5 m broad is raised to 6.5 dm. For this, earth is removed from a cuboidal hole, 27 m long and 18.2 m broad, dug by the side of the land. The depth of the hole will be-

(A)  2 m

(B)  3 m

(C)  2.2 m

(D)  2.5 m

Answer: (A)

116. If the angles of a triangle are in the ratio 2 : 3 : 5, then the measure of the least angle of the triangle is-

(A)  90°

(B)  20°

(C)  18°

(D)  36°

Answer: (D)

117. If x = 5, then the value of the expression 

(A)  625/24

(B)  24/25

(C)  24/5

(D)  576/25

Answer: (D)

118. (x3 + y6) (x3 – y6) is equal to-

(A)  x6 – y12

(B)  x9 + y36

(C)  x9 – y36

(D)  x6 + y12

Answer: (A)

119. If x = 2, y = 1 and z = −3, then x3 + y3 + z3 – 3xyz is equal to

(A)  2

(B)  8

(C)  6

(D)  0

Answer: (D)

120. The sum of  is-

(A)   

(B)   

(C)   

(D)   

Answer: (D)

121. The length of two parallel chords of a circle of radius 5 cm are 6 cm and 8 cm in the same side of the centre. The distance between them is-

(A)  1 cm

(B)  2 cm

(C)  3 cm

(D)  1.5 cm

Answer: (A)

122. AB is a diameter of a circle having centre at O. P is a point on the circumference of the circle. If ∠POA = 120°, then measure of ∠PBO is-

(A)  68°

(B)  60°

(C)  75°

(D)  70°

Answer: (B)

123. BE, CF are the two medians of ∆ABC and is their point of intersection. EF cuts AG at O. Ratio of AO : OG is equal to-

(A)  3 : 1

(B)  1 : 3

(C)  2 : 1

(D)  2 : 3

Answer: (C)

124. ABC is triangle in which ∠A = 90°. Let P be any point on side AC. If BC = 10 cm, AC = 8 cm and BP = 9 cm, then AP =

(A)  2√3 cm

(B)  3√3 cm

(C)  2√5 cm

(D)  3√5 cm

Answer: (D)

125. ABCD is a cyclic quadrilateral, AB is the diameter of the circle. If ∠ACD = 50°, then measure of ∠BAD is-

(A)  140°

(B)  130°

(C)  50°

(D)  40°

Answer: (D)

126. If 5tan A = 4, then the value of 

(A)  7/9

(B)  9/14

(C)  5/6

(D)  5/14

Answer: (D)

127. The value of (cos 0° + sin 45° + sin 30°) (sin 90° − cos 45° + cos 60°) is-

(A)  5/4

(B)  3/2

(C)  7/4

(D)  3/4

Answer: (C)

128. If x = a sec θ + b tan θ and y = a tan θ + b sec θ (a ≠ b), then the value of   is-

(A)  2

(B)  1/2

(C)  0

(D)  1

Answer: (D)

129. If 2 sin2 θ – 3 sin θ + 1 = 0, θ being positive acute angle, then the values of θ are-

(A)  45°, 50°

(B)  60°, 55°

(C)  30°, 90°

(D)  60°, 45°

Answer: (C)

130. The value of sec4 A(1 – sin4A) – 2tan2 A is-

(A)  2

(B)  1/2

(C)  1

(D)  0

Answer: (C)

131. Which one of the following is true?

(A)    

(B)   

(C)   

(D)   

Answer: (D)

132. The weight of a container completely filled with water is 2.25 kg. The container weighs 0.77 kg when its 0.2 part is filled with water. The weight (in kg) of the container when 0.4 part of it is filled with water is-

(A)  1.14

(B)  1.88

(C)  0.40

(D)  0.74

Answer: (A)

133. 

(A)  1.25

(B)  0

(C)  1/5

(D)  2.4

Answer: (B)

134. A can do half of a piece of work in 1 day, whereas B can do full. B can do half the work as C in 1 day. The ratio of their efficiencies of work is-

(A)  2 : 4 : 1

(B)  4 : 2 : 1

(C)  1 : 2 : 4

(D)  2 : 1 : 4

Answer: (C)

135. 15 men can finish a work in 20 days, however it takes 24 women to finish it in 20 days. If 10 men and 8 women undertake to complete the work, then they will take-

(A)  15 days

(B)  30 days

(C)  10 days

(D)  20 days

Answer: (D)

136. Divide 27 into two parts so that 5 times the first and 11 times the second together equals to 195. Then ratio of the first and second part is-

(A)  5 : 4

(B)  2 : 7

(C)  17 : 10

(D)  3 : 2

Answer: (C)

137. The LCM of two numbers is 12 times their HCF. The sum of the HCF and LCM is 403. If one of the number is 93, then the other is-

(A)  116

(B)  112

(C)  124

(D)  120

Answer: (C)

138. A bag contains coins of Rs 1.50 paise and 25 paise in the ratio 2 : 3 : 5. If the total value of these coins is Rs 228, then the number of 5 paise coins in that bag was-

(A)  124

(B)  144

(C)  96

(D)  112

Answer: (B)

139. Ram, Rahim and Robert started a partnership business investing Rs 30,000, Rs 50,000 and Rs 40,000 respectively. If they made an annual profit of Rs 18,504, then the share of Rahim is-

(A)  Rs 7,010

(B)  Rs 7,510

(C)  Rs 7,710

(D)  Rs 7,170

Answer: (C)

140. After deducting a commission of 5%, a T.V. set costs Rs 9,595. Its marked price is-

(A)  Rs 10,100

(B)  Rs 12,000

(C)  Rs 10,000

(D)  Rs 10,074.75

Answer: (A)

141. A discount series of 10%, 20% and 40% is equal to a single discount of-

(A)  60%

(B)  60.28%

(C)  50%

(D)  56.8%

Answer: (D)

Directions-(Q. 142-146) Read it carefully and answer the questions.

    Total cost of production of a firm is Rs 250 lakh. Following pie-chart shows the percentage of costs of production in different purposes.

Letters A, B, C, D represent the following-

A = Cost of raw materials

B = Cost of packing materials

C = Cost of labour

D = Maintenance cost

142. Cost of packing materials and raw materials together amounts to-

(A)  Rs 175 lakh

(B)  Rs 162.5 lakh

(C)  Rs 87.5 lakh

(D)  Rs 137.5 lakh

Answer: (B)

143. If the cost of production doubles in a period of 3 years, then the corresponding maintenance cost in rupees will be-

(A)  12.5 lakh

(B)  125 lakh

(C)  25 lakh

(D)  74 lakh

Answer: (C)

144. If the production increases to five times of the present cost, then the percentage increase of the cost is-

(A)  500%

(B)  400%

(C)  300%

(D)  50%

Answer: (B)

145. If the packing cost increases by 2%, then the new packing cost will be-

(A)  Rs 1.25 lakh

(B)  Rs 5 lakh

(C)  Rs 62.5 lakh

(D)  Rs 63.75 lakh

Answer: (D)

146. If the total maintenance cost increases from Rs 12.5 lakh to Rs 50 lakh, then the percentage increase of the maintenance cost is-

(A)  25%

(B)  75%

(C)  300%

(D)  400%

Answer: (C)

Directions-(Q. 147-150) Study the following histogram and answer the questions.

147. The range of marks obtained by maximum number of students is-

(A)  30 – 40

(B)  50 – 60

(C)  80 – 90

(D)  60 – 70

Answer: (B)

148. The number of students securing marks in the range 90-100 is-

(A)  40

(B)  30

(C)  20

(D)  10

Answer: (D)

149. The total number of students on whom this survey was made is-

(A)  190

(B)  200

(C)  210

(D)  220

Answer: (C)

150. The percentage of students securing marks less than 50 is-

(A)   

(B)   

(C)   

(D)   

Answer: (B)

For Visually Handicapped/Cerebral Palsy Candidates Only

142. If (x + y) : √xy = 4 : 1, then the value of x : y is-

(A)  (2 + √3) : (2 – √3)

(B)  (1 + √3) : (1 – √3)

(C)  (3 + √3) : (3 – √3)

(D)  (4 + √3) : (4 – √3)

Answer: (A)

143. If 25x2 – 20x + t becomes a perfect square, then the value of t is-

(A)  0

(B)  1

(C)  2

(D)  4

Answer: (D)

144. If a2 = a + 1, then the value of a3 – 2a is-

(A)  1

(B)  0

(C)  −1

(D)  2

Answer: (A)

145. If a, b and c are real and rational numbers and b2 – 4ac = 0, then the types of the roots of ax2 + bx + c = 0 is-

(A)  Real and equal

(B)  Real and unequal

(C)  Irrational

(D)  Complex

Answer: (A)

146. The distance between two points (−5, 0) and (7, 0) is-

(A)  12 units

(B)  7 units

(C)  5 units

(D)  13 units

Answer: (A)

147. If x sin α tan α + y cos α cot α = 1 and x sin α – y cos α = 0, then the value of x2 + y2 is

(A)  1/2

(B)  2

(C)  1

(D)  3/2

Answer: (C)

148. The angle of elevation of a ladder, leaning against a wall is 60° and the foot of the ladder is 4.6 m away from the wall. The length of the ladder is-

(A)  2.3 m

(B)  4.6 m

(C)  7.8 m

(D)  9.2 m

Answer: (D)

149. Length of each edge of a regular tetrahedron is 6 cm. The volume of the tetrahedron is-

(A)  18√3 cm3

(B)  18 √2 cm3

(C)  36√2 cm3

(D)  36√3 cm3

Answer: (B)

150. Three circles having same radius with centres at A, B and C touch externally to each other. Then ABC form a triangle which is-

(A)  right-angled

(B)  equilateral

(C)  isosceles

(D)  scalene

Answer: (B)

Part-IV

General Awareness

151. Where is Brabourne Stadium located?

(A)  Jamshedpur

(B)  Kolkata

(C)  Cuttack

(D)  Mumbai

Answer: (D)

152. Who is the founder of Homeopathy?

(A)  Hahnemann

(B)  Domagk

(C)  Lainnec

(D)  Wakesman

Answer: (A)

153. When is Human Rights Day celebrated?

(A)  10th Oct.

(B)  12th Dec.

(C)  9th Dec.

(D)  10th Dec.

Answer: (D)

154. Television was invented by-

(A)  R. A. Millikan

(B)  Lawrence

(C)  Louis Braille

(D)  J. L. Baird

Answer: (D)

155. India’s 1st Hi-Speed Rural Broadband Network has been commissioned in a district of-

(A)  Kerala

(B)  Karnataka

(C)  Andhra Pradesh

(D)  Telangana

Answer: (A)

156. Mercury thermometer was invented by-

(A)  Newton

(B)  Priestley

(C)  Galileo

(D)  Fahrenheit

Answer: (D)

157. Amjad Ali Khan is associated with which of the following musical instruments?

(A)  Sarod

(B)  Veena

(C)  Violin

(D)  Sitar

Answer: (A)

158. Reserve Bank of India was nationalized in-

(A)  1948

(B)  1949

(C)  1945

(D)  1947

Answer: (B)

159. What is Hawala?

(A)  Illegal trading of shares

(B)  Tax evasion

(C)  Full details of a subject

(D)  Illegal transactions of foreign exchange

Answer: (D)

160. Who was the first to use the term “Micro” and “Macro” in Economics in 1933?

(A)  Gurly

(B)  I. Fischer

(C)  James Tobin

(D)  Ragnar Frisch

Answer: (D)

161. Which is the final Appellate Court of Justice?

(A)  Civil Court

(B)  District Court

(C)  Supreme Court

(D)  High Court

Answer: (C)

162. The system in which the few govern many is known as-

(A)  Plutocracy

(B)  Autocracy

(C)  Monarchy

(D)  Oligarchy

Answer: (D)

163. Which of the following regulates the working of share market in India?

(A)  BIFR

(B)  SEBI

(C)  MRTP Act

(D)  FERA

Answer: (B)

164. Aggregate Monetary Resources is-

(A)  M3

(B)  M4

(C)  M1

(D)  M2

Answer: (A)

165. According to which Article, State Legislative Council can be created or abolished?

(A)  Article 169

(B)  Article 170

(C)  Article 167

(D)  Article 168

Answer: (A)

166. The last dynasty of the Delhi Sultanate was the-

(A)  Lodi dynasty

(B)  Khilji dynasty

(C)  Slave dynasty

(D)  Syed dynasty

Answer: (A)

167. The ‘Great Bath’ was found at-

(A)  Lothal

(B)  Chanhudaro

(C)  Harappa

(D)  Mohenjodaro

Answer: (D)

168. Which country earned the title “The Queen of Seas”?

(A)  Italy

(B)  Britain

(C)  Germany

(D)  France

Answer: (A)

169. Who was the first Deputy Prime Minister of India?

(A)  Maulana Abul Kalam Azad

(B)  Gulzarilal Nanda

(C)  Jawaharlal Nehru

(D)  Sardar Vallabhabhai Patel

Answer: (D)

170. When did India become a fully Sovereign Democratic Republic?

(A)  26th January, 1949

(B)  26th November, 1951

(C)  26th November, 1949

(D)  26th November, 1930

Answer: (C)

171. The Earth is nearest to the Sun on-

(A)  21st March

(B)  23rd September

(C)  3rd January

(D)  4th July

Answer: (C)

172. A sufficiently large scale map indicating the detailed surface features of an area including relief is called-

(A)  Topographical map

(B)  Wall map

(C)  Relief map

(D)  Chorographical map

Answer: (A)

173. The lowest temperature is recorded by-

(A)  Maximum reading thermometer

(B)  Minimum reading thermometer

(C)  Mercurial thermometer

(D)  Alcohol thermometer

Answer: (D)

174. To whom did Allauddin Khilji entrust the mission to conquer South?

(A)  Khizra Khan

(B)  Shaji Malik

(C)  Ulugha Khan

(D)  Malik Kafur

Answer: (D)

175. Who was the first Delhi Sultan to bream the power of the Turkish nobles known as the ‘Chahalgani’ or the ‘Forty’?

(A)  Raziya

(B)  Balban

(C)  Qutb-ud-din Aibak

(D)  Illutmish

Answer: (B)

176. Centre for Ecological Sciences is situated at-

(A)  New Delhi

(B)  Bengaluru

(C)  Karnal

(D)  Allahabad

Answer: (B)

177. The first National Park of India is-

(A)  Simlipal National Park

(B)  Hazaribagh National Park

(C)  Dachigam Wild Life Sanctuary

(D)  Corbett National Park

Answer: (D)

178. Penicillin was discovered by-

(A)  Edward Jenner

(B)  Louis Pasteur

(C)  Ian Fleming

(D)  Alexander Fleming

Answer: (D)

179. Vitamin which provides immunity-

(A)  K

(B)  C

(C)  A

(D)  E

Answer: (C)

180. Respiration is controlled by …… part of brain.

(A)  Hypothalamus

(B)  Cerebellum

(C)  Olfactory Lobes

(D)  Medulla Oblongata

Answer: (D)

181. In marine whales, the limbs are modified as-

(A)  Slippers

(B)  Flippers

(C)  Flappers

(D)  Grippers

Answer: (B)

182. The most stable measure of central tendency is-

(A)  Median

(B)  Range

(C)  Mean

(D)  Mode

Answer: (A)

183. Of the following Rapeseed belongs to-

(A)  Pepper

(B)  Linseed

(C)  Mustard

(D)  Coffee

Answer: (C)

184. A defect of vision in which the points in one plane an object appear in focus while those in another plane are out of focus is called-

(A)  Astigmatism

(B)  Distortion

(C)  Myopia

(D)  Hypermetropia

Answer: (A)

185. A plane glass slab is kept over coloured letters, the letter which appears least raised is-

(A)  Blue

(B)  Green

(C)  Red

(D)  Violet

Answer: (C)

186. An electronic path, that sends signals from one part of computer to another is-

(A)  Bus

(B)  Serial Port

(C)  Logic Gate

(D)  Modem

Answer: (A)

187. ICMP is sued for-

(A)  Forwarding

(B)  Multicasting

(C)  Error reporting

(D)  Addressing

Answer: (C)

188. Mass number is the sum of-

(A)  Electrons and neutrons

(B)  Only protons

(C)  Electrons and protons

(D)  Protons and neutrons

Answer: (D)

189. The resistance of an ideal voltmeter is-

(A)  high

(B)  low

(C)  infinite

(D)  zero

Answer: (C)

190. Which of the following works on the basis of conservation of linear momentum?

(A)  Rocket

(B)  Helicopter

(C)  Jet

(D)  Aeroplane

Answer: (A)

191. The hard enamel layer of teeth is-

(A)  Calcium hydroxyl apatite

(B)  Calcium hydroxide

(C)  Calcium phosphate

(D)  Calcium oxide

Answer: (A)

192. The two components of an eco-system are-

(A)  Plants and light

(B)  Weeds and micro-organisms

(C)  Plants and animals

(D)  Biotic and abiotic

Answer: (D)

193. There are only two metals that are non-silver in colour, they are-

(A)  Palladium & Platinum

(B)  Sodium & Magnesium

(C)  Nickel & Zinc

(D)  Copper & Gold

Answer: (D)

194. Breath analysers used by police to test drunken drivers works on the chemical basis of-

(A)  Complexation reactions

(B)  Redox reactions

(C)  Acid-base reactions

(D)  Precipitation reactions

Answer: (C)

195. Which of the following is the best indicator of SO2 pollution?

(A)  Lichen

(B)  Algae

(C)  Bryophyte

(D)  Pteridophyte

Answer: (A)

196. Which scientist wrote a book a called “A Brief History of Time”?

(A)  J. L. Baird

(B)  Edward Jenner

(C)  Stephen Hawking

(D)  Pasteur

Answer: (C)

197. Which of the following is observed as Sports Day every year?

(A)  29th August

(B)  2nd October

(C)  22nd April

(D)  26th July

Answer: (A)

198. The World Environment Day is celebrated on-

(A)  6 August

(B)  16 June

(C)  5 June

(D)  7 April

Answer: (C)

199. Which of the following species is locally extinct in India?

(A)  White Bellied Heron

(B)  Forest Owlet

(C)  Siberian Crane

(D)  The Gyps vulture

Answer: (B)

200. Which gas is responsible for depletion of ozone layer around earth which protect us from harmful ultraviolet rays?

(A)  Oxygen

(B)  Nitrogen oxide

(C)  Chlorofluro carbons

(D)  Nitrogen

Answer: (C)

SSC Combined Higher Secondary Level (10+2) Examination-2016 Tier-I Held on 7 February, 2017 Question Paper With Answer Key

SSC Combined Higher Secondary Level (10+2) Examination-2016 Tier-I Held on 7 February, 2017
SSC Combined Higher Secondary Level (10+2) Examination-2016 Tier-I Held on 7 February, 2017 Question Paper With Answer Key

Staff Selection Commission Combined Higher Secondary Level (10+2)

Examination-2016 Tier-I Held on 7 February, 2017

Directions-(Q. 1 to 4) Select the related word/letters/number from the given alternatives.

1. A Suitable Boy : Vikram Seth :: Gitanjali : ?

(A)  R. K. Narayan

(B)  Mulk Raj Anand

(C)  Rabindra Nath Tagore

(D)  Munshi Prem Chand

Answer: (C)

2. ASK : KAS :: SAW : ?

(A)  WAS

(B)  ASW

(C)  PQR

(D)  WSA

Answer: (D)

3. MALAYALAM : MALAYALAM :: SYSTEM : ?

(A)  METSYS

(B)  TEMSYS

(C)  SYSTEMS

(D)  MTESYS

Answer: (A)

4. 36 : 3 :: 108 : ?

(A)  9

(B)  13

(C)  12

(D)  16

Answer: (A)

Directions – (Q. 5 to 8) Select the odd word/letters/number pair from the given alternatives.

5.

(A)  Flamingo

(B)  Hawk

(C)  Money

(D)  Penguin

Answer: (C)

6.

(A)  AEG

(B)  END

(C)  IUC

(D)  OIL

Answer: (B)

7.

(A)  31

(B)  37

(C)  39

(D)  41

Answer: (C)

8.

(A)  236

(B)  327

(C)  248

(D)  177

Answer: (B)

Directions- (Q. 9 to 12) A series is given with one term missing. Select the correct alternative from the given ones that will complete the series.

9. Mercury, Venus, Earth ?

(A)  Jupiter

(B)  Saturn

(C)  Mars

(D)  Pluto

Answer: (C)

10. LOR, PSV, TWZ, ?

(A)  YBE

(B)  XAD

(C)  XAB

(D)  YBD

Answer: (B)

11. BCD, HIJ, OPQ, ?

(A)  WZA

(B)  WXY

(C)  VWX

(D)  XYZ

Answer: (B)

12. 28, 42, 56, ?

(A)  70

(B)  77

(C)  72

(D)  84

Answer: (A)

Directions-In the following question, two statements are given each followed by two conclusions I and II. You have to consider the statements to be true even if they seem to be at variance from commonly known facts. You have to decide which of the given conclusions, if any, follows from the given statements.

13. Statements I : The sowing of winter crops has been completed in 87% of the seasonal area and its above normal this time.

Conclusions:

(I) It is considered as delay because the potential of sowing is not completely utilized.

(II) Cultivation is based upon timings.

(A)  Only Conclusion II follows

(B)  Conclusions I and II both follow

(C)  Neither I nor II follow

(D)  Only conclusion I follows

Answer: (C)

14. Seven people are standing facing south. B is second to the right of the person in the middle and C is third to the left of B. A is to the immediate left of C. What is the position of A with respect to the person in the middle?

(A)  Second to the right

(B)  Immediate right

(C)  Immediate left

(D)  Second to the left

Answer: (D)

15. Arrange the given words in the sequence in which they occur in the dictionary.

(1) Addiction           (2) Adhesive

(3) Astonishing        (4) Adhere

(A)  1, 4, 2, 3

(B)  4, 3, 1, 2

(C)  2, 3, 1, 4

(D)  1, 4, 3, 2

Answer: (A)

16. In a certain code language, ‘DOMAIN’ is written as ‘563471’ and ‘TIRED’ is written as ‘27895’. How is ‘TENDER’ written in the code language?

(A)  267589

(B)  291589

(C)  291598

(D)  293598

Answer: (C)

Direction-In the following question, select the missing number from the given series.

17. 

(A)  289

(B)  729

(C)  512

(D)  527

Answer: (B)

18. If ‘−’ means ‘plus’, ‘×’ means ‘divided by’, ‘÷’ mean ‘multiplied by’ and ‘+’ mean minus, then- 357 × 7 ÷ 3 + 53 – 10 = ?

(A)  100

(B)  110

(C)  137

(D)  140

Answer: (B)

Direction-In the following question, which one set of letters when sequentially placed at the gaps in the given letter series shall complete it?

19. p_r_p_qr_p_

(A)  qpqrqr

(B)  qpqrpq

(C)  pqprpr

(D)  pqrpqr

Answer: (A)

20. A man is facing the west direction and from there he walks 10 km in the right and then turns left and walks another 6 km. Then, he turns 90° clockwise and walks for another 6 km and finally walks 6 km in the east. Find the distance between his initial and final position.

(A)  8 km

(B)  6 km

(C)  10 km

(D)  √244 km

Answer: (B)

21. A word is represented by only one set of numbers as given in any one of the alternatives. The sets of numbers given in the alternatives are represented by two classes of alphabets as shown in the given two matrices. The columns and rows of Matrix-I are numbered from 0 to 4 and that of Matrix-II are numbered from 5 to 9. A letter from these matrices can be represented first by its row and next by its column, for example, ‘A’ can be represented by 12, 23 etc. and ‘G’ can be represented by 67, 78 etc. Similarly, you have to identify the set for the word ‘SONNET’.

(A)  10, 57, 85, 96, 76, 00

(B)  21, 68, 24, 85, 55, 22

(C)  32, 79, 13, 67, 77, 34

(D)  43, 96, 02, 24, 99, 44

Answer: (B)

22. Pointing to a person, a man says, “His sister’s father is the uncle of my wife”. How is the person related to the man?

(A)  Uncle

(B)  Brother-in-law

(C)  Father

(D)  Father-in-law

Answer: (B)

23. If a mirror is placed on the line MN, then which of the answer figures in the right image of the given figure?

Answer: (C)

24. Identify the diagram that best represents the relationship among the given classes.

Languages, Hindi, Bengai, Ramayan

Answer: (D)

25. A piece of paper is folded and punched as shown below in the question figures. From the given answer figures, indicate how it will appear when opened.

Answer: (D)

26. Where do metabolism cell reactions take place?

(A)  vacuoles

(B)  mitochondria

(C)  nucleus

(D)  centrioles

Answer: (B)

27. Which of these is not a language in India?

(A)  Sindhi

(B)  Sikh

(C)  Kannada

(D)  Konkami

Answer: (B)

28. ‘Ranni ki vav’ in Gujarat is a famous …………. .

(A)  University

(B)  Valley of Flowers

(C)  Stepwell

(D)  Temple

Answer: (C)

29. Who among the following hasn’t received Bharat Ratna?

(A)  Atal Bihari Vajpayee

(B)  Madan Mohan Malaviya

(C)  Dr. B. R. Ambedkar

(D)  V. P. Singh

Answer: (D)

30. Which drug is used to cure Allergies?

(A)  Fexofenadine

(B)  Ketoconazole

(C)  Azithromycin

(D)  Bupropion

Answer: (A)

31. Oytes belongs to the phylum …………. .

(A)  Arthropoda

(B)  Annelida

(C)  Platyhelminthes

(D)  Mollusca

Answer: (D)

32. Atomic mass of Oxygen is 16 and atomic number is 8. What is the mass in grams of 2 moles of oxygen gas?

(A)  8

(B)  64

(C)  32

(D)  16

Answer: (B)

33. Atomic number of which of the following elements is greater than that of Potassium-

(A)  Sulphur

(B)  Chlorine

(C)  Calcium

(D)  Argon

Answer: (C)

34. Commonly used abbreviation SQL in computer science stands for?

(A)  System Query Language

(B)  Search Query Language

(C)  Structured Query Language

(D)  Single Query Language

Answer: (C)

35. Priya likes a wedding gown so much that she was willing to pay even Rs 30,000 for it. Luckly she finds the same gown marked Rs 25,000 at a local store. The store is further offering a 30% discount on the gown. Priya’s consumer surplus is-

(A)  Rs 5,000

(B)  Rs 12,500

(C)  Rs 7,500

(D)  Rs 25,000

Answer: (B)

36. An increase in the price of tomatoes will cause ………. .

(A)  an increase in the supply of tomatoes

(B)  a decrease in the supply of tomatoes

(C)  an increase in the supply of tomato ketchup

(D)  a decrease in the supply of tomato ketchup

Answer: (D)

37. Kawal Tiger Reserve is in which state?

(A)  Karnataka

(B)  Telangana

(C)  Tamil Nadu

(D)  West Bengal

Answer: (B)

38. Ranthambore National Park is in which state?

(A)  Madhya Pradesh

(B)  Rajasthan

(C)  Gujarat

(D)  Chhattisgarh

Answer: (B)

39. Which sector has the highest number of people working in India?

(A)  Manufacturing

(B)  Agriculture

(C)  IT

(D)  Services

Answer: (B)

40. An artificial navigation channel is called …………… .

(A)  Canal

(B)  Greenhouse effect

(C)  Fjord

(D)  Caprock

Answer: (A)

41. Pyongyang is the Capital city of …………. .

(A)  Saudi Arabia

(B)  North Korea

(C)  Czech Republic

(D)  Trinidad and Tobago

Answer: (B)

42. Tipu Sultan lost his life in the ………… Anglo-Mysore war.

(A)  1st

(B)  2nd

(C)  3rd

(D)  4th

Answer: (D)

43. Jodha Bai was married to ………… .

(A)  Babur

(B)  Humayun

(C)  Akbar

(D)  Aurangzeb

Answer: (C)

44. Tetris, a type of video game was invented by ………… .

(A)  Alexander Parkes

(B)  Luigi Palmieri

(C)  Alexey Pajitnov

(D)  Ransom Eli Olds

Answer: (C)

45. …………. is the mechanical transfer of energy to a system or from a system by an external force on it.

(A)  Work

(B)  Power

(C)  Intensity

(D)  Force

Answer: (A)

46. What is myopia?

(A)  Inability to see distant objects clearly

(B)  Abnormal functioning of the thyroid gland

(C)  It is a condition of enlargement of heart

(D)  It is a kidney disorder

Answer: (A)

47. When was the Election Commission of India formed?

(A)  1947

(B)  1950

(C)  1991

(D)  1994

Answer: (B)

48. The number of parliamentary seats (Lok Sabha) of Tamil Nadu is ………… .

(A)  2

(B)  13

(C)  20

(D)  39

Answer: (D)

49. Kimi Raikkonen is associated with which Sport?

(A)  Lawn Tennis

(B)  Basketball

(C)  Formula One

(D)  WWE

Answer: (C)

50. Who is the author of ‘Love Stories that Touched My Heart’?

(A)  Amitav Ghosh

(B)  Mitali Meelan

(C)  Ravinder Singh

(D)  Sudha Murty

Answer: (C)

51. If  then the value of is-

(A)  2

(B)  2/√3

(C)  √2

(D)  −√2

Answer: (A)

52. If cot(A/2) = , then the value of x is-

(A) 

(B) 

(C) 

(D) 

Answer: (C)

53. If  then the value of x is-

(A) 

(B) 

(C) 

(D) 

Answer: (B)

54. At 19% discount the selling price of a freezer is Rs 24,000, what is the selling price if the discount is 32.5%?

(A)  Rs 13,500

(B)  Rs 20,000

(C)  Rs 16,065

(D)  Rs 21,465

Answer: (B)

55. How many balls of radius 10 cm can be made by melting a bigger ball of diameter 80 cm?

(A)  64

(B)  128

(C)  32

(D)  48

Answer: (A)

56. On dividing a number by 54, we get 86 as quotient and 13 as remainder. What is the number?

(A)  4657

(B)  4631

(C)  4618

(D)  4670

Answer: (A)

57. In the first 44 overs of a cricket match, the run rate was 5.1 runs/over. Calculate the required run rate in the remaining 6 overs to reach the target of 294 runs?

(A)  12.19

(B)  12.79

(C)  11.60

(D)  10.99

Answer: (C)

58. ∆GHI is similar to ∆ If the ratio of Perimeter of ∆GHI : Perimeter of ∆KLM = 4 : 9 and length of GH is 8 cm what is the length of the corresponding side KL?

(A)  81 cm

(B)  18 cm

(C)  27 cm

(D)  54 cm

Answer: (B)

59. A(4, −3) and C(0, 7) are vertices of a square ABCD. Find equation of diagonal BD?

(A)  2x – 5y = 6

(B)  2x + 5y = −6

(C)  2x – 5y = −6

(D)  2x + 5y = 6

Answer: (C)

60. What is the measure of an exterior angle of a regular nonagon?

(A)  40°

(B)  45°

(C)  36°

(D)  60°

Answer: (A)

61. The circumference of a circle is 88 cm, find its diameter?

(A)  14 cm

(B)  7 cm

(C)  28 cm

(D)  21 cm

Answer: (C)

62. If 10A = 3B = 7C; find the A : B : C _____

(A)  30 : 70 : 21

(B)  21 : 70 : 30

(C)  70 : 21 : 30

(D)  7 : 3 : 10

Answer: (B)

63. Coefficient of x2 in (x + 5) (3 – 2x) (4x – 2) is____

(A)  24

(B)  32

(C)  −24

(D)  −32

Answer: (C)

64. On a certain Principal if the simple interest for two years is Rs 2,600 and Compound interest for the two years is 2.769, what is the rate of interest?

(A)  6.5%

(B)  26%

(C)  19.5%

(D)  13%

Answer: (D)

65. If x + y = 14 and x2 + y2 = 100, then find xy__

(A)  48

(B)  45

(C)  40

(D)  33

Answer: (A)

66. The total cost of a refrigerator with an oven was Rs 12,000. The oven was sold at a profit of 20% and the refrigerator at a loss of 28%. If the sale price was the same in both the items, then the cost price of the cheaper item was__

(A)  Rs 4,500

(B)  Rs 7,500

(C)  Rs 6,000

(D)  Rs 5,000

Answer: (A)

67. A does 75% of a work in 45 days. He then calls in B and they together finish the remaining work in 5 days. How long B alone would take to do the whole work?

(A)  15 days

(B)  45 days

(C)  60 days

(D)  30 days

Answer: (D)

68. Two cars travel from city A to city B at a speed of 48 and 60 km/hr respectively. If one car take 3.5 hours lesser time then the other car for the journey, then the distance between City A and City B is_____

(A)  1008 km

(B)  1260 km

(C)  840 km

(D)  672 km

Answer: (C)

69. The sum of the digits of a 2-digit number is 12, if we add 54 to the number, then new number obtained is a number formed by interchange of the digits. Find the number.

(A)  93

(B)  63

(C)  36

(D)  39

Answer: (D)

70. If 2x + 2(1 – x) > 3x – 2 > 2x – 5; then x can take which of the following values?

(A)  −2

(B)  2

(C)  4

(D)  −4

Answer: (A)

71. If the radius of a circle is increased by 12% then its area increases by-

(A)  24%

(B)  25.44%

(C)  12%

(D)  12.72%

Answer: (B)

72. Refer the below data table and answer the following question-

What is the value of the total stock (in lakh rupees)?

(A)  948.1

(B)  130

(C)  388

(D)  94.81

Answer: (D)

73. Refer the below data table and answer the following questions :

If the imports in 2012 was Rs 900 crores and the total exports in the years 2012 and 2013 together was Rs 4,000 crores, then the imports in 2013 was?

(A)  2875

(B)  3738

(C)  1125

(D)  2212

Answer: (B)

74. Refer the below data table and answer the following question:

What was the increase in the height of the child from the 10th Birthday to the 14th Birthday?

(A)  25 cms

(B)  20 cms

(C)  30 cms

(D)  15 cms

Answer: (B)

75. Refer the below data table and answer the following question:

Between 10 pm to 6 am, a fitness band records the following data. How long was the user in Light sleep or in Extremely light sleep?

(A)  1.7 hours

(B)  2.7 hours

(C)  3.2 hours

(D)  3.7 hours

Answer: (C)

Directions-(Q. 76 and 77) Rearrange the parts of the sentence in correct order.

76. A certified, verifiable

B would be a valuable

Q all-purpose identity card

R document for many people

(A)  QPR

(B)  RQP

(C)  PQR

(D)  PRQ

Answer: (A)

77. As for the tasks

P three important ones

Q ahead, there appear

R to be at least

(A)  RQP

(B)  QRP

(C)  PRQ

(D)  PQR

Answer: (B)

78. Select the antonym of

to contrive

(A)  to concoct

(B)  to plot

(C)  to forge

(D)  to raze

Answer: (D)

Direction-In the following question, sentence given with blank is to be filled in with an appropriate word. Select the correct alternative out of the four and indicate it by selecting the appropriate option.

79. Because a waterfall is so ……., people can sit and watch it for hours.

(A)  enslaving

(B)  deviating

(C)  magnetising

(D)  captivating

Answer: (D)

Direction-In the following question, out of the four alternatives, select the alternative which is the best substitute of the phrase.

80. The process by which air is circulated through, mixed with or dissolved in a liquid or substance.

(A)  aerating

(B)  miasma

(C)  effluvium

(D)  volatile

Answer: (A)

Direction-(Q. 81-83) In the following question, out of the four alternatives, select the alternatives which best expresses the meaning of the idiom/phrase.

81. to go down like a lead balloon

(A)  losing all the money while gambling

(B)  an idea on which one has a lot of hope, but it fails

(C)  failure in business or venture

(D)  a speech, proposal, or joke that is poorly received

Answer: (D)

82. Select the synonym of

pate

(A)  radical

(B)  abject

(C)  basal

(D)  dome

Answer: (D)

83. To get one’s walking papers

(A)  One has the right to entry within a country or community

(B)  One is discharged from a hospital or mental asylum

(C)  One can leave

(D)  One is free of imprisonment

Answer: (C)

Direction-In the following question, a sentence has been given in Direct/Indirect speech. Out of the four alternatives suggested, select the one, which best express the same sentence in Indirect/Direct speech.

84. Jay says, “I eat a guava”.

(A)  Jay said, that he ate a guava

(B)  Jay said, that he has been eating guavas.

(C)  Jay says, that he eats a guava.

(D)  Jay said, he ate a guava.

Answer: (C)

Directions-(Q. 85 and 86) Improve the bracketed part of the sentence.

85. Candidates will have to (give) the test again tomorrow.

(A)  do

(B)  take

(C)  perform

(D)  no improvement

Answer: (B)

86. Tanveer has wisdom, charm, and (he had a good) sense of humour.

(A)  a good

(B)  Tanveer has a good

(C)  he has a good

(D)  no improvement

Answer: (A)

Direction-Select the synonym of

87. ague

(A)  virile

(B)  hale

(C)  malaria

(D)  potent

Answer: (C)

Direction-Select the antonym of

88. to impair

(A)  to debilitate

(B)  to mar

(C)  to mend

(D)  to blunt

Answer: (C)

Direction-Select the word with the correct speling.

89.

(A)  sloting

(B)  dowagers

(C)  carbonyle

(D)  asigned

Answer: (B)

Direction-In the following question, out of the four alternatives, select the alternative which best expresses the meaning of the idiom/phrase.

90. To make a sharp sound or series of sounds as a result of a hard object striking another.

(A)  innuendo

(B)  clack

(C)  swat

(D)  wallop

Answer: (B)

Direction-In the following question, a sentence has been given in Active/Passive voice. Out of four alternatives suggested, select the one, which best expresses the same sentence in Passive/Active voice.

91. Priya bought the car last year

(A)  The car was being bought by Priya last year

(B)  The car has been bought by Priya last year

(C)  The car was bought by Priya last year

(D)  The car had been bought by Priya last year

Answer: (C)

Directions-(Q. 92 and 93) In the following question, some part of the sentence may have errors. Find out which part of the sentence has an error and select the appropriate option. If a sentence is free from error, select ‘No error’.

92. The airplane took (A)/ of a soon as I (B)/ arrived at the airport (C)/ No error (D)

Answer: (B)

93. This is in between (A)/ you and me, not (B)/ the entire neighbourhood. (C)/ No error (D)

Answer: (A)

Direction-Select the word with the correct spelling.

94.

(A)  stabling

(B)  grupings

(C)  lobotomi

(D)  alusive

Answer: (A)

Direction-In the following question, sentence given with blank is to be filled in the with an appropriate word. Select the correct alternative out of four and indicate it by selecting the appropriate option.

95. The approaching hurricane is sure to be a ……. that will affect the lives of millions.

(A)  wreck

(B)  misadventure

(C)  calamity

(D)  mess

Answer: (C)

Directions-(Q. 96-100) In the following passage, some of the words have been left out. Read the passage carefully and select the correct answer for the given blank out of the four alternatives.

    The ultimate objective is to ensure that each citizen participates ………… the multiple transactions of society with confidence, and with the assurance that her ‘voice’ carries ………. weight as her ……….. That is, the transition ……….. subject to agent can only be achieved if people are liberated from ……. poverty and grinding illiteracy, malnutrition, hunger and homelessness.

96. The ultimate objective is to ensure that each citizen participates ………… .

(A)  into

(B)  at

(C)  in

(D)  for

Answer: (C)

97. with the assurance that her ‘voice’ carries ……… .

(A)  as much

(B)  as much as

(C)  so much

(D)  much more

Answer: (A)

98. weight as her ……

(A)  neighbour’s

(B)  neighbours’

(C)  neighbours

(D)  neighbour

Answer: (A)

99. the transition …… subject to agent

(A)  to

(B)  so

(C)  with

(D)  from

Answer: (D)

100. people are liberated from ……. poverty and grinding.

(A)  soothing

(B)  debilitating

(C)  encouraging

(D)  strengthening

Answer: (B)

SSC Combined Graduate Level (Tier-1) Exam-2019 Held on 06-03-2020 Time 1.00-2.00 PM Question Paper With Answer Key

SSC Combined Graduate Level (Tier-1) Exam-2019 Held on 06-03-2020 Time 1.00-2.00 PM
SSC Combined Graduate Level (Tier-1) Exam-2019 Held on 06-03-2020 Time 1.00-2.00 PM Question Paper With Answer Key

SSC Combined Graduate Level (Tier-1) Exam-2019 Held on 06-03-2020 Time 1.00-2.00 AM

General Intelligence and Reasoning

1. Which of the following sets is the best represented by the given Venn diagram.

(a)   Weeks, Years, Months

(b)   Truck, Ships, Transport

(c)   Paragraph, Sentence, Pen

(d)   House, Window, Tenant

Answer: (a)

2. Which of the option figures is the exact mirror image of the given figure when the mirror is held at the right side?

Answer: (a)

3. Select the number that can replace the question mark (?) in the following series.

11, 12, 36, 41, ?, 296

(a)   287

(b)   123

(c)   48

(d)   205

Answer: (a)

4. In a certain code language ‘VERBAL’ is written as ‘QNMYXK’ and ‘REACH’ is written as ‘MNXZF’. How will ‘BRAVE’ be written as in that language?

(a)   KQNMX

(b)   YNZQM

(c)   KMQXN

(d)   YMXQN

Answer: (d)

5. Read the given statements and conclusion carefully. Assuming that the information given in the statements is true, even if it appears to be at variance with commonly known facts, decide which of the given conclusions logically follow from the statements.

Statements:

(1) All books are magazines.

(2) Some books are novels.

Conclusions:

(I) Some magazines are books.

(II) No magazine is a book.

(III) No novel is a magazine.

(IV) Some novels are books.

(a)   Only conclusion II and III follow

(b)   Only conclusion I and II follow

(c)   Only conclusion I and IV follow

(d)   Only conclusion I, II and IV follow

Answer: (c)

6. Select the option in which the numbers are related in the same way as are the numbers in the given set.

(343, 98, 21)

(a)   (1728, 288, 24)

(b)   (512, 126, 27)

(c)   (1331, 242, 33)

(d)   (217, 72, 18)

Answer: (c)

7. Select the figure that can replace the question mark (?) in the following series.

Answer: (c)

8. In a certain code language, BREATHER is coded as 29512859. How will AVIATION be coded as in that language?

(a)   12912954

(b)   14912956

(c)   12910965

(d)   14912965

Answer: (d)

9. Select the cube that can be formed by folding the given sheet (unfolded box) along the lines.

Answer: (c)

10. Select the set of letters that when sequentially placed in the blanks of the given letter series will complete the series.

_m_ln_mlmnk_k_nl

(a)   l, n, k, l, n

(b)   k, l, k, l, m

(c)   m, n, k, l, m

(d)   k, n, k, l, m

Answer: (d)

11. Four number-pairs have been given, out of which three are alike in some manner and one is different. Select the number-pair that is different.

(a)   28, 5

(b)   14, 8

(c)   56, 2

(d)   16, 7

Answer: (a)

12. Four letter-clusters have been given, out of which three are alike in some manner, while one is different. Select the odd letter-cluster.

(a)   VINE

(b)   LAIM

(c)   SWJK

(d)   NOHA

Answer: (c)

13. Select the letter-cluster that can replace the questions mark (?) in the following series.

qDx, nHs, kLn, hPi, ?

(a)   eSd

(b)   eTd

(c)   fTe

(d)   EtD

Answer: (b)

14. A group of three friends, K, L and M, are sitting in a café. Their average age is 24 years, Another friend ‘N’ joins the group and the new average age of the group becomes 23 years. If another friend ‘R’, whose age is 2 years more than that of ‘N’, replaces ‘K’, then the average age of L, M, N and R becomes 22.5 years. What is the age of K?

(a)   22  years

(b)   24 years

(c)   20 years

(d)   25  years

Answer: (b)

15. How many triangles are present in the given figure?

(a)   20

(b)   22

(c)   23

(d)   18

Answer: (b)

16. If the following words are arranged as per their order in the English dictionary, which word will come fifth in the sequence?

(1) Weekly         (2) Weather     (3) Weird

(4) Weaver         (5) Weight       (6) Wielded

(a)   Weight

(b)   Wielded

(c)   Weird

(d)   Weekly

Answer: (b)

17. Select the option that is related to the third word in the same way as the second word is related to the first word.

Crop : Farmer :: Food : ?

(a)   Utensil

(b)   Vegetables

(c)   Chef

(d)   Kitchen

Answer: (c)

18. A + B means ‘A is the father of B’;

A – B means ‘A is the sister of B’;

A × B means ‘A is the brother of B’;

A ÷ B means ‘A is the daughter of B’;

If R + S × T – V ÷ U then how is S related to U?

(a)   Son

(b)   Daughter

(c)   Husband

(d)   Brother

Answer: (a)

19. Four words have been given, out of which three are alike in some manner, while one is different. Select the odd word.

(a)   Iron

(b)   Silver

(c)   Brass

(d)   Copper

Answer: (c)

20. The sequence of folding a piece of paper and the manner in which the folded paper has been cut is shown in the following figures. How would this paper look when unfolded?

Answer: (c)

21. The two given expressions on both the side of the ‘=’ sign will have the same value if two numbers from either side or both side are interchanged. Select the correct numbers to be interchanged from the given option:

4 + 6 × 7 – 27 ÷ 3 = 7 × 8 – 4 + 39 ÷ 3

(a)   8, 7

(b)   3, 4

(c)   6, 4

(d)   6, 8

Answer: (d)

22. Study the given pattern carefully and select the number that can replace the question mark (?) in it.

(a)   16

(b)   18

(c)   10

(d)   14

Answer: (a)

23. Select the option that is related to the third number in the same way as the second number is related to the first number and the sixth number is related to the fifth number.

78 : 117 : : 90 : ? : : 108 : 162

(a)   117

(b)   126

(c)   135

(d)   144

Answer: (c)

24. Select the option figure in which the given figure is embedded (rotation is NOT allowed).

Answer: (c)

25. Select the combination of letters that when sequentially placed in the blanks of the given series will complete the series.

_uvwuv_wvtuw_uvwuv_wv_uw

(a)   t t u t v

(b)   t v w t t

(c)   t u t u t

(d)   t t t t t

Answer: (d)

General Awareness

26. Which of the following monuments is NOT a part of the Qutub Complex?

(a)   Buland Darwaza

(b)   Qutub Minar

(c)   Quwwat-ul-Islam Mosque

(d)   Alai Darwaza

Answer: (a)

27. Which ISO certification pertains to Environmental Management Systems?

(a)   9001

(b)   27001

(c)   14001

(d)   22000

Answer: (c)

28. In July 2019, which among the following operations was launched by the BSF to fortify the Pakistan border in Punjab and Jammu?

(a)   Chakravyuth

(b)   Vayuputra

(c)   Garuda

(d)   Sudarshan

Answer: (d)

29. In January 2020, with which country did India sign a MoU for content exchange programme between All India Radio and Betar?

(a)   Nepal

(b)   Bhutan

(c)   Sri Lanka

(d)   Bangladesh

Answer: (d)

30. In which year NABARD established?

(a)   1978

(b)   1979

(c)   1981

(d)   1982

Answer: (d)

31. Which of the following statements is correct with reference to the Karachi Session of the Congress (1931)?

(a)   The Gandhi-Irwin Pact was ratified

(b)   Mahatma Gandhi presided over the session

(c)   The Quit-India Resolution was passed

(d)   The Khilafat movement was launched at this event

Answer: (a)

32. Under which Article can the Parliament amend the Constitution?

(a)   Article 269

(b)   Article 74

(c)   Article 368

(d)   Article 374

Answer: (c)

33. Which among the following is the food tube?

(a)   Thymus

(b)   Larynx

(c)   Aorta

(d)   Oesophagus

Answer: (d)

34. Which of the following is an east flowing river ?

(a)   Brahmani

(b)   Sabarmati

(c)   Sharavati

(d)   Mahi

Answer: (a)

35. Which of the following pairs is associated with the taxation system of the Marathas?

(a)   Chauth and Sardeshmukhi

(b)   Zat and Sawar

(c)   Polaj and Parauti

(d)   Iqta and Jagir

Answer: (a)

36. The Act that transferred the power from the British East India Company to the British Crown in India was:

(a)   Government of India Act, 1833

(b)   Government of India Act, 1835

(c)   Government of India Act, 1947

(d)   Government of India Act, 1858

Answer: (d)

37. The SASTRA-Ramanujan Prize is awarded in the field of:

(a)   Mathematics

(b)   Literature

(c)   Chemistry

(d)   Dance

Answer: (a)

38. The Arid Forest Research Institute is located in ______.

(a)   Dehradun

(b)   Jaipur

(c)   Ahemdabad

(d)   Jodhpur

Answer: (d)

39. Which country hosted the 13th South Asian Games?

(a)   Maldives

(b)   India

(c)   Bhutan

(d)   Nepal

Answer: (d)

40. Which of the following states does NOT share its boundary with Bhutan?

(a)   Sikkim

(b)   Meghalaya

(c)   West Bengal

(d)   Arunachal Pradesh

Answer: (b)

41. Who is the author of the book ‘Hazaar Chaurasi Ki Maa’?

(a)   Rita Kothari

(b)   Sugathakumari

(c)   Krishna Sobti

(d)   Mahasweta Devi

Answer: (d)

42. Which of the following is a heritage site of Madhya Pradesh?

(a)   Hampi

(b)   Lepakshi

(c)   Bhimbetka

(d)   Sasaram

Answer: (c)

43. Ozone at the higher level of the atmosphere is a product of ______ acting on oxygen molecules.

(a)   x-rays

(b)   UV radiation

(c)   IR radiation

(d)   gamma rays

Answer: (b)

44. The Battle of Chausa was fought between Humayun and ______.

(a)   Sher Shah Suri

(b)   Nadir Shah

(c)   Hemu

(d)   Krishnadeva Raya

Answer: (a)

45. Which female Indian cricketer won the BCCI C.K. Nayudu Lifetime Achievement Award for 2019?

(a)   Shantha Rangaswamy

(b)   Diana Edulji

(c)   Anjum Chopra

(d)   Jhulan Goswami

Answer: (c)

46. In the context of rivers and their tributaries, which of the following pairs is correct?

(a)   Indus-Subansiri

(b)   Alaknanda-Pindar

(c)   Krishna-Manjra

(d)   Godavari-Kabini

Answer: (b)

47. How many degrees does the Earth rotate about its own axis in one hour?

(a)   15

(b)   20

(c)   24

(d)   10

Answer: (a)

48. Under which of the following Amendments to the Constitution of India is defection to another party after election made illegal?

(a)   86th

(b)   61st

(c)   52nd

(d)   92nd

Answer: (c)

49. The first electron shell which is the nearest to the nucleus never holds more than ‘n’ electrons, where ‘n’ is equal to:

(a)   6

(b)   2

(c)   4

(d)   8

Answer: (b)

50. With which sport is Archana Kamath associated?

(a)   Squash

(b)   Table Tennis

(c)   Lawn Tennis

(d)   Badminton

Answer: (b)

Quantitative Aptitude

51. Solve the following 113 × 87 = ?

(a)   10000

(b)   9831

(c)   10169

(d)   10026

Answer: (b)

52. In a wallet, there are notes of the denomination Rs 10 and Rs 50. The total number of notes is 12. The number of Rs 10 and Rs 50 notes are in the ratio of 1 : 2. Total money in the wallet is:

(a)   Rs 280

(b)   Rs 110

(c)   Rs 360

(d)   Rs 440

Answer: (d)

53. The coefficient of x in (x – 3y)3 is:

(a)   3y2

(b)   −3y2

(c)   −27y2

(d)   27y2

Answer: (d)

54. A manufacture sells cooking gas stoves to shopkeepers at 10% profit, and in turn they sell the cooking gas stoves to customer to earn 15% profit. If a customer gets a cooking gas stove for Rs 7,590, then what is its manufacturing cost?

(a)   Rs 5,000

(b)   Rs 5,090

(c)   Rs 6,500

(d)   Rs 6,000

Answer: (d)

55. During a medical check-up, the heights of 40 students in a class were recorded as shown in the following table

How many students have a height of 165 cm or more?.

(a)   15

(b)   25

(c)   16

(d)   10

Answer: (a)

56. In a rectangle, if the measures of two sides are 5 cm and 8 cm, then the third side can be:

(a)   3 cm

(b)   4 cm

(c)   14 cm

(d)   2 cm

Answer: (b)

57. Solve the following:

(a)   15

(b)   −2

(c)   8

(d)   3

Answer: (a)

58. If  then the value of 

(a)   10

(b)   140

(c)   110

(d)   80

Answer: (c)

59. If (cos2θ – 1) (1+ tan2θ) + 2tan2θ = 1, 0° ≤ θ ≤ 90° then θ is:

(a)   90°

(b)   30°

(c)   45°

(d)   60°

Answer: (c)

60. In a school, 4% of the students did not appear for the annual exams. 10% of the students who appeared for the exams could not pass the exam. Out of the remaining students, 50% got distinction marks and 432 students passed the exam but could not get distinction marks. The total number of students in the school is:

(a)   960

(b)   878

(c)   1000

(d)   12000

Answer: (c)

61. Amit and Sunil together can complete a work in 9 days, Sunil and Dinesh together can complete the same work in 12 days, and Amit and Dinesh together can complete the same work in 18 days. In how many days will they complete the work if Amit, Sunil and Dinesh work together?

(a)   14 days

(b)   16 days

(c)   8 days

(d)   12 days

Answer: (c)

62. In the given figure, If AB = 10 cm, CD = 7 cm, SD = 4 cm and AS = 5 cm then BC = ?

(a)   8 cm

(b)   7.5 cm

(c)   9 cm

(d)   6 cm

Answer: (a)

63. The lengths of the diagonals of a rhombus are 16 cm and 12 cm. Its area is:

(a)   96 cm2

(b)   48 cm2

(c)   69 cm2

(d)   28 cm2

Answer: (a)

64. The largest number which should replace * in the number 2365*4 to make the number divisible by 4 is:

(a)   8

(b)   9

(c)   2

(d)   0

Answer: (a)

65. In the given figure, if AB = 8 cm, AC = 10 cm, ∠ABD = 90° and AD = 17 cm, then the measure of CD is:

(a)   10 cm

(b)   8 cm

(c)   9 cm

(d)   11 cm

Answer: (c)

66. Two cyclists X and Y start at the same time from place A and go towards place B at a speed of 6 km/h and 8 km/h, respectively. Despite stopping for 15 minutes during the journey, Y reaches 10 minutes earlier than X. The distance between the places A and B is:

(a)   16.5 km

(b)   8 km

(c)   6 km

(d)   10 km

Answer: (d)

67. The following table shows the income (in Rs) for a particular month, together with their source, in respect of 5 employees (A, B, C, D and E).

How many employees got more than a total of Rs 10,000 as income other than salary?

(a)   1

(b)   4

(c)   3

(d)   2

Answer: (d)

68. The value of  is:

(a)   0

(b)   2

(c)   5

(d) 

Answer: (a)

69. Solve the following:

(a)   1

(b)   4

(c)   0

(d)   √6/8

Answer: (c)

70. A shopkeeper marked a computer table for Rs 7,200. He allows a discount of 10% on it and yet makes profit of 8%. What will be his gain percentage if he does NOT allow any discount?

(a)   18%

(b)   20%

(c)   9%

(d)   2%

Answer: (b)

71. The number of students enrolled in different streams in a college is shown in the following table.

The ratio of the total number of boys to that of girls in the college is:

(a)   27 : 23

(b)   23 : 27

(c)   13 : 12

(d)   1 : 1

Answer: (a)

72. The following table shows the daily earnings of 45 skilled workers.

How many workers earn less than Rs 1,100 in a day?

(a)   43

(b)   29

(c)   39

(d)   10

Answer: (c)

73. In the given figure, if ∠APO = 35°, then which of the following options is correct?

(a)   ∠BPO = 35°

(b)   ∠APB = 80°

(c)   ∠APB = 60°

(d)   ∠BPO = 55°

Answer: (a)

74. 24 students collected money for donation. The average contribution was Rs 50. Later on, their teacher also contributed some money. Now the average contribution is Rs 56. The teacher’s contribution is:

(a)   Rs 56

(b)   Rs 200

(c)   Rs 104

(d)   Rs 106

Answer: (b)

75. Amit borrowed a sum of Rs 25,000 on simple interest. Bhola borrowed the same amount on compound interest (interest compounded yearly). At the end of 2  years, Bhola had to pay Rs 160 more interest than Amit. The rate of interest charged per annum is:

(a) 

(b) 

(c)   8%

(d) 

Answer: (c)

English Comprehension

76. Select the correct passive form of the given sentence.

A child could understand his theory.

(a)   His theory could be understood by a child.

(b)   His theory is being understood by a child.

(c)   His theory was understood by a child.

(d)   His theory can be understood by a child.

Answer: (a)

77. Given below are four jumbled sentence. Out of the given options pick the one that gives their correct order.

(A) These lotteries organized by State Governments sell dreams.

(B) Lured by this, millions of poor people waste their hard-earned money.

(C) Crores of rupees and gold are offered as prizes.

(D) People buy lotteries hoping to become rich overnight.

(a)   DBCA

(b)   CABD

(c)   CBAD

(d)   DACB

Answer: (d)

78. Select the most appropriate meaning of the given idiom.

Pull a fast one

(a)   Believe someone easily

(b)   Progress fast

(c)   Trick someone

(d)   Take quick action

Answer: (c)

79. In the sentence identify the segment which contains the grammatical error.

We had a long discussion at a cp of coffee.

(a)   We had

(b)   of coffee

(c)   at a cup

(d)   a long discussion

Answer: (c)

80. Select the most appropriate option to fill in the blank.

The poor man was ______ entry to the restaurant.

(a)   deluded

(b)   denied

(c)   derived

(d)   devoid

Answer: (a)

81. Select the most appropriate antonym of the given word.

FICKLE

(a)   Stable

(b)   Flighty

(c)   Crafty

(d)   Subtle

Answer: (a)

Comprehension:

In the following passage some words have been deleted. Fill in the blanks with the help of the alternatives given. Select the most appropriate option for each blank.

Agra’s marble-marvel has such an overwhelming presence (1) _____ other monuments in the town have been relegated (2) ______ oblivion. Amongst such 60 odd monuments in (3) ______ town, the Red Taj is the most (4) ______ and relatively well-preserved. The tomb in red sandstone was (5) ______ by Ann Hissing, widow of Dutch officer, after his death in 1803.

82. Select the correct option to fill in the blank 1.

(a)   but

(b)   that

(c)   than

(d)   If

Answer: (c)

83. Select the correct option to fill in the blank 2.

(a)   on

(b)   to

(c)   with

(d)   by

Answer: (b)

84. Select the correct option to fill in the blank 1.

(a)   their

(b)   an

(c)   a

(d)   the

Answer: d()

85. Select the correct option to fill in the blank 4.

(a)   elegant

(b)   adequate

(c)   efficient

(d)   active

Answer: (a)

86. Select the correct option to fill in the blank 5.

(a)   building

(b)   built

(c)   to build

(d)   build

Answer: (b)

87. Select the most appropriate synonym of the given word.

DISHEARTENED

(a)   disliked

(b)   depressed

(c)   disgusted

(d)   disgraced

Answer: (b)

88. Select the most appropriate synonym of the given word.

CORDIAL

(a)   hard

(b)   cold

(c)   warm

(d)   rude

Answer: (c)

89. Select the INCORRECTLY spelt word.

(a)   Opportunity

(b)   Obsolete

(c)   Omision

(d)   Obscure

Answer: (c)

90. Select the most appropriate meaning of the given idiom.

See eye to eye

(a)   See clearly

(b)   Stare at someone

(c)   Be suspicious

(d)   Agree with someone

Answer: (d)

91. Select the most appropriate one word substitution for the given group of words.

Pertaining to an individual from birth

(a)   Habitual

(b)   Anomaly

(c)   Chronic

(d)   Congenital

Answer: (d)

92. Given below are four jumbled sentences. Out of the given options pick the one that gives their correct order.

(A) These are generated by a violent undersea disturbance or ocean activity.

(B) Once generated, they travel outward on the ocean surface in all directions.

(C) A tsunami is made up of a series of very long waves.

(D) Spreading thus, they look like ripples caused by throwing a rock in a pond.

(a)   BDCA

(b)   CBDA

(c)   CABD

(d)   BADC

Answer: (c)

93. Select the most appropriate antonym of the given word.

HASTE

(a)   Lose

(b)   Dash

(c)   Rush

(d)   Delay

Answer: (d)

94. In the sentence identify the segment which contains the grammatical error.

The both children go to the same school.

(a)   to the

(b)   same school

(c)   children go

(d)   The both

Answer: (d)

95. Select the most appropriate option to the substitute the underlined segment in the given sentence. If there is no need to substitute it, select No improvement.

They stopped to laugh as soon as the teacher entered the classroom.

(a)   stopped laughing

(b)   No improvement

(c)   stop laughing

(d)   stopped laugh

Answer: (a)

96. Select the most appropriate one word substitution for the given group of words.

To increase the speed

(a)   Assimilate

(b)   Exhilarate

(c)   Accelerate

(d)   Activate

Answer: (c)

97. Select the most appropriate option to fill in the blank.

Have you _____ for a job in this company?

(a)   approved

(b)   applied

(c)   appointed

(d)   supplied

Answer: (b)

98. Select the correct indirect form of the given sentence.

Mahesh said to Rita, “Don’t play in the sun.”

(a)   Mahesh advised Rita not to play in the sun.

(b)   Mahesh told to Rita to not play in the sun.

(c)   Mahesh requested Rita that don’t play in the sun.

(d)   Mahesh ordered Rita that not play in the sun.

Answer: (a)

99. Select the most appropriate option to substitute the underlined segment in the given sentence. If there is no need to substitute it, select no improvement.

If they will get married, they will probably settle in Mumbai.

(a)   No improvement

(b)   If they get married

(c)   If they will be getting married

(d)   If they had got married

Answer: (b)

100. Select the INCORRECTLY spelt word.

(a)   Rebel

(b)   Delegait

(c)   Monarch

(d)   Edible

Answer: (b)

SSC Combined Graduate Level (Tier-1) Exam-2019 Held on 06-03-2020 Time 10.00-11.00 AM Question Paper With Answer Key

SSC Combined Graduate Level (Tier-1) Exam-2019 Held on 06-03-2020 Time 10.00-11.00 AM
SSC Combined Graduate Level (Tier-1) Exam-2019 Held on 06-03-2020 Time 10.00-11.00 AM Question Paper With Answer Key

SSC Combined Graduate Level (Tier-1) Exam-2019 Held on 06-03-2020 Time 10.00-11.00 AM

General Intelligence and Reasoning

1. In a certain code language, ‘DKQX’ is written as ‘WPJC’. How will ‘AFKP’ be written as in that language?

(a)   ZUQJ

(b)   ZVQL

(c)   ZTOK

(d)   ZUPK

Answer: (d)

2. Read the given statements and conclusions carefully. Assuming that the information given in the statements is true, even if it appears to be at variance with commonly known facts, decide which of the given conclusions logically follow(s) from the statements.

Statements:

(1) Some machines are kites.

(2) No machine is a pigeon.

Conclusion:

(I) Some machines are pigeons.

(II) All kites are pigeons.

(III) Some kites are not pigeons.

(a)   Only conclusion III follows

(b)   Only conclusion II follows

(c)   Only conclusion I and III follow

(d)   Only conclusion I and II follow

Answer: (a)

3. Select the letter that can replace the question mark (?) in the following series.

A, E, I, ?, Q, U

(a)   K

(b)   N

(c)   M

(d)   O

Answer: (c)

4. Select the correct mirror image of the given figure when a mirror is placed on the right of the figure.

Answer: (a)

5. Select the option in which the words share the same relationship as that shared by the given pair of words.

North : South

(a)   Ductile : Bendable

(b)   Whole : Total

(c)   Frown : Smile

(d)   Integrity : Honesty

Answer: (c)

6. In the given Venn diagram, the ‘rectangle’ represents ‘travelers who like to travel by plane’, the ‘circle’ represents ‘travelers who like to travel by bus’, and the ‘triangle’ ‘represents travelers who like to travel by train’. The numbers given in the diagram represent the number of travelers in that particular category.

How many travelers like to travel either by train or plane but NOT by bus?

(a)   36

(b)   30

(c)   29

(d)   27

Answer: (b)

7. Arrange the following words in a logical and meaningful order.

(1) Quadrillion   (2) Quintillion (3) Million

(4) Octillion        (5) Trillion

(a)   4-3-5-1-2

(b)   2-3-5-1-4

(c)   3-5-1-2-4

(d)   3-5-1-4-2

Answer: (c)

8. Four letter-clusters have been given, out of which three are alike in some manner and one is different. Select the odd letter-cluster.

(a)   FBL

(b)   DDP

(c)   GCV

(d)   BJT

Answer: (c)

9. Study the given pattern carefully and select the number that can replace the question mark (?) in it.

(a)   113

(b)   95

(c)   111

(d)   77

Answer: (d)

10. In a certain group of men and horse, the total number of legs is 14 more than twice the number of heads. How many horses are there in the group?

(a)   11

(b)   9

(c)   5

(d)   7

Answer: (d)

11. Four words been given, out of which three are alike in some manner and one is different, Select the odd word.

(a)   Trumpet

(b)   Xylophone

(c)   Saxophone

(d)   Clarinet

Answer: (b)

12. Which two signs should be interchanged to make the given equation correct?

12 + 81 – 27 × 9 ÷ 3 = 36

(a)   − & ÷

(b)   × & −

(c)   ÷ & ×

(d)   + & ×

Answer: (a)

13. In a certain code language, ‘PAGE’ is coded as ‘161495’. How will ‘PART’ be coded as in that language?

(a)   16128620

(b)   16125020

(c)   16136220

(d)   16132420

Answer: (d)

14. Four number-pairs have been given, out of which three are alike in some manner and one is different. Select the number-pair that is different from the rest.

(a)   15:250

(b)   13:169

(c)   17:289

(d)   11:121

Answer: (a)

15. Select the figure that can replace the question mark (?) in the following series.

Answer: ()

16. Select the option that depicts how the given transparent sheet of paper would appear if it is folded at the dotted line.

Answer: (a)

17. Select the number that can replace the question mark (?) in the following series.

5, 10, 26, ?, 122, 170

(a)   77

(b)   65

(c)   82

(d)   50

Answer: (d)

18. Study the given pattern carefully and select the number that can replace the question mark (?) in it.

(a)   82

(b)   92

(c)   98

(d)   88

Answer: (d)

19. Select the letter-cluster that can replace the question mark (?) in the following series.

AG, CGK, EJO, ?, IPW

(a)   GNZ

(b)   GMS

(c)   GMZ

(d)   GNS

Answer: (b)

20. Select the number that can replace the question mark (?) in the following series.

62, 66, 74, 90, ?

(a)   122

(b)   106

(c)   120

(d)   116

Answer: (a)

21. How many triangles are there in the given figure?

(a)   16

(b)   22

(c)   28

(d)   25

Answer: (d)

22. A + B means ‘A is the husband of B’;

A – B means ‘B is the sister of A’;

A × B means ‘A is the mother of B;

A ÷ B means ‘B is the son of A’.

If, P + R × T – Q ÷ S + U, then how is P related to S?

(a)   Maternal grandfather

(b)   Uncle

(c)   Paternal

(d)   Father-in-law

Answer: (a)

23. ‘Astronomy’ is related to ‘Stars’ in the same way as ‘Agronomy’ is related to’

(a)   Emotions

(b)   Planets

(c)   Crops

(d)   Mines

Answer: (c)

24. Three different positions of the same dice are shown. Select the symbol that will be on the face opposite to the one showing ‘+’.

(a)   $

(b)   #

(c)   @

(d)   *

Answer: (c)

25. Select the option in the which the given figure is embedded (rotation is not allowed).

Answer: (d)

General Awareness

26. Which of the following is the primary sex organ in females?

(a)   Uterus

(b)   Ovary

(c)   Vagina

(d)   Fallopian tube

Answer: (b)

27. Which of the following diseases is NOT transmitted from one person to another?

(a)   Cirrhosis

(b)   AIDS

(c)   Hepatitis B

(d)   Syphilis

Answer: (a)

28. Which International organization released the ‘World Energy Outlook 2019’ Report?

(a)   United Nations

(b)   International Energy Agency

(c)   World Trade Organization

(d)   UNESCO

Answer: (b)

29. The Supreme Court has declared access to the internet a fundamental right under Article ______ of the Indian constitution.

(a)   19

(b)   14

(c)   21

(d)   17

Answer: (a)

30. Name the Indian wrestler who has been named the Junior Freestyle Wrestler of the year (2019) by United World Wrestling (UWW).

(a)   B. P. Raju

(b)   Kamareddy

(c)   Amita Bagchi

(d)   Deepak Punia

Answer: (d)

31. UN Women propose to partner with _____ gender park with an aim to secure gender parity is South Asia.

(a)   Kerala’s

(b)   Rajasthan’s

(c)   Punjab’s

(d)   Haryana’s

Answer: (a)

32. Which of the following is a natural flame retardant?

(a)   RNA

(b)   Skin

(c)   DNA

(d)   Flowers

Answer: (c)

33. Merchant Discount rate refers to ______.

(a)   The total discount a bank offers to the merchant for promoting online transactions

(b)   The rate charged to a merchant for payment processing services on debit and credit card transactions

(c)   The total discount a merchant offers on online transactions

(d)   Only taxes that a digital payment entails

Answer: (b)

34. Alyssa Healy who created a world record by becoming the highest scorer in women’s T20I, belongs to

(a)   Japan

(b)   India

(c)   Canada

(d)   Australia

Answer: (d)

35. In the sequence of planets in the Solar system, which planet comes in between Mercury and Earth?

(a)   Uranus

(b)   Venus

(c)   Jupiter

(d)   Mars

Answer: (b)

36. The SDG India index is developed by _____.

(a)   Ministry of Home affairs

(b)   World Bank

(c)   Ministry of Environmental, Forests and Climate Change

(d)   NITI Aayog

Answer: (d)

37. Who performs the task of capturing oxygen in the blood?

(a)   Chlorophyll

(b)   Haemoglobin

(c)   Red blood cells

(d)   White blood cells

Answer: (b)

38. If the inflation in an economy is rising steadily, the Central Bank might_____.

(a)   keep the repo rate unchanged

(b)   decrease the repo rate

(c)   increase the repo rate

(d)   decrease the reverse repo rate

Answer: (c)

39. Name the only metal that is antibacterial.

(a)   Iron

(b)   Sodium

(c)   Copper

(d)   Aluminium

Answer: (c)

40. On which river and in which state is the Jayakwadi dam situated?

(a)   Priyar river – Kerala

(b)   Banas river – Rajasthan

(c)   Ravi river – Punjab

(d)   Godavari river – Maharashtra

Answer: (d)

41. Who is the first General of the Indian Army, whose retirement day is celebrated as the ‘Armed Forces Veterans Day’ every year?

(a)   General K. S. Thimayya

(b)   General S. M. Srinagesh

(c)   General Maharaj Rajendra Sinhji

(d)   General K. M. Cariappa

Answer: (d)

42. Which India journalist was honoured with India’s Most powerful Woman in Media Award during the prestigious Confluence Excellence Award ceremony held in the British Parliament on 27 September 2019?

(a)   Kalli Puri

(b)   Anubha Bhonsle

(c)   Sunetra Chaudhary

(d)   Sona Choudhary

Answer: (a)

43. Which of the following gases is consistently seen to be most abundant in a volcanic eruption?

(a)   water vapour

(b)   Carbon Dioxide

(c)   Sulphur Dioxide

(d)   hydrogen Sulphide

Answer: (a)

44. Maharunnisa who was known as Nur Jahan was the wife of

(a)   Jahangir

(b)   Saha Jahan

(c)   Akbar

(d)   Aurangzeb

Answer: (a)

45. Which leader gave us the C R Formula?

(a)   Tilak

(b)   C. R. Das

(c)   Mahatma Gandhi

(d)   C. Rajagopalachari

Answer: (d)

46. Which of the following is the first South Asian country to cell match-fixing a crime?

(a)   Pakistan

(b)   India

(c)   Sri Lanka

(d)   Nepal

Answer: (c)

47. To which of the following Indian states does Mardani Khel, one of the famous martial art forms in India belong?

(a)   Maharashtra

(b)   Rajasthan

(c)   Uttar Pradesh

(d)   Madhya Pradesh

Answer: (a)

48. In which year did D. C. Sircar publish Indian Epigraphy and Indian Epigraphical Glossary?

(a)   1966-67

(b)   1967-68

(c)   1965-66

(d)   1964-65

Answer: (c)

49. Madhavpur Mela takes place in _____.

(a)   Uttar Pradesh

(b)   Maharashtra

(c)   Madhya Pradesh

(d)   Gujarat

Answer: (d)

50. When did the Vikrama Era Begin?

(a)   57 BC

(b)   55 BC

(c)   50 BC

(d)   47 BC

Answer: (a)

Quantitative Aptitude

51. A, B and C can individually complete a task in 24 days, 20 days and 18 days, respectively. B and C start the task, and they work for 6 days and leave. The number of days required by A alone to finish the remaining task, is:

(a) 10 day

(b)    

(c)    

(d)  

Answer: (c)

52. If (2sinA + cosec A) = 2√2, 0° < A < 90°, then the value of 2(sin4A + cos4A) is:

(a)   2

(b)   1

(c)   4

(d)   0

Answer: (b)

53. If the number 687×29 is divisible by 9, then the value of 2x is:

(a)   4

(b)   8

(c)   2

(d)   3

Answer: (b)

54. If the value of (a + b – 2)2 + (b + c – 5)2 + (c + a – 5)2 = 0, then the value of  is:

(a)   3

(b)   2

(c)   0

(d)   1

Answer: (a)

55. In the following table, the production of various crops (in tones) is given from 2015 to 2019. Study the table and answer the questions that follow.

The difference (in tones) between the average production of barley and average production of rice is:

(a)   471

(b)   549

(c)   780

(d)   231

Answer: (d)

56. An article was sold at a gain of 18%. If it had been sold for Rs 49 more, then the gain would have been 25%. The cost price of the article is:

(a)   Rs 890

(b)   Rs 700

(c)   Rs 650

(d)   Rs 570

Answer: (b)

57. The average of the marks of 30 boys is 88, and when the top two scores were excluded, the average marks reduced to 87.5. If the top of two scores differ by 2, then the highest marks is:

(a)   90

(b)   94

(c)   96

(d)   92

Answer: (c)

58. Two tangents PA and PB are drawn to a circle with centre O from an external point P. If ∠OAB = 30°, then ∠APB is:

(a)   120°

(b)   30°

(c)   60°

(d)   15°

Answer: (c)

59. If a sum of Rs 1,180 is to be divided among A, B and C, such that 2 times A’s share, 5 times B’s share and 7 times C’s share, are equal, then A’s share is:

(a)   Rs 750

(b)   Rs 500

(c)   Rs 650

(d)   Rs 700

Answer: (d)

60. In the following table, the production of various crops (in tones) is given from 2015 to 2019. Study the table and answer the questions that follow.

The percentage growth of maize in the year 2019 over the year 2015 is : (correct to two decimal places)

(a)   60.28%

(b)   65.12%

(c)   71.43%

(d)   77.77%

Answer: (c)

61. If the area of an equilateral triangle is 36√3 cm2, then the perimeter of the triangle is:

(a)   36√3 cm

(b)   18√3 cm

(c)   12 cm

(d)   36 cm

Answer: (d)

62. ABC is an equilateral triangle. P, Q and R are the midpoints of sides AB, BC and CA, respectively. If the length of the side of the triangle ABC is 8 cm, then the area of ∆PQR is:

(a)   √3/3 cm2

(b)   8√3 cm2

(c)   4√3 cm2

(d)   √3/4 cm2

Answer: (c)

63. A circular disc of area 0.64 π m2 rolls down a length of 1.408 km. The number of revolutions it makes is: (Take π = 22/7)

(a)   280

(b)   360

(c)   140

(d)   180

Answer: (a)

64. If 3a = 27b = 81c and abc = 144, then the value of  is:

(a)   18/120

(b)   33/10

(c)   17/120

(d)   18/10

Answer: (b)

65. A shopkeeper marks the price of an article in such a way that after allowing a discount of 22%, he gets a gain of 11%. If the marked price is Rs 888, then the cost price of the article is:

(a)   Rs 550

(b)   Rs 895

(c)   Rs 624

(d)   Rs 782

Answer: (c)

66. What will be the difference in compound interest on a sum of Rs 7,800 at 8% for 1 year, when the interest is paid yearly and half yearly?

(a)   Rs 19.46

(b)   Rs 12.48

(c)   Rs 29.18

(d)   Rs 24.72

Answer: (b)

67. Solve the following:

(a)   1

(b)   −2

(c)   2

(d)   −1

Answer: (b)

68. A student takes 1.25 hours to travel from home to school at a speed of 4 km/h. By what percentage should he increase his speed to reduce the time by 25% to cover the same distance from school of home?

(a)    

(b)   45%

(c)   25%

(d)   50%

Answer: (a)

69. A metallic sphere of diameter 40 cm is melted into smaller spheres of radius 0.5 cm. How many such small balls can be made?

(a)   32,000

(b)   6400

(c)   64,000

(d)   3200

Answer: (c)

70. If a + b + c = 9 and ab + bc + ca = −22, then the value of a3 + b3 + c3 – 3abc is:

(a)   783

(b)   1571

(c)   487

(d)   1323

Answer: (d)

71. If xcosA – ysinA = 1 and x sinA + y cosA = 4, then the value of 17x2 + 17y2 is

(a)   289

(b)   49

(c)   7

(d)   0

Answer: (a)

72. The population of a city increased by 30% in the first year and decreased by 15% in the next year. If the present population is 11,050 then the population 2 years ago was:

(a)   10,050

(b)   99,000

(c)   10,000

(d)   99,500

Answer: (c)

73. If ‘+’ means ‘−’ means ‘+’, ‘×’ means ‘÷’ and ‘÷’ means ‘×’ then the value of is:

(a)   2

(b)   1

(c)   −2

(d)   −1

Answer: (c)

74. In the following table, the production of various crops (in tones) is given from 2015 to 2019. Study the table and answer the questions that follow.

The highest growth of sugarcane production over its previous year is recorded in the year:

(a)   2019

(b)   2017

(c)   2016

(d)   2018

Answer: (c)

75. In the following table, the production of various crops (in tones) is given from 2015 to 2019. Study the table and answer the questions that follow.

The average production of wheat (in tones) in the period given in the table is:

(a)   3600

(b)   3482

(c)   3300

(d)   4218

Answer: (a)

English Comprehension

76. Select the correct passive form of the given sentence.

The farmer is ploughing the fields.

(a)   The fields were ploughed by the farmer.

(b)   The fields are being ploughed by the farmer

(c)   The fields are ploughed by the farmer.

(d)   The fields have been ploughed by the farmer.

Answer: (b)

77. Select the most appropriate option to substitute the underlined segment in the given sentence. If there is no need to substitute it, select No improvement.

We came back because we had ran out of money.

(a)   No improvement

(b)   we have ran

(c)   we had run

(d)   we run

Answer: (c)

78. Select the option that can be used as a one-word substitute for the given group of words/phrase.

Having two opposing feeling at the same time

(a)   Ambivalent

(b)   Contemporary

(c)   Coinciding

(d)   Equivalent

Answer: (a)

79. Identify the segment in the sentence which contains the grammatical error. If there is no error, select ‘No error’

Those who are late they will not be allowed inside the classroom.

(a)   No error

(b)   Those who are late

(c)   inside the classroom

(d)   they will not be allowed

Answer: (d)

80. Select the most appropriate ANTONYM of the given word.

Persist

(a)   Cease

(b)   Continue

(c)   Remain

(d)   Insist

Answer: (a)

81. Select the correct indirect form of the given sentence.

“What a rare flower!” she said.

(a)   She exclaimed that it was a very rare flower.

(b)   She asked if it was a rare flower.

(c)   She exclaimed what a rare flower it is.

(d)   She told that what a rare flower it was.

Answer: (a)

Comprehension:

In the following passage some words have been deleted. Fill in the blanks with the help of the alternatives given. Select the most appropriate option for each blank.

Atlas was made to hold the sky up on his shoulders as a punishment. Once, Hercules (1) ______ to hold it for Atlas while the latter went looking (2) ______ golden apples. When Atlas returned, he saw Hercules (3) _____ hard to hold the sky. The weight of the sky (4) _______ Hercules sweat and groan. Now Atlas did not want to (5) _______ a job as arduous as that!

82. Select the most appropriate option to fill in blank 1.

(a)   extended

(b)   offered

(c)   ordered

(d)   afforded

Answer: (b)

83. Select the most appropriate option to fill in blank 2.

(a)   for

(b)   after

(c)   at

(d)   on

Answer: (a)

84. Select the most appropriate option to fill in blank 3.

(a)   struggling

(b)   tackling

(c)   seeking

(d)   coping

Answer: (a)

85. Select the most appropriate option to fill in blank 4.

(a)   were making

(b)   has made

(c)   make

(d)   was making

Answer: (d)

86. Select the most appropriate option to fill in blank 5.

(a)   take in

(b)   take out

(c)   take back

(d)   take away

Answer: (c)

87. Select the most appropriate synonym of the given word.

Ovation

(a)   Preparation

(b)   Censure

(c)   Applause

(d)   Creation

Answer: (c)

88. Select the most appropriate option to fill in the blank.

The roof of the old building _____ during the storm.

(a)   demolished

(b)   destroyed

(c)   scratched

(d)   collapsed

Answer: (d)

89. Select the INCORRECTLY spelt word.

(a)   Alliance

(b)   Acquaintence

(c)   Abandon

(d)   Acquisition

Answer: (b)

90. Select the most appropriate synonym of the given word.

(a)   Feared

(b)   Favoured

(c)   Adored

(d)   Compared

Answer: (b)

91. Select the appropriate option to fill in the blank.

She _____ how the magician had performed the rope trick.

(a)   wondered

(b)   bewildered

(c)   puzzled

(d)   suspected

Answer: (a)

92. Select the most appropriate option to substitute the underlined segment in the given sentence. If there is no need to substitute, it select No improvement.

When it started raining, I ran to the nearer house for shelter.

(a)   No improvement

(b)   the nearest house

(c)   a near house

(d)   the near house

Answer: (b)

93. Select the most appropriate meaning of the given idiom.

Chicken-hearted

(a)   Generous

(b)   Miserly

(c)   Selfish

(d)   Cowardly

Answer: (d)

94. Select the most appropriate ANTONYM of the given word.

Glee

(a)   Fun

(b)   Joy

(c)   Woe

(d)   Bliss

Answer: (c)

95. Select the most appropriate meaning of the given idiom.

By and by

(a)   Gradually

(b)   Suddenly

(c)   On the whole

(d)   By any means

Answer: (a)

96. Given below are four jumbled sentences. Out of the given options pick the one that gives their correct order.

(A) The mountain range was covered with snow

(B) Napoleon had to cross the Alps in winter.

(C) Napoleon replied that the word impossible’ did not exist for him.

(D) So, someone told Napoleon that the task was impossible.

(a)   DBCA

(b)   BADC

(c)   BDCA

(d)   ADBC

Answer: (b)

97. Select the option that can be used as a one-word substitute for the given group of words/phrase.

A group of ships

(a)   squad

(b)   fleet

(c)   swarm

(d)   colony

Answer: (b)

98. Identify the segment in the sentence which contains the grammatical error. If there is no error, select ‘No error’

Everyone except she have travelled by air.

(a)   travelled by air

(b)   Everyone except

(c)   she have

(d)   No error

Answer: (c)

99. Select the wrongly spelt word.

(a)   Fierce

(b)   Sieze

(c)   Piece

(d)   Grief

Answer: (b)

100. Given below are four jumbled sentences. Out of the given options pick the one that gives their correct order.

(A) Much of this war had taken place along the Western Front.

(B) Both sides had dug in deep and each lost many men over little ground.

(C) This front was  a line of trenches across which the two sides faced each other.

(D) Andre Maginot had fought a war with the French against the Germans.

(a)   BCDA

(b)   DACB

(c)   ACDB

(d)   DBCA

Answer: (b)

SSC Combined Graduate Level (Tier-1) Exam-2019 Held on 07-03-2020 Time 1.00-2.00 PM Question Paper With Answer Key

SSC Combined Graduate Level (Tier-1) Exam-2019 Held on 07-03-2020 Time 1.00-2.00 PM
SSC Combined Graduate Level (Tier-1) Exam-2019 Held on 07-03-2020 Time 1.00-2.00 PM Question Paper With Answer Key

SSC Combined Graduate Level (Tier-1) Exam-2019 Held on 07-03-2020 Time 1.00-2.00 PM

General Intelligence and Reasoning

1. Select the option in which the given figure is embedded (rotation is not allowed).

Answer: (a)

2. Four letter-clusters have been given, out of which three are alike in some manner and one is different. Select the odd letter-cluster.

(a)   EHKN

(b)   PSVY

(c)   TWZC

(d)   NOSV

Answer: (d)

3. Read the given statements and consluions carefully. Assuming tht the information given in the statements is true, even if it appears to be at variance with commonly known facts, decide which of the given conclusions logically follows(s) from the statements.

Statements:

(1) All toys are pens.

(2) All games are pens.

Conclusions:

(I) No toy is game.

(II) No pen is a game.

(III) Some toys are games.

(a)   Either conclusion I or III follows

(b)   Only conclusions II and III follow

(c)   All conclusions I, II and III follow

(d)   Only conclusions I and II follow

Answer: (a)

4. What day of the week has 31 January 2007?

(a)   Monday

(b)   Tuesday

(c)   Wednesday

(d)   Thursday

Answer: (c)

5. If ‘A’ stands for ‘subtraction’, ‘B’ stands for ‘multiplication’, ‘C” stands for ‘addition’, and ‘D’ stands for ‘division’, then what is the value of the following expression?

27A8B5C(11C3)B5C36D6

(a)   20

(b)   63

(c)   60

(d)   24

Answer: (b)

6. Which of the following Venn diagram best represents the relationship between the following classes?

Accounts, Girls, Internet users

Answer: (b)

7. Select the correct mirror image of the given alphanumeric-cluster when a vertical mirror is placed on the right side of the cluster.

654HBMA

Answer: (c)

8. Select the letter will replace the question mark (?) in the following series.

C, B, B, C, Z, E, W, H, S, ?, N

(a)   K

(b)   P

(c)   L

(d)   I

Answer: (c)

9. Select the option that is related to the third number in the same way as the second number is related to the first number.

11 : 265 :: 20 : ?

(a)   835

(b)   840

(c)   841

(d)   838

Answer: (c)

10. Select the option in which the words share the same relationship as that shared by the given pair of words.

Office : Work

(a)   House : Bricks

(b)   Hospital : Director

(c)   School : Study

(d)   Court : Lawyer

Answer: (c)

11. Select the letter-cluster that can replace the question mark (?) in the following series.

PERSONA, ANPERSO, ANOSPER, ?

(a)   NOSAREP

(b)   ANOSERP

(c)   ANSORPE

(d)   ANOSREP

Answer: (d)

12. In a certain code language ‘PAINTS’ is written as ‘CRPKUV’. How will ‘PURITY’ be written as in that language?

(a)   VSKTAZ

(b)   WRKSNU

(c)   XSISBM

(d)   WRKTAV

Answer: (d)

13. Select the figure that can replace the question mark (?) in the following series.

Answer: (c)

14. Four words have been given, out of which three are alike in some manner and one is different. Select the odd word.

(a)   Distance

(b)   Litre

(c)   Minute

(d)   Kilogram

Answer: (a)

15. Study the given pattern carefully and select the number that can replace the question mark (?) in it.

(a)   83

(b)   71

(c)   81

(d)   79

Answer: (c)

16. ‘Diabetes’ is related to ‘Blood Sugar’ in the same way as ‘Hypertension’ is related to ‘_______’.

(a)   Blood Pressure

(b)   Heart

(c)   Body Cells

(d)   Red Blood Cells

Answer: (a)

17. Four number-pairs have been given, out of which three are alike in some manner and one is different. Select the number-pair that is different from the rest.

(a)   33 : 109

(b)   30 : 94

(c)   27 : 85

(d)   44 : 136

Answer: (a)

18. A – B means ‘A is the mother of B’;

A × B means ‘A is the sister of B’;

A ÷ B means ‘A is the daughter of B’.

Which of the following expressions means ‘U is the daughter of Q’?

(a)   K – U ÷ Z ×Q

(b)   Q – Z × U ÷ K

(c)   K – Z ÷ U × Q

(d)   Q – Z ÷ U × K

Answer: (b)

19. Select the number that can replace the question mark (?) in the following series.

35, 36, 40, ?, 83, 208, 244

(a)   73

(b)   67

(c)   55

(d)   60

Answer: (b)

20. Arrange the following words in the order in which they would appear in an English dictionary.

(1) Temper          (2) Temple

(3) Tamarind      (4) Tender

(5) Tenacity

(a)   3, 2, 1, 4, 5

(b)   1, 3, 2, 5, 4

(c)   3, 1, 2, 5, 4

(d)   3, 1, 5, 4, 2

Answer: (c)

21. The sequence of folding a piece of paper and the manner in which the folded paper has been cut is shown in the following figures. How would this paper look when unfolded?

Answer: (d)

22. In a certain code language, ‘SOLID’ is coded as ‘64’ and ‘POUR’ is coded as ‘74’. How will ‘TON’ be coded as in that language?

(a)   60

(b)   85

(c)   52

(d)   49

Answer: (c)

23. Select the option in which the numbers are related in the same way as are the numbers in the given set.

(45, 24, 441)

(a)   (35, 17, 324)

(b)   (50, 30, 480)

(c)   (41, 18, 630)

(d)   (30, 60, 1024)

Answer: (a)

24. How many triangle are there in the given figure?

(a)   17

(b)   21

(c)   20

(d)   19

Answer: ()

25. A cube is made by folding the given sheet. In the cube so formed, which of the following pairs of numbers will be on opposite sides?

(a)   3 & 6

(b)   2 & 4

(c)   1 & 5

(d)   1 & 2

Answer: (b)

General Awareness

26. Who won the ‘ICC (International Cricket Council) Spirit of Cricket, 2019’ award?

(a)   Virat Kohli

(b)   Rohit Sharma

(c)   Pat Cummins

(d)   Ben Stokes

Answer: (a)

27. The tidal mouth of a river where fresh and saline water get mixed is known as a/an___.

(a)   Gorge

(b)   Estuary

(c)   Archipelago

(d)   Fold

Answer: (b)

28. Which organization launched the Agricultural Marketing and Farmer Friendly Reforms Index (AMFFRI) in 2016?

(a)   National Institute of Public Finance and Policy

(b)   NITI Aayog

(c)   Reserve Bank of India

(d)   Indian Institute of Public Administration

Answer: (b)

29. Which of the following is one of the Articles of the Constitution of India that mentions the Comptroller and Auditor General of India?

(a)   352

(b)   148

(c)   156

(d)   280

Answer: (b)

30. A situation where the expenditure of the government exceeds its revenue is known as____.

(a)   Morbidity

(b)   Devaluation

(c)   Default

(d)   Deficit Financing

Answer: (d)

31. Which of the following martial art forms is practice in the state of Punjab and West Bengal?

(a)   Kalaripayattu

(b)   Lathi

(c)   Thang-ta

(d)   Mardaani khel

Answer: (b)

32. Which of the following can be diagnosed with the Widal Test?

(a)   AIDS

(b)   Cancer

(c)   Typhoid Fever

(d)   Tuberculosis

Answer: (c)

33. Which of the following schedules of the Constitution of India has been INCORRECTLY matched with its content?

(a)   First Schedule-The states and the union territories

(b)   Third Schedule-Forms of oath or affirmations

(c)   Fourth Schedule-Allocation of seats in the council of states

(d)   Second Schedule – Languages

Answer: (d)

34. Which of the following is sued for removing permanent hardness of water?

(a)   Baking soda

(b)   Bleaching powder

(c)   Lime

(d)   Washing soda

Answer: (d)

35. Who among the following was known as ‘Andhra Kesari’?

(a)   T. Prakasam

(b)   Khan Abdul Gaffar Khan

(c)   Mahatma Gandhi

(d)   Chittaranjan Das

Answer: (a)

36. Who is the author of the book title ‘The Anarchy: The East India Company, Corporate Violence, and the Pillage of an Empire?

(a)   Pankaj Mishra

(b)   William Dalrymple

(c)   V. P. Menon

(d)   Shashi Tharoor

Answer: (b)

37. In which part of India are the Patkai Bum hills located?

(a)   Northern part of India

(b)   Southern part of India

(c)   Western part of India

(d)   Eastern part of India

Answer: (d)

38. Where were the Khelo India Youth Games, 2020 held?

(a)   Guwahati

(b)   Bhopal

(c)   Chandigarh

(d)   New Delhi

Answer: (a)

39. Where was the 107th edition of the Indian Science Congress held?

(a)   Lucknow

(b)   Chennai

(c)   Bengaluru

(d)   Dehradun

Answer: (c)

40. Who discovered cells in the human body?

(a)   Charles Darwin

(b)   Robert Hooke

(c)   Herbert Spencer

(d)   Albert Einstein

Answer: (b)

41. Who among the following was NOT a recipient of the Arjuna Awards in 2019?

(a)   Swapana Burman

(b)   Merzban Patel

(c)   Ravindra Jadeja

(d)   Poonam Yadav

Answer: (b)

42. In which year did the Third Round Table Conference take place?

(a)   1928

(b)   1929

(c)   1932

(d)   1930

Answer: (c)

43. Who wrote the play ‘Mricchakatika’ (The Little Clay Cart), a social drama with touches of grim reality?

(a)   Magh

(b)   Raidasa

(c)   Sudraka

(d)   Kalidasa

Answer: (c)

44. Which of the following is caused by a virus named ‘Varicella zoster’?

(a)   Polio

(b)   Chickenpox

(c)   Rabies

(d)   Cholera

Answer: (b)

45. Which one of the following statements is INCORRECT?

(a)   Matter is made up of small particles

(b)   Evaporation causes cooling

(c)   Brass is a mixture of 30% zinc and 70% copper

(d)   The measurable unit of pressure is denoted in Metre

Answer: (d)

46. On which lake is the world’s only floating post office situated?

(a)   Wulr Lake

(b)   Loktak Lake

(c)   Chilika Lake

(d)   Dal Lake

Answer: (d)

47. Who among the following was NOT associated with the revolt of 1857?

(a)   Kunwar Singh

(b)   Bhagat Singh

(c)   Begum Hazrat

(d)   Mangal Pandey

Answer: (b)

48. In which year did the battle of Khanua (Khanwa) take place between Babur and the Rajput forces led by Rana Sanga?

(a)   1527

(b)   1526

(c)   1522

(d)   1529

Answer: (a)

49. What is the name of the anti-hijacking exercise conducted by the Indian Navy in collaboration with the Indian Coast Guard?

(a)   Nagah

(b)   Indra

(c)   Ekuverin

(d)   Apharan

Answer: (d)

50. What is Gotabaya Rajapaksa’s nationality?

(a)   Sri Lankan

(b)   Burmese

(c)   Bangladesh

(d)   Indonesian

Answer: (a)

Quantitative Aptitude

51. If 18 men can cut a field in 35 days, then in how many days can 21 men cut the same field?

(a)   32

(b)   27

(c)   28

(d)   30

Answer: (d)

52. The following table shows the percentage of marks obtained by seven students in six different subjects in an examination.

What are the average marks obtained by all the seven students in Physics? (rounded off to two digits after the decimal)

(a)   75.90

(b)   85.12

(c)   89.14

(d)   80.15

Answer: (c)

53. In ∆ABC, D, E and F are the midpoints of sides AB, BC and CA, respectively. If AB = 12 cm, BC = 20 cm and CA = 15 cm, then the value of 

(a)   23.5 cm

(b)   5.88 cm

(c)   11.75 cm

(d)   15.67 cm

Answer: (c)

54. The following table shows the age-wise brand ownership of mobile phone handsets.

If a total of 5000 ‘C’ mobile phone sets are sold till date, then how many are more than one year old?

(a)   4350

(b)   4000

(c)   4200

(d)   4500

Answer: (d)

55. If  then the value of  is:

(a)   5/12

(b)   −5

(c)   1

(d)   1/5

Answer: (d)

56. The two trains leave Varanasi for Lucknow at 11:00 a.m. and at 11:30 a.m., respectively and travel at a speed of 110 km/h and 140 km/h, respectively. How many kilometers from Varanasi will both trains meet?

(a) 

(b) 

(c) 

(d) 

Answer: (a)

57. If x + y + z = 10 and xy + yz + zx == 15, then find the value of x3 + y3 + z3 – 3xyz.

(a)   550

(b)   660

(c)   575

(d)   525

Answer: (a)

58. If the perimeter of a certain rectangle is 50 units and its area is 150 sq. units, then how many units is the length of its shorter side?

(a)   15

(b)   9

(c)   10

(d)   12

Answer: (c)

59. A contract is to be completed in 75 days and 187 men are to work 15 hours per day. After 65 days, 3/5 of the work is completed. How many additional men may be employed, so that the work may be completed in time, each man now working 17 hours per day?

(a)   528

(b)   495

(c)   514

(d)   532

Answer: (a)

60. If radius of a circle is decreased by 11%, then the total decrease in the area of the circle is given as:

(a)   20.79%

(b)   19.50%

(c)   20.50%

(d)   21%

Answer: (a)

61. If in 13 years a fixed sum doubles at simple interest, what will be the interest rate per year?

(a)   7.29%

(b)   7.69%

(c)   7.92%

(d)   8.69%

Answer: (b)

62. A man buys two matches ‘A’ and ‘B’ at a total cost of Rs 800. He sells both watches at the same selling price, and earns a profit of 18% on watch ‘A’ and incurs a loss of 22% on watch ‘B’. What are the cost prices of the two watches? (two place after decimal).

(a)   A = Rs 350.32 or B = Rs 450.68

(b)   A = Rs 317 or B = Rs 483

(c)   A = Rs 220 or B = Rs 580

(d)   A = Rs 318.37 or B = Rs 481.63

Answer: (d)

63. The following table shows the annual profit of a company (in Rs lakh).

The period which has the maximum percentage increase in profit over the previous year is :

(a)   2016-2017

(b)   2017-2018

(c)   2018-2019

(d)   2015-2016

Answer: (d)

64. If a2 + b2 + c2 = 300 and ab + bc + ca = 50m then what is the value of a + b + c ? (Given a, b and c are all positive).

(a)   15

(b)   20

(c)   22

(d)   25

Answer: (b)

65. The following table shows the number of employees working in various departments of an organization from 2016 to 2019.

In which year were the maximum number of employees working in the organization?

(a)   2016

(b)   2019

(c)   2017

(d)   2018

Answer: (b)

66. AB is a diameter of a circle with centre O. The tangent at a point C on the circle meets AB produced at Q. If ∠CAB = 42°, then what is the measure of ∠CQA?

(a)   17°

(b)   5°

(c)   6°

(d)   7°

Answer: (c)

67. If 7 divided a positive integer n, the remainder is 2. Which of the following numbers gives a reminder of 0 when divided by 7?

(a)   n + 5

(b)   n – 5

(c)   n + 2

(d)   n + 1

Answer: (a)

68. In quadrilateral PQRS, RM ⊥ QS, PN ⊥ QS and QS = 6 cm. If RM = 3 cm and PN = 2 cm, then the area of PQRS is:

(a)   15 cm2

(b)   14 cm2

(c)   13 cm2

(d)   11 cm2

Answer: (a)

69. The value of-

(a) 

(b) 

(c) 

(d) 

Answer: (b)

70. The cost price of goods, for a shopkeeper, was X. He marked them at 15% above the cost price. Finally, he sold the goods at a discount of 25%. ‘What is his profit/loss percentage?’

(a)   13.5% profit

(b)   13% loss

(c)   12% profit

(d)   13.75% loss

Answer: (d)

71. If 3sec2θ + tan θ = 7, 0° < θ < 90°, then the value of  is:

(a) 

(b) 

(c) 

(d) 

Answer: (c)

72. The perimeter of an isosceles triangles is 50 cm. If the base is 18 cm, then find the length of the equal sides.

(a)   16 cm

(b)   18 cm

(c)   32 cm

(d)   25 cm

Answer: (a)

73. Anu spends 68% of her monthly income. If her monthly income increase by 20% and her monthly savings increase by  then the percentage increase in her monthly expenditure is:

(a)   22%

(b)   20%

(c)   25%

(d)   32%

Answer: (c)

74. 

(a) 

(b) 

(c) 

(d) 

Answer: (c)

75. A sum lent out at compound interest amounts to Rs 1,250 in one year and to Rs 1,458 in 3 years at a certain rate percentage p.a. What is the simple interest on the same sum for  at the same rate of interest?

(a)   Rs 600

(b)   Rs 480

(c)   Rs 500

(d)   Rs 520

Answer: (c)

English Comprehension

76. Select the most appropriate Antonym of the given word.

Assist

(a)   Conceal

(b)   Obstruct

(c)   Decline

(d)   Support

Answer: (b)

77. Select the correct indirect form of the given sentence.

The guard said to the visitors, “Do not take photographs inside the museum”.

(a)   The guard warned the visitors not to be taking photographs inside the museum.

(b)   The guard told the visitors that do not take photographs inside the museum.

(c)   The guard forbade the visitors to take photographs inside the museum.

(d)   The guard requested the visitors to not took photographs inside the museum.

Answer: (c)

78. Select the most appropriate option to substitute the underlined segment in the given sentence. If there is no need to substitute it, select No improvement.

The old lady nearly dead from fright.

(a)   No improvement

(b)   dead out of fright

(c)   died with frightening

(d)   died of fright

Answer: (d)

79. Given below are four jumbled sentences. Out of the given options pick the one that gives their correct order.

(A) I took a deep breath and inhaled the air.

(B) It was a fine spring morning.

(C) As soon as I entered the garden, lush green grass welcomed me.

(D) The freshness in the air drew my feet towards a garden.

(a)   CADB

(b)   BADC

(c)   ACBD

(d)   CDAB

Answer: (b)

80. Given below are jumbled sentences. Out of the given options pick the one that gives their correct order.

(A) It hurts the heart most of all by inflating blood pressure.

(B) You can blow up and let it out or bottle it up and ignore it.

(C) The question then arises – What to do about it?

(D) Researchers have long agreed that anger hurts your health.

(a)   CADB

(b)   DBCA

(c)   BCAD

(d)   DACB

Answer: (d)

81. Select the most appropriate meaning of the given idiom.

Come to the point

(a)   To get reconciled to something

(b)   To speak plainly about the real issue

(c)   To unravel a mystery

(d)   To meet at a designated place

Answer: (b)

Comprehension:

In the following passage some words have been deleted. Fill in the blanks with the help of the alternative given. Select the most appropriate option for each blank.

There are certain spheres of one’s life in which one should go by one’s own (1) ______ rather than follow another’s counsel. The choice (2) ______ one’s subject of study is one such sphere. (3) _______ one is genuinely interested in a subject, one (4) _______ never opt for it. India is already full of (5) ______ pegs in round holes, who are unfit for their roles and are doing more them.

82. Select the most appropriate to fill in blank (1).

(a)   alternative

(b)   option

(c)   possibility

(d)   choice

Answer: (d)

83. Select the most appropriate to fill in blank (2).

(a)   of

(b)   between

(c)   among

(d)   at

Answer: (a)

84. Select the most appropriate to fill in blank (3).

(a)   Although

(b)   Even if

(c)   Since

(d)   Unless

Answer: (d)

85. Select the most appropriate to fill in blank (4).

(a)   should

(b)   could

(c)   would

(d)   ought

Answer: (a)

86. Select the most appropriate to fill in blank (5).

(a)   small

(b)   circular

(c)   square

(d)   large

Answer: (c)

87. Select the most appropriate option to substitute the underlined segment in the given sentence. If there is no need to substitute it, select No improvement.

He was greatly disappointed to be failed again.

(a)   No improvement

(b)   by being failed

(c)   on failing

(d)   to failed

Answer: (c)

88. Select the most appropriate synonym of the given word.

Affluent

(a)   Prosperous

(b)   Authentic

(c)   Hostile

(d)   Impoverished

Answer: (a)

89. Fill in the blank with the most appropriate word.

There are _____ errors in your assignment.

(a)   multifold

(b)   numerous

(c)   sufficient

(d)   abundant

Answer: (b)

90. Select the INCORRECTLY spelt word.

(a)   Actually

(b)   Persuit

(c)   Parallel

(d)   Opinion

Answer: (b)

91. Select the most appropriate synonym of the given word.

Accurate

(a)   Capable

(b)   Precise

(c)   Erroneous

(d)   Compact

Answer: (b)

92. In the sentence identify the segment which contains the grammatical error.

The present what I buy for you yesterday was very expensive.

(a)   What I buy

(b)   The present

(c)   was very expensive

(d)   for you yesterday

Answer: (a)

93. Select the correct passive from of the given sentence.

He incurred heavy losses in business.

(a)   Heavy losses was incurred in business by him.

(b)   Heavy losses are incurred in business by him.

(c)   Heavy losses were incurred in business by him

(d)   Heavy losses are being incurred in business by him.

Answer: (c)

94. Select the INCORRECTLY spelt word.

(a)   Affiliate

(b)   Exclaimation

(c)   Aesthetic

(d)   Annihilate

Answer: (b)

95. Select the most appropriate meaning of the given idiom.

Give a piece of one’s mind

(a)   To share one’s ideas

(b)   To advise someone

(c)   To rebuke someone strongly

(d)   To donate something valuable

Answer: (c)

96. Select the word which means the same as the group of words given.

A place where plants are grown for sale.

(a)   Yard

(b)   Nursery

(c)   Aviary

(d)   Garden

Answer: (b)

97. Select the most appropriate ANTONYM of the given word.

Industrious

(a)   Cautious

(b)   Diligent

(c)   Durable

(d)   Lethargic

Answer: (d)

98. Fill in the blank with the most appropriate word.

There is a _______ growth of flowers in tropical countries.

(a)   plenty

(b)   pious

(c)   powerful

(d)   luxuriant

Answer: (d)

99. In the sentence identify the segment which contains the grammatical error.

He went to work but returned back immediately.

(a)   to work

(b)   immediately

(c)   but returned back

(d)   He went

Answer: (c)

100. Select the word which means the same as the group of words given.

Putting to death painlessly to end suffering.

(a)   Blaspheme

(b)   Euthanasia

(c)   Genocide

(d)   Altruism

Answer: (b)

SSC Combined Graduate Level (Tier-1) Exam-2019 Held on 07-03-2020 Time 10.00-11.00 AM Question Paper With Answer Key

SSC Combined Graduate Level (Tier-1) Exam-2019 Held on 07-03-2020 Time 10.00-11.00 AM
SSC Combined Graduate Level (Tier-1) Exam-2019 Held on 07-03-2020 Time 10.00-11.00 AM Question Paper With Answer Key

SSC Combined Graduate Level (Tier-1) Exam-2019 Held on 07-03-2020 Time 10.00-11.00 AM

General Intelligence and Reasoning

1. Select the option figure in the which the given figure is embedded (rotation is NOT allowed).

Answer: (c)

2. Read the given statements and conclusion carefully. Assuming that the information given in the statements is true, even if it appears to be at variance with commonly known facts, decide which of the given conclusions logically follow(s)

Statements:

(1) Some cars are buses.

(2) Some buses are trucks.

(3) Some trucks are scooters

Conclusions:

(I) Some cars are scooters.

(II) Some trucks are buses.

(III) Some scooters are buses

(IV) All scooters are tucks.

(a)   Only conclusions I and II follow

(b)   Only conclusion II and IV follow

(c)   Only conclusion II follows

(d)   All the conclusions follow

Answer: (c)

3. Select the box that CANNOT be formed by folding the given unfolded box.

Answer: (c)

4. Select the option that is related to the third word in the same way as the second word is related to the first word.

Amusing : Hilarious :: Moisten : ?

(a)   Soak

(b)   Dry

(c)   Humid

(d)   Water

Answer: (a)

5. Select the set of letters that when sequentially placed in the blanks of the given letter series will complete the series.

p_rsqrps_pqs_qr_qrps_p_s

(a)   q, r, p, p, r, q

(b)   q, r, p, s, r, q

(c)   r, p, q, s, r, q

(d)   p, r, p, s, r, p

Answer: (b)

6. The sequence of folding a piece of paper and the manner in which the folded paper has been cut is shown in the following figures. How would this paper look when unfolded?

Answer: (c)

7. Select the option that is related to third number in the same way as the second number is related to the first number and the sixth number is related to the fifth number.

6:16::20:?::11:81

(a)   324

(b)   91

(c)   120

(d)   361

Answer: (a)

8. In a certain code language, CIRCULAR is coded as 24-3-9-24-1-15-5-9. How will VERTICAL be coded as in that language?

(a)   5-2-9-7-3-24-5-15

(b)   22-4-9-7-9-24-5-15

(c)   5-4-9-7-3-24-5-15

(d)   22-4-9-7-3-23-1-15

Answer: (c)

9. Four letter-clusters have been given, out of which three are alike in some manner, while one is different. Select the odd letter-cluster.

(a)   SVY

(b)   FIK

(c)   NQT

(d)   XAD

Answer: (b)

10. Select the correct combination of mathematical signs to sequentially replace the * signs, to balance the following equation.

(14 * 9 * 6) * 15 * 8

(a)   ×, =, ÷, −

(b)   −, ÷, ×, =

(c)   ÷, −, =, ×

(d)   ×, −, ÷, =

Answer: (d)

11. Select the option in which the words share the same relationship as that shared by the given pair of words.

(a)   Appraisal : Performance

(b)   Painting : Painter

(c)   Disease : Doctor

(d)   Carelessness : Errors

Answer: (d)

12. Select the option in which the numbers are related the same way as are the numbers in the given set.

17 : 102 : 153

(a)   16 : 96 : 144

(b)   23 : 162 : 207

(c)   18 : 104 : 171

(d)   13 : 78 : 108

Answer: (a)

13. Select the letter-cluster that can replace the question mark (?) in the following series.

eBd, hXi, kTn, nPs, ?

(a)   qLx

(b)   Qlv

(c)   qKx

(d)   pLw

Answer: (a)

14. Arrange the following words in the order in which they appear in an English dictionary.

(1) Decipher       (2) Decide       (3) Decline

(4) Deceive         (5) Decimal     (6) Decision

(a)   4, 5, 2, 1, 6, 3

(b)   4, 3, 2, 1, 5, 6

(c)   4, 2, 5, 1, 6, 3

(d)   3, 5, 4, 1, 6, 2

Answer: (c)

15. Select the figure that can replace the question mark (?) in the following series.

Answer: ()

16. Four number-pairs have been given, out of which three are alike in some manner and one is different. Select the number-pair that is different.

(a)   4/5 : 3

(b)   16/20 : 6

(c)   25/41 : 8

(d)   30/51 : 9

Answer: (c)

17. A + B means ‘A is the mother of B’;

A – B means ‘A is the husband of B’;

A × B means ‘A is son of B’;

A ÷ B means ‘A is the daughter of B’.

If, W × Z + Y ÷ X, then how is X related to Z?

(a)   Son

(b)   Husband

(c)   Daughter

(d)   Wife

Answer: (b)

18. A private taxi company charges fixed charge along with a per kilometer charge based on the distance covered. For a journey of 24 km, the charges paid are Rs 368 and for a journey of 32 km, the charges paid are Rs 464. How much will a person have to pay for travelling a distance of 15 km?

(a)   Rs 290

(b)   Rs 260

(c)   Rs 280

(d)   Rs 180

Answer: (b)

19. Four words have been given, out of which three are alike in some manner, while one is different. Select the odd word.

(a)   Scream

(b)   Roar

(c)   Mumble

(d)   Shout

Answer: (c)

20. The given Venn diagram represents results of a class of students:

The triangle represents students who scored 85% and above in Maths, the circle represents students who scored 85% and above in English, the rectangle represents students who scored 85% and above in Science, and the square represents students who scored 85% and above in Social Science. The numbers given in the diagram represent the number of students in that particular category.

How many students scored 85% and above in all subjects?

(a)   15

(b)   5

(c)   11

(d)   17

Answer: (b)

21. Study the given pattern carefully and select the number that can replace the question mark (?) in it.

(a)   6

(b)   5

(c)   3

(d)   4

Answer: (b)

22. In a certain code language, ‘CREATION’ is written as RCAEITNO. How will ‘SEQUENCE’ be written as in that language?

(a)   ESUQNEEC

(b)   ESUQENEC

(c)   QESEUENC

(d)   QESEUCNE

Answer: (a)

23. Select the number that can replace the question mark (?) in the following series.

17, 19, 22, 27, 34, 45, 58, ?

(a)   67

(b)   73

(c)   75

(d)   78

Answer: (c)

24. How many triangles are present in the given figure?

(a)   26

(b)   22

(c)   28

(d)   30

Answer: (*)

25. Which of the option figure is the exact mirror image of the given figure when the mirror is held at the right side?

BRWOQWRB

Answer: (b)

General Awareness

26. Pradhan Mantri Jeevan Jyoti Bima Yojana offers a protection term insurance cover of ____ to the insurer.

(a)   Rs 2 Lakh

(b)   Rs 5 Lakh

(c)   Rs 8 Lakh

(d)   Rs 4 Lakh

Answer: (a)

27. Which among the following is NOT an insulator?

(a)   Ebonite

(b)   Mercury

(c)   Glass

(d)   Dry Paper

Answer: (b)

28. Which Indian male cricketer won the BCCI CK Nayudu Lifetime Achievement Award for the year 2019?

(a)   K Srikanth

(b)   Sachin Tendulkar

(c)   Rahul Dravid

(d)   Sunil Gavaskar

Answer: (a)

29. With reference to the Vedangas, which of the following terms denotes ‘Ritual’?

(a)   Chanda

(b)   Kalpa

(c)   Vyakarana

(d)   Shiksha

Answer: (b)

30. Where is the Dharmraja Ratha monument located?

(a)   Suchindram

(b)   Kanchipuram

(c)   Mahabalipuram

(d)   Khajurahoo

Answer: (c)

31. Who was appointed as brand ambassador of Visa – the payment technology company in 2019?

(a)   PT Usha

(b)   P. V. Sindhu

(c)   Dutee Chand

(d)   Sania Mirza

Answer: (b)

32. Who is the author of the book ‘Wise and Otherwise : A salute to life?

(a)   Zoya Hasan

(b)   Amrita Pritam

(c)   Sudha Murthy

(d)   Kiran Desai

Answer: (c)

33. Which of the following states does NOT share its boundary with Bangladesh?

(a)   Mehgalaya

(b)   Tripura

(c)   Assam

(d)   Manipur

Answer: (d)

34. Pravasi Bharatiya Divas is celebrated on:

(a)   2nd January

(b)   8th January

(c)   9th January

(d)   1st January

Answer: (c)

35. The Pritzker Prize is an international award given to recognize contribution in the field of__.

(a)   medicine

(b)   literature

(c)   mathematics

(d)   architecture

Answer: (d)

36. Which of the following rivers forms the dhuandhar waterfall near Japalpur?

(a)   Tapi

(b)   Luni

(c)   Narmada

(d)   Tungabhadra

Answer: (c)

37. The power of a lens is -2.0 D.Here ‘D’ stands for:

(a)   Dilation

(b)   Distance

(c)   Dioptre

(d)   Degree

Answer: (c)

38. Select the correct pair of dance form and its state.

(a)   Padayani – Kerala

(b)   Dalkhai – Karnataka

(c)   Thang Ta – Bihar

(d)   Kalbelia – Himachal Pradesh

Answer: (a)

39. Which among the following has its refractive index closest to that of crown glass?

(a)   Sapphire

(b)   Ruby

(c)   Diamond

(d)   Canada balsam

Answer: (d)

40. The Indian National Congress session of September 1920 was held at

(a)   Calcutta

(b)   Madras

(c)   Nagpur

(d)   Lucknow

Answer: (a)

41. For which game has the Father of Leander Paes been a member of the Indian National Team?

(a)   Hockey

(b)   Basketball

(c)   Tennis

(d)   Badminton

Answer: (a)

42. What does ‘T’ stand for in ATM?

(a)   Teller

(b)   Transfer

(c)   Transaction

(d)   Trunk

Answer: (a)

43. Which of the pairs is correct with reference to mountain passess?

(a)   Rohtang – Sikkim

(b)   Lipulekh – Uttarakhand

(c)   Bomdila – Himachal Pradesh

(d)   Nathula – Arunachal Pradesh

Answer: (b)

44. In which year was the first amendment to the Constitution of India made?

(a)   1950

(b)   1953

(c)   1951

(d)   1952

Answer: (c)

45. Which among the following is a cation?

(a)   Ammonium

(b)   Iodide

(c)   Chloride

(d)   Fluoride

Answer: (a)

46. Which of the following monasteries is located in Sikkim?

(a)   Rumtek

(b)   Kye

(c)   Hemis

(d)   Tabo

Answer: (a)

47. The scientific study of a cell is called:

(a)   taxonomy

(b)   physiology

(c)   histology

(d)   cytology

Answer: (d)

48. Which of the following Articles of the Constitution of India guarantees the Right to Freedom of Religion?

(a)   Articles 19-22

(b)   Articles 23-24

(c)   Articles 14-18

(d)   Aritcles 25-28

Answer: (d)

49. In December 2019, the Rohtang passageway in Himachal Pradesh was renamed as:

(a)   Atal Tunnel

(b)   Mukherjee Tunnel

(c)   Swaraj Tunnel

(d)   Bose Tunnel

Answer: (a)

50. The council of Ministers during the time of Shivaji Maharaj was known as :

(a)   Ashta Pradhan

(b)   Navaratnas

(c)   Agraharam

(d)   Ashta Diggajas

Answer: (a)

Quantitative Aptitude

51. The value of  is :

(a)   −1

(b)   1

(c)   −2sin2θcos2θ

(d)   0

Answer: (d)

52. From the given table, what is the percentage of students scoring 40 or more, but less than 70.

(a)   48%

(b)   8%

(c)   96%

(d)   56%

Answer: (a)

53. The value of 3 – (9 – 3 × 8 ÷ 2) is :

(a)   -21

(b)   6

(c)   0

(d)   21/2

Answer: (b)

54. The coefficient of y in the expansion of (2y – 5)3, is:

(a)   150

(b)   50

(c)   -30

(d)   -150

Answer: (a)

55. In a school, the distribution of teachers is as follows:

(a)   12

(b)   10

(c)   39

(d)   18

Answer: (a)

56. Ten men or twelve women can finish the same work in 10 days. If 5 men and 2 women undertake the work together, how many days will they take to complete the work?

(a)   15

(b)   40

(c)   20

(d)   60

Answer: (a)

57. In an examination, Anita scored 31% marks and failed by 16 marks. Sunita scored 40% marks and obtained 56 marks more than those required to pass. Find the minimum marks required to pass.

(a)   3116

(b)   264

(c)   3944

(d)   7100

Answer: (b)

58. If the given number 925×85 is divisible by 11, then the smallest value of x is:

(a)   1

(b)   2

(c)   3

(d)   4

Answer: (d)

59. Two cars A and B leave Delhi at 8 : 30 a.m. and at 9 a.m. for Shimla, respectively. They travel at the speeds of 40 km/h and 50 km/h respectively. How many kilometers away from Delhi will the two cars be together?

(a)   45 km

(b)   100 km

(c)   200 km

(d)   5 km

Answer: (b)

60. What is the value of sin 30° + cos 30° − tan 45°?

(a) 

(b) 

(c) 

(d) 

Answer: (a)

61. While selling an article of marked price Rs 5,040 at a discount of 40%, if a trader gains 20%, then the profit in Rs is :

(a)   642

(b)   504

(c)   720

(d)   2520

Answer: (b)

62. By selling 18 table fans for Rs 11,664 a man incurs a loss of 10%. How many fans should be sells for Rs 17,424 to earn 10% profit?

(a)   18

(b)   22

(c)   20

(d)   23

Answer: (b)

63. In the given figure, cos θ is equal to:

(a)   5/13

(b)   12/13

(c)   5/12

(d)   12/5

Answer: (a)

64. In the given figure, MP is tangent to a circle with centre A and NQ is a tangent to a circle with centre B. If MP = 15 cm, NQ = 8 cm, PA = 17 cm and BQ = 10 cm, then AB is:

(a)   28 cm

(b)   14 cm

(c)   13.5 cm

(d)   23 cm

Answer: (b)

65. In the given figure, the measure of ∠BAC is :

(a)   56°

(b)   62°

(c)   58°

(d)   48°

Answer: (d)

66. The simple interest on a sum of Rs 50,000 at the end of two years is Rs 4,000. What would be the compound interest on the same sum at the same rate for the same period?

(a)   4040

(b)   4080

(c)   4008

(d)   8000

Answer: (b)

67. If 1 – 64x3 – 12x + 9x2 = (1 – 4x)3, then the value of p is:

(a)   48

(b)   -12

(c)   -48

(d)   16

Answer: (a)

68. In the figure if ∠A = 100° then ∠C = ?

(a)   50°

(b)   90°

(c)   80°

(d)   100°

Answer: (c)

69. The value of 1512 – 1492 is:

(a)   600

(b)   300

(c)   4

(d)   400

Answer: (a)

70. In the given figure, ∆ABC is an isosceles triangle, in which AB = AC, AD ⊥ BC, BC = 6 cm and AD = 4 cm. The length of AB is:

(a)   7 cm

(b)   6 cm

(c)   5 cm

(d)   4 cm

Answer: (c)

71. The area of the four walls of a room having length 6 m, breadth 4 m and height 4 m, is:

(a)   50

(b)   60

(c)   40

(d)   80

Answer: (d)

72. If A : B = 3 : 5, and B : C = 2 : 3, then A : B : C is equal to :

(a)   3 : 8 : 6

(b)   3 : 7 : 3

(c)   6:10 : 15

(d)   6: 15: 10

Answer: (c)

73. Study the given table carefully and answer the question that follows.

The percentage of students who have passed with distinction in  the  year 2012 is:

(a)   25%

(b)   20%

(c)   22%

(d)   27%

Answer: (a)

74. The average of five consecutive odd number is m. If the next three odd numbers are also included, then what is the increase in the average?

(a)   3

(b)   0

(c)   17

(d)   8

Answer: (a)

75. In a particular year, the number of students enrolled in different stream in a College is as follow :

The percentage of girl students is:

(a)   46%

(b)   50%

(c)   135%

(d)   54%

Answer: (d)

English Comprehension

76. Select the most appropriate synonym of the given word.

Sacred

(a)   Holy

(b)   Valued

(c)   Scarce

(d)   Precious

Answer: (a)

77. Select the INCORRECTLY spelt word.

(a)   Alien

(b)   Accross

(c)   Awful

(d)   Already

Answer: (b)

78. Select the most appropriate meaning of the given idiom.

Look down upon

(a)   To consider someone inferior

(b)   The be full of guilt

(c)   To  look for something underground

(d)   To look down from a height

Answer: (a)

79. Select the correct passive form of the given sentence.

The children sang the National Anthem with great enthusiasm.

(a)   The National Anthem is sung with great enthusiasm by the children.

(b)   The National Anthem has been sung with great enthusiasm by the children

(c)   The National Anthem was being sung with great enthusiasm by the children.

(d)   The National Anthem was sung with great enthusiasm by the children.

Answer: (d)

80. Select the option that can be used as a one word substitute for the given group of words.

A game in which neither party wins

(a)   Quit

(b)   Draw

(c)   Flop

(d)   Equal

Answer: (b)

81. Select the most appropriate ANTONYM of the given word.

Native

(a)   Rural

(b)   Rustic

(c)   Foreign

(d)   Urban

Answer: (c)

Comprehension:

In the following passage some words have been deleted. Fill in the banks with the help of the alternatives given. Select the most appropriate option for each blank.

Advertising is the promotion of goods and services, it provides an (1) ______ to the manufacturing companies to sell their (2) ______ better than their competitors. Every time we switch (3) _______ the television we find some product or the other (4) _____ pushed towards us. It cannot be denied that the (5) ______ behind commercial advertising is to increase sales and earn more profit.

82. Select the most appropriate option to fill in blank number (1).

(a)   opportunity

(b)   excuse

(c)   occasion

(d)   event

Answer: (a)

83. Select the most appropriate option to fill in blank number (2).

(a)   creation

(b)   products

(c)   compounds

(d)   outcome

Answer: (b)

84. Select the most appropriate option to fill in blank number (3).

(a)   in

(b)   on

(c)   off

(d)   at

Answer: (b)

85. Select the most appropriate option to fill in blank number (4).

(a)   be

(b)   being

(c)   been

(d)   to be

Answer: (b)

86. Select the most appropriate option to fill in blank number (5).

(a)   influence

(b)   feeling

(c)   motive

(d)   emotion

Answer: (c)

87. Given below are four jumbled sentences. Out of the given options pick the one that gives their correct order.

(A) When a calf is born under water, the mother must get it to the surface before it drowns.

(B) The young ones remain with their parents for up to fifteen years or more.

(C) Often another whale assists the mother nudging the baby gently and encouraging it to swim.

(D) Whales have highly developed maternal instincts.

(a)   DBCA

(b)   BCAD

(c)   CBDA

(d)   DACB

Answer: (d)

88. Fill in the blank with the most appropriate word.

She was loved by her employees for her _______.

(a)   malevolence

(b)   condolence

(c)   benevolence

(d)   insolence

Answer: (c)

89. Identify the segment in the sentence which contains the grammatical error. If there is no error, select ‘No error’

He married with a rich heiress last year.

(a)   last year

(b)   with a rich heiress

(c)   He married

(d)   No error

Answer: (b)

90. Select the most appropriate meaning of the given idiom.

Keep abreast of

(a)   Keep up the good work

(b)   Keep a watch on

(c)   Keep ahead of

(d)   Keep oneself updated

Answer: (d)

91. Select the most appropriate option to substitute the underlined segment in the given sentence. If there is no need to substitute it select No improvement.

Mother was watching a movie when the lights were going off.

(a)   have gone off

(b)   No improvement

(c)   going off

(d)   went off

Answer: (d)

92. Given below are four jumbled sentences. Out of the given option pick the one that gives their correct order.

(A) While he was doing this, he missed seeing a velvet purse full of gold coins lying on the road.

(B) Ramesh always worried about how he would become sick and weak in old age.

(C) To try how he would cope with blindness, Ramesh started walking with this eyes closed.

(D) He feared that he might even lose his sight and go blind.

(a)   CABD

(b)   BCAD

(c)   DBAC

(d)   BDCA

Answer: (d)

93. Select the most appropriate synonym of the given word.

Weary

(a)   Touchy

(b)   Lively

(c)   Restless

(d)   Exhausted

Answer: (d)

94. Select the most appropriate ANTONYM of the given word.

Repulsive

(a)   Revolting

(b)   Hideous

(c)   Brilliant

(d)   Attractive

Answer: (d)

95. Fill in the blank with the most appropriate word.

The minister gave an _______ that strict action would be taken against the culprits.

(a)   ambition

(b)   assurance

(c)   insurance

(d)   admission

Answer: (b)

96. Select the most appropriate option to substitute segment in the given sentence. If there is no need to substitute it, select No improvement.

Scarcely had the passenger boarded the plane when the captain welcomed them.

(a)   that the captain welcomed them

(b)   when the captain had welcomed them

(c)   than the captain welcomes them

(d)   No improvement

Answer: (d)

97. Identify the segment in the sentence which contains the grammatical error. If there is no error, select ‘No error’

The famous author and actor are being honourd at a function today

(a)   are being honoured

(b)   The famous author and actor

(c)   No error

(d)   at a function today

Answer: (a)

98. Select the option that can be used as a one word substitute for the given group of words

That which cannot be satisfied

(a)   Insane

(b)   Ingenuous

(c)   Inapt

(d)   Insatiable

Answer: (d)

99. Select the correct indirect form the given sentence.

The traveler said to passerby, “Can you help me find my way, please?”

(a)   The traveler requested to the passer by if he can help him find his way.

(b)   The traveler asked the passer by that he can kindly help him find his way.

(c)   The traveler asked the passer by if he could kindly help him find his way.

(d)   The traveler asked the passer by could you kindly help me find my way?

Answer: (c)

100. Select the INCORRECTLY spelt word.

(a)   Guarranty

(b)   Guardian

(c)   Guidance

(d)   Guilty

Answer: (a)

SSC Combined Graduate Level (Tier-1) Exam-2019 Held on 09-03-2020 Time 1.00-2.00 PM Question Paper With Answer Key

SSC Combined Graduate Level (Tier-1) Exam-2019 Held on 09-03-2020 Time 1.00-2.00 PM
SSC Combined Graduate Level (Tier-1) Exam-2019 Held on 09-03-2020 Time 1.00-2.00 PM Question Paper With Answer Key

SSC Combined Graduate Level (Tier-1) Exam-2019 Held on 09-03-2020 Time 1.00-2.00 PM

General Intelligence and Reasoning

1. A – B means ‘A is the sister of B’;

A × B means ‘A is the husband of B’;

A ÷ B means ‘A is son of B’.

If, Y × M – T – Z ÷ L × U, then how is U related to Y?

(a)   Mother

(b)   Father

(c)   Father-in-law

(d)   Mother-in-law

Answer: (d)

2. Select the number that can replace the question mark (?) in the following series.

39, 50, 63, 80, ?, 122, 151, 182

(a)   108

(b)   105

(c)   99

(d)   90

Answer: (c)

3. Which two signs should be interchanged to make the given equation correct?

14 × 3 ÷ 27 + 54 – 9 = 21

(a)   + and −

(b)   × and −

(c)   × and ÷

(d)   ÷ and −

Answer: (c)

4. Four number-pairs have been given, out of which three are alike in some manner and one is different. Select the number-pair that is different.

(a)   39 : 270

(b)   31 : 214

(c)   21 : 154

(d)   37 : 256

Answer: (c)

5. Which of the following Venn diagrams best represents the relationship between the following classes?

Lady Constables, Uncles, Mothers

Answer: (d)

6. Select the letter will replace the question mark (?) in the following series.

A, J, E, L, I, N, O, P, U, ?

(a)   B

(b)   R

(c)   K

(d)   W

Answer: (b)

7. How many triangle are there in the given figure.

(a)   13

(b)   12

(c)   15

(d)   14

Answer: (a)

8. Select the option in which the numbers are related in the same way as are the numbers in the given set.

(22, 64, 352)

(a)   (27, 52, 453)

(b)   (19, 52, 257)

(c)   (14, 68, 238)

(d)   (24, 16, 108)

Answer: (c)

9. Select the option in which the given figure X is embedded (rotation is not allowed).

Answer: (d)

10. What day of the week of 29 June 2010?

(a)   Tuesday

(b)   Sunday

(c)   Wednesday

(d)   Monday

Answer: (a)

11. The sequence of folding a piece of paper and the manner in which the folded paper has been cut is shown in the following figures. How would this paper look when unfolded?

Answer: (c)

12. Four letter-clusters have been given, out of which there are alike in some manner and one is different. Select the odd letter-cluster.

(a)   KN

(b)   LO

(c)   AZ

(d)   GT

Answer: (a)

13. Read the given statement and conclusions carefully. Assuming that the information given in the statements is true, even if it appears to be at variance with commonly known facts, decide which of the given conclusion logically follow(s)

Statements:

(1) No garden is a park.

(2) Some gardens are schools.

Conclusions

(I) No park is a garden.

(II) Some schools are gardens.

(III) Some parks are gardens

(a)   Either conclusion I or III and conclusion II follow

(b)   Only conclusions II and III follow

(c)   Either conclusion I or III follows

(d)   Only conclusions I and II follow

Answer: (d)

14. Four words have been given, out of which three are alike in some manner and one is different. Select the odd words.

(a)   Cancer

(b)   Asthma

(c)   Diabetes

(d)   Bone Marrow

Answer: (d)

15. Select the option that is related to the third number in the same way as the second number is related to the first number.

3:51::7:?

(a)   319

(b)   679

(c)   609

(d)   335

Answer: (b)

16. Select the letter-cluster that can replace the question mark (?) in the following series.

NAVIGATE, PCVIGATE, NAXKGATE, ?, NAVIGAVG

(a)   NAVIJCTF

(b)   PCVII CUG

(c)   NAXKICTE

(d)   NAVIICTE

Answer: (d)

17. Study the given pattern carefully and select he number that can replace the question mark (?) in it.

15           18        ?

8             9          12

161         243      432

(a)   24

(b)   26

(c)   22

(d)   20

Answer: (a)

18. Select the correct mirror image of the given letter-cluster when a vertical mirror is placed on the right side of the cluster.

EWGZMPK

Answer: (a)

19. Select the dice that can be formed by folding the given sheet along the lines.

(a)   Only A, B and C

(b)   Only A and B

(c)   Only A

(d)   Only B and D

Answer: (b)

20. In a certain code language, ‘FIXATION’ is written as ‘AXIFNOIT’. How will ‘GLYCERIN’ be written as in that language?

(a)   NIRECYLG

(b)   CYLGNIRE

(c)   LGZDGINR

(d)   YLGECRNI

Answer: (b)

21. ‘Photoshop’ is related to ‘Software’ in the same way as ‘Optical Mouse’ is related to ‘________’.

(a)   Touch-screen

(b)   Hardware

(c)   Hard-disc

(d)   Malware

Answer: (b)

22. Select the figure that can replace the question mark (?) in the following series.

Answer: (c)

23. Select the option in which the words share the same relationship as t hat shared by the given pair of words.

Temple : Worship

(a)   Church : Prayer

(b)   Golden Temple : Amritsar

(c)   Garden : Plants

(d)   University : President

Answer: (a)

24. Arrange the following words in a logical and meaningful order.

(1) Casting         (2) Acting        (3) Film Shooting        (4) Final Film  (5) Editing

(a)   3, 2, 1, 4, 5

(b)   4, 3, 5, 1, 2

(c)   1, 2, 3, 5, 4

(d)   3,1, 2, 5, 4

Answer: (c)

25. In a certain code language, ‘SPECIAL’ is coded as ‘20176410213’. How will ‘MACHINE’ be coded as in that language?

(a)   1424901056

(b)   1324810155

(c)   1524910146

(d)   1424910156

Answer: (d)

General Awareness

26. What does ‘C’ stand for in ITCZ?

(a)   Constant

(b)   Convection

(c)   Circulation

(d)   Convergence

Answer: (d)

27. What is the rank of India in the Global Peace Index 2019?

(a)   142

(b)   143

(c)   144

(d)   141

Answer: (d)

28. Who among the following was the mascot of the Men’s Hockey World Cup held at Bhubaneswar in 2018?

(a)   Turtle Olly

(b)   Shera

(c)   Borobi

(d)   Millie

Answer: (a)

29. Which tales are related with the painting and sculptures of the Ajanta caves?

(a)   Pentamerone Tales

(b)   Panchatantra Tales

(c)   Hitopadesha Tales

(d)   Jataka Tales

Answer: (d)

30. In 2019, IIT-Madras launched the country’s first indigenously designed standing wheelchair named is:

(a)   Arise

(b)   Optimist

(c)   Marathon

(d)   Awake

Answer: (a)

31. Who of the following authored the book ‘Revolution 2020’?

(a)   Rupa Pai

(b)   Arvind Kejriwal

(c)   Devdutt Pattnaik

(d)   Chetan Bhagat

Answer: (d)

32. How many medals did India win in the 2012 Summer Olympics?

(a)   6

(b)   5

(c)   3

(d)   4

Answer: (a)

33. Who among the following is NOT a recipient of the 2019 Mother Teresa Memorial Awards for Social Justice?

(a)   Priti Patkar

(b)   Medha Patkar

(c)   Kailash Satyarthi

(d)   Ajeet Singh

Answer: (b)

34. Which of the following ministries is NOT associated with the initiative of Beti Bachao, Beti Padhao Scheme?

(a)   Ministry of Women and Child Development

(b)   Ministry of Rural Development

(c)   Ministry of Health and Family Welfare

(d)   Ministry of Human Resource Development

Answer: (b)

35. Which of the following events did NOT occur in 1919?

(a)   Rowlatt Act was passed

(b)   Partition of Bengal took place

(c)   Montague Chelmsford Reform was announced

(d)   Jallianwala Bagh tragedy took place

Answer: (b)

36. Constantan is an alloy of copper and ______.

(a)   iron

(b)   nickel

(c)   aluminium

(d)   tin

Answer: (b)

37. Which among the following is a non-biodegradable waste?

(a)   Dead animals

(b)   Plastic

(c)   Flowers

(d)   Vegetable

Answer: (b)

38. Who signed the treaty of Alinagar with the British?

(a)   Alivardi Khan

(b)   Mir Jafar

(c)   Mir Qasim

(d)   Siraj-ud-Daula

Answer: (d)

39. Which of the following is the lowermost layer of the Atmosphere?

(a)   Troposphere

(b)   Exosphere

(c)   Thermosphere

(d)   Mesosphere

Answer: (a)

40. Which of the following states does NOT share its boundary with Nepal?

(a)   Uttar Pradesh

(b)   Assam

(c)   West Bengal

(d)   Bihar

Answer: (b)

41. How many molecules of ATP are obtained by the respiration of one molecule of glucose?

(a)   38

(b)   4

(c)   2

(d)   36

Answer: (c)

42. ‘Pedology’ is the science of:

(a)   soil

(b)   childhood illness

(c)   ground water

(d)   skin disease

Answer: (a)

43. Under which Amendment to the Constitution of India was Goods and Services Tax imposed?

(a)   97th

(b)   101st

(c)   99th

(d)   103rd

Answer: (b)

44. What is the rank of India in the Human Development Index 2019?

(a)   128th

(b)   89th

(c)   129th

(d)   130th

Answer: (c)

45. In July 2019, which of the following schemes was launched to accelerate water harvesting and conservation measures?

(a)   Jal Vikas Abhiyan

(b)   Jal Bachao Abhiyan

(c)   Jal hi dhan hai Abhiyan

(d)   Jal Shakti Abhiyan

Answer: (d)

46. Which of the following rivers was known as Purushni in the Vedic period?

(a)   Chenab

(b)   Sutlej

(c)   Beas

(d)   Ravi

Answer: (d)

47. In which state is the festival of Uttarayan uniquely celebrated?

(a)   Jharkhand

(b)   Gujarat

(c)   Uttar Pradesh

(d)   Kerala

Answer: (b)

48. The rating for a fuse used in a household electric circuit is provided on the basis of:

(a)   current

(b)   resistance

(c)   power

(d)   voltage

Answer: (a)

49. Who among the following initiated ‘Din-i-illhai’?

(a)   Babur

(b)   Humayun

(c)   Jahangir

(d)   Akbar

Answer: (d)

50. Who among the following is the Chief Minister of Mizoram as on November 2019?

(a)   Neiphiu Rio

(b)   Laldenga

(c)   PU Zoramthanga

(d)   Conrad Kongkal Sangma

Answer: (c)

Quantitative Aptitude

51. The following table shows the production of fertilizers (in lakh tonne) by six companies for 5 months (January to May).

There a continuous decrease in production over the months in:

(a)   Company I

(b)   Company II

(c)   Company IV

(d)   Company III

Answer: (a)

52. A shopkeeper buys two books from Rs 3000.. He sells the first book at a profit of 20% and the second book at a loss of 10%. What is the selling price of the first book, if, in the whole transaction there is no profit no loss?

(a)   Rs 115

(b)   Rs 125

(c)   Rs 120

(d)   Rs 110

Answer: (c)

53. When 2 is subtracted from each of the given an numbers, then the sum of the numbers so obtained is 102. When 5 is subtracted from each of them, then the sum of the numbers so obtained is 12. What is the average of the given n number?

(a)   5.8

(b)   5.4

(c)   6.6

(d)   6.2

Answer: (b)

54. A certain sum amounts to Rs 280900 in 2 years at 6% per annum, interest compounded annually. The sum is:

(a)   Rs 350000

(b)   Rs 250000

(c)   Rs 200000

(d)   Rs 550000

Answer: (b)

55. The average of 4 terms is 30 and the 1st term is 1/3 of the sum of the remaining terms. What is the first term?

(a)   40

(b)   30

(c)   20

(d)   60

Answer: (b)

56. Solve the following.

(a)   2

(b)   1

(c)   3

(d)   0

Answer: (d)

57. A wheel covers a distance of 1,100 cum in one round. The diameter of the wheel is:

(a)   100 cm

(b)   175 cm

(c)   125 cm

(d)   150 cm

Answer: (*)

58. Solve the following expression.

11 + 11 × 11 – 11 ÷ 11

(a)   22

(b)   131

(c)   121

(d)   11

Answer: (b)

59. If 5 divided the integer n, the remainder is 2. What will be the remainder if 7n is divided by 5?

(a)   1

(b)   3

(c)   2

(d)   4

Answer: (d)

60. The volumes of spheres A and B are in the ratio 125 : 64. If the sum of radii of A and B is 36 cm, then the surface area (in cm2) of A is:

(a)   800 π

(b)   512 π

(c)   1600 π

(d)   1024 π

Answer: (c)

61. Arrange the angles of the triangle from smallest to largest in the triangle, where the sides are AB = 7 cm, AC = 8 cm, BC = 9 cm. AB = 7 cm, AC = 8 cm, BC = 9 cm

(a)   A, B, C

(b)   C, B, D

(c)   C, B, A

(d)   B, A, C

Answer: (c)

62. If a = 2b = 8c and a + b + c = 13 then the value of  is:

(a)   9/2

(b)   5/6

(c)   −9/2

(d)   −5/6

Answer: (a)

63. In a circle, chords PQ and TS are produced to meet at R. If RQ = 14.4 cm , PQ = 11.2 cm, and SR = 12.8 cm, then the length of chord of TS is:

(a)   18 cm

(b)   14.2 cm

(c)   12.4 cm

(d)   16 cm

Answer: (d)

64. A, B and C can individually complete a task in 24 days, 16 days, and 32 days respectively. If A and C start the work and worked for 6 days and left, then the number of days required by B to complete the remaining task, is:

(a)   9

(b)  

(c) 

(d)  

Answer: (a)

65. The following table shows the imports and exports (in Rs 2019).

The average trade balance (in Rs crores) is:

(a)   4

(b)   8

(c)   6

(d)   12

Answer: (b)

66. Find the height of a cuboid whose volume is 330 cm2 and base area is 15 cm2.

(a)   22 cm

(b)   21 cm

(c)   19 cm

(d)   24 cm

Answer: (a)

67. If 5 cot θ = 3, find the value of 

(a)   20/41

(b)   44/21

(c)   11/40

(d)   21/44

Answer: (d)

68. If x, y, z are three numbers such that x + y = 13, y + z = 15 and z + x = 16, then the value of xy + xz is:

(a)   5/36

(b)   18/5

(c)   5/18

(d)   36/5

Answer: (c)

69. A train X travelling at 60 km/h overtakes another train Y, 225 m long, and completely passes it in 72 seconds. If the trains had been going in opposite directions, they would have passed each other in 18 seconds. The length (in m) of X and the speed (in km/h) of Y are respectively:

(a)   245 and 45/245

(b)   245 and 54/245

(c)   255 and 36/255

(d)   255 and 40/255

Answer: (c)

70. If a2 + b2 – c2 = 0, then the value of  is:

(a)   1

(b)   0

(c)   2

(d)   3

Answer: (*)

71. Study the following table and answer the question.

Expenditures of a company (in Lakh Rupees) per annum over the given Years

Total expenditure on all the items in 2014 was approximately what percent of the total expenditure in 2018?

(a)   65.32%

(b)   68.32%

(c)   66.32%

(d)   60.32%

Answer: (c)

72. A man wanted to sell his bat at a discount of 8%. His brother who was the cricketer wanted to buy the bat, so the man sells it at a discount of 10%. In this deal, the man reduces Rs 70 in profit. What was the market value of the bat?

(a)   Rs 3,000

(b)   Rs 3,500

(c)   Rs 2,500

(d)   Rs 3,200

Answer: (b)

73. Amit travelled a distance of 50 km in 9 hours. He travelled partly on foot at 5 km/h and partly by bicycle at 10 km/h. The distance travelled on the bicycle is:

(a)   12 km

(b)   13 km

(c)   10 km

(d)   11 km

Answer: (c)

74. The value (cosec A + cot A + 1) (cosec A – cot A + 1) – 2 cosec A is:

(a)   2

(b)   0

(c)   4 cosec A

(d)   2 cosec A

Answer: (a)

75. The following table shows the production of fertilizers (in lakh tonne) by six companies for 5 months (January to May).

In which months does Company II have a contribution of approximately 20% iin the total fertilizer production?

(a)   May

(b)   April

(c)   January

(d)   March

Answer: (b)

English Comprehension

76. Select the most appropriate synonym of the given word.

Abort

(a)   Continue

(b)   Begin

(c)   End

(d)   Hate

Answer: (c)

77. In the sentence identify the segment which contains the grammatical error.

We must get this post send as soon as possible.

(a)   We  must get

(b)   soon as possible

(c)   this post

(d)   send as

Answer: (d)

78. Select the most appropriate option to substitute the underlined segment in the given sentence. If there is no need to substitute it, select no improvement.

I was sitting at home doing nothing when I had heard that song again.

(a)   hears

(b)   heard

(c)   has heard

(d)   No improvement

Answer: (b)

79. Given below are four jumbled sentences. Out of the given options pick the one that gives their correct order.

(A) They also visit ailing friends and relatives, or attend to personal matters even during office hours.

(B) Once in office, they receive friends and relatives who call them any time without prior appointment.

(C) We can frequently find a large number of people sitting here and there and doing nothing.

(D) Even those who are employed often come late to office and leave early unless they are forced to be punctual.

(a)   ABCD

(b)   CDBA

(c)   DCAB

(d)   ACDB

Answer: (b)

80. Select the word which means the same as the group of words given.

Having something more than required

(a)   Surplus

(b)   Surrogate

(c)   Surroundings

(d)   Surcharge

Answer: (a)

81. Select the correct indirect form of the given sentence.

“What are  you going to call the puppy, Jane,?” the boy asked.

(a)   Jane asked the boy what is he going to call the puppy.

(b)   The boy wondered what Jane were going to call the puppy.

(c)   The boy asked Jane what she was going to call the puppy.

(d)   The boy told Jane what she was going to call her puppy.

Answer: (c)

Comprehension:

In the following passage some words have been deleted. Fill in the blanks with the help of the alternatives given. Select the most appropriate option for each blank.

In a way, adult education is an educational movement through the medium (1) _____ village school. In the beginning, the (2) ______ were suspicious. It was hard to persuade (3) _____ to realize that they were really getting something (4) _____ the schools. They were not at all sure that they wanted their (5) ______ to leave their homes.

82. Select the most appropriate option for the blank numbered 1.

(a)   by

(b)   in

(c)   from

(d)   of

Answer: (d)

83. Select the most appropriate option for the blank numbered 2.

(a)   politicians

(b)   villagers

(c)   officers

(d)   children

Answer: (b)

84. Select the most appropriate option for the blank numbered 3.

(a)   us

(b)   him

(c)   them

(d)   they

Answer: (c)

85. Select the most appropriate option for the blank numbered 4.

(a)   by

(b)   to

(c)   from

(d)   for

Answer: ()

86. Select the most appropriate option for the blank numbered 5.

(a)   teachers

(b)   children

(c)   parents

(d)   villagers

Answer: (b)

87. Select the word which means the same as the group of words given.

Anger about an unfair situation or about someone’s unfair behavior.

(a)   Indignation

(b)   Indulgence

(c)   Induction

(d)   Indigenization

Answer: (a)

88. Select the most appropriate meaning of the following idiom.

Blind alley

(a)   A person who comes to meet occasionally

(b)   A situation in which no further progress can be made

(c)   Taking first step after somebody’s approach

(d)   A state of deep thought

Answer: (b)

89. Select the most appropriate ANTONYM of the given word.

Mellow

(a)   Medium

(b)   Shiny

(c)   Soft

(d)   Hard

Answer: (d)

90. Select the most appropriate option to substitute the underlined segment in the given sentence. If there is no need to substitute it, select No improvement.

One evening, when father was coming home from work, I played in the front garden.

(a)   was playing

(b)   a playing

(c)   play

(d)   No improvement

Answer: (a)

91. Select the INCORRECTLY spelt word.

(a)   Guarantee

(b)   Dilemma

(c)   Athelete

(d)   Convenience

Answer: (c)

92. Select the correct passive from of the given sentence.

The commander order his soldiers to move forward and attack the enemy camps.

(a)   The soldiers were ordered by their commander to move forward and attack the enemy camps.

(b)   The commander ordered his soldiers to be moved forward and attacked the enemy camps.

(c)   The commander was ordered by his soldiers to move forward and attack the enemy camps.

(d)   His soldiers were ordered by the commander to be move forward and attacked the enemy camps

Answer: (a)

93. Select the word appropriate synonym of the given word.

Exceed

(a)   Improve

(b)   Decline

(c)   Surpass

(d)   Decrease

Answer: (c)

94. Select the most appropriate option to fill in the blank.

My cousin’s study on the reading habits of school children across three states has been recognized as a _______ piece research.

(a)   pioneering

(b)   negligent

(c)   useless

(d)   proud

Answer: (a)

95. Select the most appropriate meaning of the following idiom

The acid test

(a)   Throwing acid on someone’s face

(b)   A critical situation or crisis

(c)   A fact, event or situation that proves something

(d)   An unpleasant or offensive test

Answer: (c)

96. Select the most appropriate option to fill in the blank.

Suzerainty is having political control over a _______ state.

(a)   democratic

(b)   backward

(c)   dependent

(d)   unified

Answer: (c)

97. In the sentence identify the segment which contains the grammatical error.

Sita has no tate in classical dance.

(a)   in

(b)   classical dance

(c)   Sita has

(d)   no taste

Answer: (a)

98. Given below are four jumbled sentences. Out of the given options pick the one that gives their correct order.

(A) Accommodation and food were given free.

(B) This was held in a hostel for poor students built by a philanthropist.

(C) A few years ago, I was the chief guest at a small function.

(D) But the inmates of the hostel has to hear other expenses like clothing and toiletries.

(a)   ABCD

(b)   BCAD

(c)   BCDA

(d)   CBAD

Answer: (d)

99. Select the most appropriate ANOTNYM of the given word.

Emerge

(a)   Surface

(b)   Disappear

(c)   Announce

(d)   Appear

Answer: (b)

100. Select the INCORRECTLY spelt word.

(a)   Soldier

(b)   Beginning

(c)   Guidence

(d)   Shining

Answer: (c)

SSC Combined Graduate Level (Tier-1) Exam-2019 Held on 09-03-2020 Time 10.00-11.00 AM Question Paper With Answer Key

SSC Combined Graduate Level (Tier-1) Exam-2019 Held on 09-03-2020 Time 10.00-11.00 AM
SSC Combined Graduate Level (Tier-1) Exam-2019 Held on 09-03-2020 Time 10.00-11.00 AM Question Paper With Answer Key

SSC Combined Graduate Level (Tier-1) Exam-2019 Held on 09-03-2020 Time 10:00-11.00 AM

General Intelligence and Reasoning

1. Select the option in which the numbers are related in the same way as are the numbers in the given set.

(88, 60, 37)

(a)   (36, 70, 38)

(b)   (43, 49, 27)

(c)   (56, 38, 28)

(d)   (94, 42, 34)

Answer: (d)

2. ‘Ethiopia’ is related to ‘Africa’ in the same way as ‘Brazil’ is related to’

(a)   Brasilia

(b)   South America

(c)   Africa

(d)   North America

Answer: (b)

3. Four letter-clusters have been given. Out of which three are alike in some manner and one is different. Select the odd letter-cluster.

(a)   JNQS

(b)   RSTU

(c)   TVXZ

(d)   ADGJ

Answer: (a)

4. In a certain code language, ‘POUND’ is coded as ‘106’ and ‘CLEAN’ is coded as ‘41’. How will ‘MAKER’ be coded as in that language?

(a)   112

(b)   50

(c)   54

(d)   78

Answer: (c)

5. Select the option in which the given figure is embedded (rotation is not allowed).

Answer: (b)

6. Which of the following Venn diagrams best represents the relationship between the classes?

Tables, Wooden objects, Chairs

Answer: (b)

7. Four number-pairs have been given, out of which three are alike in some manner and one is different. Select the number-pair that is different from the rest.

(a)   53 : 378

(b)   45 : 322

(c)   37 : 266

(d)   43 : 318

Answer: (d)

8. Select the letter-cluster that can replace the question mark (?) in the following series.

DJQC, ELTG, FNWK, GPZO, ?

(a)   HRCS

(b)   HRBU

(c)   HQCT

(d)   GRDS

Answer: (a)

9. 60 students participated in one or more of the three competitions, i.e. Quiz, Extempore and Debate. A total of 22 students participated either in Quiz only or in Extempore only. 4 students participated in all three competitions. A total of 14 students participated in any of the two competitions only. How many students participated in Debate only?

(a)   22

(b)   11

(c)   20

(d)   14

Answer: (c)

10. Select the option in which the words share the same relationship as that shared by the given pair of words.

Audition : Selection

(a)   Competition : Poem

(b)   Machine : Fault

(c)   Examination: Pass

(d)   Loyalty : Fraud

Answer: (c)

11. Select the letter that can replace the question mark (?) in the following series.

Y, R, L, G, ?, Z

(a)   F

(b)   E

(c)   D

(d)   C

Answer: (d)

12. Four words have been given, out of which three are alike in some manner and one is different. Select the odd word.

(a)   Volume

(b)   Gallon

(c)   Distance

(d)   Weight

Answer: (b)

13. Arrange the following words in the order in which they would appear in an English dictionary.

(1) Realistic (2) Realism (3) Restore (4) Research (5) Resurge

(a)   1, 2, 4, 3, 5

(b)   2, 1, 4, 5, 3

(c)   2, 1, 3, 4, 5

(d)   2, 1, 4, 3, 5

Answer: (d)

14. Read the given statements and conclusions carefully. Assuming that the information given in the statements is true, even if it appears to be at variance with commonly known fats, decide which of the given conclusions logically follow(s) the statements.

Statements:

(1) Some dancers are writers.

(2) All writers are students.

Conclusion:

(I) Some students are dancers.

(II) Some students are writers.

(III) No student is a dancer.

(a)   Only conclusion II and III

(b)   Either conclusion I or III follows

(c)   Only conclusion I and II follows

(d)   All conclusions I, II and III follows

Answer: (c)

15. Select the option that is related to the third number in the same way as the second number is related to the first number.

56 : 30 :: 78 : ?

(a)   61

(b)   53

(c)   50

(d)   56

Answer: (d)

16. In a certain code language ‘DRONE’ is written as ‘SERFO’. How will ‘HOUSE’ be written as I that language?

(a)   PIXFT

(b)   QJXGB

(c)   PQWGS

(d)   PIVFT

Answer: (a)

17. A piece of paper is folded and cut as shown below in the question figures. Select from t he given answer figures, how it will appear when unfolded?

Answer: (a)

18. A + B means ‘A’ is the father of B’;

A × B means ‘A is the brother of B’;

A ÷ B means ‘A is the sister of B’.

Which of the following expression means ‘H is the paternal grandson of P’?

(a)   P + K + D ÷ H × M

(b)   D + H + K × M ÷ P

(c)   P + K + D × H ÷ M

(d)   H + K + D × P ÷ M

Answer: (d)

19. Identify the number that does NOT belong to the following series.

2, 6, 14, 0, 62, 126, 250

(a)   62

(b)   126

(c)   250

(d)   14

Answer: (c)

20. How many triangles are there in the given figure?

(a)   24

(b)   22

(c)   21

(d)   23

Answer: (b)

21. Select the correct mirror image of the given letter-cluster when a vertical mirror is placed on the right side of the cluster.

RQBKHNT

Answer: (b)

22. A cube is made by folding the given sheet. In the cube so formed, which of the following pairs of letters will be on opposite sides?

(a)   A & Q

(b)   Q & E

(c)   L & U

(d)   B & E

Answer: (b)

23. If ‘A’ stands for ‘subtraction’, ‘B’ stands for ‘multiplication’, ‘C’ stands for ‘addition’ and ‘D’ stands for ‘division’, then what is the value of the following expression?

32 B 4 A 12 B (35 A 24) C 52 D 4

(a)   47

(b)   9

(c)   39

(d)   19

Answer: (b)

24. Select the figure that can replace the question mark (?) in the following series.

Answer: (b)

25. Study the given pattern carefully and select the number that can replace the question mark (?) in it.

(a)   108

(b)   69

(c)   205

(d)   55

Answer: (a)

General Awareness

26. Name the Indian equestrian to qualify for the Tokyo Olympics officially after a wait/span of two decades.

(a)   Fouaad Mirza

(b)   Ghulam Mohammed Khan

(c)   Raghubir Singh

(d)   Vishal Singh

Answer: (a)

27. The India Meteorological Department (IMD) celebrated its _______foundation day on January 15, 2020.

(a)   135th

(b)   145th

(c)   140th

(d)   139th

Answer: (b)

28. In which of the following is citric acid present?

(a)   Curd

(b)   Black Gram

(c)   Lemon

(d)   Tamarind

Answer: (c)

29. The Paithan Hydro-Electric Project is on river ______.

(a)   Godavari

(b)   Brahmaputra

(c)   Narmada

(d)   Yamuna

Answer: (a)

30. Shaukat Kaifi passed away in 2019. She was the mother of which famous actress?

(a)   Dia Mirza

(b)   Shabana Azmi

(c)   Fatima Sana Shaikh

(d)   Urmila Matondkar

Answer: (b)

31. Which of the following years is called the year of the Great Divide in the demographic history of India?

(a)   1931

(b)   1911

(c)   1921

(d)   1951

Answer: (c)

32. Jai Prakash Narayn Museum of Socialism is situated in:

(a)   Amritsar

(b)   New Delhi

(c)   Lcuknow

(d)   Vadodara

Answer: (c)

33. Fatehabad district derives its name from the eponymous headquarters town founded by ______ in the 14th century, who named it after his son Fateh Khan.

(a)   Firoz Shah Tughlaq

(b)   Ghiyasuddin

(c)   Humayun

(d)   Allauddin Khilji

Answer: (a)

34. What is the yellow dust present in the middle of a flower called?

(a)   Pollen

(b)   Sperm

(c)   Stomata

(d)   Zygote

Answer: (a)

35. What does the writ of ‘Mandamus’ mean?

(a)   We Command

(b)   You May Have the Body

(c)   By What Warrants

(d)   To be Certified

Answer: (a)

36. _________ are called the ‘Powerhouses’ of the cell.

(a)   Mitochondria

(b)   Pituitary glands

(c)   Arteries

(d)   Lungs

Answer: (a)

37. Which of the following Acts was amended to provide a statutory basis for the implementation of the flexible inflation targeting (FIT) framework?

(a)   Banking Regulation Act, 1949

(b)   Reserve Bank of India (RBI) Act, 1934

(c)   Deposit Insurance and Credit Guarantee Corporation Act, 1961

(d)   The Industrial Finance Corporation of India Act, 1948

Answer: (b)

38. Deficiency of which of the following vitamins causes Night Blindness?

(a)   Vitamin B

(b)   Vitamin D

(c)   Vitamin C

(d)   Vitamin A

Answer: (d)

39. Who designed the logo for Lokpal in India?

(a)   Kamareddy

(b)   B.P. Raju

(c)   Prashant

(d)   Deepak Punia

Answer: (c)

40. The wood of which of the following trees is used to make the large plucked string instrument used in Carnatic classical music known as Bobbili Veena/Saraswathi Veena?

(a)   Sandal wood

(b)   Jackfruit wood

(c)   Teak wood

(d)   Walnut wood

Answer: (b)

41. Name the author of the book ‘Ain-i-Akbari’?

(a)   Abdul Rahim Khan-I-Khan

(b)   Dara Shikoh

(c)   Todar Mal

(d)   Abul’Fazl

Answer: (d)

42. Protium, deuterium and tritium are the naturally occurring isotopes of:

(a)   hydrogen

(b)   carbon

(c)   gold

(d)   nitrogen

Answer: (a)

43. Who launched the first phase of the Mumbai Trans Harbour Link (MTHL) on 15th January 2020?

(a)   Narendra Modi

(b)   Ajit Pawar

(c)   Devendra Fadnavis

(d)   Uddhav Thackeray

Answer: (d)

44. In which year was the first Round Table Conference held in London?

(a)   1930

(b)   1923

(c)   1907

(d)   1919

Answer: (a)

45. The Legislative Assembly of which state adopted a new logo on the 9th January 2020?

(a)   Andhra Pradesh

(b)   Tripura

(c)   Arunachal Pradesh

(d)   Assam

Answer: (c)

46. What is the pass at the southern end of the Nilgiri hills called?

(a)   Goran ghat pass

(b)   Malshej ghat pass

(c)   Palakkad gap

(d)   Chorla ghat pass

Answer: (c)

47. On which country has the World Anti-doping Agency recently imposed a 4-year ban?

(a)   China

(b)   Pakistan

(c)   Brazil

(d)   Russia

Answer: (d)

48. Name the first actor to win the Oscar Awards.

(a)   Lionel Barrymore

(b)   Warner Baxter

(c)   Emil Jannings

(d)   Charlie Chaplin

Answer: (c)

49. What does GNP stand for?

(a)   Gramin Nigam Parishad

(b)   Gramin Nisak Praman

(c)   Gross National Product

(d)   Gross National Produce

Answer: (c)

50. In which year did the Portuguese capture Goa?

(a)   1610 (AD)

(b)   1510 (AD)

(c)   1540 (AD)

(d)   1475 (AD)

Answer: (b)

Quantitative Aptitude

51. In the figure, PA is a tangent from an external point P to the circle with centre O. If ∠POB = 110°, then the measure of ∠APO is:

(a)   30°

(b)   25°

(c)   20°

(d)   40°

Answer: (c)

52. If 6 tan θ – 5√3 sec θ + 12 cot θ = 0, 0° < θ < 90°, then the value of (cosec θ + sec θ) is:

(a)     

(b)    

(c)    

(d)    

Answer: (d)

53. X and Y are two stations which are 280 km apart. A train starts at a certain time from X and travels towards Y at 60 km/h. After 2 hours, another train starts from Y and travels towards X at 20 km/h. After how many hours does the train leaving from X meets the train which left from Y?

(a)   3 hours

(b)   6 hours

(c)   2 hours

(d)   4 hours

Answer: (d)

54. The following table represents the number of candidates that appeared and qualified in a competitive examination from different states over five years. Study the table and answer the questions that follow.

What was the total number of candidates appearing from all the states in the  years 1997?

(a)   22,700

(b)   27,200

(c)   27,000

(d)   72,200

Answer: (b)

55. Expand : (4a + 3b + 2c)2

(a)   4a2 + 3b2 + 2c2 + 4ab + 12c + 16 ca

(b)   16a2 – 9b2 + 4c2 – 24ab + 12bc – 16 ca

(c)   16a2 + 9b2 + 4c2 + 24ab + 12bc + 16 ca

(d)   16a2 + 9b2 + 4c2 – 24ab – 12bc – 16ca

Answer: (c)

56. Ravi scores 72% marks in examinations. If these are 360 marks, then the maximum marks are:

(a)   350

(b)   450

(c)   400

(d)   500

Answer: (d)

57. (3a – 4b)3 is equal to:

(a)   27a3 – 64b3

(b)   27a3 – 64b3 – 108a2b + 144ab2

(c)   9a2 – 24ab + 16b2

(d)   9a2 – 16b2

Answer: (b)

58. Triangle PDC is drawn inside the square ABCD of side 24 cm where P lies on AB. What is the area of the triangle?

(a)   280 cm2

(b)   200 cm2

(c)   298 cm2

(d)   288 cm2

Answer: (d)

59. The following table represents the number of candidates that appeared and qualified in a competitive examination from different state over five years. Study the table and answer the questions that follow.

The total number of candidates that qualified from all the states together in 1998 is approximately what percentage of the total number of candidates that qualified from all the states together in 2001? (Correct to two decimal points)

(a)   94.7%

(b)   97.72%

(c)   96.70%

(d)   90.72%

Answer: (b)

60. What is the smallest integer that is a multiple of 5, 8 and 15?

(a)   40

(b)   60

(c)   600

(d)   120

Answer: (d)

61. If A + B = 12 and AB = 17, what is the value of A3 + B3?

(a)   1116

(b)   1106

(c)   1166

(d)   1213

Answer: (a)

62. The following table represents the number of candidates that appeared and qualified in a competitive examination from different states over five years. Study the table and answer the questions that follow.

What is the difference between the number of candidates qualifying in the year 1998, in the states M and P?

(a)   60

(b)   40

(c)   50

(d)   44

Answer: (b)

63. The single discount equivalent to two successive discounts of 20% and 15% is:

(a)   32%

(b)   30%

(c)   28%

(d)   22%

Answer: (a)

64. The length, breadth and height of a cuboidal box are in the ratio 7 : 5 : 3 and its whole surface area is 27832 cm2. Its volume is:

(a)   208120 cm3

(b)   280120 cm3

(c)   288100 cm3

(d)   288120 cm3

Answer: (d)

65. In the given figure ∆ABC, if θ = 80°, the measure of each of the other two angles will be:

(a)   60°

(b)   40°

(c)   80°

(d)   50°

Answer: (d)

66. If 3A = 4B = 5C, then A : B : C is equal to:

(a)   10 : 7 : 6

(b)   10 : 5 : 4

(c)   20 : 15 : 12

(d)   20 : 15 : 16

Answer: (c)

67. The average age of A, B and C is 20 years, and that of B and C is 25 years. What is the age of age of A?

(a)   20  years

(b)   10 years

(c)   15 years

(d)   25 years

Answer: (b)

68. Ram and Shyam can complete a task in  and 15 days, respectively. They work together for 4 days, and then Ram leaves. In how many days after Ram leaves, will Shyam complete the remaining task alone?

(a)     

(b)   2 day

(c)   3 day

(d)   4 day

Answer: (b)

69. Seema flies a kits on a 16 m string at an inclination of 60°. What is the height (h) of the kite above the ground?

(a)   4√3 m

(b)   16√3 m

(c)   6√3 m

(d)   8√3 m

Answer: (d)

70. The simple interest on a sum of money for 3 years at an interest rate of 6% p.a. is Rs 6,750. What will be the compound interest rounded off on the same sum at the same rate for the same period, compounded annually is closest to:

(a)   Rs 7,103

(b)   Rs 7,133

(c)   Rs 7,663

(d)   Rs 7,163

Answer: (d)

71. The following table represents the number of candidates that appeared and qualified in a competitive examination from different states over five years. Study the table and answer the questions that follow.

What is the average number of candidates that appeared from State Q during the given years?

(a)   8880

(b)   9000

(c)   8990

(d)   8980

Answer: (c)

72. The value of 1800 ÷ 20 × {(12 – 6) + (24 – 12)} is:

(a)   2720

(b)   840

(c)   1720

(d)   1620

Answer: (d)

73. What is the area of a sector of a circle of radius 14 cm and central angle 45°? (Take π = 22/7)

(a)   67 cm2

(b)   77 cm2

(c)   70 cm2

(d)   11 cm2

Answer: (b)

74. In the right angle triangle shown in the figure, what is the value of cosec θ?

(a)   13/5

(b)   12/13

(c)   5/13

(d)   5/11

Answer: (a)

75. Ram makes a profit of 30% by selling an article. What would be the profit percent if it were calculated on the selling price instead of the cost price? (Correct to one decimal place)

(a)   20.1%

(b)   24.2%

(c)   23.1%

(d)   22.4%

Answer: (c)

English Comprehension

76. Select the most appropriate option to substitute the underlined segment in the given sentence. If there is no need to substitute it, select no improvement.

Our new office will become operational from a weeks time.

(a)   No  improvement

(b)   for a weeks time

(c)   by the week’s time

(d)   in a week’s time

Answer: (d)

77. Select the most appropriate option to substitute the underlined segment in the given sentence. If there is no need to substitute it, select No improvement.

You will not learn cycling Unless you don’t try.

(a)   until  you are not trying

(b)   until  you don’t try

(c)   unless you try

(d)   unless you didn’t try

Answer: (c)

78. Select the most appropriate synonym of the given word.

Solemn.

(a)   Serious

(b)   Ordinary

(c)   Ignorant

(d)   Furious

Answer: (a)

79. Select the most appropriate meaning of the given idiom.

Kill two birds with one stone

(a)   To achieve two results with a single effort.

(b)   To try something impossible

(c)   To plan two murders simultaneously

(d)   To be an expert shooter

Answer: (a)

80. Select the correct passive form of the given sentence.

Fortunately, the fire did not cause much damage.

(a)   Fortunately, not much damage is being caused by the fire.

(b)   Fortunately not much damage was caused by the fire.

(c)   Fortunately, not much damage was being caused by the fire.

(d)   Fortunately, not much damage is caused by the fire.

Answer: (b)

81. Select the correct indirect form of the given sentence.

Rahul said to his mother, “Can I go out and play now?”

(a)   Rahul asked his mother if he could go out and play then.

(b)   Rahul asked his mother that could he go out and play then.

(c)   Rahul asked his mother if can he go out and play now.

(d)   Rahul asked to his mother could he go out and play now.

Answer: (a)

Comprehension:

In the following passage some words have been deleted. Fill in the blanks with the help of the alternatives given. Select the most appropriate option for each blank.

Tourism can cause social, cultural or environmental disruption. The greatest concern is (1) ________ damage to environment. In order to attract more tourist, (2) ________ resorts are built which take neither the local architecture (3) _______ the ecology into consideration. Natural systems get (4) _______ as a result of indiscriminate construction to provide water (5) ________ waste disposal facilities along with recreational arrangements to tourists.

82. Select the most appropriate option to fill in blank number (1).

(a)   an

(b)   one

(c)   the

(d)   a

Answer: (c)

83. Select the most appropriate option to fill in blank number (2).

(a)   spreading

(b)   stretching

(c)   scheming

(d)   sprawling

Answer: (d)

84. Select the most appropriate option to fill in blank number (3).

(a)   none

(b)   no

(c)   or

(d)   nor

Answer: (d)

85. Select the most appropriate option to fill in blank number (4).

(a)   destroyed

(b)   distracted

(c)   dismayed

(d)   displayed

Answer: (a)

86. Select the most appropriate option to fill in blank number (5).

(a)   and

(b)   also

(c)   but

(d)   as

Answer: (a)

87. Select the most appropriate ANTONYM of the given word.

Persuade.

(a)   Dissuade

(b)   Pervade

(c)   Induce

(d)   Impress

Answer: (a)

88. Given below are four jumbled sentences. Out of the given options pick the one that gives their correct order.

(A) Their prestige and glory spread far and wide.

(B) The first Olympics were held at Olympia in Greece in 776 B.C.

(C) It was because the Christians believed that they encouraged pagan worship in temples.

(D) However, with the advent of Christianity, the games lost their importance.

(a)   BADC

(b)   DCAB

(c)   BCDA

(d)   ACDB

Answer: (a)

89. Select the most appropriate ANTONYM of the given word.

Rear

(a)   Front

(b)   Lower

(c)   Hind

(d)   Back

Answer: (a)

90. Select the most appropriate synonym of the given word.

Eradicate

(a)   Approve

(b)   Magnify

(c)   Condense

(d)   Uproot

Answer: (d)

91. Select the most appropriate meaning of the given idiom.

Make off with

(a)   To discover

(b)   To transfer

(c)   To run away

(d)   To understand

Answer: (c)

92. Fill in the blank with the most appropriate word.

Do you want to _______ to Reader’s Digest magazine?

(a)   Subscribe

(b)   transcribe

(c)   prescribe

(d)   describe

Answer: (a)

93. In the sentence identify the segment which contains the grammatical error.

If a few drops of lemon are put in the milk, it has turned sour.

(a)   drops of lemon

(b)   are put in the milk

(c)   If few

(d)   it has turned sour

Answer: (d)

94. Select the INCORRECTLY spelt word.

(a)   Journy

(b)   Furious

(c)   Failure

(d)   Delicious

Answer: (a)

95. Select the word which means the same as the group of words given.

Person or animal living on another

(a)   Parasite

(b)   Heir

(c)   Successor

(d)   Pupil

Answer: (a)

96. Select the INCORRECTLY spelt word.

(a)   Potraite

(b)   Persevere

(c)   Pageant

(d)   Pamphlet

Answer: (a)

97. Fill in the blank with the most appropriate word.

Your demands are bound to lead to _____ in the family.

(a)   discord

(b)   barrier

(c)   absence

(d)   decrease

Answer: (a)

98. In the sentence identify the segment which contains the grammatical error.

Mohit did not went to school yesterday as he was unwell.

(a)   went to school

(b)   was  unwell

(c)   yesterday as he

(d)   Mohit did not

Answer: (a)

99. Given below are four jumbled sentences. Out of the given options pick the one that gives their correct order.

(A) These baits were temptations that people couldn’t resist.

(B) He evolved a theory that the world was a rat-trap.

(C) Once, a peddler sat looking at his rat-traps.

(D) Here, people lay baits for others.

(a)   CBDA

(b)   BDAC

(c)   DACB

(d)   CADB

Answer: (a)

100. Select the word which means the same as the group of words given.

Central character in a story or play.

(a)   Hedonist

(b)   Cartoonist

(c)   Opportunist

(d)   Protagonist

Answer: (d)

SSC Combined Graduate Level Tier-I Examination Held on 08-08-2017 Question Paper With Answer Key

SSC Combined Graduate Level Tier-I Examination Held on 08-08-2017
SSC Combined Graduate Level Tier-I Examination Held on 08-08-2017 Question Paper With Answer Key

SSC Combined Graduate Level Tier-I Examination Held on 08-08-2017

Part A

Reasoning

Directions – (Q. 1 to 6) Select the related word from the given alternatives.

1. Night : Stars : : Day : ?

(A)  Sun

(B)  Blue

(C)  Work

(D)  Planet

Answer: (A)

2. BEH : KNQ : : FIL : ?

(A)  ONM

(B)  NLJ

(C)  ORU

(D)  OMK

Answer: (C)

3. 6 : −3 : : − 18 : ?

(A)  3

(B)  9

(C)  −9

(D)  −6

Answer: (B)

4. 

(A)  Cyan

(B)  Crimson

(C)  Indigo

(D)  Sky blue

Answer: (B)

5.

(A)  JPV

(B)  UIO

(C)  KQW

(D)  LRX

Answer: (B)

6.

(A)  8

(B)  27

(C)  100

(D)  125

Answer: (C)

Directions – (Q. 7 to 9) A series is given, with one word missing. Choose the correct alternative from the given ones that will complete the series.

7. employ, oyster, errors, ornate, tennis, ?

(A)  neptune

(B)  nature

(C)  terminate

(D)  isomer

Answer: (D)

8. XXXXXO, XXXXOX, XXXOXX, XXOXXX, XOXXXX, ?

(A)  OXXXXX

(B)  OXXXXO

(C)  OXXXOX

(D)  XXXXXX

Answer: (A)

9. −7/4, −1, −25, ? 5/4 2

(A)  0.5

(B)  0.75

(C)  0.25

(D)  1

Answer: (A)

10. Dhruv’s birthday is on Sunday 28th May. On what day of the week will be Sahil’s birthday in the same year if Sahil was born on 19th October?

(A)  Saturday

(B)  Wednesday

(C)  Thursday

(D)  Sunday

Answer: (C)

11. The weights of 4 boxes are 30, 20, 60 and 70 kilograms. Which of the following cannot be the total weight, in kilograms, of any combination of these boxes and in a combination o box can be used only once?

(A)  180

(B)  170

(C)  120

(D)  150

Answer: (B)

Directions – From the given words, select the word which cannot be formed using the letters of the given word.

12. PROXIMAL

(A)  MOLAR

(B)  AXIOM

(C)  REALM

(D)  APRIL

Answer: (C)

13. If PONDERS is coded as ONCDQR, then how will MAT be coded as?

(A)  LZS

(B)  OLJ

(C)  AEG

(D)  LDZ

Answer: (A)

14. In a certain code language, ‘+’, ‘×’ represents ‘÷’ and ‘÷’ represents ‘−’. What is the answer to the following question?

80 × 16 ÷ 4 + 2 – 8 = ?

(A)  11

(B)  64

(C)  30

(D)  5

Answer: (B)

15. If 35% 31 = 12, 92% 30 = 14, then what is the value of 15% 24 = ?

(A)  12

(B)  25

(C)  33

(D)  28

Answer: (A)

16. Select the missing number from the given responses-

(A)  172

(B)  75

(C)  77

(D)  170

Answer: (C)

17. X and Y both start from a same point. X walks 17 m West, then turns to his right and walks 13 m. At the same time, Y walks 9 m North, then turns East and walks 7 m, then turns to his left and walks 4 m. Where is Y now with respect to the position of X?

(A)  24 m West

(B)  10 m East

(C)  14 m West

(D)  24 m East

Answer: (D)

Directions – In the question a statement is given, followed by two arguments, I and II. You have to consider the statement to be true even if it seems to be at variance from commonly known facts. You have to decide which of the given arguments, if any, is a strong argument.

18. Statement : Should songs be eliminated from Indian movies.

Arguments:

(I) Yes, Hollywood movies are hit despite having no songs.

(II) No, songs help increase length of the movie.

(A)  If only argument I is strong

(B)  If only argument II is strong

(C)  If both I and II are strong

(D)  If neither I nor II is strong

Answer: (D)

19. Which of the following cube in the answer figure cannot be made based on the unfolded cube in the question figure?

Answer: (A)

20. In the following figure, square represents Pharmacists, triangle represents Singers, circle represents Surgeons and rectangle represents Mothers. Which set of letters represents surgeons who are either mothers or singers?

(A)  E, D, G

(B)  A, F, C

(C)  A, D, C

(D)  H, B, C

Answer: (B)

21. Which answer figure will complete the pattern in the question figure?

Answer: (B)

22. From the given answer figures, select the one in which the question figure is hidden/embedded.

Answer: (A)

23. A piece of paper is folded and punched as shown below in the question figures. From the given answer figures, indicate how it will appear when opened.

Answer: (D)

24. If a mirror is placed on the line MN, then which of the answer figures is the right image of the given figure?

Answer: (A)

25. A word is represented by only one set of numbers as given in any one of the alternatives. The sets of numbers given in the alternatives are represented by two classes of alphabets as shown in the given two matrices.

The columns and rows and Matrix I are numbered from 0 to 4 and that of Matrix II are numbered from 5 to 9. A letter from these matrices can be represented first by its row and next by its column, for example ‘X’ can be represented by 42, 34 etc. and ‘Z’ can be represented by 76, 88 etc. Similarly, you have to identify the set for the word ‘RIDE’.

(A)  30, 23, 85, 66

(B)  89, 30, 10, 13

(C)  10, 24, 68, 78

(D)  10, 11, 88, 78

Answer: (B)

Part B

General Knowledge

26. A ……… deficit is financed by net capital flows from the rest of the world, thus by a capital account surplus.

(A)  Current Account

(B)  Savings Account

(C)  Capital Account

(D)  Asset Account

Answer: (A)

27. …….. is defined as the output per unit of variable input.

(A)  Marginal product

(B)  Production function

(C)  Total product

(D)  Average product

Answer: (D)

28. “Taxes on lands and buildings” is listed in the ……….. list given in the Seventh Schedule in the Constitution of India.

(A)  Union

(B)  State

(C)  Global

(D)  Concurrent

Answer: (B)

29. There are total ……….. parliamentary seats (Rajya Sabha constituency) in Maharashtra.

(A)  11

(B)  19

(C)  10

(D)  1

Answer: (B)

30. Akbar was ………… years old when he became emperor.

(A)  16

(B)  19

(C)  13

(D)  10

Answer: (C)

31. Sultan Mahmud was a ruler of ……. .

(A)  Persia

(B)  Ghazni

(C)  Lahore

(D)  Arab

Answer: (B)

32. As the river enters the plain it twists and turns forming large bends known as ……….

(A)  crooks

(B)  flections

(C)  rounds

(D)  meanders

Answer: (D)

33. The method of soil conservation in which stones, grass, soil are used to build barriers along contours and trenches are made in front of the barriers to collect water is called?

(A)  Mulching

(B)  Contour barriers

(C)  Rock dam

(D)  Terrace farming

Answer: (B)

34. In male reproductive system the tests are situated outside the abdominal cavity within a pouch called ……. .

(A)  Glands

(B)  Scrotum

(C)  Testicular Lobules

(D)  Seminiferous Tubules

Answer: (B)

35. Which of the following is not among the 3 main classes of Algae?

(A)  Chlorophyceae

(B)  Rhodophyceae

(C)  Phaeophyceae

(D)  Gymnosperms

Answer: (D)

36. Sycon (Scypha), Spongilla (Fresh water sponge) and Euspongia (Bath sponge) are examples of which Phylum?

(A)  Coelenterata

(B)  Platyhelminthes

(C)  Ctenophora

(D)  Porifera

Answer: (D)

37. If the force applied on the project is in the direction opposite to the direction of motion, the speed of the object ……… .

(A)  increases

(B)  stops

(C)  decreases

(D)  no effect

Answer: (C)

38. The SI unit of acceleration is ………. .

(A)  metres per seconds squared

(B)  metres per second

(C)  seconds per metre

(D)  seconds per metre squared

Answer: (A)

39. In Microsoft Excel, the ……….. () function returns the smallest value among the values passed as arguments.

(A)  LEAST

(B)  LESS

(C)  MIN

(D)  LOW

Answer: (C)

40. Fire extinguishers emit which gas?

(A)  Carbon monoxide

(B)  Chlorine

(C)  Carbon dioxide

(D)  Nitrogen

Answer: (C)

41. What is formed when Magnesium is burnt?

(A)  Baking Soda

(B)  Calcium Carbonate

(C)  Ash

(D)  Vinegar

Answer: (C)

42. The salt concentration (measured as salinity in parts per thousand), is less than ………… % in inland waters.

(A)  5

(B)  20

(C)  50

(D)  75

Answer: (A)

43. ………. scheme launched by the Central Government aims to improve rural livelihoods and promote rural development and strengthen the Panchayati Raj across the country.

(A)  Pradhan Mantri Fasal Bima Yojana

(B)  Gram Uday Se Bharat Uday Abhiyan

(C)  Stand up India scheme

(D)  National RU URBAN Mission

Answer: (B)

44. Who discovered Photon?

(A)  George Crum

(B)  Albert Einstein

(C)  Henry Cavendish

(D)  Humphry Davy

Answer: (B)

45. Who is the winner of 2016-17 Premier League (Football)?

(A)  Leicester City

(B)  Manchester City

(C)  Chelsea

(D)  Manchester United

Answer: (C)

46. Gol Gumbaz was designed by?

(A)  Ustad Ahmad of Lahouri

(B)  George Wittet

(C)  Henry Irwin

(D)  Yaqut of Dabul

Answer: (D)

47. Which of the following was the winner of the Grammy Awards 2016 “Country Song of the Year”?

(A)  Chances Are

(B)  Girl Crush

(C)  Hold My Hand

(D)  Traveller

Answer: (B)

48. (1) The author of the novel ‘Bird Box’ is Victor Lavalle.

(2) The author of the novel ‘The Devil in Silver’ is Josh Malerman.

(3) The author of the novel ‘Fellside’ is Victor LaValle.

Which of the statements given above are not correct?

(A)  1 and 2

(B)  2 and 3

(C)  1 and 3

(D)  1, 2 and 3

Answer: (D)

49. In April 2017, to address growing unemployment, which country has abolished the 457 Visa Programme used by thousands of temporary foreign workers, a majority of the Indians?

(A)  USA

(B)  Canada

(C)  Australia

(D)  UK

Answer: (C)

50. SAARC Agriculture Centre (SAC) is based in which city?

(A)  Islamabad

(B)  Dhaka

(C)  Colombo

(D)  Kathmandu

Answer: (B)

Part C

Quantitative Aptitude

51. What leas number must be subtracted from 3401, so that the sum is completely divisible by 11?

(A)  3

(B)  1

(C)  2

(D)  0

Answer: (C)

52. M is thrice as good as workman as N and together they finish a piece of work in 30 days. In how many days will M alone finish the work?

(A)  50

(B)  40

(C)  60

(D)  45

Answer: (B)

53. What is the area (in sq cm) of a regular hexagon of side 14 cm?

(A)  147√3

(B)  441√3

(C)  196√3

(D)  294√3

Answer: (D)

54. If 2 T-shirts are offered free on purchase of 5 T-shirts, what is the effective discount (in %) on each T-shirt?

(A)  40

(B)  20

(C)  30

(D)  50

Answer: (A)

55. The ratio of present ages of R and S is 11 : 17. Before 11 years the ratio of their ages was 11 : 20. What is R’s present age (in years)?

(A)  51

(B)  33

(C)  22

(D)  40

Answer: (B)

56. The average marks of 40 students in an examination was 25. It was later found that the marks of one student had been wrongly entered as 73 instead of 37. What is the value of correct average?

(A)  24.3

(B)  24.1

(C)  24.5

(D)  24.7      

Answer: (B)

57. A wholesaler salles a jacket to a retailer at a profit of 5% and he retailer sells it to a customer at a profit of 10%. If the customer pays Rs 4,158, what had it cost (in Rs) to the wholesaler?

(A)  3,500

(B)  3,400

(C)  3,300

(D)  3,600

Answer: (D)

58. A number is increased by 84, it becomes 107% of itself, What is the number?

(A)  600

(B)  900

(C)  1500

(D)  1200

Answer: (D)

59. If a boat goes a certain distance of 30 km/hr and comes back the same distance at 60 km/hr. What is the average speed (in km/hr) for the total journey?

(A)  45

(B)  50

(C)  40

(D)  35

Answer: (C)

60. An amount fetched a total simple interest of Rs 3,200 at the rate of 6.25%/yr in 4 years. What is the amount (in Rs)?

(A)  13,800

(B)  11,800

(C)  12,800

(D)  14,800

Answer: (C)

61. If  then what is the value of x?

(A)  −10

(B)  9/8

(C)  10

(D)  −9/8

Answer: (C)

62. If a3 + b3 = 152 and a + b = 8, then what is the value of ab?

(A)  2

(B)  11

(C)  −10

(D)  15

Answer: (D)

63. A fraction is greater than its reciprocal by 9/20. What is the fraction?

(A)  5/4

(B)  4/5

(C)  3/4

(D)  4/3

Answer: (A)

64. What is the sum of the first 9 terms of an arithmetic progression if the first term is 7 and last term is 55?

(A)  219

(B)  137

(C)  231

(D)  279

Answer: (D)

65. What is the reflection of the point (5, −2) in the line x = −1 ?

(A)  (−7, −2)

(B)  (5, 0)

(C)  (7, −2)

(D)  (5, 2)

Answer: (A)

66. Point A divides segment BC in the ratio 4 : 1. Co-ordinates of B are (6, 1) and C are (7/2, 6). What are the co-ordinates of point A?

(A)  (4, 3)

(B)  (4, 5)

(C)  (2, 5)

(D)  (3, 5)

Answer: (B)

67. What is the slope of the line parallel to the line passing through the points (5, −1) and (4, −4) ?

(A)  −3

(B)  −1/3

(C)  3

(D)  1/3

Answer: (C)

68. ∆XYZ is similar to ∆ If ratio of perimeter of ∆XYZ and perimeter of ∆PQR is 16 : 9 and PQ = 3.6 cm, then what is the length (in cm) of XY?

(A)  4.8

(B)  3.2

(C)  6.4

(D)  8.6

Answer: (C)

69. What is the value of (1/2) sec 30° + √2 tan 60°?

(A)   

(B)   

(C)   

(D)    

Answer: (A)

70. ∆DEF is right angled at E. If m∠D = 45°, then what is the value of cosec F X cot D?

(A)  1/√2

(B)  2

(C)  1/2

(D)  √2

Answer: (D)

71. If  then what is the value of sin θ?

(A)  24/25

(B)  7/25

(C)  24/7

(D)  25/7

Answer: (B)

Directions – (Q. 72 to 75) Students from different countries (A, B, C, D, E, F) participated in a certain seminar. The pie chart shows how many students come from each of the six participating countries. Study the diagram and answer the following questions.

72. The biggest contingent of students was from which country?

(A)  A

(B)  C

(C)  B

(D)  D

Answer: (C)

73. What is the angular measure (in degrees) of the sector representing country A?

(A)  100

(B)  25

(C)  50

(D)  120

Answer: (A)

74. By what count (in %) students from Country B at the seminar were more than the students from Country E?

(A)  40

(B)  200

(C)  20

(D)  18

Answer: (C)

75. If the cost of total spending on transport for the seminar was Rs 9 lakhs and the cost of hosting the students was Rs 15,000 per student when what was the ratio of hosting cost of all the students to that of spending on transport?

(A)  2 : 5

(B)  3 : 2

(C)  1 : 1

(D)  1 : 2

Answer: (B)

Part D

English Comprehension

Directions – (Q. 76 and 77) In the following question, some part of the sentence may have errors. Find out which part of the sentence has an error and select the appropriate option. If a sentence is free from error, select ‘No error’.

76. He was such a (A) / wonderful person (B) / into so many ways. (C) / No error (D)

Answer: (C)

77. The magician placed the ball (A) / underneath of his hat and (B) / made a mystic sign above it. (C) / No error (D)

Answer: (B)

Directions – (Q. 78 and 79) In the following questions, the sentence given with blank to be filled in with an appropriate word. Select the correct alternative out of the four and indicate it by selecting the appropriate option.

78. Our nation’s ………. into another country’s was could pull us into the crisis.

(A)  intervention

(B)  intention

(C)  perfection

(D)  invention

Answer: (A)

79. As a fitness ………., Deepti is always preaching to her friends about the importance of working out

(A)  enthusiasm

(B)  enthusiast

(C)  enthusiastic

(D)  enthusiastically

Answer: (B)

Directions – (Q. 80 to 83) In the following questions, out of the four alternatives, select the word similar in meaning to the word given-

80. Phonetic

(A)  Mute

(B)  Reticent

(C)  Silent

(D)  Spoken

Answer: (D)

81. To Astound

(A)  To Bewilder

(B)  To Tranquil

(C)  To Placid

(D)  To Serene

Answer: (A)

82. Disdain

(A)  Antipathy

(B)  Admiration

(C)  Derision

(D)  Scorn

Answer: (B)

83. Canonical

(A)  Approved

(B)  Official

(C)  Sanctioned

(D)  Unorhodox

Answer: (D)

Directions – (Q. 84 and 85) In the following questions, out of the four alternatives, select the alternative which best expresses the meaning of the idiom/phrase.

84. It goes without saying-

(A)  To silently bear the injustice

(B)  Something which is implied to be obvious

(C)  There is no point in doing something after you are told to do it

(D)  Break long relationship with someone

Answer: (B)

85. To let someone off-

(A)  To let someone fall

(B)  To leave someone in his present state

(C)  To release someone from blame

(D)  To refuse to answer

Answer: (B)

Directions – (Q. 86 and 87) In the following questions, out of the four alternatives, select the alternative which will improve the bracketed part of the sentence. In case no improvement is needed, select ‘no improvement’.

86. May be they (has had) a fight.

(A)  were had

(B)  was having

(C)  were having

(D)  no improvement

Answer: (C)

87. We wouldn’t want them to think we (doing) anything immoral.

(A)  was doing

(B)  were doing

(C)  done

(D)  no improvement

Answer: (B)

Directions – (Q. 88 and 89) In the following question, out of the four alternatives, select the alternative which is the best substitute of the words/sentence.

88. Excessively lengthy speech-

(A)  Concision

(B)  Verbiage

(C)  Curt

(D)  Succinct

Answer: (B)

89. An ornamented staff carried by rulers on ceremonial occasions as a symbol of sovereignty –

(A)  Spectacle

(B)  Receptacle

(C)  Sceptre

(D)  Zephyr

Answer: (C)

Directions – (Q. 90 and 91) In the following questions, four words are given out of which one word is correctly spelt. Select the correctly spelt word.

90.

(A)  Contusion

(B)  Contution

(C)  Cuntusion

(D)  Cuntution

Answer: (A)

91.

(A)  Scemitars

(B)  Scimitars

(C)  Scimetars

(D)  Scemetars

Answer: (B)

Directions – (Q. 92 and 93) The questions below consist of a set of labelled sentences. Out of the four options given, select the most logical order of the sentences to form a coherent paragraph.

92. Three times this was

X : only be compared to the deepest notes of thunder

Y : Shook with the noise, that can

Z : repeated, and each time the earth

(A)  ZXY

(B)  YZX

(C)  ZYX

(D)  YXZ

Answer: (C)

93. The causes and their

X : in the same series

Y : one order, they stand

Z : effects belong to the

(A)  ZXY

(B)  YZX

(C)  YXZ

(D)  ZYX

Answer: (D)

Directions – (Q. 94 and 95) In the following questions, a sentence has been given in Active/Passive voice. Out of the four alternatives suggested, select the one which best expresses the same sentence in Passive/Active voice.

94. Sunita will bake two dozen cupcakes for the bake sale.

(A)  For the bake sale, two dozen cookies is baked by Sunita.

(B)  For the bake sale, two dozen cookies is baked by Sunita.

(C)  Baking of two dozen cookies by Sunita will be done for the bake sale.

(D)  Baking of two dozen cookies by Sunita is done for the bake sale.

Answer: (A)

95. My mother said to my father, “I am very busy now”.

(A)  My mother told my father that I am very busy now.

(B)  My mother told my father that she is very busy then.

(C)  My mother told my father that she was very busy then.

(D)  My mother told my father that I was very busy now.

Answer: (C)

Directions – (Q. 96 to 100) In the following passage, some of the words have been left out. Read the passage carefully and select the correct answer for the given blank out of the four alternatives.

  This was enough ……….. the book store-owner-turned librarian Husain to asses its valuable contents. “The preface of the book reads ‘painters ………. technical knowledge to paint, but lack of understand nature, ………. they fail to create a masterpiece,” points out Husain, adding, “All I ……….. in my life is about books and from books. The grand library …… the palace is a cache of knowledge.”

96. This was enough ………… the book store-owner-turned librarian.

(A)  for

(B)  of

(C)  to

(D)  from

Answer: (A)

97. Painters ………… technical knowledge to paint.

(A)  has

(B)  has had

(C)  have

(D)  to have

Answer: (C)

98. To understand nature, ……. they fail to create a masterpiece.

(A)  if

(B)  that

(C)  hence

(D)  this

Answer: (C)

99. All I ……….. in my life is about books and from books.

(A)  learning

(B)  learned

(C)  to learn

(D)  learns

Answer: (B)

100. The grand library ……….. the palace is a cache of knowledge.

(A)  from

(B)  at

(C)  to

(D)  off

Answer: (B)

SSC CGL (Tier-2) Main Examination 2017 Held on March 9, 2018 English Language Paper-2 Question Paper With Answer Key

SSC CGL (Tier-2) Main Examination 2017 Held on March 9, 2018
SSC CGL (Tier-2) Main Examination 2017 Held on March 9, 2018 English Language Paper-2 Question Paper With Answer Key

SSC CGL (Tier-2) Main Examination 2017 Held on March 9, 2018

English Language Paper-2

Directions (Q. Nos. 1-3) In the following questions, out of the given four alternatives select the one which best expresses the meaning of the given word.

1. Verbatim

(a)  Exactly

(b)  Virtually

(c)  Carelessly

(d)  Loosely  

Answer: (a)

2. Temerity

(a)  Boldness

(b)  Cowardice

(c)  Distrust

(d)  Uncertainty

Answer: (a)

3. Ruck

(a)  Straighten

(b)  Mass

(c)  Unbend

(d)  Leave

Answer: (b)

Directions (Q. Nos. 4-6) In the following questions, out of the given four alternatives, select the one which is opposite in meaning of the given word.

4. Rueful

(a)  Contrite

(b)  Regretful

(c)  Defiant

(d)  Dolent

Answer: (c)

5. Penury

(a)  Dearth

(b)  Need

(c)  Wealth

(d)  Adversity

Answer: (c)

6. Overweening

(a)  Arrogant

(b)  Haughty

(c)  Brash

(d)  Modest

Answer: (d)

Directions (Q. Nos. 7-16) In the following questions, out of the given four alternatives, select the alternative which best expresses the meaning of the Idiom/phrase.

7. Play one’s ace

(a)  To fight vehemently

(b)  To use one’s best weapon or resource

(c)  To win something

(d)  To depend on chance

Answer: (b)

8. Aid and abet

(a)  To relieve someone of their duties

(b)  To help someone to commit a crime

(c)  To be charitable

(d)  To waste money

Answer: (b)

9. All over hell’s half acre

(a)  Degrading oneself

(b)  Everywhere

(c)  To own a large piece of land

(d)  To be sinful

Answer: (b)

10. An arrow in the quiver

(a)  Strategies that can be followed

(b)  Being vengeful

(c)  Extremely old fashioned

(d)  Types of weapons in one’s possession

Answer: (a)

11. Like a shag on a rock

(a)  Completely alone

(b)  Being sharp

(c)  Highly respected

(d)  To be confused

Answer: (a)

12. Asleep at the wheel

(a)  Feeling sleepy during travelling

(b)  Making money easily

(c)  Not attentive to one’s duties

(d)  To meet with an accident

Answer: (c)

13. The ayes have it

(a)  Everything is in synchronization

(b)  The affirmative votes are in the majority

(c)  The agreement of the superior

(d)  Mob rule

Answer: (b)

14. Dust and ashes

(a)  Something very simple

(b)  Something that is very disappointing

(c)  The death of someone

(d)  Explosion

Answer: (b)

15. Be full of beans

(a)  To be productive

(b)  Full of energy and life

(c)  To have a lot of food prepared

(d)  To work at a farm

Answer: (b)

16. To get more kicks than half pence

(a)  To be physically strong

(b)  Harsh treatment than rewards

(c)  To give more than what one have

(d)  To be happy

Answer: (b)

Directions (Q. Nos. 17-28) In the following questions, out of the given four alternatives, select the alternative which is the best substitute of the given phrase.

17. Fear of riding in a car

(a)  Astrophobia

(b)  Amaxophobia

(c)  Cacophobia

(d)  Genophobia

Answer: (b)

18. Loss of speech due to medical problem

(a)  Anorexia

(b)  Anosmia

(c)  Aphasia

(d)  Hypogeusia

Answer: (c)

19. Place where cows are sheltered

(a)  Pyre

(b)  Dyke

(c)  Byre

(d)  Convey

Answer: (c)

20. Palm reading

(a)  Bibliomancy

(b)  Logomancy

(c)  Chiromancy

(d)  Amniomancy   

Answer: (c)

21. To atone for one’s sins

(a)  Deprecate

(b)  Effeminate

(c)  Expiate

(d)  Expatriate

Answer: (c)

22. A low level tract of uncultivated grassland

(a)  Island

(b)  Peninsula

(c)  Meadow

(d)  Oasis

Answer: (c)

23. Egg-laying mammal

(a)  Corpus

(b)  Platypus

(c)  Rhizopus

(d)  Octopus

Answer: (b)

24. Food for farm animals

(a)  Glutton

(b)  Leer

(c)  Provender

(d)  Parasol

Answer: (c)

25. Existing only in name

(a)  Titular

(b)  Gratis

(c)  Intestate

(d)  Indelible

Answer: (a)

26. A stand having three legs

(a)  Triptan

(b)  Tripod

(c)  Triver

(d)  Tricot

Answer: (b)

27. One extremely fond of one’s wife

(a)  Umbrian

(b)  Uxorious

(c)  Uriel

(d)  Uther

Answer: (b)

28. Evening prayer in a church

(a)  Vespid

(b)  Vespers

(c)  Vesture

(d)  Vestiges

Answer: (b)

Directions (Q. Nos. 29-31) In the following questions, four words have been given out of which one word is incorrectly spelt. Select the incorrectly spelt word.

29. 

(a)  Camarderie

(b)  Camouflage

(c)  Capricious

(d)  Carnivorous

Answer: (a)

30.

(a)  Desicration

(b)  Disparaging

(c)  Dilapidated

(d)  Diminution

Answer: (a)

31.

(a)  Exasperated

(b)  Excruchisating

(c)  Exaggeration

(d)  Exultation

Answer: (b)

Directions (Q. Nos. 32-36) In the following questions, the sentence given with blank to be filled in with an appropriate word. Select the correct alternative out of the given four and indicate it by selecting the appropriate option.

32. We must insist that free oratory is only the beginning of free speech, it is not the end, but a ………… to an end.

(a)  medium

(b)  middle

(c)  trail

(d)  means

Answer: (d)

33. India is rushing headlong towards economic success and modernization, counting on high-tech industries such as information technology and biotechnology to …….. the nation to prosperity.

(a)  protect

(b)  protract

(c)  propel

(d)  prob

Answer: (c)

34. It is true that transgenic plants can offer a range of benefits which are above and beyond those who ………… from more traditional innovations in cultivation.

(a)  enacted

(b)  entwined

(c)  elected

(d)  emerged

Answer: (d)

35. There is absolutely no point in complaining that over the years, there has been ………… for increased productivity and higher earnings for workers in industry.

(a)  urge

(b)  force

(c)  pressure

(d)  limit

Answer: (c)

36. Pidgins are languages that are not acquired as mother tongue and that are used for a ……….. set of communicative functions.

(a)  restricted

(b)  spacious

(c)  still

(d)  many

Answer: (a)

Directions (Q. Nos. 37-56) In the following questions, some part of the sentence may have errors. Find out which part of the sentence has an error and select the appropriate option. If a sentence is free from error, select ‘No Error’.

37. (1) Terminals are locations where passengers

(2) and freight can be transferred

(3) from one vehicle or mode to others

(4) No Error

(a)  1

(b)  2

(c)  3

(d)  4

Answer: (c)

38. (1) Public attention in the developed world has

(2) recently focused on asthma because of its

(3) rapidly increasing prevalence, effecting up to one in four urban children

(4) No Error

(a)  1

(b)  2

(c)  3

(d)  4

Answer: (c)

39. (1) In Spiritualism, it is a part of services

(2) to communicate with life

(3) personalities in the spirit world

(4) No Error

(a)  1

(b)  2

(c)  3

(d)  4

Answer: (b)

40. (1) In Marxists philosophy, the bourgeoisie

(2) are the social class that came to own

(3) the means of production during industrialization

(4) No Error

(a)  1

(b)  2

(c)  3

(d)  4

Answer: (b)

41. (1) An atom is a smallest constituent unit

(2) of ordinary matter that has the

(3) properties of a chemical element

(4) No Error

(a)  1

(b)  2

(c)  3

(d)  4

Answer: (a)

42. (1) Unlike a liquid, a solid object

(2) do not flow to take on

(3) the shape of its container

(4) No Error

(a)  1

(b)  2

(c)  3

(d)  4

Answer: (b)

43. (1) Sulphur, while usually polycrystalline

(2) may also occur of other

(3) allotropic forms with completely different properties.

(4) No Error

(a)  1

(b)  2

(c)  3

(d)  4

Answer: (b)

44. (1) Metals can be hammered or press permanently

(2) out of shape

(3) without breaking or cracking

(4) No Error

(a)  1

(b)  2

(c)  3

(d)  4

Answer: (a)

45. (1) In nineteenth century, ,astronomical research

(2) was focused on the routine work of compute

(3) the motions of astronomical objects

(4) No Error

(a)  1

(b)  2

(c)  3

(d)  4

Answer: (b)

46. (1) Clean air scatters blue light much than red wavelength

(2) and so the midday

(3) sky appears blue

(4) No Error

(a)  1

(b)  2

(c)  3

(d)  4

Answer: (a)

47. (1) Most optical phenomena

(2) can be accounting for using

(3) the classical electromagnetic description of light

(4) No Error

(a)  1

(b)  2

(c)  3

(d)  4

Answer: (b)

48. (1) A telescope’s light gathering power and ability

(2) to resolve small detail

(3) is directly related to the diameter of their objective.

(4) No Error

(a)  1

(b)  2

(c)  3

(d)  4

Answer: (c)

49. (1) Aeronautical charts are for visual meteorological conditions

(2) depicts terrain, geographic features

(3) navigational aids and other aids to navigation

(4) No Error

(a)  1

(b)  2

(c)  3

(d)  4

Answer: (b)

50. (1) The imperial period of Rome

(2) lasted approximately 1500 year compared

(3) to the 500 years of the Republican era

(4) No Error

(a)  1

(b)  2

(c)  3

(d)  4

Answer: (b)

51. (1) Horror movies tends to be a vessel

(2) for showing audiences eras of issues across the globe

(3) visually and in the most effective manner.

(4) No Error

(a)  1

(b)  2

(c)  3

(d)  4

Answer: (a)

52. (1) It was during the reign of Elizabeth

(2) that ‘Britannia’ came to be view

(3) as a personification of Britain

(4) No Error

(a)  1

(b)  2

(c)  3

(d)  4

Answer: (b)

53. (1) Hoplite soldiers utilized the phalanx formation

(2) of order to be effective in war

(3) with fewer soldiers

(4) No Error

(a)  1

(b)  2

(c)  3

(d)  4

Answer: (b)

54. (1) Evidence from 30000 years ago in Europe

(2) reveal starch residue on rocks

(3) used for pounding plants.

(4) No Error

(a)  1

(b)  2

(c)  3

(d)  4

Answer: (b)

55. (1) Gluten gives elasticity to dough,

(2) keeps its shape and often give

(3) the final product a chewy texture

(4) No Error

(a)  1

(b)  2

(c)  3

(d)  4

Answer: (b)

56. (1) Regional anaesthesia blocks transmission of nerve impulses

(2) among a targeted part of the body

(3) and the central nervous system

(4) No Error

(a)  1

(b)  2

(c)  3

(d)  4

Answer: (b)

Directions (Q. Nos. 57-61) In the following passage, some of the words have been left out. Read the passage carefully and select the correct answer for the given blank out of the four alternatives.

The need of the hour is to ……….. value based education. The stress should be on the values such as patience, honesty, tolerance, sympathy and love for fellow brethren and sisters. The students must be indoctrinated to keep values as ………. priority. The youngsters need not only to be taught how to develop their skills, talents and abilities, they must also be taught how to use these skills, talents and abilities for the ………. and betterment of all. Once values become everyone’s priority in life all the negative aspects of life will automatically ……. The world …….. needs people with high values to make it a better place to live in.

57. The need of the hour is to ………. value based education.

(a)  impart

(b)  suspect

(c)  retreat

(d)  reject

Answer: (a)

58. The students must be indoctrinated to keep values as ……. priority.

(a)  mere

(b)  top

(c)  low

(d)  rare

Answer: (b)

59. They must also be taught how to use these skills, talents and abilities for the ………… and betterment of all.

(a)  loss

(b)  misfortune

(c)  welfare

(d)  detriment

Answer: (c)

60. All the negative aspects of life will automatically ……..

(a)  dwindle

(b)  develop

(c)  expand

(d)  enlarge

Answer: (a)

61. The world …… needs people with high values.

(a)  direly

(b)  hardly

(c)  abnormally

(d)  absently

Answer: (a)

Direction (Q. Nos. 62-66) In the following passage, some of the words have been left out. Read the passage carefully and select the correct answer for the given blank out of the four alternatives.

Education is the most important and ………. tool for women empowerment. It makes women aware of their rights and responsibilities. Educational achievement of a woman can have …….. effects for the family and across generations. Most of the girls drop out of schools due to the unavailability of separate toilets for them. The recently launched ‘Swachh Bharat Mission’ focusing on …………. sanitation facilities in schools and every rural household by 2019, can prove to be very ……….. in bring down the ……….. of girls dropping out of school.

62. Education is the most important and ………. tool for

(a)  indispensable

(b)  needless

(c)  redundant

(d)  peripheral

Answer: (a)

63. achievements of a woman can have ………. effects for the

(a)  ripple

(b)  relentless

(c)  harsh

(d)  stringent

Answer: (a)

64. focusing on ………….. sanitation facilities in schools

(a)  depriving

(b)  improving

(c)  residing

(d)  resisting

Answer: (b)

65. can prove to be very ………… in

(a)  significant

(b)  meaningless

(c)  dominant

(d)  buoyant

Answer: (a)

66. bringing down the ………. to girls dropping out of school.

(a)  cost

(b)  rate

(c)  profit

(d)  price

Answer: (b)

Directions (Q. Nos. 67-71) In the following passage, some of the words have been left out. Read the passage carefully and select the correct answer for the given blank out of the four alternatives.

When it comes to thinking about how to be happier, many of us …….. about taking more vacations or finding ways to avoid ……. tasks. We may dream about ……….. housework and instead doing something fun and ………. However, tasks which don’t make us happy can, over the time, add up to a meaningful life. Even routine activities like talking on the phone, cooking, cleaning, housework, meditating, emailing praying, waiting for others and balancing finances appear to bring more ……. to people’s lives. but not happiness in the moment.

67. many of us …….. about taking more vacations

(a)  fantasise

(b)  realise

(c)  downsize

(d)  synthesize

Answer: (a)

68. taking more vacations or finding ways to avoid ……. tasks.

(a)  exciting

(b)  mundane

(c)  fresh

(d)  special

Answer: (b)

69. We may dream about ………. housework

(a)  skipping

(b)  interloping

(c)  clipping

(d)  unwrapping

Answer: (a)

70. housework and instead doing something fun and …….

(a)  pleasurable

(b)  regrettable

(c)  inimitable

(d)  excusable

Answer: (a)

71. balancing finances appear to bring more………. to people’s lives

(a)  meaning

(b)  denotation

(c)  inference

(d)  quotation

Answer: (a)

Directions (Q. Nos. 72-91) The question below consists of a set of labelled sentences. These sentences, when properly sequenced form a coherent paragraph. Select the most logical order of sentences from among the options.

72. P : The triumph came against formidable hurdles and in the absence of any expectations.

Q : Indian women were never the favourites, going into the tournament ranked 12th in the world and fourth in Asia – behind World No. 8 China, Korea and Japan.

R : India may have won the final of the women’s Asia Cup hockey tournament against China in a penalty shootout, but it was a fully deserving victory.

S : Throughout their campaign in Japan, team members played out of their skin to register the win, which has secured them a spot in next year’s World Cup emphatically on merit, not as wild-card entrants.

(a)  RSPQ

(b)  SPQR

(c)  PQRS

(d)  SQPR

Answer: (a)

73. P : Chennai and its sprawl extending to two neighbouring districts should return to the traditional wisdom of creating tanks and lakes for water storage and rejuvenating old silted ones, in order to harvest the floods and replenish depleted groundwater.

Q : The finding from one study in 2013 shows that 27 tanks have totally disappeared and another 400 have lost almost their entire capacity.

R  : This underscores the need to revive such natural sponges.

S : Inviting the community to monitor the health of the tanks and lakes can keep out encroachers, who are often protected by parton-politicians.

(a)  SQRP

(b)  PQRS

(c)  RPQS

(d)  RPSQ

Answer: (b)

74. P : However, Indian girls have not matched this rate of access to education.

Q : Still the Indian girl child who has been deprived of this gift as largely girls are not considered worthy of literacy.

R : The power of education in transforming communities is a gift that has changed India in the last few decades.

S : India has seen a substantial increase in primary school enrolment due to the Right to Education (RTE) Act 2009.

(a)  QPRS

(b)  PRQS

(c)  RQSP

(d)  SRQP

Answer: (c)

75. P : All Indian water bodies within and near population centres are now grossly polluted with organic and hazardous pollutants.

Q : India is now facing a water situation that is significantly worse than any that previous generations had faced.

R : That’s why Interstate disputes over river waters are becoming intense and widespread.

S : Not a single Indian city can provide clean water that can be consumed from the tap on a 24 × 7 basis.

(a)  RPSQ

(b)  RQPS

(c)  RSPQ

(d)  QPSR

Answer: (d)

76. P : There is a glimmer of hope with India registering a slight drop in the number of new tuberculosis cases and TB deaths in 2016 compared with 2015.

Q : Incidence estimates for India were considered interim, pending a national TB prevalence survey scheduled for 2017-18.

R : In terms of mortality, the drop was from 0.51 million in 2015 to 0.43 million in 2016.

S : From an estimated 2.84 million new cases in 2015, the number dropped marginally to 2.79 million in 2016, according to the World Health Organizations Global tuberculosis report, 2017.

(a)  RQPS

(b)  PSQR

(c)  SPQR

(d)  SRQP

Answer: (b)

77. P : People today are so driven towards their personal goals that civic sense as an ethic has become a low priority, almost nuisance.

Q : This attitude cuts across all ranks and sections.

R : We don’t think it deserves much importance.

S : Civic sense or rather the lack of it has been widely discussed and somehow it is an undisputed fact that we Indians don’t seem to care much for it.

(a)  RQPS

(b)  QSRP

(c)  RPQS

(d)  SQRP

Answer: (d)

78. P : First impressions count and the first impression that a potential employer will have of you is going to depend on how you present your resume.

Q : The statistics are that the average employer only spends a few seconds looking at it, so it is imperative that it makes an impression.

R : Nobody can ever underestimate how important it is to have a good resume.

S : This is going to be your one and only chance to capture a potential employer’s attention, or for your resume to be tossed into the file of those they don’t want to pursue.

(a)  RPQS

(b)  QSPR

(c)  SRPQ

(d)  SQRP

Answer: (a)

P : The growing popularity of computer, video games and television makes the children very inactive in their lifestyles.

Q : The time spent in these things may be spent in some physical activities.

R : Parents should be a role model for their children.

S : If the parents are looking very active, the children are more likely to be more active and will stay active for the rest of their life.

(a)  SQRP

(b)  RPQS

(c)  PQRS

(d)  QSRP

Answer: (c)

80. P : However, a persons specific daily calorie intake can vary depending on their age, gender and physical activity level.

Q : The average person needs to eat about 2000 calories everyday to maintain their weight.

R : Your body uses calories from food for walking, thinking, breathing and other important functions.

S : The number of calories in food is a measurement of the amount of energy stored in that food.

(a)  RQPS

(b)  PQRS

(c)  PSRQ

(d)  SRQP

Answer: (d)

81. P : How do you find a mate when you are just 2 cm in size and there aren’t many who math your profile?

Q : Now, scientists have discovered that the loudspeakers they make are almost maximally optimized for the purpose at hand: transform any given leaf into the best ‘amplifier’ it could be.

R : But the tiny tree crickets have taken this a step further: they amplify their mating calls using loudspeakers that they themselves build using leaves.

S : An old trick is to draw attention to oneself by creating a lot of noise.

(a)  PSRQ

(b)  QRSP

(c)  PQRS

(d)  SRQP

Answer: (a)

82. P : The list also includes antibiotics and disinfectants, the use of which can stave off killers like sepsis and meningitis.

Q : The 10 critical products that hospitals must stock to save newborns include a piece of cloth to keep a baby warm and close to the mother to encourage breast-feeding.

R : The biggest cause of death is premature birth, while the second is complications like asphyxia during delivery.

S : But other solutions will need greater investment.

(a)  SPQR

(b)  QPSR

(c)  PSRQ

(d)  SRPQ

Answer: (b)

83. P : First, with the discovery of the planet Kepler 90i, orbiting the star Kepler 90, we now know of another star besides the Sun that has eight planets orbiting it.

Q : But this announcement of the discovery of two new exoplanets, Kepler-90i and Kepler-80g by the National Aeronautics and Space Administration is particularly significant.

R : Second Christopher Shallue, a software engineer at Google, and Andrew Vanderburg, of the University of Texas, Austin, have discovered it using a deep learning neural network-an artificial intelligence tools that mimics the workings of a human brain.

S : Exoplanets, or planets outside our solar system, are routinely being discovered, with the number of those that have already been found now standing at 3567.

(a)  SRQP

(b)  PQRS

(c)  QRSP

(d)  SQPR

Answer: (d)

84. P : The funds are intended to offset the loss of wages due to TB and to help with travel and nutrition.

Q : The Central TB Division has said the government would hand over a sum of Rs 500 a month to each of India’s 35 lakh diagnosed TB patients in order to strengthen the fight against the disease.

R : An early study from a prisoner-of-war camp in Germany in the 1940s showed that Soviet inmates, who didn’t receive extra rations from the Red Cross as their British counterparts did, were around 16 times more likely to develop the disease.

S : Yet, much more needs to be done to protect TB patients from the effects of malnutrition, which has a complicated relationship with TB.

(a)  PQSR

(b)  RSQP

(c)  QPSR

(d)  SPRQ

Answer: (c)

85. P : This time the court will have to do more than pronounce on whether the provision discriminates against men on the basis of gender and gives an unconstitutional exemption to women.

Q : It has further noted that in a case of adultery, one person is liable for the offence, but the other is absolved and that the concept of gender neutrality, on which criminal law

R : By agreeing to have another look at the constitutional validity of Section-497 of the Indian Penal Code, under which men can be prosecuted for adultery, the Supreme Court has re-opened a question that has been decided thrice in the past.

S : While agreeing to issue the notice to the government, the Bench has observed that the provision is archaic.

(a)  PRQS

(b)  RPSQ

(c)  SQRP

(d)  QRSP

Answer: (b)

86. P : One lakh rupees invested in Bitocoin in 2010 would be worth a few hundred crore rupees today.

Q : In fact, all it took for the currency to reach $ 11000 after breaching the $ 10000 ark was a single day.

R : In 2017 alone, Bitocoin price has increased by over 1000%.

S : That is the kind of extraordinary return the digital currency has given to investors as its price has witnessed a meteoric rise, from just a few cents in 2010 to hit a lifetime high of over $ 11000 last week.

(a)  PSRQ

(b)  QRSP

(c)  SPQR

(d)  QPRS

Answer: (a)

87. P : Even the periodic directions of the Supreme Court in a public interest case, Dr. S. Rajasekharan vs. Union of India, have not produced any dramatic change in the official attitude.

Q : But as anyone who uses India’s roads knows only too well, that is not an administrative priority.

R : The most effective measure to keep roads safe is enforcement of rules with zero tolerance to violations.

S : Despite the court setting up the Committee on Road Safety and appointing an amicus curiae to help implement its recommendations, it is mostly business as usual for the police in enforcing road rules, for engineers tasked with forming roads and pavements, and transport officials in charge of licensing.

(a)  RQPS

(b)  PQRS

(c)  PSRQ

(d)  QRSP

Answer: (a)

88. P : But it is the conflict between the massive scope for progress provided by the digital era and the fear of loss of individual autonomy that is foregrounded in any debate about data protection laws.

Q : It also enhanced the perils of unregulated and arbitrary use of personal data.

R :Unauthorized leaks, hacking and other cyber crimes have rendered data bases vulnerable.

S : The dawn of the information age opened up great opportunities for the beneficial use of data.

(a)  QSPR

(b)  PQRS

(c)  SQRP

(d)  QRSP

Answer: (c)

89. P : A few months ago, the court had sought the Centre’s response to a petition filed in public interest by former Union Law Minister Ashwani Kumar, who complained about the delay in India ratifying the UN Convention, which it had signed in 1997.

Q : The Union government has informed the Supreme Court that it is seriously considering the 273rd Report of the Law Commission, which has recommended that India ratify the United Nations Convention against Torture and punish its perpetrators.

R : The court disposed off the matter without any direction after being informed that the matter was under serious consideration.

S; The petition had also favoured a standalone legislation to prohibit torture.

(a)  SRQP

(b)  RQPS

(c)  SPQR

(d)  QPSR

Answer: (d)

90. P: The are moving ahead with specific instruments for loss and damage they suffer due to destructive climate-linked events.

Q : Early studies also suggest that it is on track to achieve the national pledge under the 2015 Paris Agreement for a 33-35% cut in emissions intensity per unit of growth from the same base year by 2030, and thus heed the 2°C warming goal.

R : As the 23rd conference of the UN Framework Convention on Climate Change in Bonn shifts into high gear, developing countries including India are focusing on the imperatives of ensuring adequate financing for mitigation and adaptation.

S : India’s progress in reducing the intensity of its greenhouse gas emissions per unit of GDP by 20-25% from 2005 levels by 2020, based on the commitment made in Copenhagen in 2009 has been positive.

(a)  RPSQ

(b)  PQRS

(c)  SRPQ

(d)  QRSP

Answer: (a)

91. P : Delhi’s air quality deteriorates with unfailing regularity, with large swathes of north India in the grip of suffocating smog, but the State government that can make it easier for millions to breathe do not act with any sense of urgency.

Q : The smog that envelops the region is exacerbated by the burning of biomass in Punjab and Haryana and the winter atmosphere is marked by weak ventilation.

R : It is unconscionable for governments, through indifference and inaction, to subject citizens to such toxic air and cause extreme suffering especially among people with respiratory ailments and impaired lung function.

S : It has turned into a public health emergency in the capital, with the air quality index touching extremely hazardous levels in some parts, necessitating the closure of primary schools, has further lowered its standing.

(a)  RSQP

(b)  PSRQ

(c)  SRQP

(d)  RQPS

Answer: (b)

Directions (Q Nos. 92-113) Improve the bracketed part of the sentence.

92. The research efforts by the Danish Carlsberg scientists (great accelerated) the increase in knowledge about yeast and brewing.

(a)  great acceleration

(b)  greatly accelerate

(c)  greatly accelerated

(d)  No improvement

Answer: (c)

93. The Mesopotamia, the brewer’s craft was (the only) profession which derive social sanction and divine protection from female deities.

(a)  only

(b)  an only

(c)  a only     

(d)  No improvement

Answer: (d)

94. Amylase is present in the saliva of humans and some mammals, where (it began) the chemical process of digestion.

(a)  it begins

(b)  it beginning

(c)  it begin

(d)  No improvement

Answer: (a)

95. The phobia typically results in a rapid onset of fear and is present (for much than) six months.

(a)  for much then

(b)  from more than

(c)  for more than

(d)  No improvement

Answer: (c)

96. Systematic desensitization is a process in which the patients seeking help slowly (became accustomed) to their phobia.

(a)  becomes accustomed

(b)  becomes accustom

(c)  become accustomed

(d)  No improvement

Answer: (c)

97. Organic farming in general features practices that (strive in) cycle resources, promote ecological balance and conserve biodiversity.

(a)  strive of

(b)  strive off

(c)  strive to

(d)  No improvement

Answer: (c)

98. Contemporary and historic view regarding diagnosis, prognosis and treatment of medical conditions (have been document) for thousands of years.

(a)  have been documented

(b)  have document

(c)  have being documented

(d)  No improvement

Answer: (a)

99. In open access publishing, a journal article (were made) available free for all on the web by the publisher.

(a)  is make

(b)  making

(c)  is made

(d)  No improvement

Answer: (c)

100. Some forms of feminism have been criticised (for take into) account only white, middle class and college-educated perspectives.

(a)  to take in

(b)  for taking into

(c)  to take into

(d)  No improvement

Answer: (b)

101. Full cultural assimilation occurs when new members of a society become indistinguishable (off) members of the other group.

(a)  of

(b)  from

(c)  for more than

(d)  No improvement

Answer: (b)

102. Human activity since the Industrial Revolution (have increased) the amount of greenhouse gases in the atmosphere.

(a)  has increased

(b)  have increasing

(c)  have increase

(d)  No improvement

Answer: (a)

103. Nitrous oxide (has significant) medical uses, especially in surgery and dentistry, for its anaesthetic and pain reducing effects.

(a)  have significant

(b)  significance

(c)  has significance

(d)  No improvement

Answer: (d)

104. Unlike a merchant ship, which carries cargo, a warship typically (only carry) weapons, ammunition and supplies for its crew.

(a)  only carrying

(b)  carrying only

(c)  carries only

(d)  No improvement

Answer: (c)

105. The modern torpedo is a self-propelled weapon with an explosive warhead, (launched) above or below the water surface.

(a)  were launch

(b)  launching

(c)  are launching

(d)  No improvement

Answer: (d)

106. Oxygen is an oxidizing agent that readily forms oxides (with more) elements as well as with other compounds.

(a)  with most

(b)  for more

(c)  of more

(d)  No improvement

Answer: (a)

107. Reports on Indian rhinoplasty performed by a Kumhar vaidya(were published) in the Gentleman’s Magazine in 1794.

(a)  publishing

(b)  was published

(c)  is been published

(d)  No improvement

Answer: (d)

108. Eastern North American tribes historically (smoked) tobacco both socially and ceremonially, to seal a peace treaty or trade agreement.

(a)  smokes

(b)  has smoked

(c)  was smoking

(d)  No improvement

Answer: (d)

109. Much of the progress in India, especially in higher education and scientific research, (have been credited) to various public institutions.

(a)  have credited

(b)  has credited

(c)  has been credited

(d)  No improvement

Answer: (c)

110. In physiology, dehydration is a deficit of total body water (with a accompanying) disruption of metabolic processes.

(a)  with an accompanying

(b)  to a accompanying

(c)  with accompany

(d)  No improvement

Answer: (a)

111. The greatest legacy one can pass on to one’s children and grand children is not money or other material things (accumulate in) one’s life, but rather a legacy of character and faith.

(a)  accumulate of

(b)  accumulating

(c)  accumulated in

(d)  No improvement

Answer: (c)

112. When a person is angry and the (instant become) silent, the mind than takes a new pace.

(a)  instantly becomes

(b)  instant becoming

(c)  instantly become

(d)  No improvement

Answer: (a)

113. Sri Ramakrishna always (stress on) realization or spiritual awakening where every dispute on God and religion gets entirely silenced.

(a)  stressed on

(b)  stressed

(c)  stressing on

(d)  No improvement

Answer: (a)

Directions (Q. Nos. 114-118) A passage is given with five questions following it. Read the passage carefully and select the best answer to each question out of the given four alternatives.

In 1979, there was no liberalization; there were few multinationals and not many billionaires. ‘Social work’ was done by activists in the field and affluent men and women in cities ‘gave to charity’. The norm in the social sector was to look at everything from the lens of ‘but India is a poor country’. Child labour, e.g., was accepted as necessary evil that helped poor families survive. Concepts like social entrepreneurship, philanthropy and impact investing were unknown. Into that world came Rippan-no connections, no wealth, not even a freedom struggle background, a regular middle-class guy, with a regular middle-class job but with an unshakeable conviction that Indian were India’s responsibility. He was about to set-up an organization that would seek to engage every citizen in the struggle for justice for children. Here was a group of people who talked of large-scale impact, of funding individuals and groups that would work all over India.

CRY not only survived Rippan’s premature death, it grew and changed. In 1989, CRY’s mission and values also struck a chord with top-notch professionals, who gave up jobs to take home salaries of Rs 2000 to Rs 3500 a month.

Today, the development sector in India and abroad is full of people who grew up in what I like to call the CRY School of Social Work and Management. People who discovered their calling in CRY and have gone on to do seminal work across the globe with their own initiatives for change.

Shantha Sinha, who pioneered work in eradicating child labour in Andhra Pradesh, came to Rippan after every funding door had closed. She got a ‘yes’ and an immediate cheque for the funds she needed to start her work which , today, has been replicated globally. Andal Damodaran of the Indian Council for Child Welfare in Chennai, Gloria D’Souza, who founded Parisar Asha in Mumbai, Zakiya and John Kurrien of the Centre for Learning Resources in Pune, innovators in transforming education for children from adverse backgrounds-all of them were similarly supported.

Rippan’s approach was simple: “What I can do, I must do.” His allegiance clear: “CRY is my home, family and life.” His heart belonged to children. Just an hour or so before he died, when one of his oldest and dearest colleagues asked him how he was feeling, he said, “I can see the faces of smiling children outside my window.” I cannot think of a better way to leave the planet.

114. Which world is being talked about in the first sentence of the second para of the passage?

(a)  World of rich people

(b)  World of religious people

(c)  World with no liberalization, less millionaires and lesser ways to help others

(d)  World where people used to help needy ones enthusiastically and with much reverence

Answer: (c)

115. CRY was formed to help

(a)  children

(b)  old age people

(c)  acid attack survivors

(d)  widows

Answer: (a)

116. What is the better way for the author to leave this planet?

(a)  To be able to see the people smiling because of you before you are dying.

(b)  To enjoy all luxuries of life before dying.

(c)  To be able to get recognition of successful entrepreneur before dying.

(d)  To die without any expectations.

Answer: (a)

117. What Rippan’s conviction intended to?

(a)  Help children reach school

(b)  Form organization for the justice for children

(c)  Protest against the government

(d)  Get better care for children

Answer: (b)

118. How Rippan felt about his life at his death?

(a)  Disappointing life full of regrets

(b)  Anger against the government

(c)  Yearning for more

(d)  Satisfied with the happiness of children

Answer: (d)

Directions (Q. Nos. 119-123) A passage is given with five questions following it. Read the passage carefully and select the best answer to each question out of the given four alternatives.

There is exciting news for butterfly enthusiasts. By studying 207 species of butterflies, scientists have created an evolutionary tree painting the detailed picture of butterfly relationships and evolution over time.

An international team of lepidopterists carried out DNA studies and carbon dating analysis to understand the age and characteristics of butterflies. By comparing and merging previous studies on butterflies, the researchers were able to create the new bigger and better evolutionary tree.

“We still have a long way to go, but his is the first comprehensive map of butterfly evolution”, said Akito Y. Kawahara, associated professor at the Florida Museum of Natural History in a release. “Lots of previous studies cover butterfly evolution on smaller scales – by locality or taxon – but surprisingly few have reached across the breadth of butterfly diversity”. Dr. Kawahara is the corresponding author of the paper published in Current Biology.

The report also supported previous studies that butterflies originated around 119 million years ago in the late Cretaceous period. After the mass extinction (65 million  years ago), most of the butterflies diverged into many different groups. The butterfly species were placed in seven groups – Papilionidae, Hedyliadae, Hesperiidae, Pieridae, Riodinidae, Lycaenidae and Nymphalidae.

“Our analyses support swallowtails (Papilionidae) as sister to all other butterflies, followed by skippers (Hesperiidae) and the nocturnal butterflies (Hedylidae) as sister to the remainder”, says the report.

The whites (Pieridae) were supported as sister to brush-footed butterflies (Nymphalidae), blues and metalmarks (Lycaenidae and Riodinidae). Previously, swallowtails and birdwings were believed to have a common ancestor but the new study showed that they feed on different plants. “That tells us that butterflies and plants may have evolved together”, Dr. Kawahara added. They also studied the association of butterflies with ants. Some butterfly larvae secrete sugars that serve as a meal for ants and the ant in return protects the larva from other predators.

This is a well-studied symbiotic relationship. The scientists report that most of the blue butterflies and hairstreaks and some of the metalmark butterflies exhibit this beahviour. “We [India] have about one fifth of the known moths and butterflies represented in Indian collections (3800 out of an estimated 20000 species). The only way they could undertake the study was the fact that they have access to a good collection. We lack this in India,” explained Smetacek.

119. Who are referred to as Lepidopterists?

(a)  Those who study or collect butterflies and moths.

(b)  Those who do study on ants.

(c)  Those who do study on birds.

(d)  Those who do study on plant eaters.

Answer: (a)

120. When did the mass extinction of butterflies happen?

(a)  65 million years ago

(b)  207 million years ago

(c)  119 million years ago

(d)  380 million years ago

Answer: (a)

121. Which group of butterflies is known as nocturnal butterflies?

(a)  Hesperiidae

(b)  Nymphalidae

(c)  Hedylidae

(d)  Papilionoidae

Answer: (c)

122. Which group of butterflies along with bridwings feed on different plants which were believed to have common ancestor in past?

(a)  Riodinidae

(b)  Lycaenidae

(c)  Hesperiidae

(d)  Papilionidae

Answer: (d)

123. Symbiotic relationship is shown most by

(a)  Lycaenidae and Ridnidae

(b)  Ridnidae and Hairstreaks

(c)  Lycaenidae and Hairstreaks

(d)  Hesperiidae and Hedylidae

Answer: (c)

Directions (Q. Nos. 124-128) A passage is given with five questions following it. Read the passage carefully and select the best answer to each question out of the given four alternatives.

A plant species that has outlived the dinosaurs, the Cycas circinalis, a ‘living fossil’ found only in the Western Ghats, faces the threat of extinction due to the over-harvesting of its leaves for commercial purposes.

While there are nine species of Cycas found across the country, almost all of which are classified as being endangered, the Cycas circnalis, found at altitudes between 1500 and 1800 m, has traditionally been  used by tribal communities for a variety of purposes. In the Nilgiris and other p arts of the Western Ghats, where the plant is found, the seed is sometimes soaked in water (to remove a strong poison contained within) and ground into flour, after which it is eaten, while the seed is also said to be used in traditional, indigenous medicine.

However, over the last few decades, the leaves of the circinalis, which are used for ornmanetal purposes like for setting up ‘pandals’ have been harvested in bulk from the remaining plants, often leading to them becoming stunted and unable to reproduce and increase in number. With a single leaf costing as much as Rs 25, the economic incentive to completely strip the plant of all its leaves in one harvest remains high for most people.

Shiny Miriam Rehel, Programme Co-ordinator, for the Biodiversity Restoration Programme at Keystone Foundation speaking to The Hindu, said that circinalis plants do not grow in large numbers in the wild and the over exploitation of the leaves from the plant has caused a further decline over the last few decades.

The Keystone Foundation has been trying to promote the conservation of the species by encouraging forest-dependent communities to sustainably harvest the leaves from the circinalis. “We collect the seeds from the wild and raise it at our nursery on the Keystone campus. We then encourage farmers and tribal communities to grow the pants near to their dwellings or land h oldings,” said Ms. Rehel, who added that the conservation of the species needs to be assured through working with local communities, creating awareness of sustainably harvesting the leaves and ensuring that the plant itself remain healthy. Speaking of their importance, Ms. Rehel said not only the conservation of the endangered circinalis important to protect the livelihood of tribal communities, but also for biodiversity.

She said that the circinalis is an ideal host plant for ants, cupid butterflies and bats, and added that their protection will be a vital aspect of protecting the ecology of the Western Ghats in the long run.

124. Why Cycas circinalis faces the threat of extinction?

(a)  Because of its low production

(b)  Because of excessive cattle grazing

(c)  Because of over-harvesting of its leaves

(d)  Because of pests

Answer: (c)

125. Which word in the passage means ‘native’?

(a)  Indigenous

(b)  Tribal

(c)  Incentive

(d)  Dwelling

Answer: (a)

126. How many uses of Cycas circinalis for humans are mentioned in the passage?

(a)  One

(b)  Four

(c)  Two

(d)  Three

Answer: (d)

127. What else other than over-harvesting has contributed in decline of Cycas circinalis?

(a)  Consumption by wildlife

(b)  The small scale growth of the plant

(c)  The high cost of the plant

(d)  The pollution of air and water

Answer: (b)

128. Why the conservation of Cycas circiinalis important?

(a)  To ensure the livelihood of tribal communities and to protect biodiversity.

(b)  To establish a business of selling it

(c)  To prevent its population from increasing

(d)  To prevent its stunted growth

Answer: (a)

Directions (Q. Nos. 129-133) A passage is given with five questions following it. Read the passage carefully and select the best answer to each question out of the given four alternatives.

It is interesting to note contemporary storytellers such as Vikram Sridhar have stepped out of the traditional boundaries of storytelling to embrace drama. Whiles storytelling might bring to mind grandma’s tales complete with myths and morals, Sridhar believes these stores can be tweaked for a contemporary  retelling. Sridhar, who has been a theatre practitioner for 15 years felt stories could be used to draw attention to social issues.

“I had to think how I could be different from informal storytellers and mould my stories to interest adults and children. I had a strong theatre background, had worked in a corporate set-up and was deeply interested in heritage and conservation. I decided to bring my interests and strengths together into performance storytelling,” says Sridhar. Based in Bengaluru, Vikram followed his passion for storytelling after doing his engineering and MBA.

“I had to travel a lot. In many places in Europe and Singapore, I witnessed ‘Open Night’ where a story, poetry or music was performed at public spaces or coffee shops in 10-minute slots. This made me think that we need larger versions of open night and stores are a strong medium to get a message across.” Sridhar is having a story session at Tipu’s armoury thanks to Indian National Trust for Arts and Cultural Heritage (INTACH), who had done restoration work at the armoury more than 15 years ago. “The armoury is totally in neglect, says Meera lyer, convenor, INTACH-Bangalore Chapter. “Storytelling is also a way to bring attention to the monument and how it can be given a focus and use. A building that is used often, automatically has the greater chance of preservation.”

“I will not be using mike or gadgets,” Sridhar says. “The audience will gets stories of the armoury and more. The history of Karnataka’s Mysore pak will be traced as well as the tale of the iron lady, Onake Obavva who fought the forces of Hyder Ali in the 8th century single-handedly with a pestle (Onake) in the kingdom of Chitradurga”.

129. The transition from stories to drama was due to?

(a)  Sridhar’s conviction that stores can be used to concentrate on social issues.

(b)  His background in theatre.

(c)  His interest in drama.

(d)  His need to step out of traditional boundaries

Answer: (a)

130. How story telling is helping INTACH?

(a)  It will help in generating money

(b)  It will make the monument more popular

(c)  It will help in preservation of the monument.

(d)  It will help to get the attention of the government

Answer: (c)

131. What stories are mentioned in the passage?

(a)  Travel stories of Sridhar

(b)  The process of transition from storytelling to drama

(c)  Story of Tipu’s Armoury, Karnatka’s Mysore Pak, Tale of Iron lady and fight of ONake Obavva with Hyder Ali

(d)  No option is correct

Answer: (c)

132. What is the meaning of the word ‘Armoury’?

(a)  Place where weapons are kept

(b)  Place where clothes are kept

(c)  Place where soldiers sleep

(d)  Place where prisoners are kept

Answer: (a)

133. What was Sridhar interested in?

(a)  Performance story telling

(b)  Heritage and conservation

(c)  Travelling

(d)  Armoury

Answer: (b)

Directions (Q. Nos. 134-160) In the following questions, a sentence has been given Direct/Indirect speech. Out of the four alternatives suggested, select the one which best expresses the same sentence in Indirect/Direct speech.

134. The host said to his guests, “Please consider this place as your home.”

(a)  The host requested his guests to have considered that place as their home.

(b)  The host requests his guests for considering that place as their home.

(c)  The host requested his guests to consider that place as your home.

(d)  The host requested his guests to consider that place as their home.

Answer: (d)

135. The vendor said to us, “These mangoes will ripen in three days.”

(a)  The vendor says that those mangoes would ripen in three days.

(b)  The vendor informed us that those mangoes will ripen in three days.

(c)  The vendor says that those mangoes will be ripe in three days.

(d)  The vendor informed us that those mangoes would ripen in three days.

Answer: (d)

136. He said to the physician, “I need to get well soon.”

(a)  He told the physician that he needed to get well soon.

(b)  He told he physician that I needed to get well soon.

(c)  He says to the physician that he needed to get well soon.

(d)  He told the physician that he need to get well soon.

Answer: (a)

137. My father said to me, “We will be going to Jaipur this summer.”

(a)  My father says that we would be going to Jaipur that summer.

(b)  My father told me that we will be going to Jaipur that summer.

(c)  My father says that we will be going to Jaipur this summer.

(d)  My father told me that we would be going to Jaipur that summer.

Answer: (d)

138. Grandfather says, “Health is wealth.”

(a)  Grandfather said that health was wealth.

(b)  Grandfather says that health was wealth.

(c)  Grandfather says that health is wealth.

(d)  Grandfather said that being healthy was being wealthy.

Answer: (c)

139. The candidate said to the panel, “I promise to live up to your expectations.”

(a)  The candidate told the panel that he promises to live up to their expectations.

(b)  The candidate told the panel that he is promising to live up to their expectations.

(c)  The candidate told the panel that he was promising to live up to their expectations.

(d)  The candidate told the panel that he promised to live up to their expectations.

Answer: (d)

140. He said to his partner. “You have to share the responsibilities if you want to share the profits.”

(a)  He told his partner that he had to share the responsibilities if he wanted to share the profits.

(b)  He told his partner that he has to share the responsibilities if he wanted to share the profits.

(c)  He tells his partner that he had to share the responsibilities if he wanted to share the profits.

(d)  He told his partner that he has to share the responsibilities if he wants to share the profits.

Answer: (a)

141. I said to my father, “If you won’t get angry I would like to confess something.”

(a)  I told my father that if he wouldn’t get angry I would like to confess something.

(b)  I told my father that he wouldn’t get angry I will like to confess something.

(c)  I told my father that he wouldn’t get angry when I confess something.

(d)  I told my father that if he wouldn’t get angry I would have liked to confess something.

Answer: (a)

142. The manager said, “We have to have the plan ready by tomorrow.”

(a)  The manager said that they have to have the plan ready by the following day.

(b)  The manager said that they had to have the plan ready by the following day.

(c)  The manager says that they had to have the plan ready by tomorrow.

(d)  The manager told that they have to have the plan ready by the following day.

Answer: (b)

143. Aunt said to mother, “We need to find some more information about the boy’s family.”

(a)  Aunt told mother that they needed to find some more information about the boy’s family.

(b)  Aunt told mother that we need to have found some more information about the boys’s family.

(c)  Aunt told mother that they are needed to find some more information about the boy’s family.

(d)  Aunt told mother that we are in need to find some more information about the boy’s family.

Answer: (a)

144. My brother said, “I can dance better than my friends.”

(a)  My brother said that he can dance better than his friends.

(b)  My brother said that he could dance better than his friends.

(c)  My brother said that he could be dancing better than hits friends.

(d)  My brother said that his dances were better than his friends.

Answer: (b)

145. My son said to me, “I will do my best.”

(a)  My son told me that he would do his best.

(b)  My son told me that he will do his best.

(c)  My son tells me that he would do h is best.

(d)  My son told me this that he will do his best.

Answer: (a)

146. Vaibhav said, “I am reading ‘Great Expectations’ by Charles Dickens.”

(a)  Vaibhav said that he is reading ‘Great Expectations’ by Charles Dickens.

(b)  Vaibhav says that he was reading ‘Great Expectations’ by Charles Dickens.

(c)  Vaibhav said that he was reading ‘Great Expectations’ by Charles Dickens.

(d)  Vaibhav said that he has been reading ‘Great Expectations’ by Charles Dickens.

Answer: (c)

147. “I think I have been too lenient with you,” father said to me.

(a)  My father told me that he thinks that he had been too lenient with me.

(b)  My father told me that he thought that he has been too lenient with me.

(c)  My father told me that he thought that he has been too lenient with me.

(d)  My father tells me that he thinks that he has been too lenient with me.

Answer: (b)

148. “The delivery boy has arrived,” said the watchman.

(a)  The watchman informed that the delivery boy has arrived.

(b)  The watchman informed that the delivery boy had arrived.

(c)  The watchman informs that the delivery boy has arrived.

(d)  The watchman informed that the delivery boy has had arrived.

Answer: (b)

149. The student said, “I am finding it difficult to memorise”

(a)  The student said that he is finding it difficult to memorise.

(b)  The student said that he was finding it difficult to memorise.

(c)  The student says that he was finding it difficult to memorise.

(d)  The student said that he has found it difficult to memorise.

Answer: (b)

150. She said to her friend, “We should go for a picnic.”

(a)  She told her friend that they shall go for a picnic.

(b)  She told her friend that they should go for a picnic.

(c)  She tells her friend that they should go for a picnic.

(d)  She told her friend that was should go for a picnic.

Answer: (b)

151. My friend said to me, “I am angry with you.”

(a)  My friend told me that he is angry with me.

(b)  My friend told me that he was angry with me.

(c)  My friend tells me that he was angry with me.

(d)  My friend told me that he is being angry with me.

Answer: (b)

152. My friend said to me, “I think cooking is more of an art than science.”

(a)  My friend told me that he thinks that cooking was more of an art than science.

(b)  My friend tells me that the thought that cooking is more of an art than science.

(c)  My friend told me that he thought that cooking was more of an art than science.

(d)  My friend told me this that he thinks that cooking was more of an art than science.

Answer: (c)

153. I said to the taxi driver, “Do you known this address?”

(a)  I asked the taxi driver if you knew that address.

(b)  I asked the taxi driver if he knows that address.

(c)  I inquired with the taxi driver if you knew that address.

(d)  I asked the taxi driver if he knew that address.

Answer: (d)

154. The teacher said to me, “Draw the chart on the board.”

(a)  The teacher says me to draw the chart on the board.

(b)  The teacher is asking me to draw the chart on the board.

(c)  The teacher says me to be drawing the chart on the board.

(d)  The teacher told me to draw the chart on the board.

Answer: (d)

155. He said to her, “I have an interview scheduled for tomorrow.”

(a)  He informed her that he has an interview scheduled for tomorrow.

(b)  He informs her that he had an interview scheduled for the next day.

(c)  He informed her that he had an interview scheduled for the next day.

(d)  He informed her that he has had an interview scheduled for tomorrow.

Answer: (c)

156. The receptionist said, “How can I help?”

(a)  The receptionist inquired how she can help.

(b)  The receptionist inquired how she could help.

(c)  The receptionist is asking how she could help.

(d)  The receptionist inquired how she can helping.

Answer: (b)

157. Manjeet said to his friend, “It is easy to find fault with others.”

(a)  Manjeet told his friend that it was easy to find fault with others.

(b)  Manjeet told his friend that it is easy to find fault with others.

(c)  Manjeet has told his friend that it easy to have found fault with others.

(d)  Manjeet told his friend that it was easy to find fault with others.

Answer: (a)

158. “I will be taking an extra class tomorrow.” the teacher said to us.

(a)  The teacher informed us that he will be taking an extra class the next day.

(b)  The teacher informed us that he shall be taking an extra class the next day.

(c)  The teacher informed us that he will be taking an extra class tomorrow.

(d)  The teacher informed us that he would be taking an extra class the next day.

Answer: (d)

159. “My new car will be delivered today,’ he said,.

(a)  He said that this new car would be delivered that day.

(b)  He said that his new car will be delivered that day.

(c)  He says that his new car would be delivered today.

(d)  He said that his new car will be delivered today.

Answer: (a)

160. I said to my wife, “We have brace for the coming storm.”

(a)  I told my wife that we had to brace for the coming storm.

(b)  I told my wife that we have had to brace for the coming storm.

(c)  I tell my wife that we had to brace for the coming storm.

(d)  I told my wife that we had to be bracing for the coming storm.

Answer: (a)

Directions (Q. Nos. 161-180) In the following questions a sentences has been given in Active/Passive voice. Out of the four alternatives suggested select the one which best expresses the same sentence in Passive/Active voice.

161. You have to submit the assignment before the deadline.

(a)  The assignment has to be submitted by you before the deadline.

(b)  Submission of the assignment has to be done by you before the deadline.

(c)  Before the deadline you have to be done with the assignment submission.

(d)  The assignment is to be submitted by yourself before the deadline.

Answer: (a)

162. The mechanic will repair the scooter by noon.

(a)  Repairing of the scooter by the mechanic will be done by noon.

(b)  The scooter by noon by the mechanic would have been repaired.

(c)  By noon the mechanic will have repaired the scooter.

(d)  By scooter will be repaired by the mechanic by noon.

Answer: (d)

163. Her performance did not impress the judges.

(a)  Impressing the judges by her performance was not done by her.

(b)  Her performance was not impressive for the judges.

(c)  She failed to impress the judges by her performance.

(d)  The judges were not impressed by her performance.

Answer: (d)

164. The agent showed us many residential properties.

(a)  Showing of many residential properties to us has been done by the agent.

(b)  We were shown many residential properties by the agent.

(c)  Many residential properties the agents had shown us.

(d)  We ourselves were shown by the agent himself many of the residential properties.

Answer: (b)

165. I have been asking you since days to replace the leaking tap.

(a)  You have been asked by me since days to replace the leaking tap.

(b)  Asking you to replace the leaking tap has been done by me since days.

(c)  Replacement of the leaking tap is being asked by me to you since days.

(d)  Since days I have been asking you for replacement of the tap that is leaking.

Answer: (a)

166. This medicine gives me a heartburn.

(a)  Giving of heartburn is done by this medicine.

(b)  I have got heartburn by that medicine.

(c)  A heartburn is given to me by this medicine.

(d)  Heartburning has been caused by this medicine.

Answer: (c)

167. You need to bake it in the over for 15 minutes.

(a)  It needs to be baked in the oven for 15 minutes by you.

(b)  Baking it in the oven for 15 minutes is needed be done by you.

(c)  For 15 minutes in the oven it needs to be baked by yourself.

(d)  It is in need of being baked in the oven for 15 minutes by you.

Answer: (a)

168. The driverless car turned the corner just the way it should.

(a)  Turning of the corner just the way it should has been done by the driverless car.

(b)  The car which is driverless has turned the corner just the way it should.

(c)  The corner was turned by the driverless car just the way it should.

(d)  Just the way it should be doing it the driverless car has been turning the corner.

Answer: (c)

169. I will bake for you your birthday cake.

(a)  Baking of your birthday cake will be done by me.

(b)  You birthday cake will be baked by me.

(c)  For you the cake of your birthday I would be baking.

(d)  It will be me who will bake your birthday cake.

Answer: (b)

170. I adore movie stars.

(a)  Adoring movie stars is being done by me.

(b)  Movie stars are adored by me.

(c)  I am in adoration of movie stars.

(d)  Movie stars are what that is adored by me.

Answer: (b)

171. The management appreciates your efforts.

(a)  Appreciating of your efforts is being done by the management.

(b)  Efforts which are made by you will be appreciated by the management.

(c)  Appreciation of your efforts is done by the management.

(d)  Your efforts are appreciated by the management.

Answer: (d)

172. She eats too much fast food.

(a)  Eating of too much fast food is being done by her.

(b)  Too much fast food is eaten by her.

(c)  Too much of eating fast food she has done.

(d)  Fast food has been eaten by her in too much.

Answer: (b)

173. She will never admit her mistakes.

(a)  Admission of her mistakes will never be done by her.

(b)  Her mistakes will never be admitted by her.

(c)  Her mistakes is what she will never be admitting.

(d)  Admitting her mistakes has never been done by her.

Answer: (b)

174. You need to brush your teeth twice daily.

(a)  Your teeth are needed to be brushed twice by you daily.

(b)  Brushing twice of your teeth daily is needed by you.

(c)  It is twice that you should be brushing your teeth daily.

(d)  Daily you are needed to brush your teeth twice.

Answer: (a)

175. You are making a bad situation worse.

(a)  Making a bad situation worse is being done by you.

(b)  A bad situation is being made worse by you.

(c)  A situation which is bad you are making is worse.

(d)  Worsening a bad situation is being done by you.

Answer: (b)

176. You should talk to your mother more often.

(a)  Your mother should be talked to by you more often.

(b)  Talking to your mother more often should be done by you.

(c)  It is more often that you should be talking to your mother.

(d)  You shall talk to y our mother more than often.

Answer: (a)

177. I always carry a bottle of water with me.

(a)  Carrying a bottle of water is always done by me.

(b)  A bottle of water is always carried by me.

(c)  It is with me that I always am carrying a bottle of water.

(d)  Always I carried a bottle of water with myself.

Answer: (b)

178. I have known this place since I was a kid.

(a)  This p lace has been known to me since I was kid.

(b)  Knowing this place since I was a kid is being done by me.

(c)  Since I was a kid that is a known place to me.

(d)  I have been knowing this place since the time I was a kid.

Answer: (a)

179. She dances the Salsa as if she were twenty something.

(a)  Dancing of the Salsa was done by her as if she were twenty something.

(b)  She was twenty something when she danced the Salsa.

(c)  She becomes twenty something while dancing the Salsa.   

(d)  The Salsa is danced by her as if she were twenty something.

Answer: (d)

180. If you hit a century the selectors will surely notice you.

(a)  If hitting a century is done by you the selectors will surely be noticing you.

(b)  If a century is hit by you, you will surely be noticed by the selectors.

(c)  Noticing of you will surely be done by the selectors if you hit a century.

(d)  When you will be hitting a century then the selectors will be surely noticing you.

Answer: (b)

Directions (Q. Nos. 181-185) In the following passage, some of the words have been left out. Read the passage carefully and select the correct answer for the given blank out of the four alternatives.

Never write down a phonetic ………. without fully understanding its meaning in every detail. A key to the various marks will be ……… at the bottom of every page of the dictionary and the student should refer to this ……… . In the front part o the dictionary there will also be found an ……. of all possible sounds that any letter may have; and every sound that any letter may have may be indicated by a peculiar mark, ……….. several letters may represent the same sound there are a variety of symbols for the same sound.

181. down a phonetic ……….. without fully

(a)  from

(b)  form

(c)  for

(d)  of

Answer: (b)

182. various marks will be …….. at the bottom of every

(a)  found

(b)  find

(c)  finding

(d)  to find

Answer: (a)

183. should refer to this …….. . In the front part

(a)  frequency

(b)  frequently

(c)  frequent

(d)  frequenting

Answer: (b)

184. be found an …….. of all possible

(a)  explained

(b)  explaining

(c)  explanation

(d)  explain

Answer: (c)

185. peculiar mark, ……… several letters may represent

(a)  of

(b)  form

(c)  off

(d)  since

Answer: (d)

Directions (Q. Nos. 186-190) In the following passage, some of the words have been left out. Read the passage carefully and select the correct answer for the given blank out of the four alternatives.

Sometimes fate is like a small sandstorm that keeps ……. directions. You change direction but the sandstorm chases you. You turn again, but the storm adjusts. Over ….. over you play this out, like some ominous dance with death just before dawn. Why? Because this storm isn’t something that blew in ………. far away, something that blew in …….. far away, something that has nothing to do with you. This storm is you. Something ………. of you. So all you can do is given in to it, step right inside the storm, closing your eyes and plugging up your ears so the sand doesn’t get in and walk ………. it, step by step.

186. sandstorm that keeps ………. directions.

(a)  change

(b)  changing

(c)  changed

(d)  to change

Answer: (b)

187. Over …….. over you pay this out.

(a)  and

(b)  since

(c)  but

(d)  or

Answer: (a)

188. something that blew in ……. far away,

(a)  for

(b)  of

(c)  from

(d)  form

Answer: (c)

189. Something ……… of you.

(a)  into

(b)  on

(c)  inner

(d)  inside

Answer: (d)

190. and walk ………. it, step by step.

(a)  through

(b)  though

(c)  threw

(d)  thought

Answer: (a)

Directions (Q. Nos. 191-195) A passage is given with 5 questions following it. Read the passage carefully and choose the best answer to each question out of the four alternatives.

Volcanic eruptions create beautiful surreal panorama like the unique landscape in Turkey. Here, rock structures of different shapes and sizes are a result of volcanic outbursts called “Fairy Chimneys” of Goreme in the Cappadocia region, Turkey, these rock sites are a major tourists attraction. Cappadocia lies in Central Antolia in Turkey with the Taurus Mountains in the south separating if from the Mediterranean Sea. The rock formations that we see today can be traced back to millions of years. Several volcanic eruptions through deep fractures threw up rocks, lava, ash and other materials, After the eruption, a dense ash covered the entire area. Soon the ejected ash solidified into a soft/porous rock called ‘tuff’.

The soft rocks were exposed to sub aerial erosion by wind, water and climatic changes (temperature changes). The rocks expanded in summer as they heated up, and broke up in winter as they froze. Added to this, the icy water collected in the cracks led to disintegration. Cappadocia gets heavy rain, and this too has played its part in altering the landscape. Rainwater has created sweeping curves on the sides of the valleys. The rivers washed away the eroded pieces of rocks and the harder rocks stayed on. Today, you see them in different shapes-capped chimneys, pillars mushrooms and conical formations. As erosion constantly changes the landscape of the cork structures in Cappadocia, it can pose a danger to the habitation.

191. What is ‘tuff’?

(a)  Solidified ash

(b)  A type of volcano

(c)  Local name for the chimneys

(d)  Dense rocks

Answer: (a)

192. What is to the south of Cappadocia?

(a)  Goreme

(b)  Volcanoes

(c)  Turkey

(d)  Taurus Mountains

Answer: (d)

193. When do the rocks break-up?

(a)  Summer

(b)  During rains

(c)  When volcanoes erupt

(d)  Winter

Answer: (d)

194. Why are there curves on the sides of the valley?

(a)  Due to winds

(b)  Due to changing temperature

(c)  Due to rain water

(d)  Due to human activity

Answer: (c)

195. How old are these rock formations?

(a)  Billions of years

(b)  Thousands of years

(c)  Millions of years

(d)  Hundred of years

Answer: (c)

Directions (Q. Nos. 196-200) A passage is given with 5 questions following it. Read the passage carefully and choose the best answer to each question out of the four alternatives.

The Berber are a group of people indigenous to North Africa. In the Nalut district of the Tripolitania region, Ghadames, an oasis town is known as the “pearl of the desert”. It is one of the oldest pre-Saharan cities and a grand example of a traditional settlement. Flying over, I was surprised to see just houses and more houses. There didn’t seem to be any pathways or alleys. Puzzled I few down to investigate. I found that each house is built in such a way that it hangs over the alleys creating an underground network of passages. The ground floor of the house is used to store supplies, the next floor is for the use of the family and the open-air terraces are reserved for women. They were talking a language I did not understand. I later learnt it was Ghadam’s, a Berger language. Towards the end of the month, for three days the old city of Ghadames comes alive with feasting, singing and dancing. It happens at the end of the date harvest. When all the work is done, the people head to the city’s World Heritage-listed old quarter to eat date and celebrate. The people of the old quarter moved out in the mid-1980s, and live in the modern town. But, on this occasion they return to their family homes in the old city. They throw open the doors for singing, dancing and other festivities. The covered alleys provide shelter from the hot Sahara sun.

196. ‘The Berber’ refers to

(a)  A group of settlements in Tripolitania region

(b)  A group of people

(c)  A species of native animals of North Africa

(d)  An Oasis town

Answer: (b)

197. While flying over why we won’t see and pathways in Ghadames?

(a)  The people walk from roof top to roof top.

(b)  Houses are built in such a way that they cover the pathways.

(c)  The pathways are built underground.

(d)  In those days there was no concept of pathways.

Answer: (b)

198. Which part of the house is kept specially for the women folk?

(a)  The terrace

(b)  The ground floor

(c)  The floor above the ground floor

(d)  The floor below the terrace

Answer: (a)

199. When are the 3 days of celebration?

(a)  At the beginning of the date invest

(b)  At the beginning of the holy month

(c)  At the end of the date harvest

(d)  At the end of the holy year

Answer: (c)

200. Which of the following is true?

(a)  Tripolitania is a part of Nalut

(b)  Nalut is a Berber language

(c)  Ghadames town is a part of Nalut district

(d)  The Berber is a part of Nalut

Answer: (c)

SSC CGL (Tier 1) Exam 2018 Held on March 28, 2018 Question Paper With Answer Key

SSC CGL (Tier 1) Exam 2018 Held on March 28, 2018
SSC CGL (Tier 1) Exam 2018 Held on March 28, 2018 Question Paper With Answer Key

SSC CGL (Tier 1) Exam 2018 Held on March 28, 2018

Directions (Q. Nos. 1-2) In the following questions, some part of the sentence may have errors. Find out which part of the sentence has an error and select the appropriate option. If a sentence is free from error, select ‘No Error’.

1. The of January 26 was chosen (1)/ to commemorate the Purna Swaraj (2)/ declaration for the complete independence, of 1930 (3) No error (4)

(a)  1

(b)  2

(c)  3

(d)  4

Answer: (a)

2. A Creole language is (1)/stable natural language develop from (2)/ a mixture of difference languages. (3) No error (4)

(a)  1

(b)  2

(c)  3

(d)  4

Answer: (b)

Directions (Q. Nos. 3-4) In the following questions, the sentence given with blank of filled in with an appropriate word. Select the correct alternative out of the four and indicate it by selecting the appropriate option.

3. She was discharged ………. the hospital after treatment.

(a)  from

(b)  of

(c)  with

(d)  as

Answer: (a)

4. The mechanic …….. the car according to his won liking.

(a)  tweaking

(b)  is tweaked

(c)  will tweak

(d)  have tweaked

Answer: (c)

Directions (Q. Nos. 5-6) In the following questions, out of the given four alternative, select the one which best expresses the meaning of the given word.

5. Dour

(a)  Cheery

(b)  Pleasant

(c)  Morose

(d)  Approachable

Answer: (c)

6. Scrumptious

(a)  Stale

(b)  Crushable

(c)  Devotedness

(d)  Tasty

Answer: (d)

Directions (Q. Nos. 7-8) In the following questions, out of  the given four alternatives, select the one which is opposite in meaning of the given word.

7. Exquisite

(a)  Elaborate

(b)  Consummate

(c)  Sensitive

(d)  Rough

Answer: (d)

8. Dogmatic

(a)  Flexible

(b)  Arrogant

(c)  Formal

(d)  Stubborn

Answer: (a)

9. Rearrange the parts of the sentence in correct order.

A serious critic

P : meaning of a work of art

Q : has to comprehend the particular

R : content, unique structure and the special

(a)  QRP

(b)  RQP

(c)  PQR

(d)  QPR

Answer: (a)

10. A sentence has been given in Active/Passive Voice. Out of the four given alternatives, select the one which best expresses the same sentence in Passive/Active Voice.

The boss made the assistant work late.

(a)  The assistant was made to work late by the boss.

(b)  The assistant is being urged to work late.

(c)  The assistant was make to work late by the boss.

(d)  The assistant is working late due to the boss.

Answer: (a)

11. A sentence has been given in Direct/Indirect Speech. Out of the four given alternatives, select the one which best expresses the same sentence in Indirect/Direct Speech.

Sneha said, “I am buying a gaming console today”.

(a)  Sneha said that she is buying a gaming console today.

(b)  Sneha says she was buying a gaming console that day.

(c)  Sneha say that she is buying a gaming console today.

(d)  Sneha said that she was buying a gaming console that day.

Answer: (b)

12. In the following question, a word has been written in four different ways out of which only one is correctly spelt. Select the correctly spelt word.

(a)  Handkurchief

(b)  Handkerchief

(c)  Handkerchef

(d)  Handkurchef

Answer: (b)

Directions (Q. Nos. 13-17) In the following passage, some of the words have been left out. Read the passage carefully and select the correct answer for the given blank out of the four alternatives.

It is this fact which makes it ……… necessary that our interest shall be broad and pure if our thoughts are to be of this type. It is …….. enough that we have the strength to drive from our minds a wrong or  impure thought which seeks. …………. . To stand guard as a policeman over our thoughts to see that no unworthy one enters, requires ……. much time and energy. Our interest must be of a such a nature as to lead us away from the field of unworthy thoughts ……. we are to be free from their tyranny.

13. Which makes it ………. necessary that our intersects.

(a)  vital

(b)  vitality

(c)  vitally

(d)  vitalism

Answer: (c)

14. It is ………. enough that we have

(a)  not

(b)  no

(c)  nor

(d)  neither

Answer: (a)

15. Which seeks ……… . To stand guard

(a)  entrance

(b)  enter

(c)  entering

(d)  enters

Answer: (a)

16. Enters, requires ……….. much time and energy.

(a)  to

(b)  very

(c)  of

(d)  too

Answer: (d)

17. Unworthy thoughts ………… we are to be

(a)  if

(b)  when

(c)  why

(d)  then

Answer: (a)

Directions (Q. Nos. 18-19) In the following questions, out of the four alternative, select the alternative which best expresses the meaning of the idiom/phrase.

18. A shot in the arm

(a)  Get hit by something.

(b)  Lose physical strength.

(c)  Lose motivation to do a task.

(d)  An encouraging stimulus.

Answer: (a)

19. You number in up

(a)  When you win by betting on your lucky number

(b)  Be the chosen one from a large group

(c)  Win a lottery or a lucky draw.

(d)  The time has come when someone is doomed to suffer a disaster.

Answer: (d)

Directions (Q. Nos. 20-21) In the following questions, out of the four alternatives, select the alternative which is the best substitute of the words/sentence.

20. The study and classification of speech sounds

(a)  Twang

(b)  Resonance

(c)  Oratory

(d)  Phonetics

Answer: (d)

21. Feeling or expressing pensive sadness

(a)  Mirthful

(b)  Exultant

(c)  Apathetic

(d)  Melancholic

Answer: (d)

Directions (Q. Nos. 22-23) In the following questions, out of the four alternatives, select the alternative which will improve the bracketed part of the sentence. In case no improvement is needed, select ‘no improvement.

22. You might be (to incline) to put the blame of infrastructure and officials, which is understandable.

(a)  inclined

(b)  incline

(c)  inclines

(d)  No improvement

Answer: (a)

23. LPG quickly (gain) popularity as it was easy to use and did not tarnish vessels.

(a)  gaining

(b)  will gain

(c)  gained

(d)  No improvement

Answer: (c)

24. The question below consists of a set of labelled sentences. Out of the four options given, select the most logical order of the sentences to form a coherent paragraph.

So we did get

(A) to read a whole lot

(B) this day onwards

(C) on how we can save the sparrow from

(a)  ABC

(b)  CBA

(c)  ACB

(d)  CAB

Answer: (c)

25. In the following question, four words are given of which one word is correctly spelt. Select the correctly spelt word.

(a)  frenatically

(b)  frenetically

(c)  freneticaly

(d)  frenatically

Answer: (c)

Directions (Q. Nos. 26-27) In the following questions, select the related word pair/number from the given alternatives.

26. Wine : Grapes : : ? : ?

(a)  Curd : Milk

(b)  Thread : Cloth

(c)  Wood : Table

(d)  Chair : Sit

Answer: (a)

27. 36 : 9 : : 49 : ?

(a)  24

(b)  13

(c)  32

(d)  15

Answer: (b)

28. In the following question, select the related letter/letters from the given alternatives

GHJ : JKM : : NOQ : ?

(a)  QRT

(b)  PQS

(c)  RST

(d)  OPR

Answer: (a)

29. In the following questions, select the odd word pair from the given alternatives.

(a)  Crop-Rice

(b)  Crop-Rubber

(c)  Crop-Wheat

(d)  Crop-Bajra

Answer: (b)

30. In the following question. four number pairs are given. The number of left side of (−) is related to the of the right side of (−) with some Logic/Rule/Relation. Three are similar on basis of same Logic/Rule/Relation. Select the odd one out from the given options.

(a)  17-34

(b)  19-38

(c)  23-46

(d)  27-56

Answer: (d)

31. In the following question, select the odd letter/letters from the given alternatives.

(a)  DHM

(b)  GKO

(c)  IMQ

(d)  LPT

Answer: (a)

32. From the given alternatives, according to dictionary, which word will come at second position?

(1) Charity    (2) Cardigan

(3) Caravan   (4) Carton

(5) Challenge

(a)  Charity

(b)  Challenge

(c)  Cardigan

(d)  Caravan

Answer: (c)

33. In the following questions, select the missing number from the given series.

1, 2, 6, 42, ?

(a)  1696

(b)  1036

(c)  1806

(d)  984

Answer: (c)

34. A series is given with one term missing. Select the correct alternative from the given ones that will complete the series.

L, O, Q, T, V, ?

(a)  X

(b)  W

(c)  Y

(d)  Z

Answer: (c)

35. ‘R + S’ means ‘R is mother of S’, ‘R – S’ means ‘R is wife of S’, ‘R × S’ means ‘R is son of S’ and ‘R is father of S’.

Which of the following represents K as the grandson of T?

(a)  K + T × J × R

(b)  J + K × L × T

(c)  R + T – S ÷ K

(d)  K + L × R ÷ T

Answer: (b)

36. From the given alternatives, select the word which cannot be formed using the letters of the given word.

Recalcitrant

(a)  Can

(b)  Rent

(c)  Recall

(d)  Calcite

Answer: (c)

37. In a certain code language, ‘FRANK’ is written as ‘GQBM’. How is ‘MARCO’ written in that code language?

(a)  NYTPB

(b)  MZSCQ

(c)  NZSBP

(d)  NQPBT

Answer: (c)

38. In a certain code language, ‘×’ represents ‘+’, ‘÷’ represents ‘×’, ‘−’ represents ‘÷’ and ‘+’ represents ‘−’. Find out the answer to the following question.

20 + 16 × 6 ÷ 10 – 4 = ?

(a)  13

(b)  12

(c)  30

(d)  19

Answer: (d)

39. The following equation is incorrect. Which two signs should be interchanged to correct the equation?

13 × 8 + 25 – 6 ÷ 10 = 20

(a)  ÷ and −

(b)  × and −

(c)  + and ÷

(d)  − and +

Answer: (b)

40. If 30%3 = 100, 70%14 = 50 and 20%8 = 25, then find the value of 50%5 = ?

(a)  110

(b)  30

(c)  100

(d)  20

Answer: (c)

41. Which of the following terms follows the trend of the given list?

CBABABAB, BACBABAB, BABACBAB, BABABACB, BCABABAB, ……… .

(a)  BABABCAB

(b)  CBABABAB

(c)  BACBABAB

(d)  BABCABAB

Answer: (d)

42. Two football players A and B from opposing teams start from the same corner. A runs 35 m East, then 15 m south, turns to his right and runs 60 m. Meanwhile B runs 20 m North, then 25 m West, turns left and runs 20 m. What is the position of B with respect to A?

(a)  15 m North

(b)  5 m North

(c)  16 m South

(d)  5 m South

Answer: (a)

43. In the question two statements are given, followed by two conclusions, I and II. You have to consider the statements to be true even if it seems to be at variance from commonly known facts. You have to decide which of the given conclusions, if any, follows from the given statements.

Statement (I) All sticks are canes

(II) Some wands are sticks

Conclusions (I) Some canes are wands

(II) No wands are canes

(a)  Only conclusions I follows

(b)  Only conclusions II follows

(c)  Both conclusions I and II follow

(d)  Neither conclusions I nor II follows

Answer: (a)

44. In the following figure, rectangle represents Authors, circle represents Art critics, triangle represents Tattooists and square represents Joggers. Which set of letters represents joggers who are either Art critics or Authors?

(a)  BAH

(b)  CBA

(c)  AHF

(d)  BH

Answer: (d)

45. A series is given with one term missing. Select the correct alternative from the given ones that will complete the series.

FNG, KSL, PXQ, UCV, ?

(a)  YIB

(b)  ZHA

(c)  AIB

(d)  BJC

Answer: (b)

46. In the following question, select the missing number from the given series.

9, 11, 14, 18, 23, ?

(a)  31

(b)  27

(c)  29

(d)  33

Answer: (c)

47. In the following question, four groups of three numbers are given. In each group the second and third number are related to the first number by a Logic/Rule/Relation. Three are similar on basis of same Logic/Rule/Relation. Select the odd one out from the given alternatives.

(a)  (8, 17, 33)

(b)  (3, 7, 12)

(c)  (9, 19, 37)

(d)  (15, 31, 61)

Answer: (b)

48. If a mirror is placed on the line MN, then which of the answer figures is the right image of the given figure?

Answer: (c)

49. Which of the following cube in the answer figure cannot be made based on the unfolded cube in the question figure?

Answer: (b)

50. A word is represented by only one set of numbers as given in any one of the alternatives. The sets of numbers given in the alternatives are represented by two classes of alphabets as shown in the given two matrices. The columns and rows Matrix-I are numbered from 0 to 4 and that of Matrix-II are numbered from 5 to 9. A letter from these matrices can be represented first by its row and next by its column for example ‘F’ can be represented by 31, 21 etc and ‘Y’ can be represented by 97, 98 etc. Similarly, you have to identify the set for the word ‘TONE’.

(a)  34, 75, 23, 75

(b)  30, 59, 43, 98

(c)  14, 75, 23, 69

(d)  68, 78, 95, 34

Answer: (d)

51. If the value √3 = 1.732, then calculate the value of 

(a)  1.732

(b)  −1.464

(c)  3.464

(d)  −1.732

Answer: (d)

52. Find the difference between place value and face value of 7 in 589573212.

(a)  693

(b)  6993

(c)  69993

(d)  699993

Answer: (c)

53. If  then find the value of 5y – 15.

(a)  110

(b)  130

(c)  150

(d)  160

Answer: (c)

54. If  then find the value of 

(a)  0

(b)  1

(c)  2

(d)  3

Answer: (b)

55. Calculate the distance of the chord from the centre of the circle, if the chord of length 48 cm is drawn in the circle of radius 25 cm.

(a)  7 cm

(b)  14 cm

(c)  49 cm

(d)  56 cm

Answer: (a)

56. If the angle subtended by the radius of the circle and the vertex B of the triangle ABC is 40°. Then calculate the value of ∠BAC

(a)  35°

(b)  45°

(c)  50°

(d)  65°

Answer: (c)

57. Weights of two friends A and B are in the ratio of 7 : 8. A’s weight increase by 10% and then total weight of A and B increase by 20%. What is the percentage increase in the B’s weight?

(a)  28.75

(b)  11.55

(c)  32.85

(d)  24.25

Answer: (a)

58. The income of X, Y and Z are in the ratio of 3 : 4 : 2 and their expenses in the ratio 5 : 5 : 2. If X saves Rs 3000 out of an income of Rs 18000, then what will be the savings of Y?

(a)  4500

(b)  9000

(c)  6000

(d)  7500

Answer: (b)

59. P and Q are two alloys which are prepared by mixing tin and lead in the ratio of 12 : 5 and 4 : 3 respectively. If equal quantities of alloys are melted to form third alloy R, then what is the ratio of tin and lead in alloy R?

(a)  84 : 65

(b)  57 : 35

(c)  76 : 43

(d)  78 : 47

Answer: (c)

60. What will be the average of the first 5 positive even numbers divisible by 9?

(a)  50

(b)  54

(c)  48

(d)  62

Answer: (b)

61. The simple interest on a sum for 20 years is 5/4 of the principal. What is the per annum rate of interest?

(a)  6.25%

(b)  12.5%

(c)  2.5%

(d)  5%

Answer: (a)

62. If Mohan purchase 13 articles for Rs 12 and sells al the articles at the rate of 12 articles for Rs 13, then what will be the profit percentage?

(a)  19.47

(b)  17.36

(c)  11.11

(d)  12.5

Answer: (b)

63. A tyre is sold at three successive discounts of 20%, 30% and 20%. What is the net discount?

(a)  62.8%

(b)  59.6%

(c)  55.2%

(d)  51.4%

Answer: (c)

64. What is the value of 

(a)  5/12

(b)  5/19

(c)  4/9

(d)  4/15

Answer: (a)

65. Robin can do a work alone in 9 days. Amit can do the same work alone in 6 days. Both of them finish the work together and they get a total Rs 2500. What is the share of Robin?

(a)  Rs 1700

(b)  Rs 1200

(c)  Rs 1500

(d)  Rs 1000

Answer: (d)

66. A boat covers 63 km upstream in 9 hours and covers the same distance downstream in 7 hours. What is the speed of the boat in still water?

(a)  8.7 km/h

(b)  8.5 km/h

(c)  7 km/h

(d)  8 km/h

Answer: (d)

Directions (Q. Nos. 67-70) The line graph shows the sales per employee of a certain company. Study the diagram and answer the following questions.

67. In which year were the Sales per employee greater than that of the previous year?

(a)  2014

(b)  2015

(c)  2017

(d)  2016

Answer: (d)

68. What was the difference in the Sales per employee between the years 2011 and 2015?

(a)  15

(b)  10

(c)  20

(d)  5

Answer: (c)

69. The Sales per employee in 2012 were greater than that in 2015 by

(a)  16.7%

(b)  20%

(c)  40%

(d)  28.5%

Answer: (c)

70. If the number of employees of the company was 80 in the year 2015 then what was the sales of the company in the year 2015

(a)  Rs 30 crore

(b)  Rs 40 crore

(c)  Rs 20 crore

(d)  Rs 50 crore

Answer: (b)

71. The lengths of the two diagonals of rhombus are 5 cm and 12 cm. Find its area.

(a)  30 cm2

(b)  15 cm2

(c)  60 cm2

(d)  45 cm2

Answer: (a)

72. What is the sum of the measures of all the interior angles of a regular polygon of 9 sides?

(a)  1260

(b)  1440

(c)  1800

(d)  2340

Answer: (a)

73. The surface area of a sphere is 1386 cm2. Find its diameter.

(a)  10.5 cm

(b)  42 cm

(c)  21 cm

(d)  31.5 cm

Answer: (c)

74. ∆PQR is right angled at Q. If m∠R = 60°, then find the value of (sec P – 1/√3)

(a)  (√6 – 6)/3√3

(b)  (1 – 3√2)/3

(c)  1/√3

(d)  2/√3

Answer: (c)

75. ∆PQR is right angled at Q. If cosec P = 25/24, then what is the value of cot R?

(a)  25/7

(b)  24/25

(c)  7/24

(d)  24/7

Answer: (d)

76. National Income refers to

(a)  money value of goods and services produced in a country during a year

(b)  money value of stocks and shares of a country during a year

(c)  money value of capital goods produced by a country during a year

(d)  money value of consumer goods produced by a country during a year

Answer: (a)

77. State Land Reform is under which part of the Constitution?

(a)  7th Schedule

(b)  8th Schedule

(c)  9th Schedule

(d)  10th Schedule

Answer: (c)

78. The remains of the resistant rocks found in the river eroded plain or peneplain are called

(a)  Monadnock

(b)  Jukenberg

(c)  Hum

(d)  No option is correct

Answer: (a)

79. Which of the following rivers does not make an estuary?

(a)  Narmada

(b)  Tapi

(c)  Mandovi

(d)  Mahanadi

Answer: (d)

80. Which ruler of Bengal had relations with China?

(a)  Ghiyasuddin Azamshah

(b)  Sikander Shah

(c)  Nusrat Shah

(d)  Hussein Shah

Answer: (a)

81. When was the Arya Samaj divided on principles?

(a)  1893

(b)  1895

(c)  1903

(d)  1913

Answer: (a)

82. Ustad Mansur was a famous painter of which Mughal ruler’s engime?

(a)  Akbar

(b)  Jahangir

(c)  Shah Jahan

(d)  Aurangzeb

Answer: (b)

83. Which country on October 23, 2017 signed the Paris agreement on climate change, leaving the United States and Syria as the only two countries holding out to sign this global climate pact?

(a)  Nicaragua

(b)  Colombia

(c)  Peru

(d)  Uganda

Answer: (a)

84. Which singer was in January, 2018, chosen for the 5th Edition of Yash Chopra Memorial Award 2018 for her outstanding contribution to the Hindi film industry?

(a)  Alka Yagnik

(b)  Asha Bhosle

(c)  Sunidhi Chauhan

(d)  Kavita Krishnamurthy

Answer: (b)

85. Where is the tomb of Mughal Emperor Jahangir in Pakistan?

(a)  Islamabad

(b)  Hyderabad

(c)  Lahore

(d)  Karachi

Answer: (c)

86. What is the number of groups and periods in the modern periodic table?

(a)  18, 7

(b)  18, 8

(c)  16, 7

(d)  16, 8

Answer: (a)

87. Which among the following is an example of mixture?

(a)  Milk

(b)  Gasoline

(c)  Liquefied petroleum gas

(d)  All options are correct

Answer: (d)

88. Which Schedule of Indian Constitution was added by the 74th Constitutional Amendment Act?

(a)  Ninth Schedule

(b)  Tenth Schedule

(c)  Eleventh Schedule

(d)  Twelfth Schedule

Answer: (d)

89. Which among the following comes under the elements of Basic Structure of Indian Constitution?

(I) Supremacy of Constitution

(II) Free and Fair Election

(III) Judicial Review

(a)  Only I

(b)  Only II

(c)  Both I and II

(d)  All of these

Answer: (d)

90. The plasma of the blood contains which among the following constituents?

(I) Proteins    (II) Salts      (III) Hormones

(a)  I and II

(b)  I and III

(c)  II and III

(d)  All of these

Answer: (d)

91. What is the basis of first level of classification among plants?

(a)  Differential plant body

(b)  Specialized tissues

(c)  Ability to bear seeds

(d)  No option is correct

Answer: (a)

92. On September 5, 2017 who launched an online quiz competition named ‘Prakriti Khoj’?

(a)  Health Ministry

(b)  Commerce Ministry

(c)  Environment Ministry

(d)  Home Ministry

Answer: (c)

93. Researchers at the University of illinoise have discovered new from of matter called Excitonium which was coined by ………. in the 1960’s.

(a)  Bertrand Halperin

(b)  Eugene Wigner

(c)  Paul Dirac

(d)  Felix Bloch

Answer: (a)

94. On October 24, 2017, the government of India announced a massive of …………. package to boost economy.

(a)  Rs 7 lakh crore

(b)  Rs 9 lakh crore

(c)  Rs 15 lakh crore

(d)  Rs 5 lakh crore

Answer: (b)

95. On November 22, 2017 cabinet approved expansion of which mission for a Pan India coverage in all the 640 districts of the country?

(a)  Make in India

(b)  Swatchh Bharat

(c)  Beti Bachao Beti Padhao

(d)  Skill India

Answer: (c)

96. A body is initially moving with a velocity of 5 m/sec. It undergoes an acceleration of 2 m/sec2 for 4 seconds. Find the displacement of this body in these 4 seconds.

(a)  4 m

(b)  72 m

(c)  36 m

(d)  8 m

Answer: (c)

97. If the velocity of a body is halved its kinetic energy

(a)  become 4 times

(b)  becomes 1/4th

(c)  is also doubled

(d)  becomes half

Answer: (b)

98. Plants utilize ……….. from the atmosphere of photosynthesis.

(a)  oxygen

(b)  nitrogen

(c)  carbon dioxide

(d)  hydrogen

Answer: (c)

99. Which of the statements given below are correct?

(A) Russia hosted the Golf 2017 Players Championship.

(B) Elina Svitolina won the Tennis 2017 Italian Open Women’s Singles.

(C) Henri Kontinen and John Peers won the Tennis 2017 ATP World Tour Finals Men’s Doubles.

(a)  Only A

(b)  Only B

(c)  A and B

(d)  A and C

Answer: (d)

100. In Microsoft Excel, left arrow key stroke leads to ……… cell movement on spreadsheet.

(a)  One cell up

(b)  One cell down

(c)  One cell left

(d)  One cell right

Answer: (c)

SSC Combined Graduate Level Tier-II Examination Held on 08-08-2017 Question Paper With Answer Key

SSC Combined Graduate Level Tier-II Examination Held on 08-08-2017
SSC Combined Graduate Level Tier-II Examination Held on 08-08-2017 Question Paper With Answer Key

SSC Combined Graduate Level Tier-II Examination Held on 08-08-2017

1. In the following question, select the related word from the given alternatives.

Goiter : Iodine : : Anemia : ?

(A)  Vitamin D

(B)  Iron

(C)  Vitamin E

(D)  Calcium

Answer: (B)

2. In the following question, select the related letter from the given alternatives.

DE : O : : AF : ?

(A)  H

(B)  K

(C)  J

(D)  I

Answer: (B)

3. In the following question, select the related number from the given alternatives.

56 : 41 : : 94 : ?

(A)  49

(B)  53

(C)  60

(D)  89

Answer: (A)

4. In the following question, select the odd word from the given alternatives.

(A)  Beneath

(B)  On top

(C)  Huge

(D)  Above

Answer: (C)

5. In the following question, select the odd letters from the given alternatives.

(A)  MN

(B)  NM

(C)  OL

(D)  ET

Answer: (D)

6. In the following question, select the odd number from the given alternatives.

(A)  253

(B)  473

(C)  143

(D)  633

Answer: (D)

7. Arrange the given words in the sequence in which they occur in the dictionary.

(1) Flinching  (2) Flintlock

(3) Flinpites  (4) Flintlocks

(5) Flinchers

(A)  15243

(B)  51243

(C)  51324

(D)  51342

Answer: (C)

8. In the following question, which one set of letters when sequentially placed at the gaps in the given letter series shall complete it?

_yzaa_y_xy_aazy_

(A)  xxzzx

(B)  xxaza

(C)  xzxzx

(D)  aazzx

Answer: (C)

9. In the following question, select the missing number from the given series.

19, 38, ?, 228, 684, 1368

(A)  108

(B)  113

(C)  114

(D)  138

Answer: (C)

10. There are five students P, Q, R, S and T having different heights in a class. P’s height is more than only one student. Q’s height is more than S and P but not more than R. S’s height is more than P, R is not the smallest. Who is having the maximum height in the class?

(A)  Q

(B)  R

(C)  S

(D)  T

Answer: (B)

11. In a row of 74 girls, Shweta is 27th from left end. Palak is 7th to the right of Shweta. What is Palak’s position from the right end of the row?

(A)  40

(B)  41

(C)  42

(D)  44

Answer: (B)

12. In the following question, from the given alternative words, select the word which can be formed using the letters of the given word-

CAPITULATE

(A)  CAPABLE

(B)  LUPIN

(C)  PITTY

(D)  TALE

Answer: (D)

13. In a certain code language, ‘BIOLOGY’ is written as ‘YRLOLTB’. How is ‘PHYSICS’ written in that code language?

(A)  KSXRBHH

(B)  KSBHRXH

(C)  XHRHBSK

(D)  KSBHXRH

Answer: (B)

14. In the following question, correct the equation by interchanging two signs.

6 + 8 ÷ 4 – 4 = 8

(A)  ÷ and =

(B)  ÷ and +

(C)  ÷ and −

(D)  + and −

Answer: (D)

15. If 4 × 9 × 3 = 4 and 5 × 3 × 1 = 3, the 9 × 9 × 7 = ?

(A)  5

(B)  6

(C)  7

(D)  9

Answer: (A)

16. In the following question, select the number which can be placed at the sign of question mark (?) from the given alternatives.

(A)  108

(B)  112

(C)  118

(D)  120

Answer: (D)

17. How many rectangles are there in the given figure?

(A)  9

(B)  10

(C)  11

(D)  12

Answer: (C)

18. In the following question below are given some statements followed by some conclusions. Taking the given statements to be true even if they seem to be at variance from commonly known facts, read all the conclusions and then decided which of the given conclusion logically follows the given statements.

Statements:

(I) All cups are plates.

(II) Some plates are glass.

Conclusions:

(I) Some glasses are cups

(II) All glasses are cups.

(A)  Only conclusion I follows

(B)  Only conclusion II follows

(C)  Neither conclusion I nor conclusion II follows

(D)  Both conclusions follow

Answer: (C)

19. From the given options, which figure can be formed by folding the figure given in the question?

Answer: (B)

20 Identify the diagram that best represents the relationship among the given classes.

Answer: (B)

21. Which answer figure will complete the pattern in the question figure?

Answer: (B)

22. From the given answer figures, select the one in which the question figure is hidden/embedded.

Answer: (B)

23. A piece of paper is folded and punched as shown below in the question figures. From the given answer figures, indicate how it will appear when opened?

Answer: (D)

24. If a mirror is placed on the line AB, then which of the answer figure is the right image of the given figure?

Answer: (D)

25. A word is represented by only one set of numbers as given in any one of the alternatives. The sets of numbers given in the alternatives are represented by two classes of alphabets as shown in the given two matrices. The columns and rows of Matrix-I are numbered from 0 to 4 and that of Matrix-II are numbered from 5 to 9. A letter from these matrices can be represented first by its row and next by its column, for example. ‘H’ can be represented by 34, 41, etc., and ‘T’ can be represented by 59, 97, etc. Similarly, you have to identify the set for the word “STRAW”.

(A)  00, 78, 13, 67, 23

(B)  12, 59, 01, 55, 10

(C)  24, 97, 20, 86, 31

(D)  43, 66, 44, 98, 43

Answer: (A)

26. ‘Neo-Malthusian Theory’ is associated with which of the following?

(A)  Employment

(B)  Poverty

(C)  Resource scarcity

(D)  Income

Answer: (C)

27. Match the following :

Sector                      Example

(a) Primary             (1) Consultancy

(b) Secondary        (2) Apiculture

(c) Tertiary             (3) Shoe Factory

(A)  (a) – 2; (b) – 3; (c) – 1

(B)  (a) – 3; (b) – 1; (c) – 2

(C)  (a) – 1; (b) – 3; (c) – 2

(D)  (a) – 3; (b) – 2; (c) – 1

Answer: (A)

28. How many Fundamental Duties are mentioned in Indian Constitution ?

(A)  Five

(B)  Seven

(C)  Nine

(D)  Eleven

Answer: (D)

29. Part IV of Constitution of India deals with which of the following?

(A)  The Union

(B)  The States

(C)  Fundamental Rights

(D)  Directive Principles of State Policy

Answer: (D)

30. Who gave the slogan ‘Back to the Vedas (Vedo ki aur lauto)’ ?

(A)  Guru Nanak Dev Ji

(B)  Dayanand Saraswati

(C)  Swami Vivekananda

(D)  Raja Ram Mohan Roy

Answer: (B)

31. Todarmal was the famous revenue minister of which Mughal Emperor?

(A)  Shah Jahan

(B)  Bahadur Shah Jafar

(C)  Akbar

(D)  Aurangzeb

Answer: (C)

32. Gravitational force is maximum at which of the following place?

(A)  At equator

(B)  At tropic of cancer

(C)  At tropic of Capricorn

(D)  At poles

Answer: (D)

33. Which of the following devices is used to measure humidity?

(A)  Hydrometer

(B)  Hygrometer

(C)  Psychometer

(D)  Anemometer

Answer: (B)

34. Which of the following diseases is caused by female Anopheles mosquito?

(A)  Chicken Pox

(B)  Malaria

(C)  Black Fever

(D)  Cholera

Answer: (C)

35. Which part of the plant gives us saffron?

(A)  Roots

(B)  Petals

(C)  Stem

(D)  Stigma

Answer: (D)

36. Which of the following transports water from the roots of the plant to its leaves?

(A)  Xylem

(B)  Phloem

(C)  Both xylem and phloem

(D)  Cortex

Answer: (A)

37. Which of the following is not a vector quantity?

(A)  Momentum

(B)  Displacement

(C)  Torque

(D)  Speed

Answer: (D)

38. At what temperature (in Fahrenheit) pure water freezes?

(A)  32

(B)  0

(C)  48

(D)  37

Answer: (A)

39. In which graphics, digital photos and scanned images are typically stored with extensions such as .bmp, .png, .jpg, .tif or .gif?

(A)  Bitmap

(B)  Pixels

(C)  Plane

(D)  Both Bitmap and Pixels

Answer: (A)

40. Process of gaining electrons is known as……….

(A)  oxidation

(B)  reduction

(C)  radiation

(D)  both oxidation and reduction

Answer: (B)

41. Which of the following metal (shown by its symbol) is generally used for making filaments of bulb?

(A)  Fe

(B)  An

(C)  Ag

(D)  W

Answer: (D)

42. Which of the following trees shed their leaves once in a year?

(A)  Deciduous trees

(B)  Coniferous trees

(C)  Evergreen trees

(D)  Both deciduous and coniferous trees

Answer: (A)

43. The Entrepreneurship Development Scheme (EDS) has been launched under New Economy Development Policy (NEDP) on January 19, 2017 at –

(A)  New Delhi

(B)  Mizoram

(C)  Gujarat

(D)  Uttar Pradesh

Answer: (B)

44. Who invented Dynamite?

(A)  J. B. Dunlop

(B)  Alfred Nobel

(C)  James Simons

(D)  Peter Hargreaves

Answer: (B)

45. ‘Rovers cup’ is associated with which of the following sport?

(A)  Football

(B)  Hockey

(C)  Golf

(D)  Basketball

Answer: (A)

46. Dandia is a form of dance associated with which of the following state?

(A)  Haryana

(B)  Punjab

(C)  Gujarat

(D)  Rajasthan

Answer: (C)

47. Who among the following was awarded with Dhronacharya Award 2016 in the field ‘Swimming (Lifetime)’ ?

(A)  Shri Raj Kumar Sharma

(B)  Shri Sagar Mal Dhayal

(C)  Shri S. Pradeep Kumar

(D)  Shri Nagapuri Ramesh

Answer: (C)

48. ‘Ace Against Odds’ is an autobiography of which sports person?

(A)  Vishwanathan Anand

(B)  Sania Mirza

(C)  Abhinav Bindra

(D)  Anjali Bhagwat

Answer: (B)

49. Under the framework of the new partnership with which country, Victory University will help India to Establish a National Sports University?

(A)  Portugal

(B)  Australia

(C)  Vietnam

(D)  USA

Answer: (B)

50. With which country India exchanged its border maps?

(A)  China

(B)  Sri Lanka

(C)  Pakistan

(D)  Bangladesh

Answer: (D)

51. What least value which should be added to 1812 to make it divisible by 7, 11 and 14?

(A)  12

(B)  36

(C)  72

(D)  154

Answer: (B)

52. A is 1.5 times efficient than B therefore takes 8 days less than B to complete a work. If A and B work on alternate days and A works on first day, then in how many days the work will be complete?

(A)  17

(B)  19

(C)  19.5

(D)  21

Answer: (B)

53. One of the diagonal of a rhombus is 70% of the other diagonal?

(A)  3 : 10

(B)  3 : 20

(C)  7 : 20

(D)  7 : 10

Answer: (C)

54. A shopkeeper sells a table at a discount of 20% and earns a profit of 60%. If he sells the same table at 40% discount, then what will be his new profit per cent?

(A)  20

(B)  30

(C)  35

(D)  40

Answer: (A)

55. If A/3 = B/2 = C/5, then what is the value of ratio (C + A)2 : (A + B)2 : (B + C)2 ?

(A)  9 : 4 : 25

(B)  25 : 4 : 9

(C)  64 : 25 : 49

(D)  49 : 25 : 64

Answer: (C)

56. 5 years ago the average age of a family which includes father, mother and a son was 35 years. 3 years ago the average age of father and mother was 46 years. What is the present age (in years) of the son?

(A)  20

(B)  24

(C)  26

(D)  22

Answer: (D)

57. The cost price of 60 articles is same as the selling price of x articles. If there is a profit of 20%, then what is the value of x?

(A)  15

(B)  30

(C)  50

(D)  80

Answer: (C)

58. A person scores 45% of the total marks in the exam and still fails by 40 marks. The passing percentage of the exam is 55%. What is the maximum marks of the exam?

(A)  300

(B)  350

(C)  400

(D)  500

Answer: (C)

59. A man starts running from point P at 11 : 00 a.m. with a speed of 10 km/hr. He runs for 2 hours and then takes 1 hours rest. He continues this till he is caught by another man who starts at 2 : 00 p.m. from point P and runs nonstop at a speed of 15 km/hr towards the first man. At what time (in p.m.) will the first man be caught?

(A)  6 : 20

(B)  4 : 40

(C)  6 : 00

(D)  5 : 30

Answer: (B)

60. The difference of compound interest and simple interest for 3 years and for 2 years are in ratio 23 : 7 respectively. What is rate of interest per annum (in %) ?

(A)  200/7

(B)  100/7

(C)  300/7

(D)  400/7

Answer: (A)

61. If (x2/yz) + (y2/zx) + (z2/xy) = 3, then what is the value of (x + y + z)3?

(A)  0

(B)  1

(C)  2

(D)  3

Answer: (A)

62. If x1/4 + x1/4 = 2, then what is the value of x81 + (1/x81) ?

(A)  −2

(B)  0

(C)  1

(D)  2

Answer: (D)

63. If a(a + b + c) = 45, b(a + b + c) = 75 and c(a + b + c) = 105, then what is the value of (a2 + b2 + c2)?

(A)  75

(B)  83

(C)  217

(D)  225

Answer: (B)

64. If x2 + (1/x2) = 1, then what is the value of x48 + x42 + x36 + x30 + x24 + x18 + x12 + x6 + 1 ?

(A)  −9

(B)  0

(C)  1

(D)  9

Answer: (C)

65. In the given figure, PQRS is a trapezium in which PM/SN, NR = 9 cm, PS = 12 cm, QM = NR and NR = SN. What is the area (in cm2) of trapezium?

(A)  170

(B)  182

(C)  189

(D)  191

Answer: (C)

66. In the given figure, PQR is an equilateral triangle and PS is the angle bisector of ∠ What is the value of RT : RQ?

(A)  1 : 2

(B)  1 : 1

(C)  2 : 1

(D)  1 : 3

Answer: (B)

67. Two chords of length 20 cm and 24 cm are drawn perpendicular to each other in a circle of radius is 15 cm. What is the distance between the points of intersection of these chords (in cm) from the center of the circle?

(A)  √114

(B)  √182

(C)  √206

(D)  √218

Answer: (C)

68. In the given figure, QRTS is a cyclic quadrilateral. If PT = 5 cm, SQ = 4 cm, PS = 6 cm and ∠PQR = 63°, then what is the value (in cm) of TR?

(A)  3

(B)  7

(C)  9

(D)  15

Answer: (B)

69. What is the simplified value of sin2(90 – θ) – [{sin (90 – θ) sin θ}/tan θ]?

(A)  1

(B)  cosec θ

(C)  0

(D)  cos θ

Answer: (C)

70. What is the simplified value of 

(A)  sin θ

(B)  1 – sin 2θ

(C)  1 +  sin 2θ

(D)  1 – sin θ

Answer: (B)

71. If 5 sec θ – 3 tan θ = 5, then what is the value of than θ – 3 sec θ?

(A)  1

(B)  2

(C)  3

(D)  4

Answer: (C)

Directions – (Q. 72-75) The table below shows the distribution of number of people living in 8 different countries and the per capita income of each of the countries. The total population of these countries taken together is 200 crores.

Per capita income = Total GDP of country/Population of the country.

72. What is the difference (in crores) in population of the most and the least populated country?

(A)  18

(B)  32

(C)  34

(D)  36

Answer: (D)

73. What is the total GDP (in crore dollars) of country?

(A)  27120

(B)  31640

(C)  38280

(D)  44660

Answer: (B)

74. What is the total GDP (in crore dollars) for the country with the third lowest per capita income?

(A)  181900

(B)  108460

(C)  145200

(D)  166560

Answer: (D)

75. Which country has the highest total GDP?

(A)  Country 1

(B)  Country 2

(C)  Country 3

(D)  Country 8

Answer: (C)

Directions – (Q. 76 and 77) In the following questions, some part of the sentence may have errors. Find out which pat of the sentence has an error and select the appropriate option. If a sentence is free from error, select ‘No error’.

76. Hardly had I reached the (A) / exhibition where I learnt (B) / about the major robbery. (C) / No error (D)

(A)  A

(B)  B

(C)  C

(D)  D

Answer: (B)

77. Noe sooner did the sun rise (A) / when we resumed the journey (B) / after having a hasty breakfast. (C) / No error (D)

(A)  A

(B)  B

(C)  C

(D)  D

Answer: (B)

Directions – (Q. 78 and 79) In the following question, the sentence given with blank to be filled in with an appropriate word. Select the correct alternative out of the four and indicate it by selecting the appropriate option.

78. He ………… to the problem of air pollution in his speech.

(A)  averted

(B)  adverted

(C)  exclaimed

(D)  mentioned

Answer: (B)

79. Rohan is so magnanimous that everyone is always ……….. to help him in his project.

(A)  eager

(B)  enthusiastic

(C)  reluctant

(D)  ignorant

Answer: (A)

Directions – (Q. 80-81) In the following question, out of the four alternatives, select the word similar in meaning to the word given –

80. Succulent

(A)  Sucking

(B)  Soft

(C)  Juicy

(D)  Pale

Answer: (C)

81. Congregation

(A)  Discussion

(B)  Attention

(C)  Contraction

(D)  Assembly

Answer: (D)

Directions – (Q. 82-83) In the following question, out of the four alternatives, select the word opposite in meaning to the word given-

82. Evanescent

(A)  Enticing

(B)  Fleeting

(C)  Erratic

(D)  Elusive

Answer: (A)

83. Penegyric

(A)  Noxious

(B)  Criticism

(C)  Fantasy

(D)  Grandeur

Answer: (B)

Directions – (Q. 84-85) In the following question, out of the four alternatives, select the alternative which best express the meaning of the Idiom/Phrase-

84. To put a spoke in one’s wheel

(A)  To be advantageous at the time of difficulty

(B)  to maintain under all circumstances

(C)  To blame the other party

(D)  To put a difficulty in the way of progress

Answer: (D)

85. At loggerheads

(A)  To suffer

(B)  In conflict with someone

(C)  To face tough competition

(D)  To be in do or die situation

Answer: (B)

Directions – (Q. 86-87) Improve the bracketed part of the sentence.

86. She is (the best and wisest) girl in the class.

(A)  best and wisest

(B)  a best and a wisest

(C)  the best and the wisest

(D)  No improvement

Answer: (C)

87. The people of Japan are (wiser than America.)

(A)  wiser than that of America

(B)  wiser than those of America

(C)  wiser to that of America

(D)  No improvement

Answer: (B)

Directions – (Q. 88-89) In the following question, out of the four alternatives, select the alternative which is the best substitute of the phrase.

88. One skilled in telling stories

(A)  Ventral

(B)  Fanatic

(C)  Raconteur

(D)  Tyro

Answer: (C)

89. Fear of Fire

(A)  Arsonphobia

(B)  Astraphobia

(C)  Astrophobia

(D)  Arrhenphobia

Answer: (A)

Directions – (Q. 90-91) In the following question, four words are given out of which one word in incorrectly spelt. Find the incorrectly spelt word.

90. 

(A)  Hygeine

(B)  Fascist

(C)  Career

(D)  Apparel

Answer: (A)

91.

(A)  Reside

(B)  Revise

(C)  Advise

(D)  Demice

Answer: (D)

Directions – (Q. 92-93) The question below consist of a set of labelled sentences. Out of the four options given, select the most logical order of the sentences to form a coherent paragraph.

92. P : He raised his voice against idolworship.

Q : People are generally very open and accept social changes with a positive attitude.

R : Swami Dayanand Saraswati is remembered with revounce and affection among the social reformers of the nineteenth century.

S : India is a country which respect spiritualism much more than materialism.

(A)  QRSP

(B)  PRSQ

(C)  RSPQ

(D)  SQRP

Answer: (D)

93. P : For what we need who possess not only high skills but high value as well.

Q : So the need of the hour is not only skill-based education but also value-inspired and value-based education.

R : When we talk of investing in education, a question arises as to what kind of education needs to be given to the youngster.

S : Our aim is to evolve not only an affluent society, but an egalitarian, just, humane and compassionate society.

(A)  SPRQ

(B)  PRQS

(C)  RQSP

(D)  RSPQ

Answer: (D)

94. In the following question, a sentence has been given in Active/ Passive voice. Out of four alternatives suggested, select the one which best expresses the same sentence in Passive/ Active voice.

Please guide me.

(A)  You are requested to guide me

(B)  You have been requested to guide me

(C)  You were requested to guide me

(D)  You are being requested to guide me

Answer: (A)

95. In the following question, a sentence has been given in Direct/Indirect speech. Out of four alternatives suggested, selected the one which best expresses the same sentence in Indirect/Direct speech.

The teacher said, “The Earth revolves around the sun.”

(A)  The teacher said that the Earth is being revolving around the sun.

(B)  The teacher said that the Earth revolves around the sun.

(C)  The teacher said that the Earth revolved around the sun.

(D)  The teacher said that the Earth had been revolving around the sun.

Answer: (B)

Directions – (Q. 96-100) In the following passage some of the words have been left out. Read the passage carefully and select the correct answer for the given blank out of the four alternatives.

The ……….. of energy in India are met from both commercial and non-commercial sources. The most ……. sources of energy today are coal and natural gas, hydro electricity and nuclear power. On the other hand, firewood, cow-dung cakes and vegetable waste etc. …….. non-commercial forms of energy, while non-commercial forms of energy ……. demand, coal is the …… source of commercial energy in India.

96. The ……. of energy in India are met from both commercial.

(A)  requirements

(B)  sources

(C)  availability

(D)  lessen

Answer: (A)

97. and non-commercial sources. The most …….. sources of energy today are coal and natural gas.

(A)  vital

(B)  insignificant

(C)  important

(D)  expensive

Answer: (C)

98. On the other hand, firewood, cow-dung cakes and vegetable waste etc. …… non-commercial forms of energy.

(A)  constitutes

(B)  combines

(C)  mixes

(D)  collaborates

Answer: (A)

99. While non commercial forms of energy …….. demand.

(A)  need

(B)  meet

(C)  shed

(D)  rests

Answer: (B)

100. Coal is the …… source of commercial energy in India.

(A)  smallest

(B)  hardest

(C)  largest

(D)  heaviest

Answer: (C)

© Copyright Entrance India - Engineering and Medical Entrance Exams in India | Website Maintained by Firewall Firm - IT Monteur